You are on page 1of 192

STRICTLY BASED ON LATEST SYLLABUS

(30% LESS DURING COVID-19)

2023
NCERT Based

EXAM time
FOR 100% SURE SUCCESS

Chemistry
Class-12
ØekeâeMekeâ :
Strictly based on latest (one paper)
examination patterns as adopted
by UPMSP, Prayagraj
27-Gulshan Vihar, Jansath Road,
Muzaffarnagar-251001 (U.P.)
Phone : 0131-2660440
e-mail : aviralpublications@gmail.com EXAM time
Your faithful partner on way of success...
Dear Students,
is based on latest examination
© Publisher pattern as adopted by UPMSP w.e.f. 2018-
19. Each chapter includes objective type,
very short answer type, short answer type
and long answer type questions alongwith
5 unsolved test papers for practice.
To check the level of preparation five
Pre-Board Online Question Papers will be
available on shribalajibooks.com before
Board Examination. Students may
download these papers time to time by
scanning the QR code printed at the title.
Topics deleted for session 2022-23 due
to Covid-19, have been deleted.

We hope that will cater your


needs and will ensure your grand success.
efkeâmeer Yeer $egefš kesâ efveJeejCe nsleg hee"keâeW kesâ megPeeJe meeoj
Deeceefv$ele nw~
efkeâmeer Yeer heefjJeeo kesâ efueS vÙeeefÙekeâ #es$e cegpeheäheâjveiej
— Publisher
nesiee~
Fme hegmlekeâ keâes $egefšjefnle ØekeâeefMele keâjves keâe hetCe&
ØeÙeeme efkeâÙee ieÙee nw, efheâj Yeer Ùeefo DeefveÛÚe mes FmeceW
keâesF& $egefš jn ieF& nes, lees Gmemes Glhevve efkeâmeer Yeer #eefle
kesâ efueS uesKekeâ, ØekeâeMekeâ leLee cegõkeâ keâe keâesF& oeefÙelJe
veneR nesiee~
LATEST SYLLABUS
Note : Topics under Bold area have been deleted due to Covid-19 for 2022-23 Board Examinations.
Unit-1: Solid State (3 M) Lanthanoids : Electronic configuration, oxidation states, chemical
Classification of solids based on different binding forces : reactivity and lanthanoid contraction and its consequences.
molecular, ionic, covalent and metallic solids amorphous and Actinoids : Electronic configuration, oxidation states and
crystalline solids (elementary idea). Unit cell in two dimensional comparison with lanthanoids.
and three dimensional lattices, calculation of density of unit cell, Unit-9 : Coordination Compounds (4 M)
packing in solids, packing efficiency, voids, number of atoms per Coordination compounds : Introduction, ligands, coordination
unit cell in a cubic unit cell, point defects, electrical and magnetic number, colour, magnetic properties and shapes, IUPAC
properties. Band theory of metals, conductors semiconductors nomenclature of mononuclear coordination compounds. Bonding,
and insulators and n & p type semiconductors. Werner’s theory, VBT and CFT; structure and stereo isomerism,
Unit-2 : Solutions (5 M) importance of coordination compounds in qualitative inclusion,
Types of solutions, expression of concentration of solutions of solids extraction of metals and biological system.
in liquids, solubility of gases in liquids, solid solutions, colligative
properties-relative lowering of vapour pressure, Raoult’s law, Unit-10 : Haloalkanes and Haloarenes (4 M)
elevation of boiling point, depression of freezing point, osmotic Haloalkanes : Nomenclature, nature of C—X bond, physical and
pressure, determination of molecular mass using colligative chemical properties, mechanism of substitution reactions, polarized
properties, abnormal molecular mass, van’t Hoff factor. rotation.
Haloarenes : Nature of C—X bond, substitution reactions (Directive
Unit-3 : Electrochemistry (5 M) influence of halogen in monosubstituted compounds only).
Redox reactions, conductance in electrolytic solutions, specific and Dichloromethane, Trichloromethane, Tetrachloromethane,
molar conductivity, variations of conductivity with concentration. Iodoform, Freons and uses & effects on environment of DDT.
Kohlrausch’s law, electrolysis and law of electrolysis (elementary
idea), dry cell, electrolytic cells and Galvanic cells, lead Unit-11: Alcohols, Phenols and Ethers (5 M)
accumulator, EMF of a cell, standard electrode potential, Nernst
equation and its application to chemical cells, Gibbs free energy and Alcohols : Nomenclature, methods of preparation, physical and
relation between changes in EMF of cells, fuel cells, corrosion. chemical properties (only monohydric alcohols), identification of
primary, secondary and tertiary alcohols, mechanism of
Unit-4 : Chemical Kinetics (5 M) dehydration, uses of methanol and ethanol. Phenols :
Rate of a reaction (average and instantaneous), factors affecting rate of Nomenclature, methods of preparation, physical and chemical
reaction — concentration, temperature, catalyst; order and molecularity properties, acidic nature of phenols, electrophilic substitution
of a reaction, rate law and specific rate constant, integrated rate reaction, uses of phenols. Ethers : Nomenclature, methods of
equations and half life (only for zero and first order reactions), concept preparation, physical and chemical properties, uses.
of collision theory (elementary idea, no mathematical treatment),
activation energy, Arrhenious equation. Unit-12 : Aldehydes, Ketones & Carboxylic Acids (5 M)
Aldehydes and Ketones : Nomenclature, nature of carbonyl group,
Unit-5 : Surface Chemistry (4 M) methods of preparation, physical and chemical properties,
Adsorption : Physisorption and chemisorption, factors affecting mechanism of nucleophillic addition reactions, reactivity of alpha
adsorption of gases on solids, catalysis, homogeneous and hydrogen in aldehydes, uses.
heterogeneous, activity and selectivity; enzyme catalysis. Colloidal
Carboxylic Acids : Nomenclature, acidic nature, methods of
state : Distinction between true solutions and suspensions, lyophilic,
preparation, physical and chemical properties, uses.
lyophobic multimolecular and macromolecular colloids; properties of
colloids; Tyndall effect, Brownian movement, electrophoresis, Unit-13 : Organic Compounds Containg Nitrogen (4 M)
coagulation, emulsion and types of emulsions.
Amines : Nomenclature, classification, structure, methods of
Unit-6 : General Principles and Processes (4 M) preparation, physical and chemical properties of ethyl amine and
aniline, uses, distinction among primary, secondary and tertiary
of Extraction of Elements amines.
Principles and methods of extraction — concentration, oxidation, Cyanides and Isocyanides : Will be studied at suitable places as
reduction — electrolytic method and refining : occurrence and references.
principles of extraction of aluminium, copper, zinc and iron. Diazonium salts : Methods of preparation, chemical reactions
and synthetic, importance in organic chemistry.
Unit-7 : p-Block Elements (7 M)
Group 15 Elements : General introduction, electronic configuration, Unit-14 : Biomolecues (6 M)
occurrence, oxidation states, trends in physical and chemical
properties; nitrogen preparation properties and uses; compounds of Carbohydrates : Classification (aldoses and ketoses),
nitrogen, preparation and properties of ammonia and nitric acid, monosaccharides (glucose and fructose), D-L configuration,
oxides of nitrogen (structure only); Phosphorus – allotropic forms, oligosaccharides (sucrose, lactose, maltose), polysaccharides
compounds of phosphorus : preparation and properties of (starch, cellulose, glycogen) importance.
phosphine, preparation and properties of halides (PCl3, PCl5) and Proteins : Elementary idea of amino acids, peptide bond,
oxoacids (elementary idea only). Group 16 Elements : General polypeptides, proteins, structure of proteins—primary structure,
introduction, electronic configuration, oxidation states, occurrence, secondary structure, tertiary structure and quaternary structure
trends in physical and chemical properties, dioxygen : preparation, (qualitative idea only), denaturation of proteins.
properties and uses, classification of oxides, ozone, Sulphur – Vitamins : Classification and functions, Enzymes.
allotropic forms; compounds of sulphur : preparation properties and Nucleic Acids : DNA and RNA.
uses of sulphur dioxide, sulphuric acid : industrial process of
manufacture, properties and uses; oxoacids of sulphur (structures Unit-15 : Polymers (3 M)
only). Group 17 Elements : General introduction, electronic
configuration, oxidation states, occurrence trends in physical and Classification : Natural and synthetic, methods of polymerization
chemical properties; compounds of halogens, preparation, properties (Addition and condensation), copolymerization, some important
and uses of chlorine and hydrochloric acid, interhalogen compounds, polymers: natural and synthetic like polythene, nylon polyesters,
bakelite, rubber. Bio-degradable and non-biodegradable polymers.
oxoacids of halogens (structures only). Group 18 Elements:General
introduction, electronic configuration, occurrence, trends in physical
and chemical properties, uses. Unit-16 : Chemistry in Everyday Life (3 M)
Chemistry in Drugs : Antipyretics, Analgesics, antiseptics,
Unit-8 : d- and f-Block Elements (3 M) disinfectants, tranquilizers, antibiotic, antimicrobials,
General introduction, electronic configuration, occurrence and antihistamines, antacids, antifertility, drugs, antioxidents drugs.
characteristics of transition metals. General trends in properties of
the first series transition metals – metallic character, ionization Chemistry in Food : Preservative and synthetic sweetener.
enthalpy, oxidation states, ionic radii, colour, catalytic properties, Intitial introduction of antioxide.
magnetic properties, interstitial compounds, alloy formation, Cleansing Agents : Soaps, synthetic detergents, cleansing
preparation and properties of K2Cr2O7 and KMnO4 . process.
1. Solid State 5-13

2. Solutions 14-26

3. Electrochemistry 27-38

4. Chemical Kinetics 39-54

5. Surface Chemistry 55-64

6. Principles and Processes of Extraction of Elements —


(Deleted for 2020-21 Examinations)

7. p-Block Elements 65-80

8. d- and f-Block Elements 81-89

9. Co-ordination Compounds 90-98

10. Haloalkanes and Haloarenes 99-111

11. Alcohols, Phenols and Ethers 112-130

12. Aldehydes, Ketones and Carboxylic Acids 131-157

13. Organic Compounds Containing Nitrogen 158-168

14. Biomolecules 169-184

15. Polymers (Deleted for 2020-21 Examinations) —

16. Chemistry in Everyday Life (Deleted for 2020-21 Examinations) —

5 Model Paper (Unsolved) 185-192


5

Chapter

1 SOLID STATE
Syllabus: Classification of solids based on different binding forces, molecular, ionic, covalent and metallic solids, amorphous and
crystalline solids (elementary idea). Unit cell in two dimensional and three dimensional lattices, calculation of density of unit cell,
packing in solids, packing efficiency, voids, number of atoms per unit cell in a cubic unit cell, point defects, *electrical and
magnetic properties.

* Topics Deleted for Examination 2021

Objective Questions
1. Graphite is : (U.P. 2014) æ 3ö æ 2ö
(a) r = ç ÷ (b) r = ç ÷
(a) ionic solid (b) metallic solid ç 4 ÷a ç 4 ÷a
(c) covalent solid (d) molecular solid è ø è ø
2. In a crystal the nature of forces is : (U.P. 2014) a æ 4 ö÷
(c) r = (d) r = çç
(a) long range (b) short range 2 ÷a
(c) irregular (d) all of these è 6 ø
3. Plastic is : (U.P. 2014) 10. Which of the following has a total number of 2 atoms
(a)ionic solid (b) metallic solid per unit cell? (U.P. 2019)
(c) covalent solid (d) molecular solid (a) Face centred cubic unit cell
[Note : Plastic is amorphous solid. No answer is (b) Body centred cubic unit cell
correct] (c) Primitive cubic unit cell
4. The structure of sodium chloride crystal is: (d) None of these.
(U.P. 2014) 11. The axial distances or edge lengths in cubic crystal is:
(a)body-centred cubic lattice (U.P. 2019)
(b)face-centred cubic lattice (a) a = b = c (b) a = b ¹ c
(c) orthorhombic (c) a ¹ b ¹ c (d) a = b = g = 120°
(d)tetragonal 12. The compound which exhibits both Frenkel and
5. In sodium chloride the co-ordination number of Na + Schottky defects is: (U.P. 2019)
is: (U.P. 2014, 15) (a) NaCl (b) KCl (c) CsCl (d) AgBr
(a)4 (b) 6 13. Graphite is: (U.P. 2020)
(c) 8 (d) Unpredictable (a) ionic solid (b) metallic solid
6. Which type of solids are electrical conductor, (c) covalent solid (d) molecular solid
malleable and ductile ? (U.P. 2015) 14. Solid A is very hard, solid and non-conductor in
(a) Molecular (b) Ionic molten state and melts at high temperature. Which
(c) Metallic (d) Covalent type of solid is this? (U.P. 2020)
7. Solid A is very hard and is insulator of electricity in (a) Molecular (b) Ionic
molten state. It melts at very high temperature. Which (c) Metallic (d) Covalent
type of solid it is ? (U.P. 2015)
15. Potassium sulphate is : (U.P. 2020)
(a) Molecular (b) Ionic
(c) Metallic (d) Covalent (a) Ionic solid (b) Metallic solid
8. Due to Frenkel’s defect, the density of ionic crystal: (c) Covalent solid (d) Molecular solid
(U.P. 2015) 16. In a crystal system a = b = c and a = b = g ¹ 90° , this
(a) decreases (b) increases system is : (U.P. 2020)
(c) changes (d) remains unchanged (a) Tetragonal (b) Hexagonal
9. The length of core of an fcc unit cell is a and its atomic (c) Rhombohedral (d) Monoclinic
radius is r. The relationship between them is :
(U.P. 2019)

Answers
1. (c) 2. (a) 3. (*) 4. (b) 5. (b) 6. (c) 7. (b) 8. (d) 9. (a) 10. (b)
11. (a) 12. (d) 13. (c) 14. (d) 15. (a) 16. (c)
6

Very Short Answer Type Questions


Q.1 Write the co-ordination number of each ion present in or Explain the unit cell with suitable example.
a crystal having rock salt like structure. (U.P. 2015) or Define unit cell by giving an example. (U.P. 2014)
Ans. In rock salt like crystal, each ion has co-ordination Ans. The smallest repeating unit of a crystals is called unit
number equal to six. e.g., In NaCl, both Na + and Cl - cell. Unit cells are arranged in a regular manner.
have co-ordination number equal to six. Constituting units (atoms/ions etc.) remain situated at
Q.2 Define crystal lattice and unit cell. (U.P. 2015) corners/faces or centre of unit cell.

Short Answer Type Questions


Q.1 Piece of sodium chloride is harder than sodium metal. Ans. Z = 1 (for primitive unit cubic cell)
Why? (U.P. 2015) In this unit cell each corner atom, atom is show with 8
Ans. In sodium chloride, Na + and Cl - ions are held together unit cells and so contributes 1/8 to each unit cell.
with ionic bond (electrostatic force) while in Na-metal, Because total number of atoms at 8 corners = 8
Na-atoms are held together by metallic bonds. So, total number 8 atoms per unit cell
Electrostatic forces are stronger than metallic bond, 1
that is why sodium chloride is harder than sodium = 8´ = 1
8
metal.
Q.2 Write the formula which is used to calculate the
density of crystals. (U.P. 2015)
Z´M
Ans. r = 3
a ´NA
Here, r = density, Z = no. of atoms/unit cell
M = atomic/molar mass, a = edge length Fig. Space-filling representation of cubic unit cell.
N A = Avogadro’s number Corner atom is being shared among eight unit cell.
Q.3 How number of particles are determined in a primitive
cubic unit cell? Explain with the help of a diagram.
(U.P. 2020)

Numerical Problems
Problem 1: In a solid, potassium atoms are at the centre w 58.1
(ii) Moles of NaCl = =
of cubic lattice. Calculate the approximate number of unit cells m 58.5
in 4.0 g potassium. (At. mass of K = 39) (U.P. 2020) 58.1
\ Number of Na + Cl - units = ´ 6.023 ´ 10 23
Solution: For bcc unit cell of potassium z = 2, i. e., each 58.5
unit cell contains two atoms. Because each fcc unit cell involves 4 Na + Cl - units.
w 58.1 ´ 6.023 ´ 10 23
Also, the number of atoms in 4g potassium = ´ N A \ Number of unit cell =
m 58.5 ´ 4
4
= 23 22
´ 6.02 ´ 10 = 6.17 ´ 10 atoms = 1.49 ´ 10 23 unit cell
39
Total number of atoms Problem 3: An element has bcc structure having edge
\ No. of unit cells = length 288 pm and density 7.2 g cm -3 . Calculate the number of
2 atoms in 208 g element. (U.P. 2014)
6.17 ´ 10 22 Z´M
= = 3.088 ´ 10 22 unit cells Solution: Density (r ) = 3
2 a ´NA
Problem 2: (i) Calculate the number of unit cells in 58.5 g For fcc Z = 2 , a = 288 pm = 2.88 ´ 10 -8 cm, r = 7.2 g cm -3
NaCl. (U.P. 2014, 19)
r ´ a3 ´ N A
(ii) Calculate the number of unit cell in 58.1 g NaCl. \ M =
Z
(U.P. 2015) 7.2 ´ ( 2.88 ´ 10 -8 ) ´ 6.023 ´ 10 23
Solution: (i) \58.5 g NaCl = 1 mole = = 51.79 g mol -3
2
= 6.023 ´ 10 23 Na + Cl - units Q 51.79 g element contain = 6.023 ´ 10 23 atoms
Each fcc unit cell has four 4 Na + Cl - units. 6.023 ´ 10 23 ´ 208
6.023 ´ 10 23 \ 208 g element will contain =
\ Number of unit cells = = 1.505 ´ 10 23 51.79
4 = 2.419 ´ 10 24 atoms
7

Problem 4: Calculate the number of atoms per Z´M 4 ´ 107.9


and r = 3
= 23
(a) cubic unit cell , (b) bcc unit cell, (c) fcc unit cell NA ´a 6.023 ´ 10 ´ (408.6 ´ 10 -10 ) 3
(U.P. 2014, 17)
= 10.504 g/cm 3
(Or) How is number of particles in a primitive cubic unit cell
determined? Explain with the help of diagram. (U.P. 2020) Problem 7: Edge length of fcc unit cell of ionic solids is
Solution : (a) In cubic unit cell, 8 corners are occupied 508 pm. If radii of cation is 110 pm, then calculate the radii of
by 8 atoms and each atom shares 8 unit cells. anion. (U.P. 2015)
1 Solution: r + + r - = a
Z = 8 ´ = 1 atom/unit cell a 508
8 For fcc structure, ( r + + r - ) = = = 254 pm
1 3 2 2
2 4 r + = 110 Þ r - = 254 - 110 = 144 pm
Problem 8: Sodium metal crystallises in the cubic lattice
and edge of the unit cell is 430 pm. Calculate the number of
atoms in the unit cell.
(Density of sodium = 0.9623 g × cm -3 , At.wt. = 23, Avogadro
No. N A = 6.023 ´ 10 23 ) (U.P. 2019)
Z´ A
5 7 Solution: r =
6 8 N A ´ a3
Corner atom is shared among 8 unit cells r = 0.9623gcm -3 , A = 23, N A = 6.023 ´ 10 23
(b) In bcc unit cell, 8 atoms are at corners and one at a = 430 ´ 10 -10 cm
centre. Z ´ 23
0.9623 =
æ 1ö
Z = ç 8 ´ ÷ + 1 = 2 atom/unit cell 6.023 ´ 10 ´ ( 430 ´ 10 -10 ) 3
23

è 8ø 0.9623 ´ 6.023 ´ 10 23 ´ ( 4.3) 3 ´ 10 -24


\Z = = 2.003
23
Problem 9: Find out the number of unit cells present in
1 1 58.5 gm of NaCl. ( Na = 23, Cl = 35.5) (U.P. 2019)
2 2
Solution:Total number of Na + Cl - ions in 58.5g NaCl,
= moles ´ 6.02 ´ 10 23
w 58.5
= ´ 6.02 ´ 10 23 = ´ 6.02 ´ 10 23
Face atom is shared between m 58.5
two unit cells
= 6.02 ´ 10 23 (Q m NaCl = 58.5)
(c) In fcc unit cell, 8 atoms Q Each NaCl unit-cell (fcc) contains 4Na + Cl - units thus
Central atom number of unit-cells in 58.5g NaCl
are at corners and 6 atoms are at contributes to
six faces, each face atom is only one cell 6.02 ´ 10 23
= = 1.505 ´ 10 23
shared between two unit cells. 4
æ 1ö æ 1ö
Z = ç8´ ÷ + ç6´ ÷ Problem 10: In a metallic gold crystal, there is cubic
è 8ø è 2ø centered lattice. What is the approximate number of unit cells
= 4 atom/unit cell present in 2.0 gm of gold ? Atomic weight of gold is 197.
Problem 5: In a cubic crystal of type A x B y of a compound (U.P. 2019)
atoms A are at corners and B are at face centres. Predict the Solution: For cubic centred lattice (also known as face
formula of compound. (U.P. 2014) centred cubic lattice), number of atoms per unit cell (Z) is
Solution: Atoms A are at eight corners, thus equal to four, thus
8 Number of atoms w N A
Number of atoms A per unit cell = = 1 Number of unit cells = = ´
8 4 m 4
Atoms B are at six faces, thus 2.0 6.02 ´ 10 23
6 = ´
Number of atoms B per unit cell = = 3 197 4
2 21
= 1.52 ´ 10 units
Thus, formula is AB 3 . Problem 11: An element occurs in the BCC structure with
Problem 6: Silver crystallizes as ccp lattice. X-ray analysis cell edge of 288 pm. The density of the element is 2.7 g / cm 3 .
showed that edge length of unit cell is 408.6 pm. Calculate the How many atoms will be present in 208 gm of the elements ?
density of silver. (Ag = 108) (U.P. 2014) (U.P. 2019)
Solution: Given that, a = 408.6 pm = 408.6 ´ 10 -10 cm
N A = 6.023 ´ 10 23 , Z = 4 (for ccp)
8
ZM unit cell is 408.6 pm. Calculate the density of silver
Solution: Density of unit cell (r ) = 3
a NA (Ag = 107.9). (U.P. 2020)
Z´M
For bcc Z = 2 , a = 288pm = 2.88 ´ 10 -8 cm Solution: Density(r ) =
N A ´ a3
2.7 ´ ( 2.88 ´ 10 -8 ) 3 ´ 6.023 ´ 10 23
M= For ccp lattice, Z = 4, M Ag = 107.9,
2
a = 408.6 ´ 10 -10 cm, N A = 6.02 ´ 10 23
= 19.41 g mol -1
4 ´ 107.9
19.41g element contains = 6.023 ´ 10 23 atoms \ r= 23 -10 3
= 10.5 gcm -3
6.02 ´ 10 ´ (408.6 ´ 10 )
6.023 ´ 10 23
208 g element contains = ´ 208 Problem 14: Crystal lattice of a metallic element is cubic.
19.41
= 6.44 ´ 10 24 atoms The length of each side of unit cell is 2 ´ 10 -8 cm. The density of
Problem 12: The volume of NaCl unit cell is metal is 2.5 gm per cm 3 . What will be the number of unit cells in
2.01 ´ 10 -21 cm 3 . Calculate the density of unit cell of NaCl. 200 gm of metal? (U.P. 2020)
Na = 23, Cl = 35.5 (U.P. 2019) Solution: Volume of cubic unit cell
ZM V = a 3 = ( 2 ´ 10 -8 ) 3 = 8 ´ 10 -24 cm 3
Solution: Density(r ) = NaCl has a fcc structure
NV Density of unit cell, r = 2.5 g cm -3
4 ´ 58.5 \ Mass of one unit cell = 8 ´ 10 -24 ´ 2.5
= (Q Z = 4)
6.023 ´ 10 23 ´ 2.01 ´ 10 -21 = 20.0 ´ 10 -24 g
200
\ Number of unit cell in 200 g =
= 0.1932 g cc 20.0 ´ 10 -24
Problem 13: Silver forms cubic closed pack (CCP) lattice = 10 25 unit cell
and X- ray studies of its crystal shows that the edge length of its

Solution of NCERT Text Book Problems


Q.1 Define the term ‘amorphous’. Give a few examples of e. g ., In NaCl, each Na + is surrounded with 6Cl -
amorphous solids. and each Cl - with 6Na + .
Ans. Those solids in which constituents are not arranged (ii) Coordination number (a) 12, (b) 8.
in regular 3D pattern are called amorphous solids. Q.5 How can you determine the atomic mass of an
They are better called ‘super cooled liquids’ than true unknown metal if you know its density and the
solids, e. g ., glass, plastic, rubber etc. dimension of its unit cell ? Explain.
Q.2 What makes a glass different from a solid such as r ´ a3 ´ N A
quartz? Under what conditions could quartz be Ans. M =
Z
converted into glass?
Q.6 (i) ‘Stability of a crystal is reflected in the magnitude
Ans. The range of force of attraction between constituent
of its melting point’. Comment.
units. When quartz is melted and then cooled rapidly,
(ii) Collect melting points of solid water, ethyl
it gets converted into glass.
alcohol, diethyl ether and methane from a data
Q.3 Classify each of the following solids as ionic, metallic,
book. What can you say about the intermolecular
molecular, network (covalent) or amorphous.
forces between these molecules ?
(i) Tetra phosphorus decoxide (P4 O10 )
Ans. (i) Ionic solids have high lattice energy and so their
(ii) Ammonium phosphate (NH 4 ) 3 PO 4
melting points are high.
(iii) SiC (iv) I 2 (v) P4 (ii) The order is explained on the basis of nature and
(vi) Plastics (vii) Graphite (viii) Brass strength of interparticle forces such as H-bonds,
(ix) Rb (x) LiBr (xi) Si van der Waals’ forces etc.
Ans. Ionic — (ii), (x) Q.7 How will you distinguish between the following pairs
Metallic — (viii), (ix) of terms :
Molecular — (i), (iv), (v) (i) Hexagonal close packing and cubic close packing
Covalent — (vii), (iii), (xi)
Amorphous — (vi) (ii) Crystal lattice and unit cell
Q.4 (i) What is meant by the term ‘coordination number’? (iii) Tetrahedral void and octahedral void ?
(ii) What is the coordination number of atoms : Ans. (i) Hexagonal close packing and cubic close
(a) in a cubic close packed structure ? packing : Hexagonal close packing is called
(b) in a body centred cubic structure ? ABABAB¼ type arrangement also because in it,
Ans. (i) Co-ordination number is equal to the number of spheres of second layer are placed over spheres
constituents (atoms, ions etc.) present amount a of first layer in such a way that they cover
lattice point (atom, ions etc.). interstitial voids formed by spheres of first layer.
9

Then third layer is placed over second layer in Radius of tetrahedral void and radius of sphere
such a way that its spheres cover tetrahedral are related to each other as
voids of second layer. This structure is called
hexagonal close packed structure (hcp), e. g ., Mg, rv = 0.225 rs
Zn, Mn, Be etc. An octahedral void is formed by 6 spheres.
Six-fold axis Octahedral
void is
A o
surrounded o
B by six spheres
arranged as
A
double Octahedral void
B triangle and
A
centres of these spheres form an octahedron
when joined to each other. Radius of octahedral
(a) (b) (c) void and radius of sphere are related to each
ABABAB.... system of close packing or hexagonal other as follows.
close packing (hcp) of spheres.
rv = 0.414 rs
In cubic close packed structure, third layer is Q.8 How many lattice points are there in one unit cell of
placed over second layer in such a way that it each of the following lattice :
covers its octahedral voids and first row is (i) Face centred cubic
aligned with fourth row. It is ABCABCABC ¼type
(ii) Face centred tetragonal
arrangement.
(iii) Body centred ?
Three-fold axis
Ans. (i) Lattice points in face centred cubic or face
C centred tetragonal
B = 8 (at corners) + 6 (at face centres) = 14.
A 1 1
\ Particles per unit cell = 8 ´ + 6´ = 4.
8 2
C (ii) Face centred tetragonal lattice is not observed in
B crystalline solids.
A (iii) Lattice points in body centred cube
= 8 (at corners) + 1 (at body centre) = 9
1
(a) (b) (c) \ Particles per unit cell = 8 ´ + 1 = 2.
8
ABCABCABC... system of close packing or cubic
Q.9 Explain : (i) The basis of similarities and differences
close packing (ccp) of spheres.
between metallic and ionic crystals.
(ii) Crystal lattice and (ii) Ionic crystals are hard and brittle.
unit cell : The smallest Ans. (i)
repeating unit of a
Unit Metallic crystals Ionic crystals
crystal lattice is called
cell
unit cell. A number of Constituting Metal atom (kernels Cations and anions
unit cells are arranged in unit embedded in sea of
regular 3D pattern to electrons)
form crystal lattice. Interparticle Metallic bonds High electrostatic forces of
Cubic lattice and forces attraction
its unit cell
(iii) Tetrahedral Examples Metals (Cu, Fe, Ag, Mg) NaCl, K C l, KNO 3,
and alloys CuSO 4 ×5H2O, CaF2, ZnS,
and Octahedral
K 2SO 4 , MgO, BaO etc.
voids : Such t
voids are obtained t P h y s i c a l Soft to hard, low to high Hard rigid, brittle, high
when spheres of state enthalpy of fusion m.pt.
second layer are E l e c t r i c a l Conductor Conductor in fused state or
placed over three Tetrahedral void properties in solution (some solid
spheres touching conductors are also known
each other. Tetrahedral void is formed by four like AgCl)
spheres.
10

(ii) Because ionic bonds are stronger than metallic From D ABC, AC = 2 a and from D ADC,
bonds. CD = 3 a and we know from the unit cell that
Q.10 Calculate the efficiency of packing in case of a metal three spheres on CD are touching each other.
crystal for : 4
Thus, CD = 4 rs = 3 a or a = rs
(i) simple cubic, (ii) body centred cubic, 3
(iii) face centred cubic (with the assumptions that Percentage efficiency of body-centred unit cell
atoms are touching each other) Volume of two sphere
= ´ 100
Or Calculate the packing efficiency of a cubic lattice Volume of cube
when atoms located at corners of the cube touch each 4
2 ´ p( rs ) 3
other. (U.P. 2019) 3
or = ´ 100
Or Calculate the packing efficiency of a cubic lattice a3
when an atom located at the diagonal remains in 4
touch with the other two atoms. (U.P. 2019) 2 ´ p( rs ) 3
or = 3 ´ 100 = 68.05
Ans. Efficiency (or packing fraction or density of packing) 3
of a unit cell is defined in terms of the percentage æ 4 ö
çç rs ÷÷
volume occupied by spheres in that unit cell, out of è 3 ø
total available volume. It is calculated by the Thus, the volume occupied by spheres in
following formula. body-centred unit cell is 68.05 % and void volume
No. of spheres/unit cell ´ % efficiency is 31.95 %.
Volume occupied by each sphere (iii) Efficiency of packing in face-centred unit
= ´ 100
Total volume of unit cell cell: Consider a face-centred unit cell with edge
(i) Efficiency of packing in simple cubic unit length ‘ a’ and volume
cell: A simple cube is having 1 sphere per unit a 3 . Radius of sphere is A
cell as discussed earlier. Let us consider the edge taken as rs . It is clear B
of a simple cubic unit as a, then the volume of a
from the figure that
unit cell is a 3 . We known from the earlier atom at the face centre
discussion that in simple cube a = 2rs where ‘ rs ’ will be in touch with
is the radius of sphere. the two other atoms C
Percentage efficiency of simple cubic unit cell diagonally arranged.
Volume of one sphere
= ´ 100
Volume of unit cell From D ABC,
4 4 BC = 2a = 4rs
p ´ ( rs ) 3 p ´ ( rs ) 3
4
or = 3 ´ 100 = 3 ´ 100 or a= rs = 2 2 rs
a3 ( 2 ´ rs ) 3 2
4 We know that there are four spheres per unit cell
p
in a face-centred unit cell and thus per cent
or = 3 ´ 100 = 52.3 efficiency of face centred unit cell
8
Volume of four spheres
Thus, the volume occupied by sphere in simple = ´ 100
cubic unit cell is 52.3% and void volume is 47.7 Volume of cube
4
per cent. 4 ´ p( rs ) 3
(ii) Efficiency of = 3 ´ 100
packing in 2a A a3
body-centred 4
C B 4 ´ p( rs ) 3
unit cell: a
or = 3 ´ 100 = 74.06
Consider a (2 2 rs ) 3
body-centred unit Thus, the volume occupied by spheres in
cell with edge D face-centred unit cell (ccp arrangement) is
length ‘ a’ as shown 3a
74.06% and void volume is 25.94%.
in the figure and
with radius of Note: (i) Packing efficiency of ccp, hcp and fcc unit cells are same
sphere ‘ rs ’. (74.06%)
As discussed earlier, body-centred unit cell is (ii) Rhombohedral unit cell also have packing efficiency
having two spheres per unit cell and a 3 is the same as hcp i.e., 74.06%
volume of unit cell by taking edge length
a.
11

Q.11 Silver crystallises in fcc lattice. If edge length of the 1 R


or =
cell is 4.077 ´ 10 -8 cm and density is 10.5 g cm –3 , 2 R +r
calculate the atomic mass of silver. On inverting on both the sides,
Z´ M R+r
Ans. Density = 3
gram / cm 3 = 2
NA´ a R
4´ M r
or 10.5 = or 1+ = 2
R
6.02 ´ 10 ´ ( 4.077 ´ 10 –8 ) 3
23
r
or M = 107.1 or = 2 - 1 = 0.414
R
Q.12 A cubic solid is made of two elements P and Q. Atoms or r = 0.414 R
of Q are at the corners of the cube and P at the body i. e., the limiting radius of an octahedral void is 0.414
centre. What is the formula of the compound ? What times of the radius of sphere.
are the co-ordination numbers of P and Q ? Q.15 Copper crystallises into a fcc lattice with edge length
Ans. Q atoms are at the corners, so the number of Q atoms
1 3.61 ´ 10 –8 cm. Show that the calculated density is
per unit cell = 8 ´ = 1 in agreement with its measured value of 8.92 g cm –3 .
8
Z´ M
Number of P atoms per unit cell = 1 Ans. Density(r ) = 3
gram / cm 3 (for fcc, Z = 4)
\ Formula of compound = PQ NA ´ a
and Coordination number of P and Q = 8 4 ´ 63.5
= = 8.97 g/ cm 3
Q.13 Niobium crystallises in body centred cubic structure. ( 3.61 ´ 10 ) ´ 6.02 ´ 10 23
–8 3

If density is 8.55 g cm –3 , calculate atomic radius of The measured value is 8.92 g / cm 3 . It is in close
niobium using its atomic mass 93 u. agreement with the calculated value.
Z´ M
Ans. Density (r ) = gram / cm 3 Q.16 Analysis shows that nickel oxide has the formula
N A ´ a 3 ´ 10 –30 Ni 0.98O1.00 . What fractions of nickel exist as Ni 2+ and
2 ´ 93 Ni 3+ ions ?
or 8.55 =
6.02 ´ 10 23 ´ a 3 ´ 10 –30 Ans. Let the number of oxygen ions = 100
or a = 3.3 ´ 10 -8 cm = 33 nm so, the number of nickel ions = 98
For body centred cubic unit cell, Let number of Ni 2+ ions = x
a 3 33 3 \ Number of Ni 3+ ions = 98 - x
r= = = 14.29 nm \ Total charge on 98 nickel ions = 2 ´ x + 3 ´ ( 98 - x )
4 4
This charge is equal to total negative charge on oxide
Q.14 If the radius of the octahedral void is r and radius of ions i. e., 100 ´ 2 = 200 units.
the atoms in close packing is R, derive relation or 2 ´ x + 3( 98 - x ) = 200
between r and R. 2 x + 294 - 3 x = 200
Ans. Limiting radius of octahedral void: Figure x = 94
represents the cross-section through an octahedral 2+ 94
site. \ % Ni = ´ 100 = 96%
96
or % Ni 3+ = 100 - 96 = 4%
a
Q.17 What is a semiconductor ? Describe the two main
C B types of semiconductors and contrast their
conduction mechanism.
r
Octahedral Ans. Topic deleted for examination 2021.
void Q.18 Non-stoichiometric cuprous oxide, Cu 2O can be
prepared in laboratory. In this oxide, copper to
D A oxygen ratio is slightly less than 2 : 1. Can you
account for the fact that this substance is a p-type
semiconductor?
Octahedral void Ans. Some of the Cu + ions are replaced by Cu 2+ ions. Two
Assume distance between the centre of two adjacent Cu + ions are replaced by one Cu 2+ ion to maintain
spheres is ‘ a’, radius of sphere is ‘ R ’ and radius of the electrical neutrality. This creates hole. Since
void is ‘ r ’. conduction is due to the presence of positive holes, it
AB = BC = CD = AD = a = 2R is p-type semiconductor.
and BD = 2 a = 2( R + r ) Q.19 Ferric oxide crystallises in a hexagonal close packed
AB a 2R array of oxide ions with two out of every three
Now = =
BD 2 a 2( R + r )
12

octahedral holes occupied by ferric ions. Derive the A+ B– A+ B– A+


formula of the ferric oxide. Positive
Ans. Hexagonal close packing having array of oxide ions void
with two out of three octahedral holes occupied by B– B– A+ B–
Negative
Fe 3+ ions has one octahedral void or holes void
corresponding to each atom constituting the close A+ B– A+ A+
packing. In iron oxide only 2 3 of octahedral holes
are occupied by Fe 3+ ions. It means corresponding to
B– A+ B– A+ B–
each oxide ion, there are 2 3 Fe 3+ ions. Thus,
molecular formula of iron oxide is Fe 2O 3 . Schottky defect
Q.20 Classify each of the following as being either a p-type (4) Schottky defect appears generally in strongly
or a n-type semiconductor : ionic compounds having a high co-ordination
(i) Ge doped with In (ii) B doped with Si. number and in which the radius ratio ( r + r - ) is
Ans. (i) Ge is group 14 element and In belongs to group not far below unity i. e., in which the positive and
13. Hence an electron deficit hole is created and negative ions are of almost similar size.
therefore, it is p-type semiconductor. (5) Example: NaCl, CsCl, KCl, AgBr, KBr, CsCl etc.
(ii) B is group 13 element and Si belongs to group (ii) Frenkel defect: (1) It is also called
14, so there will be a free electron. Hence, it is dislocation effect.
n-type semiconductor. (2) This defect arises when an ion (generally cation)
Q.21 Gold (atomic radius = 0.414 nm) crystallises in a face occupies an interstitial position between the
centred unit cell. What is the length of a side of the lattice points.
cell ? (3) It may be noted again that the crystal remains
Ans. For fcc, a = 2 r 2 = 2 ´ 0.144 ´ 2 = 0.407 nm. neutral since the number of positive ions is equal
to the negative ions.
Q.22 In terms of band theory, what is the difference : (4) Positive ions being smaller than negative ions
(i) between a conductor and an insulator ? and, hence, are more likely to occupy interstitial
(ii) between a conductor and a semiconductor ? positions.
Cation void Cation in interstitial site
Ans. Topic deleted for examination 2021.
Q.23 Explain the following terms with suitable examples : B– B–
A+ A+ A+
(i) Schottky defect, (ii) Frenkel defect, +
A
(iii) interstitials and (iv) F-centres. – –
B–
B B A+
Ans. Stoichiometric compounds contain positive and
negative ions exactly in the ratio as indicated by their
chemical formula. For simplicity, we may consider the A+ B– A+ B– A+
compound of the type, AB having equal number of
A + and B - ions. In these defects, stoichiometry of B– A+ B– A+ B–
crystal is maintained. In these compounds, two types
of defects are generally observed. These are called: Frenkel defect
(i) Schottky defect and (ii) Frenkel defect (5) This defect is common in compounds where
(i) Schottky defect: (1) This defect arises when anions are much larger than the cations, i. e., the
same number of lattice points are unoccupied. radius ratio ( r + r - ) is low. The co-ordination
(2) The points which are unoccupied are lattice number in such compounds is also low. Hence,
vacancies or ‘holes’. The existence of two holes, fewer attractive forces between oppositely
one due to a missing positive ion and the other charged ions have to be broken in causing
due to a missing negative ion, in a crystal lattice, movement of a positive ion from its appropriate
is illustrated in figure. site to the interstitial positions.
(3) The crystal, as a whole, remains electrically (6) Example : AgBr, ZnS, AgCl, AgI.
neutral because the number of missing positive (iii) Interstitials : The region between lattice
and negative ions is the same. points where ions are trapped to create a defect.
13

(iv) F-centre (F-fabre colour): They are present Ans. Doping of SrCl 2 to NaCl brings in replacement of two
in solids showing defect due to anionic vacancy. Na + ion by each Sr 2+ ion, but each Sr 2+ occupies only
They are called F-centres as they are responsible one lattice point. This produces one cation vacancy.
for colour 8 solid. Thus, doping of 10 -3 mole of SrCl 2 in 100 mole NaCl
Q.24 Aluminium crystallises in a cubic close packed will produce cation vacancies = 10 -3
structure. Its metallic radius is 125 pm. \ 100 mole NaCl will have cation vacancies after
(i) What is the length of the side of the unit cell ? doping = 10 -3
(ii) How many unit cells are there in 1.00 cm 3 of \ 1 mole NaCl will have cation vacancies
aluminium ? 10 -3
= = 10 -5
Ans. (i) For unit cell fcc, a = 2 2 r 100
= 2 2 ´ 125 = 353.55 pm » 354 ´ 10 -10 cm \ Total cationic vacancies = 10 -5 ´ Av. no.
(ii) Q Volume of one unit cell = a 3 = ( 354 ´ 10 -10 ) 3 = 6.02 ´ 10 18
= 4.44 ´ 10 -23 cm 3 Q.26 Explain the following with suitable examples :
1 (i) ferromagnetism, (ii) paramagnetism,
\ Number of unit cells in 1.00 cm 3 Al =
4.44 ´ 10 -23 (iii) ferrimagnetism, (iv) anti-ferromagnetism
= 2.25 ´ 10 22 Ans. Topic deleted for examination 2021.
–3
Q.25 If NaCl is doped with 10 mole % of SrCl 2 . What is
the concentration of cation vacancies ?

vvv
14

Chapter

SOLUTIONS
Syllabus : Types of solutions, Expression of concentration of solutions of solids in liquids, Solubility of gases in liquids, Solid
solutions, Colligative properties-relative lowering of vapour pressure, R aoult’s law, Elevation in boiling point,Depression in freezing
point, Osmotic pressure, determination of molecular mass usin g colligative properties, *abnormal molecular mass, van’t
Hoff factor and Calculations based upon it.

* Topics Deleted for Examination 2021

Objective Questions
1. Mole fraction of solute in 1 molal solution is: (d) Osmotic pressure
(U.P. 2010, 17) 10. 5 mg NaCl per litre is : (U.P. 2019)
(a) 0.009 (b) 0.018 (c) 0.027 (d) 0.045 (a) 5 ppm (b) 50 m gmL-1
2. Molarity of pure water: (U.P. 2010, 13, 14, 16, 17) (c) 0.25 m gmL-1 (d) 0.066 mol mL-1
(a) 55.56 mol L-1 (b) 5.556 mol L-1 11. 8 g NaOH is dissolved in 450 mL solution. The
-1
(c) 0.18 mol L (d) 81.00 mol L-1 molarity of solution is : (U.P. 2019)
3. If 18g of glucose is dissolved in 1000g of solvent, then (a) 0.444 M (b) 0.492 M
molarity of solution is : (U.P. 2018) (c) 0.0286 M (d) 0.0392 M
(a) 1 (b) 0.1 (c) 0.5 (d) –0.1 12. The depression of freezing point is directly
4. Which of the following is not a colligative property? proportional to : (U.P. 2019)
(U.P. 2015) (a) Mole fraction of the solution
(a) Osmotic pressure (b) Molarity of the solution
(b) Lowering in vapour pressure (c) Molality of the solution
(c) Elevation in boiling point (d) Molality of the solvent
(d) Optical activity 13. Which of the following is not a colligative property of
5. Mole fraction of water in a mixture obtained by mixing the solution ? (U.P. 2019)
36 g water and 46 g ethyl alcohol is: (a) Surface tension
(U.P. 2017) (b) Osmotic pressure
(a) 0.667 (b) 0.538 (c) 0.462 (d) 0.333 (c) Lowering of vapour pressure
6. Concentration of 0.2 M H 2SO 4 (in g L-1 ) is : (d) Depression in freezing point.
(U.P. 2017) 14. At same temperature, pair of solutions which are
(a) 21.4 (b) 39.2 (c) 9.8 (d) 19.6 isotonic is : (U.P. 2020)
7. Which of the following is a colligative property? (a) 0.1 M NaCl and 0.1 M Na 2SO 4
(U.P. 2009, 18) (b) 0.1 M urea and 0.1 M NaCl
(a) Surface tension (b) Viscosity (c) 0.1 M urea and 0.2 M MgCl 2
(c) Osmotic pressure (d) Optical activity (d) 0.1 M Ca( NO 3 ) 2 and 0.1 M Na 2SO 4
8. Mole fraction of glucose in 90 g of water is : 15. In which of the following lowering of vapour pressure
(U.P. 2018) would be maximum? (U.P. 2020)
(a) 0.19 (b) 0.019 (c) 0.0019 (d) 0.00019 (a) 0.1M BaCl 2 (b) 0.1 M Glucose
9. Which of the following properties of a solution does (c) 0.2 M urea (d) 0.1 M NaCl
not depend on number of molecules? (U.P. 2018) 16. How many moles of water are present in 180 gm of
(a) Lowering in vapour pressure water ? (U.P. 2020)
(b) Depression in freezing point (a) 1 mole (b) 18 mole (c) 10 mole (d) 100 mole
(c) Surface tension
Answers
1. (b) 2. (a) 3. (a) 4. (d) 5. (a) 6. (d) 7. (c) 8. (c) 9. (c) 10. (a)
11. (a) 12. (d) 13. (a) 14. (d) 15. (a) 16. (c)
15

Very Short Answer Type Questions


Q.1 Define molarity with one example. (U.P. 2010, 18) Q.3 What do you understand by osmosis? Write the
Ans. Molarity of a solution is equal to number of gram expression of osmotic pressure? (U.P. 2012, 14)
moles of solute dissolved in one litre solution. Ans. Spontaneous, net flow of solvent molecules from
n solvent (or dilute solution) towards solution of higher
M =
V ( L) concentration.
For example: 1 M H 2SO 4 solution means, 1 L of this Osmotic pressure is calculated by following formula.
pV = nST
H 2SO 4 solution contains 1 mol H 2SO 4 dissolved in it.
p = O.P., V = volume of solution, n = moles of solute
Q.2 Define mole fraction. (U.P. 2011, 18)
Ans. It is equal to the ratio of moles of a substance present S = solution constant, T = absolute temperature
in a mixture divided by total moles present. Q.4 What is reverse osmosis? Write its one use also?
n (U.P. 2017)
X = Ans. When an external pressure, higher than osmotic
Sn
pressure is applied on solution side, solvent particles
For example: In a mixture containing 1 mole A and 2 start to flow from concentrated solution towards
mole B. solvent (or less concentrated solution). Because in this
1 2
X A = , XB = process, solvent particles flow in reverse direction as
3 3 in normal osmosis, that is why it is called reverse
osmosis. It is used in purification of water.

Short Answer Type Questions


Q.1 What do you understand by elevation in boiling point?
How does it related to molality? (U.P. 2014) 3. In molarity moles of solute In molality moles of solute are
Ans. When a non-volatile solute is added in a solvent, its are dissolved in 1 litre of dissolved in 1 kg of solvent.
vapour pressure decreases and boiling point solutions.
increases. This rise in boiling point is referred as
elevation in b.pt. Q.4 What do you understand by molal elevation constant ?
DTb = Tb¢ - Tb How is it related to molality of the solution ?
DTb = elevation in boiling point (U.P. 2014)
Tb¢ = elevated boiling point, Tb = normal boiling point Ans. When 1 mole non-volatile solute is dissolved in 1000
DTb = K b¢ ´ molality gm solvent, the elevation in boiling point is called
molal elevation constant (K ¢b ).
K b¢ = molal elevation constant
Mathematically it can be represented by the formula
Q.2 What is relation between depression in f.pt. and molar 1000 ´ K ¢b ´ w
mass of solute? (U.P. 2007, 08, 18) m=
1000 ´ K ¢f ´ w 1 DTb ´ W
Ans. DT f = or DT f µ
m ´ W (g ) m Where K ¢b = molal elevation constant
DT f = depression in f.pt. m = molecular mass of solute
K ¢f = molal depression constant w = mass of solute
w = mass of solute, m = molar mass of solute W = mass of solvent
Relation between molal elevation constant and
W = mass of solvent (in g) molality :
Q.3 Define molarity and molality and explain the DTb µ m
difference between them. (U.P. 2015, 18)
or DTb = K ¢b m
Ans. Differences between molarity and molality
Q.5 What are isotonic solutions? Explain with example.
S.No Molarity Molality (U.P. 2020)
Ans. ‘‘The solutions having same osmotic pressure at the
1. Molarity involves the total Molality involves the mass of same temperature are known as isotonic solutions.’’
volume of solution. solvent. Characteristics :
2. M o l ari t y c hang es w it h Molality is independent of (i) Isotonic solutions have equal molar concentration.
temperature because volume temperature. (ii) No osmosis occurs when the isotonic solutions are
changes with temperature. separated by semipermeable membrane.
16

Long Answer Type Questions


Q.1 What is osmotic pressure? (U.P. 2014) on the solution with the help of piston. When
How is osmotic pressure determines by Berkeley and water level is contant, the external pressure
hartley’s method? applied through the pistion is equal to the
Ans. Osmotic pressure : The excess of hydrostatic pressure osmotic pressure.
which is developed as a result of osmosis when Q.2 Discuss any four methods of expressing concentration
solution has been separated from solvent through of any aqueous solution. Give one example of each.
semipermeable membrane is called osmotic pressure. (U.P. 2018)
Determination of osmotic pressure by Berkeley and Ans. Four methods of expressing concentration are :
Hartley’s method : (i) Molarity (ii) Molality
The best method is Berkeley-Hartley method is (iii) Normality (iv) Mole fraction
described as. (i) Molarity : It expresses the number of moles of
(i) In this method, the apparatus consists of two solute dissolved in one litre of a solution. It is
concentric tube A and B. Tube A is made of calculated by
w 1000
porous pot in which copper ferrocyanide M = ´ (unit = mol L-1 )
semipermeable membrane is deposited. Porous m VmL
pot is connected with a capillary tube on one side One molar solution means, one mole of solute is
and a water reservoir on the other side. Tube B is dissolved in 1000 mL of solution.
made of gun metal and fitted with pressure (ii) Molality : It expresses the number of moles of
gauge and a pistion P. solute dissolved in one kilogram solvent. Its unit is
mol kg -1 solvent. It is calculated by
w 1000
m= ´
m¢ W g
One molal solution means one mole solute is dissolved
in 1 kg solvent.
B (iii) Normality : It expresses the number of
A
g-equivalents of solute dissolved in one litre solution.
It is calculated by
W 1000
N = ´
E VmL
-1
Its unit is eq × L . One normal solution means one
g-equivalent of solute is dissolved in one litre solution.
Berkeley-Hartley method (iv) Mole fraction : It expresses the ratio of moles
The porous pot and gun metal vessel are filled of component divided by total moles present in the
with water and solution respectively. Osmosis mixture. Let n moles of solute and N moles of solvent
takes place and water moves into gun metal are present in a mixture then
n N
vessel from the porous pot through the X solute = and X solvent =
semipermeable membrane. This is shown by fall n+N n+N
in water level in the capillary tube. This flow of
water is stopped by applying external pressure

Numerical Problems
Problem 1: Calculate mole fractions of each in a mixture And, mole fraction of alcohol = 1 - 0.667 = 0. 333
of 72 g of water and 92 g of ethyl alcohol. (C 2 H 5OH) Problem 2: At 27 ° C , calculate the osmotic pressure of 2%
(U.P. 2017) urea (NH 2 CONH 2 ) solution. Solution constant is 0.0821 litre.
Solution: Mole fraction of water in solution atm. deg -1 .mol -1 and molecular mass of urea is 60.
Mole of water n (U.P. 2016)
= =
Mole of water + Mole of alcohol n + N w. S. T .
Solution: Q P=
Mass of water 72 V. m
And, mole of water = = =4
Molecular mass of water 18 100
Given, w = 12 g, V = 100 mL = litre, S = 0.0821,
Mass of alcohol 92 1000
Mole of alcohol = = =2
Molecular mass of alcohol 46 m = 60, T = 27 + 273 = 300K
4 2 ´ 0.0821 ´ 300 ´ 1000
\ Mole fraction of water = = 0.667 \ P= = 8.21 atm
4+ 2 100 ´ 60
17

Problem 3: When 0.816 g of a substance A, were dissolved 1000 ´ K b¢ ´ 12


180 =
in 7.5 g benzene, its freezing point was found equal to 1.59°C. If 0.34 ´ 100
molal depression constant of benzene is 4.9 k kg mol -1 and its 180 ´ 0.34 ´ 100 18 ´ 0.34
f.pt. is 5.51°C, then calculate the molecular mass of A. or K ¢b = =
1000 ´ 12 12
(U.P. 2016) = 0.51 K kg mol -1
1000 ´ K ¢f ´ w
Solution: DT f = Problem 9: A solution of 0.816 gm of a compound A
m´W dissolved in 7.5 gm of benzene freezes at 1.59ºC. If the molal
w = 0.816 g, W = 7.5 g, K ¢f = 4.9 K kg mol -1 freezing point depression constant of benzene (C 6 H 6 ) is 4.90 K
DT f = 5.51 - 1.59 = 3.92° C = 3.92 K kg mol -1 and its melting point is 5.51ºC, find the molecular
1000 ´ 4.9 ´ 0.816 weight of compound A. (U.P. 2016)
3.92 = Þ m = 136
m ´ 7.5 Solution: w = 0.816 gm ( A ), W = 7.5 gm
Problem 4: 10 gm caustic soda dissolved in 500 gm of K ¢f = 4.90 K kg mol -1 , DT f = 5.51 - 1.59 = 3.92º C
water. Find the molality of solution. (U.P. 2017) mA = ?
w ´ 1000 10 1000 1000 ´ K ¢f ´ w 1000 ´ 4.90 ´ 0.816
Solution: Molality ( m) = = ´ = 0.5 m mA = =
m¢ ´ W (g ) 40 500 DT f ´ W 3.92 ´ 7.5
Problem 5: Calculate the osmotic pressure of decimolar = 136 g mol -1
urea solution at 27 ° C . (U.P. 2017)
1 Problem 10: Determine osmotic pressure of 2% urea
Solution: n = mol, V = 1 litre, T = 27 + 273 = 300K solution at 27ºC. (Solution constant = 0.082 L atm /degree
10 /mol). (U.P. 2016)
n 1 ´ 0.0821 ´ 300
P = × S. T . = = 2 .46 atm Solution: Molecular wt. of urea (NH 2CONH 2 )
V 10 ´ 1
= 14 + 2 + 12 + 16 + 14 + 2 = 60
Problem 6: On dissolving 12 gram glucose (C 6 H 12 O 6 ) in Q 2 gm urea is present in 100 mL
100 gram water, 0.34 ° C elevation in boiling point was 100 ´ 60
observed. Calculate molal elevation constant of water. \60 gm urea is present in = = 3000 mL = 3.0 L
(U.P. 2015, 16) 2
1000K b¢ . w T = 273 + 27 = 300 K and PV = ST
Solution: DT = ST 0.082 ´ 300
m. W P= = = 0.082 ´ 100 = 8.2 atm
V 3
Given, DT = 0.34 C, w = 12 g, W = 100 g, m = 180
1000 ´ K b¢ ´ 12 Problem 11: A solution of urea in water is 6% by weight.
\ 0.34 = Calculate the mole fraction of urea and water in the solution
100 ´ 180 (Molecular wt. of urea = 60). (U.P. 2016)
or, K ¢b = 0.51 kelvin molality -1 6
Solution: Moles of urea ( n1 ) = = 0.1
Problem 7: Calculate the osmotic pressure of 0.5% 60
solution of glucose at 18ºC. (mol. mass = 180).Value of (100 - 6) 94
solution constant is 0.082 litre atm degree -1 mol -1 . Moles of H 2O ( n 2 ) = = = 5.22
18 18
(U.P. 2015) n1 0.1
Solution: Molecular mass of glucose Mole fraction of urea ( x1 ) = =
n1 + n 2 0.1 + 5.22
C 6 H12O 6 = 6 ´ 12 + 12 ´ 1 + 6 ´ 16 0.1
= 72 + 12 + 96 = 180 = = 0.0188
Q 0.5 g glucose is present in 100 mL. 5.32
100 ´ 180 n2 5.22
\180 g glucose is present in = = 20 ´ 10 ´ 180 Mole fraction of water ( x 2 ) = =
0.5 n1 + n 2 0.1 + 5.22
= 36000 mL = 36.0 litre 5.22
= = 0.9812
PV = ST 5.32
P ´ 36 = 0.082 ´ ( 273 + 18) Problem 12: Calculate the elevation in boiling point of
0.082 ´ 291 water on dissolving 1.5 g urea in 50 g water. The value of molal
P= = 0.6628 atm
36 elevation constant for water is 0.52 K kg mol -1 . (U.P. 2016)
Problem 8: An aqueous solution of glucose containing 12 Solution: Given that, w = 1.5 g, W = 50 g
gm in 100 gm of water was found to boil at 100. 34ºC. molecular wt. of urea ( m) = 60
Calculate molar elevation constant of water. (U.P. 2016) K b = 0.52 K kg mol -1 , DTb = ?
Solution: Given that, w = 12 gm ; W = 100 gm 1000 ´ K ¢b ´ w 1000 ´ 0.52 ´ 1.5
or DTb = =
DTb = 100.34 - 100 = 0.34º C, K ¢b = ? m´W 60 ´ 50
molecular mass of glucose C 6 H12O 6 = 0.26º C
m = 6 ´ 12 + 12 ´ 1 + 6 ´ 16 = 72 + 12 + 96 = 180 Problem 13: A solution of urea is 6% (by mass). Calculate
1000 ´ K ¢b ´ w the mole fractions of urea and water in the solution.
m=
DTb ´ W (U.P. 2016, 17)
18

Solution: Given that, conc. of solution = 6% (by mass) DT f = 0.136° C, K f = 18.6 K 100 g solvent mol -1
i. e., 100 g solution contains 6 g urea and 94 g water w = 1.5 g, W = 60 g
6 94 100 ´ 18.6 ´ 1.5
\ n Urea = and n H 2O = (Q m Urea = 60) 0.136 = Þ m = 341.9
60 60 m ´ 60
6 60
\ X Urea = = 0.0188 Problem 20: Boiling point of a solution obtained by
6 94 adding 6 g urea ( NH 2CONH 2 ) in 200 g water is 100.28°C.
+
60 60 What will be the freezing point of this solution? Molal elevation
Also, X Urea + X H 2O = 1 constant and molal depression constants for water are 0.52°C
\ X H 2O = 1 - X Urea = 1 - 0.0188 = 0.9812 molal -1 and 1.86°C respectively. (U.P. 2017)
Problem 14: Calculate mole fractions of ethyl alcohol and Solution: DTb = K ¢b ´ molality …(1)
water in a solution containing 46 gram ethyl alcohol and 36 and DT f = K ¢f ´ molality …(2)
gram water. (U.P. 2015, 17) Dividing eq. (1) by (2),
Solution: According to the data given, DTb K¢
w 46 = b
n EtOH = = =1 (Q m EtOH = 46) DT f K ¢f
m 46 DTb = 100.28 - 100 = 0.28° C
w 36
and n H 2O = = =2 (Q m H 2O = 18) K ¢b = 0.52° C molal -1 , K ¢f = 1.86° C molal -1
m 18
1 1 0.28 0.52 1.86 ´ 0.28
\ X EtOH = = = 0.333 = Þ DT f = = 1.0015° C
1+ 2 3 DT f 1.86 0.52
2 2 DT f = 0 - 1.0015 (Q F.pt. of H 2O = 0° C )
and X H 2O = = = 0.667
1+ 2 3 = -1.0015 ° C
Problem 15: a mixture of 72 gm water and 92 gm ethyl Problem 21: Boiling point of a solution obtained by
alcohol, calculate the mole fraction for both. adding 4.18 g of a substance into 240 g water at one
(U.P. 2019) atmospheric pressure is 100.65°C. Calculate the molecular
Solution: Solve as Problem 14. mass of compound. (Molecular elevation constant of 100 g
Problem 16: One mole of a substance was dissolved in water ( K ) = 5.31) (U.P. 2017)
500 mL water. Calculate the molarity of solution.(U.P. 2017) 100 ´ K b ´ w
Solution: DTb =
n ´ 1000 1 ´ 1000 m´W
Solution: Molarity = = =2 M
V ( mL) 500 w = 4.18 g, W = 240 g, DTb = 100.65 - 100 = 0.65° C
1 K b = 5.31 K 100 g solvent mol -1 , M = ?
Problem 17: mole of a solute is dissolved in 100 g 100 ´ 5.31 ´ 4.18
10 0.65 =
solvent. Calculate the molality of solution. (U.P. 2017) m ´ 240
n 1 1000 100 ´ 5.31 ´ 4.18
Solution: Molality = ´ 1000 == ´ =1m m= = 14.228
W( g ) 10 100 0.65 ´ 240
Problem 22: 15 g of potassium chloride is dissolved in 1L
Problem 18: At a particular temperature, vapour of water. Calculate the molarity of the solution. (U.P. 2018)
pressure of pure benzene is 640 mm Hg. On adding 2.75 g of a
non- volatile solute in 39 g benzene, vapour pressure is reduced (K = 39, Cl = 35.5)
to 600 mm Hg. Calculate the molecular mass of solute. w 1000
Solution: Q M = ´
(U.P. 2017) m VmL
Solution: According to Raoult’s law, w = 15 g, m = 39 + 35.5 = 74.5, VmL = 1000 mL
P ° - Ps n P ° - Ps n 15 1000
= or = \ M = ´ = 0.2 M
P° n+N Ps N 74.5 1000
P° = 640 mm Hg, w = 2.75 g, W = 39 g Problem 23: 5.85 g of NaCl is present in 200 mL solution.
Ps = 600 mm Hg, m = ?, M = 78 What is molarity of the solution? (Na = 23, Cl = 35.5)
P ° - Ps wm 640 - 600 2.75 m (U.P. 2018)
= or = Solution: w = 5.85 g, m = 58.5, VmL = 200 mL
Ps W M 600 39 78
40 2.75 ´ 2 w 1000 5.85 1000
= Þ m = 88 \ M = ´ = ´ = 0.5 M
600 m m VmL 58.5 200
Problem 19: When 1.5 gram of a substance is dissolved in Problem 24: Calculate the molality of 20% (by weight)
60 gram water, its freezing point is lowered by 0.136°C. potassium carbonate solution. (K = 39, C = 12, O = 16)
Calculate molecular mass of the substance. Molar depression (U.P. 2018)
constant of water is 18.6. (U.P. 2017) Solution: 20% by wt. means 100 g solution contains
100 ´ K f ´ w 20 g K 2CO 3
Solution: DT f =
m´W Thus, w = 20 g, Wg = 100 - 20 = 80 g
m¢ = ( 2 ´ 39) + 12 + (16 ´ 3) = 138
19
w 1000 20 1000 1000 ´ K ¢f ´ w
Q m= ´ = ´ = 1.81 m Solution: DT f =
m¢ Wg 138 80 m´W
Problem 25: Relative density of 96% (w/w) sulphuric DT f = 0.60° C, K ¢f = ?, w = 1.822 g, m = 155, W = 100 g
acid is 1.84. Calculate its molarity and normality. 1000 ´ K ¢f ´ 1.822
0.6 = Þ K ¢f = 5.104 ° C molal -1
(U.P. 2018) 155 ´ 100
Solution: Q Relative density = 1.84 g mL-1 Problem 29: The boiling point of pure acetone is 56.38°C.
\ Mass of 100 mL solution = 1.84 ´ 100 = 184 g When 0.707 g of an organic compound is dissolved in 10 g of
Conc. of solution = 96% (w/w) acetone, there is elevation upto 56.88°C in boiling point. What
i. e., 100 g solution contains 96 g H 2SO 4 . is the molecular weight of the compound? Molecular elevation
\ Acid present in 184 g (or 100 mL) solution constant ( K m ) for acetone is 16.7°C. (U.P. 2018)
96 100 ´ K b ´ w
w= ´ 184 = 176.64 g Solution: DTb =
100 m´W
w 1000 (K b = molar elevation constant, don’t confuse with K ¢b ,
and Molarity, ( M ) = ´ i. e., molal elevation constant)
m VmL
176.64 1000 DTb = 56.88 - 56.38 = 0.5° C, w = 0.707 g, w = 10 g
= ´ = 18.02 M m = ?, K b = 16.7 ° C molar -1
98 100
100 ´ 16.7 ´ 0.707
and N = M ´ V.f. = 18.02 ´ 2 = 36.04 N 0.5 = Þ m = 236.138
Problem 26: A solution of sugar in water 5% (by mass) m ´ 10
has freezing point 271 K. Calculate the freezing point of Problem 30: At 20°C the osmotic pressure of 45 g per litre
solution of glucose in water, 5% (by mass) solution. Given solution of a substance is 3.2 atmosphere. Calculate the value
freezing point of pure water is 273.15 K. (U.P. 2015, 18) of solution constant. The molecular weight of the substance is
1000 ´ K ¢f ´ w 342. (U.P. 2019)
Solution: DT f = Solution: Given that, T = 20º C Þ 293K ,
m´W w = 45g, V = 1L, P = 3.2atm., m = 342
For sugar solution, DT f = 273.15 - 271 = 2.15 K w
w = 5 g, W = 100 - 5 = 95 g, m = 342 (C 12 H 22O11 ) PV = S. T.
1000 ´ K ¢f ´ 5 m
2.15 = …(i) 45
342 ´ 95 3.2 ´ 1 = ´ S ´ 293
342
For glucose solution, DT f = ?, w = 5 g 3.2 ´ 1 ´ 342
W = 100 - 5 = 95 g, m = 180 (C 6 H12O 6 ) S= = 0.083L atm K –1 mol -1
1000 ´ K ¢f ´ 5 45 ´ 293
DT f = …(ii) Problem 31: Calculate the mole fraction of glycerol
180 ´ 95 (C 3 H 8O 3 ) solution in 25% by weight of glycerol. (U.P. 2019)
By dividing eq. (i), by (ii) Solution: Given that, solution of glycerol is 25% by
1000 ´ K ¢f ´ 5
mass. So 100g solution contains 25g glycerol and 75g water.
2.15 342 ´ 95 180 w 25 75
= = \ nglycerol = = = 0.271, n H 2O = = 4.16
DT f 1000 ´ K ¢f ´ 5 342 m 92 18
180 ´ 95 nglycerol
\ mole fraction of glycerol =
342 nglycerol + n H 2O
DT f = 2.15 ´ = 4.08 K
180 0.271
\ F.pt. of glucose solution = 273.15 - 4.08 K = = 0.0611
0.271 + 4.16
= 269.07 K Problem 32: 9 g glucose (C 6 H 12O 6 ) is dissolved in 500 g
Problem 27: At what temperature a 5% W/V solution of of water. Find out the boiling point of the solution at a pressure
glucose will produce of osmotic pressure of 7 atm? of 1.013 bar
(R = 0.0821L atm/degree/mol) (U.P. 2018) (kb for water = 0.52 K kg mol -1 ). (U.P. 2019)
w ST Solution: w = 9g, W = 500g, K b = 0.52K kg mol -1
Solution: P = ´
m V m = 180(C 6 H12O 6 )
w = 5 g, V = 100 mL = 0.1 L, m = 180, T = ?, P = 7 atm 1000 ´ K b ´ w
S = 0.0821 L × atm K -1 mol -1 DTb =
m´W
5 0.0821 ´ T 1000 ´ 0.52 ´ 9
7= ´ Þ T = 306.94 K DTb = = 0.052º C
180 0.1 180 ´ 500
Problem 28: On dissolving 1.822 g of an organic At 1.013 bar (or 1 atm) b.pt. of water = 100º C
compound (mol. wt. 155) in 100 g of benzene, the depression \ Boiling point of solution = 100 + 0.052 = 100.052º C
in freezing point was 0.60°C. Calculate molal depression Problem 33: Calculate the molality of a solution when
constant of benzene. (U.P. 2018) 20 g NaOH is dissolved in 440 g of solvent. (U.P. 2019)
20
w 1000 Problem 39: A solution of ethyl alcohol in water is 46%
Solution: Molality = ´
m W(g) by weight. Calculate mole fraction of ethyl alcohol and water in
the solution. (C = 12, H = 1, O = 16) (U.P. 2019)
Given that, w = 20g , m(NaOH) = 40 , W = 440g
20 1000 Solution: 46% by weight solution of ethyl
Molality = ´ = 1.14m alcohol in water means :
40 440 46
Problem 34: At any temperature the vapour pressure of wt. of C 2 H 5OH, w = 46g , n = = 1 mole
solvent is 0.416 bar. 0.5 g of a non-volatile non-electrolyte solid 46
54
was dissolved in 42 g solvent (molar mass = 84 g mol -1 ). The wt. of H 2O, w = 54g , n = = 3 mole
vapour pressure of the resultant solution was round to be 0.414 18
bar. Calculate the molar mass of the solid. (U.P. 2019) 1
\ X C2H 5OH = = 0.25
Solution: Pº = 0.416 bar, w = 0.5g , W = 42g , 113
M = 84gmol -1 ; P s = 0.414 bar and X H 2O = 1 - 0.25 = 0.75
Problem 40: 2.5 g Ethanoic acid was dissolved into 75 g
According to Raoult’s law
benzene. Calculate the concentration of ethanoic acid in terms
P º-P s n w M
= = ´ of (i) Parts per million (ii) Mole fraction (iii) Percentage by
s N m W
P weight (iv) Molality (75 g benzene = 85.6 mL) (U.P. 2020)
0.416 - 0.414 0.5 84 0.414 Solution: Given that w (ethanoic acid) = 2.5g
= ´ Þm= = 207
0.414 m 42 0.416 - 0.414 W (benzene) = 75g ( V = 85.6 mL)
Problem 35: Calculate the elevation of boiling point in 2.5
n (ethanoic acid) = = 0.042 mole
water by dissolving 3.0 gm urea in 100 gm of water. The molal 60
elevation constant for water is 0.52 K kg/mol. (U.P. 2019) 75
N (benzene) = = 0.96 mole
Solution: Given that, w = 3g, W = 100g 78
K b¢ = 0.52K kg mol –1 , m urea = 60 Mass of solute 2.5
(i) ppm = ´ 10 6 = ´ 10 6
1000 ´ K ¢b ´ w 1000 ´ 0.52 ´ 3 Mass of solution 75 + 2.5
DTb = = = 0.26º C = 3.22 ´ 10 4 ppm
m´W 60 ´ 100
(ii) Mole of fraction:
Problem 36: A decrease of 1.2°C in freezing point took n 0.042
place on dissolving 7.5 gm of substance in 75 gm of water. Find X (ethanoic acid) = = = 0.041
out the molecular weight of the substance. (Molal depression n + N 0.042 + 0.96
constant for water k¢f = 1.86° C ). (U.P. 2019) and X (benzene) = 1 - 0.041 = 0.959
Mass of solute 2.5
Solution: DT f = 1.2º C, w = 7.5g, W = 75g, (iii) Mass % = ´ 100 = ´ 100
K ¢f = 1.86º C molal -1 , m = ? Mass of solution 75 + 2.5
= 3.22%
1000 ´ K ¢f ´ w Moles of solute 0.042
DT f = (iv) Molality = ´ 1000 = ´ 1000
m´W Mass of solvent 75
1000 ´ 1.86 ´ 7.5 = 0.56 m
1.2 =
m ´ 75 Problem 41: 45 g Ethylene glycol (C2 H6 O2 ) was mixed
1000 ´ 1.86 ´ 7.5 with 600 g water. Calculate depression in freezing point and
m= = 155
1.2 ´ 75 freezing point of the solution. (U.P. 2020)
Problem 37: In 214.2 gm syrup of sugar, 34.2 gm sugar Solution: Freezing point depression
(C 12 H 22O11 ) is present. What will be the molality of sugar w ´ K f ´ 1000 45 ´ 1.86 ´ 1000
syrup ? (C = 12 , H = 1, O = 16) (U.P. 2019) DT = = = 2.25 K
M ´W 62 ´ 600
Solution: Mass of solution = 214.2g Freezing point of pure water = 273.15 K
Mass of sugar (w) = 34.2g Therefore, freezing point of solution = 273.15 - 2.25
\ Mass of water (W) = 214.2 - 34.2 = 180g = 270.9 K
m sugar = 342(C 12 H 22O11 ), [Note M for ethylene glycol (C 2 H 6O 2 ) = 62]
w 1000 34.2 1000 Problem 42: The vapour pressure of water at 293 K is
Molality = ´ = ´ = 0.55m 17.535 mmHg. If 25 gm of glucose is dissolved in 450 gm of
m W 342 180 water, calculate the vapour pressure of solute (water) at 293 K.
Problem 38: Calculate the elevation in boiling point of (U.P. 2019)
water when 3 gm of urea is dissolved in 100 gm water. Molal Solution: Raoult’s law :
elevation constant for water is 0.52 K kg mol -1 . (U.P. 2019) P o - Ps n
Solution: o
=
K ´ w ´ 1000 0.52 ´ 3 ´ 1000 P n + N
DT = b = = 0.26º C o
P = 17.535 mm Hg, Ps = ?
m´W 60 ´ 100
25 450
n= = 0.1388 , N = = 25
180 18
21
17.535 - Ps 0.1388 m = 342 (C 12 H 22O11 ), w = 25g
=
17.535 25 + 0.1388 100 ´ K b ´ 1.17
Ps = 17.44mm Hg 0.017 = Þ K b = 1.24
342 ´ 25
Problem 43: At 20°C, on dissolving 10 g of a non-volatile Problem 47: Calculate mole fraction of Ethylene Glycol
non-electrolyte substance in 100 g of water, vapour pressure
(C2 H6 O2 ) if the mass of C2 H6 O2 in the aqueous solution is
decreases from 17.535 mm to 17.235 mm. Calculate the 20%. (U.P. 2020)
molecular weight of the solute. (U.P. 2019)
Solution: Let the total mass of solution is 100 g
Solution: Given that
Q Mass of ethylene glycol (C 2 H 6O 2 ) = 20% = 20 g
w = 10g, W = 100 g, P º = 17.535mm
\ Mass of water = 80% = 80 g
P s = 17.235mm 20 80
w n Glycol = = 0.323, N Water = = 4.44
s 62 18
P -P
º n w M
Using* = = m = ´ Mole fraction, X Ethylene glycol =
n
Ps N W m W n+N
M 0.323
17.535 - 17.235 10 18 = = 0.0677
= ´ 0.323 + 4.44
17.235 m 100 Problem 48: An aqueous solution of an organic
31.023 substance, in which 3 gm of substance is dissolved in 50 gm
m= = 103.41
0.3 water, has a boiling point of 100.51° C. Calculate the molecular
P º -P s n weight of the substance (kb for water = 0.51° C molal -1 ).
* Use of = is not advisable as it gives wrong (U.P. 2020)
P º N
result. 1000 ´ K ¢b ´ w
Solution: DTb =
Problem 44: When 2.5 g of a non-volatile solute is m´W
dissolved in 100 g benzene, elevation in b.pt. is 0.4°C. If molal DTb = 100.51 - 100 = 0.51° C , K b = 0.51° C molal -1 ,
elevation constant for benzene is 2.67, calculate the molar w = 3g, W = 50g, m = ?
mass of solute. (U.P. 2020)
1000 ´ K ¢b ´ w 1000 ´ 0.51 ´ 3
Solution: DTb = 0.51 = Þ m = 60
m´W m ´ 50
Given that, DTb = 0.4° C, w = 2.5g, W = 100g, Problem 49: A solution of glucose containing 0.45 gm in
K ¢b = 2.67 K -1 molal -1 250 ml water was prepared at 0º C. What is its osmotic
1000 ´ 2.67 ´ 2.5 pressure?
0.4 = Þ m = 166.88 (R = 0.0821 litre atm/degree/mole) (U.P. 2020)
m ´ 100 w ST
Problem 45: A sample of H 2 SO 4 is 94% ( w V ) and its Solution: P = ´
m V
density is 1.84 g/mL. Calculate its molality. (U.P. 2020)
P = ?, w = 0.45 g, m = 180 (C 6 H12O 6 ),
( H = 1, O = 16, S = 32) 250
Solution: Molecular mass of H 2SO 4 = 2 + 32 + 64 = 98 V= = 0.25 L, T = 0° C = 273 K,
1000
94% (w/V) H 2SO 4 means 94 gm H 2SO 4 is dissolved in S = 0.0821 L.atm -1 K -1 mol -1
100 mL solution, thus 0.45 0.0821 ´ 273
P= ´ = 0.224 atm
Mass of solution = d ´ V = 1.84 ´ 100 = 184.0 g 180 0.25
Mass of solvent = 184 - 94 = 90g Problem 50: The depression in freezing point on
Molality of solution dissolving 17.1 gm sucrose (mol. wt. = 342) in 100 gm water is
1000 ´ wt. of solute 0.93° C. Calculate molal depression constant ( k f ) of water.
( m) =
mass of solvent ´ molecular mass of H 2SO 4 (U.P. 2020)
1000 ´ 94 Solution: DT f = 0.93° C, w = 17.1 g,
= = 10.65 m
90 ´ 98 m = 342 (C 12 H 22O11 ), W = 100 g
Problem 46: On dissolving 1.17 gm of cane sugar in 25 1000 ´ K ¢f ´ w
gm of water, the boiling point of solvent is increased by 0.017° C. DT f =
What will be the molar elevation constant of the solvent? m´W
(Molecular weight of cane sugar = 342) (U.P. 2020) 1000 ´ K ¢f ´ 17.1
100 ´ K ¢b ´ w 0.93 = = 1.86 K × molal -1
Solution: DTb = 342 ´ 100
m´W
Given that, w = 1.17 g, K b = ?, DTb = 0.017 ° C,
22

Solution of NCERT Text Book Problems

Q.1 Concentrated nitric acid used in laboratory work is 106


\ x= = 0.558g
68% nitric acid by mass in aqueous solution. What 190
should be the molarity of such a sample of the acid if So, mass of Na 2CO 3 in mixture = 0.558g
the density of the solution is 1.504g mL–1 \ mass of NaHCO 3 in mixture = 1 – 0.558
Ans. 68% HNO 3 means, 68g HNO 3 present in 100g = 0.442g
solution. Na 2CO 3 + 2HCl ¾® 2NaCl + CO 2 + H 2O
100 1 mole 2 mole
Q Volume of solution ( V ) = = 66.49 mL
1.504 NaHCO 3 + HCl ¾® NaCl + CO 2 + H 2O
1 mole
w ´ 1000 68 ´ 1000 1 mole
\ Molarity = = = 16.23 M
m ´ V (in mL) 63 ´ 66.49 Q For complete neutralisation of 106g Na 2CO 3
required mole of HCl = 2
Q.2 A solution of glucose in water is labelled as 10% w/w,
\ For complete neutralisation of 0.558 g Na 2CO 3
what would be the molality and mole fraction of each 2 ´ 0.558
component in the solution ? If the density of solution required mole of HCl = = 0.01053 mole
106
is 1.2g mL–1 , then what shall be the molarity of the
Q For complete neutralisation of 84g NaHCO 3
solution ? required mole of HCl = 1
Ans. 10 %( w w ) means, 10g glucose present in 100g \ For complete neutralisation of 0.442 g NaHCO 3
glucose solution. 1 ´ 0.442
weight of solution required mole of HCl = = 0.00526 mole
\ Volume of solution = 84
density So, total mole of HCl = 0.01053 + 0.00526
100 = 0.01579 mole
= = 83.33 mL
1.2 1000 ´ 0.01579
\ Required volume of 0.1 M HCl =
Weight of solvent = 100 - 10 = 90g 0.1
w ´ 1000 10 ´ 1000 = 157.9 mL
\Molality = = = 0.617m
m ´ W(in gm) 180 ´ 90 Q.4 A solution is obtained by mixing 300g of 25%
w ´ 1000 10 ´ 1000 solution and 400g of 40% solution by mass. Calculate
Molarity = = = 0.67 M the mass percentage of the resulting solution.
m ´ V(in mL) 180 ´ 83.33
300 ´ 25
10 Ans. Mass of solute in 25% solution = = 75g
\ Moles of glucose ( N ) = = 0.055 100
180 Mass of solute in 40% solution
90 400 ´ 40
\ Moles of water ( n ) = =5 = = 160g
18 100
Total moles = 0.055 + 5 = 5.055 Total mass of solute = 75 + 160 = 235g
5
Mole fraction of water = = 0.989 Total mass of solution = 300 + 400 = 700g
5.055 235 ´ 100
Mole fraction of glucose = 1 - 0.989 = 0.011 % of solute in final solution = = 33.57 %
700
Q.3 How many mL of 0.1 M HCl are required to react % of water in final solution = 100 - 33.57 = 66.43 %
completely with 1g mixture of Na 2CO 3 and NaHCO 3 Q.5 An antifreeze solution is prepared from 222.6g of
containing equimolar amounts of two ? ethylene glycol (C 2 H 6O 2 ) and 200g of water.
Ans. Mass of mixture Na 2CO 3 and NaHCO 3 = 1g Calculate the molality of the solution. If the density of
Suppose, mass of Na 2CO 3 in mixture = x g the solutions is 1.072 g mL-1 , then what shall be the
So, amount of NaHCO 3 = (1 - x ) g molarity of the solution ?
x w ´ 1000
\ mole of Na 2CO 3 = Ans. Molality =
106 m ´ mass of solvent (in gm)
(Q mol. mass of Na 2CO 3 = 106) 222.6 ´ 1000
1– x = = 17.95 m
\ mole of NaHCO 3 = 62 ´ 200
84
(Q mol. mass of NaHCO 3 = 84) (m for ethylene glycol = 62)
mass of solution
Q Their moles are equal, Volume of solution =
x 1- x density
= 222.6 + 200
106 84 = = 394.22 mL
or 84 x = 106 - 106 x 1.072
or 190 x = 106
23
w ´ 1000 w 35
Molarity = Moles of octane = = = 0.31
m ´ volume of solution (in mL) m 114
222.6 ´ 1000 Total moles = 0.26 + 0.31 = 0.57
= = 9.11m 0.26
62 ´ 394.2 Mole fraction of heptane = = 0.456
Q.6 A sample of drinking water was found to be severely 0.57
contaminated with chloroform (CHCl 3 ) supposed to 0.31
Mole fraction of octane = = 0.544
be a carcinogen. The level of contamination was 15 0.57
ppm (by mass) : Q Solution is ideal, so
(i) express this in percent by mass Ps = PA X A + PB X B
(ii) determine the molality of chloroform in the = (105.2 ´ 0.456) + ( 46.8 ´ 0.544) = 73.43 kPa
water sample. Q.10 The vapour pressure of water is 12.3 k Pa at 300 K.
Ans. Mass of solute chloroform = 15 gram Calculate vapour pressure of 1 molal solution of non-
Mass of solution = 10 6 gram volatile solute in it.
15 ´ 100 Ans. In 1 molal solution, moles of solute = 1
(i)% mass = = 1.5 ´ 10 –3
10 6 Mass of water 1000g
w ´ 1000 1000
(ii) Molality = \ moles of water = = 55.55
m ´ mass of solvent (in g) 18
w = 15 gram, Total moles = 1 + 55.55 = 56.55
Mass of solvent = 10 6 - 15 = 999985 g 1
Mole fraction of solute = = 0.0177
For CHCl 3 ,m = 119.5 56.55
15 ´ 1000 According to Raoult's law,
\ Molality = = 1.25 ´ 10 –4 m P º– Ps
119.5 ´ 999985 = x solute
Q.7 The partial pressure of ethane over a solution Pº
12.3 - Ps
containing 6.56 ´ 10 –2 g of ethane is 1 bar. If the or = 0.0177 Þ Ps = 12.08 kPa
12.3
solution contains 5.00 ´ 10 -2 g of ethane, then what
Q.11 Calculate the mass of a non-volatile solute (molar
shall be the partial pressure of the gas ? mass 40 g mol -1 ) which should be dissolved in 114g
Ans. According to Henry law, octane to reduce its vapour pressure to 80%.
P = KH × x Ans. Since the vapour pressure is reduced to 80%, so
In Ist case, 1 = K H ´ 6.56 ´ 10 –2 ...(1) P ° - Ps w ´ M
=
In IInd case, P = K H ´ 5.0 ´ 10 –2 ...(2) Ps m´ W
Eq.(2) divided by eq. (1), P° = 100, Ps = 80 , w = ?,
5.0 ´ 10 –2 M octane = 114, m= 40 , W = 114
P= = 0.762 bar
6.56 ´ 10 –2 100 - 80 w ´ 114
= Þ w = 10 g
Q.8 An aqueous solution of 2% ( w V ) non-volatile solute 80 40 ´ 114
exerts a pressure of 1.004 bar at the normal boiling Q.12 A solution containing 30g of non-volatile solute
point of the solvent. What is the molar mass of the exactly in 90 g of water has a vapour pressure of 2.8
solute ? kPa at 298K. Further, 18 g of water is then added to
Ans. Vapour pressure of water at boiling point the solution and the new vapour pressure becomes
( P º ) = 1 atm = 1.013 bar 2.9 kPa at 298 K. Calculate :
Vapour pressure of solution ( Ps ) = 1.004 bar (i) molar mass of the solute,
P ° - Ps w ´ M (ii) vapour pressure of water at 298 K
= P ° - Ps
Pº m´ W w´ M
Ans. =
w = 2 g, M = 18 m = ?, W = 100 - 2 = 98 g Ps m´ W
1.013 - 1.004 2 ´ 18 w = 30g, W = 90g M H 2 O = 18, Ps = 2.8
= Þ m = 41.35
1.013 m ´ 98 P ° - 2.8 30 ´ 18 6
= = ...(1)
Q.9 Heptane and octane form an ideal solution. At 373 K, 2.8 m ´ 90 m
the vapour pressures of the two liquid components Again, w = 30g, W = 90 + 18 = 108g
are 105.2 kPa and 46.8 kPa respectively. What will be M H 2 O = 18, Ps = 2.9
the vapour pressure of a mixture of 26.0g of heptane P ° - 2.9 30 ´ 18 5
= = …(2)
and 35g of octane ? 2.9 m ´ 108 m
Ans. Molar mass of heptane (C 7 H16 ) = 100 From equations (1) and (2),
Molar mass of octane (C 8 H18 ) = 114 P° = 3.53 kPa, m = 23g mol
w 26 Q.13 A 5% solution (by mass) of cane sugar in water has
Moles of heptane = = = 0.26
m 100 freezing point of 271 K. Calculate the freezing point
24

of 5% glucose in water if freezing point of pure water Q.16 If the density of some lake water is 1.25g mL–1 and
is 273.15 K. contains 92g of Na + ions per kg of water, calculate
K f ´ w ´ 1000
Ans. DT f = , the molality of Na + ions in the lake.
m´ W Ans. Mass of solvent, W = 1000 g
w = 5, W = 95, m = 342 for sugar, Mass of solute, w = 92g
DT f = 273.15 - 271 = 2.15 K '
m Na = 23
K f ´ 5 ´ 1000 w ´ 1000 92 ´ 1000
2.15 = Þ K f = 13.97 \ Molality = = = 4m
342 ´ 95 m' ´ W(in g) 23 ´ 1000
w = 5, W = 95, m = 180 for glucose, Q.17 If the solubility product of CuS is 6 ´ 10 -16 , calculate
K f = 13.97 , DT = ? the maximum molarity of CuS in aqueous solution.
DT =
13.97 ´ 5 ´ 1000
= 4.085 Ans. CuS Cu 2+ + S 2–
180 ´ 95 Suppose solubility of CuS is S mol/litre.
Freezing point of glucose solution = 273.15 – 4.085 [Cu 2+ ] = S, [S 2– ] = S ,
= 269.065 K K sp = [Cu 2+ ] [S 2 – ] = S ´ S = S 2
Q.14 Two elements A and B form compounds having \ S = K sp = 6 ´ 10 –16
molecular formula AB2 and AB4 . When dissolved in
= 2.45 ´ 10 –8 mol / litre
20 g of benzene, 1 gram of AB2 lowers the freezing
point by 2.3°C whereas 1 gram of AB4 lowers the Q.18 Calculate the mass percentage of aspirin (C 9 H 8O 4 ) in
freezing point by 1.3°C. The molal depression acetonitrile (CH 3CN) when 6.5g of C 9 H 8O 4 is
constant for benzene in 1000 g is 5.1, calculate the dissolved in 450g of CH 3CN.
atomic masses of A and B. Ans. Mass of aspirin = 6.5g,
Ans. Suppose, atomic mass of A and B is a and b Mass of acetonitrile = 450g,
respectively. Total mass = 6.5 + 450 = 456.5
6.5 ´ 100
For AB 2 : w = 1 g , W = 20 g, M = a + 2b, DT = 2.3 % of aspirin = = 1.424%
K ¢f ´ w ´ 1000 456.5
DT = Q.19 Nalorphene (C 19 H 22 NO 3 ), similar to morphine, is
M´ W used to combat withdrawal symptoms in narcotic
5.1 ´ 1 ´ 1000 users. Dose of nalorphene generally given is 1.5 mg.
2.3 =
( a + 2b) ´ 20 Calculate the mass of 1.5 ´ 10 –3 m aqueous solution
or a + 2b = 110.87 …(1) required for the above dose.
For AB 4 : w = 1 g, W = 20 g, M = a + 4b, DT = 1.3 Ans. Mass of solute, w = 1.5 mg = 0.0015g
K ¢f ´ w ´ 1000 Mol. mass of solute (C 19 H 21 NO 3 ), M = 311
DT = w ´ 1000
M´ W Q molality, m =
5.1 ´ 1 ´ 1000 m ´ W(in g)
1.3 =
( a + 4b) ´ 20 0.0015 ´ 1000
\ W = = 3.2154g
or a + 4b = 196.15 …(2) 311 ´ 1.5 ´ 10 –3
Solving equations (1) and (2), Q Mass of solvent = 3.2154g
a = 25.59, b = 42.64 \ Mass of solution = 3.2154 + 0.0015 = 3.2169 g
Q.15 At 300 K, 36g of glucose present per litre of its Q.20 Calculate the amount of benzoic acid (C 6 H 5COOH)
solution has an osmotic pressure of 4.98 bar. If the required for preparing 250 mL of 0.15 M solution in
osmotic pressure of the solution is 1.52 bar at the methanol.
same temperature, what would be its concentration? w ´ 1000
w Ans. Molarity ( M )= ,
Ans. For Ist case : pV= ST m ´ V (in mL)
m Molar mass of benzoic acid = 122
36ST
or 4.98 ´ 1 = …(1) w = ?, M = 0.15, V = 250 mL
180 w ´ 1000
For IInd case : p = CST 0.15 = Þ w = 4.575 g
122 ´ 250
æ w ö Q.21 Calculate the depression in the freezing point of
or 1.52 = CST ç\C = ÷ …(2)
è m´ V ø water when 10g of CH 3CH 2CHClCOOH is added to
From equation (1) and (2), 250g of water.
1.52 C ´ 180 ( K a = 1.4 ´ 10 –3 , K f = 1.86 K kg mol –1 )
=
4.98 36 Ans. Deleted for examination 2021.
or C = 0.061 mol/litre
25

Q.22 19.5g of CH 2 FCOOH is dissolved in 500g of water. Plot this data also on the same graph paper. Indicate
The depression in the freezing point of water whether it has positive deviation or negative
observed is 1.0°C. Calculate the van’t Hoff factor and deviation from the ideal solution.
dissociation constant of fluoroacetic acid. Ans.
Ans. Deleted for examination 2021. X acetone 0.0 0.118 0.234 0.360 0.508 0.582 0.645 0.721
Q.23 Vapour pressure of water at 293 K is 17.535 mm Hg. Pacetone 0.0 54.9 110.1 202.4 322.7 405.9 454.1 521.1
Calculate the vapour pressure of water at 293 K when Pchloroform 632.8 548.1 469.4 359.7 257.7 193.6 161.2 120.7
25g of glucose is dissolved in 450g of water. Ptotal 632.8 603.0 579.5 562.1 580.4 599.5 615.3 641.8
P ° - Ps w´ M X acetone is given; X chloroform = 1 - X acetone . The total
Ans. =
Ps m´ W vapour pressure of the solution is less than the ideal
P° = 17.535 mm, Ps = ?, w = 25g, W = 450g, vapour pressure (PAo X A + PBo X B ) at each point. The
m for glucose = 180 , M for water = 18 total V.P. decreases upto 562.1 and then increases.
17 .535 - Ps 25 ´ 18 There is a dip in the curve. Thus the solution shows
= –ve deviation from Raoult’s law.
Ps 180 ´ 450
or Ps = 17.44 mm
Q.24 Henry’s law constant for the molarity of methane in 700
benzene at 298 K is 4.27 ´ 10 5 mm Hg. Calculate the
600 Ptotal
solubility of methane in benzene at 298 K under 760

Vapour pressure (mm)


mm Hg.
500
Ans. P = K H × x
P 760
x= = 5
= 1.78 ´ 10 -3 400
KH 4.27 ´ 10 Pchloroform

Q.25 100g of liquid A (molar mass 140g mol –1 ) was 300


dissolved in 1000g of liquid B (molar mass 180
200
g mol –1 ). The vapour pressure of pure liquid B was Pacetone
found to be 500 torr. Calculate the vapour pressure of 100
pure liquid A and its vapour pressure in the solution if
the total vapour pressure of the solution is 475 torr. 0
0 0.1 0.2 0.3 0.4 0.5 0.6 0.7 0.8
w 100 Mole fraction of acetone (Xacetone)
Ans. Moles of liquid A = = = 0.714
m 140 Q.27 Benzene and toluene form ideal solution over the
w 1000 entire range of composition. The vapour pressure of
Moles of liquid B = = = 5.56
m 180 pure benzene and toluene at 300 K are 50.71 mm Hg
n 0.714 and 32.06 mm Hg respectively. Calculate the mole
Mole fraction of A : X A = =
n + N 0.714 + 5.56 fraction of benzene in vapour phase if 80g of benzene
= 0.1132 is mixed with 100g of toluene.
Mole fraction of B : X B = 1 - 0.1132 = 0.8868 w 80
Ans. Moles of benzene ( n B ) = = = 1.026,
Ps = PAo X A + PBo X B m 78
Ps = 475, PBo = 500 w 100
Moles of toluene ( n T ) = = = 1.087
475 = PAo ´ 0.1132 + 500 ´ 0.8868 m 92
or PAo = 280.7 torr Total moles = 1.026 + 1.087 = 2.113
1.026
Now PA = PAo X A Mole fraction of benzene ( X B ) = = 0.486
2.113
PA = 280.7 ´ 0.1132 = 31.78 torr Mole fraction of toluene ( X T ) = 1 - 0.486 = 0.514
Q.26 Vapour pressure of pure acetone and chloroform at PAo (for benzene) = 50.71,
328 K are 741.8 mm Hg and 632.8 mm Hg
respectively. Assuming that they form ideal solution PBo (for toluene) = 32.06
over the entire range of composition, plot Ps = PAo X A + PBo X B
Ptotal , Pchloroform and Pacetone as a function of X acetone . = (50.71 ´ 0.486) + (32.06 ´ 0.514)
The experimental data observed for different = 24.65 + 16.48 = 41.13
compositions of mixture is Mole fraction of benzene in vapour phase
100 ´ X acetone 0.0 11.8 23.4 36.0 50.8 58.2 64.5 72.1 vapour pressure of benzene in solution
=
Pacetone 0.0 54.9 110.1 202.4 322.7 405.9 454.1 521.1 total vapour pressure
Pchloroform 632.8 548.1 469.4 359.7 257.7 193.6 161.2 120.7 50.71 ´ 0.486
= = 0.60
41.13
26

Q.28 The air is a mixture of number of gases. The major Ans. Since we are to calculate the mass of the solute,
components are oxygen and nitrogen with i wST
p=
approximate proportion of 20% is to 79% by volume m´ V
at 298 K. The water is in equilibrium with air at a p = 0.75 , i = 2.47 , S = 0.0821 , T = 300 K,
pressure of 10 atm. At 298 K, if the Henry’s law m CaCl 2 = 111, V = 2.5 L, w = ?
constant for oxygen and nitrogen at 298 K, are 2.47 ´ w ´ 0.0821 ´ 300
0.75 =
3.30 ´ 107 mm and 6.51 ´ 107 mm respectively, 111 ´ 2.5
calculate the composition of these gases in water. \ w = 3.42 g
Ans. Total pressure of air with water at equilibrium 3.42
or = 0.03 mole
\ Partial pressure of O 2 , 111
20 Q.30 Determine the osmotic pressure of a solution
PO2 = ´ 10
100 prepared by dissolving 25 mg of K 2SO 4 in 2 litre of
= 2 atm = 2 ´ 760 = 1520 mm water at 25°C, assuming that it is completely
\ Partial pressure of N 2 , dissociated.
79 Ans. K 2SO 4 ¾® 2K+ + SO 2–
PN 2 = ´ 10 4
100 Q Ions produced by one mole of K 2SO 4 on complete
= 7.9 atm = 7.9 ´ 760 = 6004 mm dissociation = 3., \ i = 3
According to Henry law, Mass of the solute ( w ) is given
PO2 = K H × x O2 iwST
PO 1520 p=
x O2 = 2 = 7
= 4.61 ´ 10 -5 m´ V
KH 3.30 ´ 10 w = 25 ´ 10 –3 g, V = 2L, T = 298K, S = 0.0821,
PN 2 6004 m K 2 SO4 = 174
xN2 = = 7
= 9.22 ´ 10 -5
KH 6.51 ´ 10 3 ´ 25 ´ 10 –3 ´ 0.0821 ´ 298
Q.29 Determine the amount of CaCl 2 (i = 2.47) dissolved p=
174 ´ 2
in 2.5 litre of water such that its osmotic pressure is = 5.27 ´ 10 –3 atm
0.75 atm at 27°C.
vvv
27

Chapter

ELECTROCHEMISTRY
Syllabus: Oxidation-Reduction reactions, Conductivity of electrolytic solutions, Specific and molar conductivity, Variation of
conductivity with concentration, Kohlrausch law, Electrolysis and *Laws of electrolysis (elementary idea), Dry cell,
Electrolytic and Galvanic cell, Lead storage battery, emf of cell, Standard electrode potential Nernst equation and its
application in chemical cells, *Fuel cell, Corrosion.
* Topics Deleted for Examination 2021

Objective Questions
1. The ion having highest conductance in aqueous solution (a) Mg 2+ (b) Ba 2+ (c) Na + (d) Li +
is : (U.P. 2014, 15) 8. The correct order of reactivity of Mg, Cu, Na and Au is:
(a) Li + (b) Na + (c) K + (d) Cs + (U.P. 2012, 16)
2. At 25°C specific conductance of N 50 KCl solution is (a) Au > Cu > Mg > Na (b) Na > Mg > Cu > Au
0.002765 mho cm -1 . If resistance of cell (with solution) (c) Mg > Cu > Au > Na (d) Cu > Mg > Na > Au
is 400 ohm, then cell constant is : (U.P. 2017) 9. The standard electrode potentials* of four metals
(a) 0.553cm -1 . cm -1
(b) 1106 A, B , C and D are + 1.5 V, - 2.0 V, + 034 . V and
-0.76 V respectively. The order of decreasing reactivity
(c) 2.212cm -1 (d) none of these
of these metal is: (U.P. 2014, 20)
3. Standard reduction potentials for three metallic cations (a) A > C > D > B (b) A> B > D>C
X , Y and Z are + 0.52, -3.03 and -1.18 respectively. The (c) B > D > C > A (d) D> A> B >C
correct order of reducing powers of corresponding 10. Standard reduction potentials of elements A, B , C and D
metals is : (U.P. 2017, 18) are -290
. , +150. , - 0.74 and 034
. V respectively. Among
(a) Y > Z > X (b) X > Y > Z these, the strongest oxidant is: (U.P. 2016)
(c) Z > Y > X (d) Z > X > Y (a) A (b) B (c) C (d) D
4. Standard reduction potential of following half cell 11. Which of the following oxide will be reduced by
reactions are given below : (U.P. 2010, 15) hydrogen ? (U.P. 2010, 17)
Zn 2( aq
+
.) + 2 e -
º Zn ( s) , - 0.762 (a) Na 2O (b) MgO (c) Al 2O 3 (d) Ag 2O
Cr (3aq
+
.) + 3e
-
º Cr ( s) , - 0.740 12. The standard reduction potentials of Li, Ba, Na and Mg
+ 2e º H
at 25°C are -3.05, -2.73, -2.71 and - 2.37 volts
2 H +( aq.) -
2 ( g ) , 0.00 respectively. The strongest oxidising agent is:
Fe 3( aq
+
.) + e º Fe
- 2+
( aq.) , 0.770 (U.P. 2019)
Which of them is strongest reducing agent ? (a) Ba 2+ (b) Mg 2+ (c) Na + (d) Li +
(a) Zn ( s) (b) H 2 ( g ) (c) Fe 2+
( aq.) (d) Cr ( s) 13. Following four colourless salt solutions are placed in
separate test tubes and a strip of copper is dipped in each.
5. Which of the following metal is easily oxidised ?
Which solution does finally turns blue? (U.P. 2019, 20)
(U.P. 2003, 17)
(a) Zn(NO 3 ) 2 (b) Cd(NO 3 ) 2
(a) Cu (b) Al (c) Ag (d) Pt
(c) AgNO 3 (d) Pb(NO 3 ) 2
6. Which of the following reaction is not possible ?
14. The electrode potential of a half cell (electrode) depends
(U.P. 2004, 17)
upon : (U.P. 2018, 19)
(a) Cu + 2AgNO 3 ¾® Cu(NO 3 ) + 2Ag
(a) nature of metal
(b) CaO + H 2 ¾® Cu + H 2O
(b) conc. of metal ions in the solution
(c) CuO + H 2 ¾® Cu + H 2O
(c) temperature
(d) Fe + H 2SO 4 ¾® FeSO 4 + H 2
(d) all of the above
7. At 25ºC, standard oxidation potential of Li | Li + , 15. Correct representation of following cell
Ba | Ba 2+ , Na | Na + and Mg | Mg 2+ pairs are +3.05, Zn + 2Ag + ¾® Zn 2+ + 2Ag is : (U.P. 2018)
+2.73, +2.71 and +2.37 V. Strongest oxidising agent is:
(U.P. 2009, 13, 18) * E Ro . P. values
28

(a) Ag | Ag + || Zn | Zn 2+ (b) Zn | Zn 2+ || Ag + | Ag (c) Al gets reduced


(c) 2Ag | Ag + || Zn | Zn 2+ (d) Zn | Zn 2+ || 2Ag | Ag + (d) CuSO 4 gets decomposed
18. The standard reduction potential ( E º red ) values of A, B
16. Unit of specific conductance is : (U.P. 2018)
(a) cm -2 ohm -1 (b) cm × ohm -1 eq -1 and C are + 0.68 V, –2.54 V and – 0.50 V respectively.
The order of their reducing power is : (U.P. 2019)
(c) cm -1 ohm -1 (d) cm -2 ohm (a) A > B > C (b) A > C > B
17. CuSO 4 is not stored in Aluminium bottle, because : (c) C > B > A (d) B > C > A
(U.P. 2019) 19. On increasing the dilution the specific conductance :
(a) Cu gets oxidised (U.P. 2020)
(b) Cu 2+ gets reduced (a) increases (b) decreases
(c) remains constant (d) none of these
Answers
1. (d) 2. (b) 3. (a) 4. (a) 5. (b) 6. (b) 7. (a) 8. (b) 9. (c) 10. (b)
11. (d) 12. (b) 13. (c) 14. (d) 15. (b) 16. (c) 17. (b) 18. (d) 19. (b)

Very Short Answer Type Questions


Q.1 Can we store a solution of CuSO 4 in an iron vessel? e. g., Zn ¾® Zn 2+ + 2e - (anode half cell)
On addition of Fe turnings in CuSO 4 solution, blue Cu 2+ + 2e - ¾® Cu (cathode half cell)
colour slowly disappears, why ? (U.P. 2006, 16, 17) Q.8 Explain with reason, which one of the following
Ans. No. Fe displaces Cu from CuSO 4 . reactions is oxidation and which one is reduction.
Q.2 Among Mg, Zn, Cu, Ag, which elements react with Zn - 2e - ¾® Zn 2+ …(i)
acids liberating H 2 ? (U.P. 2016) Cl 2 + 2e - ¾® 2Cl - …(ii) (U.P. 2018)
Ans. Since Mg and Zn are placed above hydrogen in Ans. (i) is oxidation because it is de-electronation process
electrochemical series, they react with acid to liberate while (ii) is reduction as it is reduction process.
H 2 gas. Cu and Ag do not liberate H 2 gas. Q.9 Discuss the displacement of metals from salt solutions,
Q.3 Amongst Na 2O, MgO, Al 2O 3 , CuO and Ag 2O, which on the basis of electrochemical series. (U.P. 2018)
oxide may be reduced by H 2 ? Why? (U.P. 2017) Ans. In ECS, metals are arranged in decreasing order of their
Ans. According to electrochemical series, oxides of metals reactivity (or ability of emit electron). A metal kept
placed below hydrogen can be reduced by H 2 . Thus, above in ECS, will displace the ions of metal, (kept
CuO and Ag 2O will be reduced by H 2 . below it in ECS) from its aqueous solution.
Q.4 H 2 is liberated when dil. HCl reacts with Zn but no H 2 e. g., Zn will displace Cu 2+ , Cu will displace Ag + .
is liberated when dil. HCl reacts with Ag, why ? Q.10 Can Br - be oxidised into Br 2 by Fe 3+ ? (U.P. 2018)
o o
(U.P. 2017) E 3 + 2 + = 0.771 V, E - = 1.09 V
Fe / Fe Br2 / Br
Ans. In ECS, Zn is above H while Ag is below H, so Zn can
displace H 2 from HCl but Ag can’t. Ans. Because E Ro . P.
of Br 2 /Br is higher than E Ro . P. of Fe 3+ /
-

Q.5 What happens when an iron nail is placed in copper Fe 2+ , so Fe will not oxidise Br - to Br 2 . Higher is the
3+

sulphate solution ? (U.P. 2016) reduction potential, more is tendency to get reduced.
Ans. In electrochemical series, Fe is placed above copper. So, Br 2 will be reduced before Fe 3+ .
Thus, it is more reactive than Cu. Therefore iron
displaces the Cu from CuSO 4 solution. Q.11 Explain strong and weak electrolytes with examples.
Fe + CuSO 4 ¾® FeSO 4 + Cu (U.P. 2019)
Q.6 Though aluminium is placed above hydrogen in Ans. Strong and weak Electrolytes : The electrolytes
electrochemical series but it is stable in air and water. which are almost completely ionized in solution are
Why? (U.P. 2016) classified as strong electrolytes. For example :
Ans. In air, a thin film of Al 2O 3 gets deposited over H 2SO 4 ,HNO 3 , salts.
On the other hand, the electrolytes which are feebly
aluminium surface and that is why Al is not affected by dissociated and have low degree of dissociation are
air and water. classified as weak electrolytes.
Q.7 Define half cell giving an example. (U.P. 2017) e.g., NH 4 OH,CH 3 COOH,HCN etc.
Ans. Electrode of a cell is also called half cell because two
halfcells (or electrodes) anode and cathode are
combined to form a cell.
29

Short Answer Type Questions


Q.1 When Cu reacts with HCl, H 2 is not liberated but when Ans. We know that the metals which are above hydrogen in
Zn reacts with HCl, H 2 is liberated, why? electrochemical series readily liberate hydrogen from
(U.P. 2016, 17) dilute acids. Mg occupies its position above hydrogen
Ans. In electrochemical series, Cu is placed below the in the electrochemical series. Thus, Mg reacts with acid
hydrogen but zinc is placed above hydrogen. Thus, Cu liberating H 2 .
is less reactive than hydrogen while zinc is more Mg + 2HCl ¾® MgCl 2 + H 2 ­
reactive than hydrogen. Q.7 When zinc rod is dipped in blue coloured CuSO 4
Q.2 Fe displaces Cu from CuSO 4 solution but Ag does not, solution it decolourises, explain why? (U.P. 2018)
why? (U.P. 2013, 17) Ans. Blue colour of CuSO 4 solution is due to the presence of
Ans. In electrochemical series, Fe is placed above Cu thus, it Cu 2+ ions in it. When zinc rod is dipped in CuSO 4
is more reactive than Cu while Ag is placed below Cu,
thus, it is less reactive than Cu. solution, then Zn 2+ ion displace Cu 2+ ion of solution
Q.3 On the basis of spontaneity of following reactions, due to which colour of solution decolourises.
arrange Mg, Zn, Cu and Ag in decreasing order of their Zn + CuSO 4 ¾® ZnSO 4 + Cu
electrode potentials.* (U.P. 2014) or Zn + Cu 2+ ¾® Zn 2+ + Cu
Cu + 2Ag + ¾® Cu 2+ + 2Ag Q.8 What is electrode potential ? Mention the factors
Mg + Zn 2+ ¾® Mg 2+ + Zn affecting it. (U.P. 2005, 07, 15)
Ans. When a metal (or an electrode) is dipped in a solution of
Zn + Cu 2+ ¾® Zn 2+ + Cu its salts a potential difference is developed at the
Ans. (i) Cu + 2Ag + ¾® Cu 2+ + 2Ag junction of surfaces of metal (electrode) and solution.
° °
ECu > E Ag ( -0. 34 > -080
. ) O. P. This potential difference is called as electrode potential
( E ).
(ii) Mg + Zn 2+ ¾® Mg 2+ + Zn
° °
Factors affecting electrode potential : The electrode
E Mg > E Zn (237
. > 0.76) O. P. potential depends on certain factors.
(iii) Zn + Cu 2+ ¾® Zn 2+ + Cu (i) Nature of the metal.
° ° (ii) Concentration of the ions in solution.
E Zn > ECu (0 .76 > -0. 34)
(iii) Temperature
thus the decreasing order of E ° is Q.9 How potential of half cells is determined? Write its
Mg > Zn > Cu > Ag uses. (U.P. 2020)
( 2. 37) ( 0. 76) ( -0. 34) ( -0. 80)
Ans. Half cell (an electrode) whose potential ( E ° ) is to be
*Comparison has been made on the basis EO° . P. values.
determined is attached to primary reference electrode
Q.4 Explain standard electrode potential with example. (standard hydrogen electrode, SHE) or any secondary
(U.P. 2012, 15) reference electrode. E ° for SHE is 00 . V , so the value
Ans. The potential difference developed between metal obtained gives E ° value of half cell.
electrode and the solution of its ions of unit molarity If secondary electrode is being used then
(1 M ) at 25°C is called standard electrode potential. It o o o
E Cell = E Anode + E Cathode
is denoted by E °.
For example: Zn Zn 2+ (1 mol L-1 ), standard electrode voltmeter Electron flow if metal M
has a negative reduction
potential is -0.76 volt. electrode potential
Q.5 When copper is added to silver nitrate solution, the Electron flow if metal M
colour of the solution turns blue, why? Explain: has a positive reduction
Salt bridge electrode potential
(U.P. 2010, 16) Metal M
Hydrogen
Ans. The position of copper in the electrochemical series is
above silver. Hence copper is more reactive than Ag.
Thus, copper metal can displace silver from the solution Molar M n+
Molar H+
of silver nitrate.
Cu + 2AgNO 3 ¾® Cu ( NO 3 ) 2 + 2Ag Platinum plate
or Cu + 2Ag + ¾® Cu 2+ + 2Ag
Measurement of electrode potential
The solution becomes blue due to Cu 2+ ion.
E ° value of a half cell expression its ability to
Q.6 Among Mg, Zn, Cu, Ag which element reacts with acid
oxidise/reduce.
liberating H 2 ? (U.P. 2015)
30

Long Answer Type Questions


Q.1 Define standard electrode potential. Write relation where C is molar concentration
between electrode potential ( E ) and standard electrode 1000
or = k´
potential ( E ° ). (U.P. 2015, 16) C
Ans. At 25°C when a metal electrode is dipped in an 1000
electrolytic solution of one molar concentration (1M), Lm = k ´
M
the potential developed is referred to as standard Here M = molarity of electrolytic solution.
electrode potential of that half cell. This is represented Effect of change in Concentration :
by E °. 1. Conductivity : Upon dilution (i.e., on lowering the
On the basis of nature of reaction taking place at an concentration) conductivity of electrolytic solution
electrode, electrode potentials are of following two decreases as the number of ions per unit volume
types : decreases.
(i) Oxidation potential : When oxidation occurs at 2. Molar Conductivity : For solutions of weak
electrode, the potential difference developed at the electrolytes, upon dilution (i.e., on lowering
junction of electrode and solution surface is called as concentration) the molar conductivity increases
oxidation potential. This is represented as EO.P. due to increase in the number of ions due to
developed M ¾® M n + + ne - dissociation.
(ii) Reduction potential : When reduction occurs at Q.3 What is electrochemical series ?
electrode, the potential difference developed at the Explain also characteristics of electrochemical series.
junction of electrode and solution surfaces is called as Write its two applications.
reduction potential. This is represented as E R.P. (U.P. 2010, 12, 14, 16, 18)
M n + + ne - ¾® M Or What is electrochemical series? Explain whether
For a chemical change, EO.P. = -E R.P. following reaction will take place. (U.P. 2018)
and, E °O.P. = -E °R.P. (i) Fe is heated with steam.
Electrode potential and standard electrode potential are (ii) Cu is kept in hydrochloric acid.
correlated by Nernst’s equation. Ans. Electrochemical series : When different elements are
2.303RT [O. S. ] arranged in increasing order of standard reduction
EO. P. = EOo. P. - log potential, the series thus obtained is called
nF [ R. S. ]
electrochemical series. It is also called activity series.
o 2.303RT [O. S. ] Metal situated above in this series are good reducing
Also, E R . P. = E R . P. + log
nF [ R. S. ] agent than element placed below it in aqueous solution.
Q.2 Define specific conductance and molar conductivity of Characteristics of electrochemical series :
an electrolytic solution. (U.P. 2014, 17, 18) (1) The metal which has high negative value (or
Or Define conductivity and molar conductivity of an smaller positive value) of standard reduction
electrolyte. What will be the effect of change in potential readily loses the electron. These are
concentration on them? Explain. (U.P. 2019) active metals. The activity of metals decreases
Or Define conductivity and describe the relationship from top to bottom in the series.
between conductivity and resistance. (U.P. 2019) (2) A metal higher in the series will displace the
Ans. Specific conductance : The reciprocal of specific another metal from its solution which is lower in
resistance (r) is called specific conductance (or the series.
conductivity). It is denoted by k (kappa). Thus, For example: Zinc displaces copper from copper
1 sulphate because, zinc occupies its position above
k= …(i) copper in the electrochemical series.
r
RA Zn + CuSO 4 ¾® ZnSO 4 + Cu
But we know r = …(ii) (3) The metals occupying positions above hydrogen
l readily liberate hydrogen from dilute acids. But
From eq. (i) and (ii), we get metals which are below hydrogen in
l electrochemical series like Cu, Hg, Au, Pt etc. do
k=
RA not evolve hydrogen from dilute acids.
Molar conductivity : Molar conductivity is defined as For example:
the conductivity power of all the ions produced by (i) Zn + 2HCl ¾® ZnCl 2 + H 2
dissolving one mole of an electrolyte in V mL solution. (ii) Cu + H 2SO 4 ¾® No reaction
It is denoted by L m . Thus, (4) The element having highest positive value of
1 standard reduction potential is the strongest
Lm = k ´V = Lm = k ´
C oxidising agent, while the element having highest
negative value of standard reduction potential is
31

the strongest reducing agent. Thus, the oxidising anions each multiplied with the number of the ions
nature increases from top to bottom in present in one formula unit of electrolyte.” i. e.,
electrochemical series, while the reducing nature L¥m = v + l ¥+ + v - l ¥- …(i)
decreases from top to bottom in the ¥ ¥
Where l + and l - are ionic conductance of cation and
electrochemical series.
(5) A non-metal higher in series having high value of anion while v + and v i are the number of cations and
reduction potential will displace another non-metal anions in the formula unit of electrolyte.
with lower reduction potential, i. e., occupying Since we know that at infinite dilution, concentration of
position above the series. solution remains zero the Kohlrausch equation can also
For example: be written as:
Cl 2 + 2KI ¾® 2KCl + I 2 L0m = v + l 0+ + v - l 0- …(ii)
(6) Metal oxides which are placed below the Cu in For example:
electrochemical series, decomposes on heating. (i) L0m BaCl 2 = l ¥ 2 + + 2 ´ l ¥ -
Ba Cl
For example:
(i) D (ii) L m Al 2 (SO 4 ) 3 = 2l 3 + + 3 ´ l ¥ 2 -
¥ ¥
2Ag 2O ¾ ¾ ® 4Ag + O 2 Al SO
4
D
(ii) 2HgO ¾ ¾® 2Hg + O 2 Applications of Kohlrausch’s Law :
(7) Oxides of iron and other metals which are placed 1. Calculation of molar conductance at infinite
below the iron, can be reduced by hydrogen and dilution for weak electrolyte : The molar conductivity
liberate metal. of weak electrolytes can not be obtained graphically by
For example: extrapolation method, due to feebly ionized. However,
CuO + H 2 ¾® H 2O + Cu L¥m values for weak electrolyte can be determined by
Application of Electrochemical Series : using the Kohlrausch equation.
(1) Metal extraction : More active metal can replace For example: The molar conductivity of acetic acid can
the less active metals from the solution of their salt. be obtained from the knowledge of molar conductances
This property is used in extraction of metals like at infinite dilution of HCl, CH 3COONa and NaCl which
Ag, Au, etc. are strong electrolyte.
2Na [ Ag (CN ) 2 ] + Zn ¾® Na 2 [ Zn (CN ) 4 ] + 2Ag From Kohlrausch’s law,
(2) In electroplating : Deposition of thin layer of L¥m (CH 3COOH ) = l ¥ ¥
- +l +
gold, silver, nickel, chromium etc. on copper, iron, m (CH 3COO ) H
¥
bronze etc. electrolytically is known as = [l + l¥ + ] + l¥ + +
m (CH 3COO- ) m( Na ) H
electroplating. An active metal is coated on less ¥ ¥
active metal to check corrosion. l ] - [l + l¥ - ]
m(Cl - ) m( Na + ) m(Cl )
(3) Predicting the reactivity of elements : Using or L m (CH 3COOH ) = L m(CH 3COONa ) + L m( HCl ) - [ L¥( NaCl ) ]
¥ ¥ ¥
electrochemical series, a given set of elements can
arranged in order of reactivity. An element with Thus, L¥m(CH 3COOH ) can now be calculated from the
lower value of reduction potential is more reactive knowledge of molar conductivities at infinite dilution
in series. of strong electrolytes like CH 3COONa, HCl and NaCl.
(4) Displacement of hydrogen from an acid by 2. Dissociation constant for weak electrolyte: Let AB
metals : Metals placed above hydrogen in be a weak electrolyte which dissociates as follows:
AB a A + + B -
electrochemical series liberate hydrogen gas on
reaction with acids, while those placed below
hydrogen (Cu, Hg, Pt, Au, Ag etc.) does not Initial concentration C 0 0
At equilibrium C (1 - a ) Ca Ca
liberate hydrogen with acids.
(i) Fe displaces H 2 from steam. According to law of mass action
3Fe + 4H 2O ¾® Fe 3O 4 + 4H 2 ( g ) [ A + ][ B - ] Ca ´ Ca
K= =
(ii) Cu is below H in ECS, so it does not react with HCl. [ AB ] C (1 - a )
Cu + HCl ¾® no reaction Ca 2
Q.4 What is Kohlrausch’s law ? Write its two applications or K= …(i)
for weak electrolyte. (U.P. 2014, 15, 16, 17, 18, 19) 1-a
Or What is Kohlrausch's law? How the degree of For weak electrolytes a is very less and so 1 - a = 1,
dissociation of weak electrolyte is calculated by this Hence
law? (U.P. 2020) K
K = Ca 2 Þ a = …(ii)
Ans. According to Kohlrausch’s law, “the molar C
conductivity of an electrolyte at infinite dilution is the LC
sum of the ionic conductivities of the cations and But a= m
L¥m
32

LCm K 0. 059
Similarly, E R.P. = E °R.P. + log 10
[O. S. ]
L ( vi)
Hence, = …(iii)
Lm¥ C n [ R. S. ]
According to Kohlrausch’s law Q.6 Explain standard hydrogen electrode (SHE) with the
L¥m = l ¥C + l ¥a …(iv) help of proper figure and equation. (U.P. 2005)
Or Describe the standard hydrogen electrode giving
From equation (iii) and (iv), we get diagram and write its one use. (U.P. 2018)
LCm K Ans. Standard hydrogen electrode (SHE) : The absolute
= …(v)
¥ ¥ C standard electrode potential of an electrode can not
lC + l a
From eq. (iv) the value of dissociation constant ( K ) can determined directly by experiments. To solve the
be calculated. problem a reference electrode is needed. The
Q.5 What is Nernst equation? Write the relation between commonly used reference electrode is standard
standard electrode potential and electrode potential. hydrogen electrode (SHE) and its standard
(U.P. 2006, 15, 16, 18) electrode potential is taken as zero.
Ans. According to Nernst equation, Preparation of SHE : Standard hydrogen electrode
RT [O. S. ] consists of a platinum wire coated with divided
EO.P. = E °O.P. - log e K ( i) platinum called platinum black, is sealed in a glass
nF [ R. S. ] tube. It is placed in beaker containing an aqueous
RT [O. S. ] solution of some acid having one molar concentration
and, E R.P. = E °R.P. + log e K ( ii)
nF [ R. S. ] of H + ions (1 MHCl). At one atmospheric pressure H 2
and, EO.P. = -E R.P. and E °O.P. = – E °R.P. K ( iii) gas is passed into solution at 25°C. The hydrogen
In above equation, electrode thus constructed forms a half-cell which on
EO.P. = Oxidation potential coupling with any other half-cell begins to work either
E °O.P. = Standard oxidation potential as anode or as cathode.
E R.P. = Reduction potential Following reactions occurs at this electrode.
E °R.P. = Standard reduction potential.
R = Molar gas constant (8.314 joule mol -1 K -1 )
T = Temperature (in kelvin)
n = The number of electrons taking part in oxidation or
reduction.
F = Faraday's coefficient (96500 coulomb).
[O.S. ] = Concentration of oxidised state in the solution.
[R.S. ] = Concentration of reduced state in the solution.
Since, log e = 2.303 log 10
Equation (i) can also be written as :
2.303RT [O.S. ]
EO.P. = E R.P. - log 10 K ( iv )
nF [R.S. ] (i) If SHE acts as anode, oxidation occurs on it as
H 2 ¾® 2H + + 2e -
2.303RT In such case, it is represented as
Calculated value of , at 25°C (298K) = 0.058
F 1
Pt, H 2 | H + (1 M )
at 30°C (303K) = 0.059 2
at 35°C (308K) = 0.060 (ii) If SHE acts as cathode, reduction occurs on it as
Taking 0.059 as the standard value, 2H + + 2e - ¾® H 2
0.059 [O. S. ]
EO.P. = E °O.P. - log 10 K (v ) In such case, it is represented as
n [ R. S. ] 1
H + (1M )| H 2 , Pt
2

Numerical Problems
Problem 1: Calculate the EMF of following cell.
Cu | Cu 2 + || Ag + | Ag Solution:
(1 M) (1M) Q E° = +0.34 V \E° = -0.34 V
R . P. ( cu 2+ |Cu ) O. P. (Cu|Cu 2+ )
E ° 2+ = 0.34 V, E ° + = + 0.80 V (U.P. 2017)
Cu |Cu Ag | Ag Q E° = +080
. V \E° = -080
. V
R . P.( Ag + | Ag ) O. P.( Ag| Ag + )
33

because E° > E° Solution: Given that,


O. P.(Cu|Cu 2+ ) O. P.( Ag| Ag + )
l ¥HCl = 380.5 S × eq -1 , l ¥NaCl = 109.8 S × eq -1
So Cu will act as anode and Ag as cathode. The cell
reaction is l ¥CH 3COONa = 78.5 S × eq -1
Cu + 2Ag + ¾® Cu 2+ + 2Ag According to Kohlrausch’s law,
°
ECell = E° 2+ + E
°
+
l ¥CH 3COOH = l ¥CH 3COONa + l ¥HCl - l ¥NaCl
O. P.(Cu|Cu ) R . P.( Ag | Ag )
= -0.34 + 080 . = 0.46 V = (78.5 + 380.5) - 109.8 = 349.2 S × eq -1
Problem 2: Calculate the EMF of following cell and also Problem 5: Calculate L¥m( NH 4 OH ) from the following
select the positive and negative poles of it. Also write the half data.
cells and overall cell reaction. (U.P. 2015) L¥m( NH 4 Cl) = 129.0 S cm 2 mol -1
Ni | Ni 2 + || Ag + | Ag
0.1 M 0.1 M L¥m ( NaOH ) = 248.0 S cm 2 mol -1 and
and E ° 2+ = -0.25 V and E ° + = +0.80 V L¥m ( NaCl ) = 126.0 S cm 2 mol -1
Ni / Ni Ag / Ag
Solution: Given that Solution: According to Kohlrausch’s law
E° = -0.25 V and E ° = +0.80 V L¥m ( NH 4OH ) = L¥m(NH 4 Cl) + L¥m ( NaOH ) - L¥m ( NaCl )
Ni 2 + / Ni Ag + / Ag
Because standard reduction potential of Ni 2+ /Ni electrode = 129 + 248 - 126 = 251 S cm 2 mol -1
is more negative (or standard oxidation potential is more Problem 6: Calculate the EMF of the cell :
positive) so Ni/Ni 2+ electrode will act as anode and Ag + /Ag Mg(s)| Mg 2 + (0.1 M)||Cu 2 + (1.0 ´ 10 -3 M)|Cu(s)
electrode will act as cathode. Given E o = +0.34 V and E o = -2.37 V
Cu 2 + Cu Mg 2 + Mg
On the basis of above values, reaction taking place at
Solution: The cell reaction is
anode ( i. e., negative pole)
Mg ( s) + Cu 2( aq
+ 2+
.) ¾® Mg ( aq.) + Cu ( s)
Ni ¾® Ni 2+ + 2e - E° 2 + = +025
. V
O. P. ( Ni / Ni )
According to Nernst equation
Reaction taking place at cathode (positive pole) [ Mg 2( aq
+
o 00591
. .) ]
2Ag + + e - ¾® 2Ag E ° + = 080
. V E cell = E cell - log
R . P. ( Ag / Ag ) n [Cu 2( aq
+
.) ]
Overall reaction Ni + 2Ag ¾® Ni 2+ + 2Ag +
00591
. (010
. )
° = E Cathode - E Anode - log
ECell = E° 2+ + E
°
+ 2 . ´ 10 -3 )
(10
O. P. ( Ni / Ni ) R . P. ( Ag / Ag )
= 025
. + 080. = 1.05 V 00591
.
. - ( -2.37) -
= 034 log 100
° 00591
. [ Ni 2+ ] 2
and ECell = ECell - log 00591
.
n [ Ag + ]2 . + 2.37 -
= 034 ´2
° 2
n = 2 , ECell . V, [ Ni 2+ ] = 01
= 105 . M , [ Ag + ] = 01
. M
= 2.71 - 00591
. = 2.65 volt
00591
. 01
. Problem 7: Conductivity of 0.20M KCl solution at 298K
ECell = 105
. - log
2 . )2
(01 is 0.025 S cm -1 .Calculate its molar conductivity. (U.P. 2018)
= 105
. - 00295
. = 1.0205 V Solution: Given that,
Problem 3: At 25ºC, specific conductance of N/50 KCl k = 0.025 Scm -1 , c = 0.20 M , L m = ?
solution is 0.002765 mho cm -1 . If resistance of cell is 400 ohm, 1000 1000
Lm = k ´ = 0.025 ´ = 125 S cm 2 mol -1
then calculate the cell constant. (U.P. 2017) C 0.2
Solution: For given N/50 solution, Problem 8: Calculate the e. m. f . of the cell.
k = 0.002765 mho cm -1 ; R = 400 ohm Mg ( s)| Mg 2+ (01
. M )|| Cu 2+ (1 ´ 10 -1 M )| Cu( s).
1 l Given E °Cu 2+ | Cu = 034
. V
Q k= ´
R a 2+
E ° Mg | Mg = -237 . V. (U.P. 2019)
1
0.002765 = ´ Cell constant Solution: Given that :
400
Mg / Mg 2+ (0.1M )|| Cu 2+ (1 ´ 10 -1 M )|Cu
\ Cell constant = 1.106 cm -1
Problem 4: At infinite dilution, equivalent conductivities Eº . V, E º
= 034 . V
= -237
R.P.(Cu 2+ / Cu) R.P.(Mg 2+ / Cu)
of HCl, NaCl and CH 3COONa are 380.5, 109.8 and 78.5 As per cell reaction and more negative reduction potential,
S . eq -1 . Calculate the equivalent conductance of CH 3COOH . Mg / Mg 2+ electrode will act as anode and Cu 2+ / Cu as
(U.P. 2017) cathode. So,
34

E ºcell = E º +E º So E ºcell = E º +E º
O.P.(Mg/ Mg 2+ ) R.P.(Cu 2+ / Cu) O.P.(Zn|Zn 2+ ) R.P.(Co 2+|Co)
= [- ( E º ]+ Eº
R.P.(Mg 2+ / Mg) R.P.(Cu 2+ / Cu ) = -(E º ) + Eº
R.P.(Zn 2+|Zn) R.P.(Co 2+|Co)
= - ( -237 . = +2.71V
. ) + 034
= -( -0.76) + 028. =1.04V
Now according to Nernst’s equation
According to Nernst equation
0059
. [Mg 2+ ] 0.059 [O.S.]anode
E cell = E ºcell - log E cell = E ºcell - log
n [Cu 2+ ] n [R.S.]cathode
but [Mg 2+ ] = [Cu 2+ ] = 0.1M 0.059 [Zn 2+ ]
= E ºcell - log
So, E cell = E ºcell = 2.37V n [Co 2+ ]
Problem 9: The standard electrode potential of Daniel [Zn 2+ ]
cell is 1.1V. Calculate the standard Gibbs energy for the because [Zn 2+ ] = [Co 2+ ] = 1,so log =0
following reaction : [Co 2+ ]
Zn( s) + Cu 2+ ( aq ) ¾® Zn 2+ ( aq ) + Cu( s) (U.P. 2019) \ E cell = E ºcell = 1.04V
2+ 2+ Problem 12: Calculate the EMF of following cell :
Solution: Zn( s ) + Cu ( aq.) ¾® Zn ( aq.) + Cu( s )
But DG º = -nFE º Cd | Cd 2+ (1 M ) || Ag + (1 M ) | Ag
Given that n = 2 , F = 96500 , E º =11V. Standard reduction potentials of half cells are given
DG º = -2 ´ 96500 ´ 11 . eV / mol. below:
= -212,300J / mol Cd 2+ | Cd; E ° = -0.40V
Problem 10: Calculate the EMF of the following cell : and Ag + | Ag; E ° = +0.80V (U.P. 2020)
Zn | Zn 2+ (1M )|| Cu 2+ | Cu Solution: Given cell is
When the values of E ° for Zn 2+ | Zn and Cu 2+ | Cu are Cd | Cd 2+ (1M ) || Ag + (| M )| Ag
-0.76 volt and +0. 34 volt respectively. Their concentration is 1
molar (U.P. 2019) Cell reaction is
Solution: E º 2+ = -0.76V; Cd + 2Ag + ¾® Cd 2+ + 2Ag
R.P.(Zn / Zn)
Eº = +0.34V Eo 2+ = -0.40 V E o + = + 080
. V
R.P.(Cu 2+ / Cu) R. P. (Cd | Cd ) R. P. ( Ag | Ag )
E ºcell = E ºO.P. (anode) +E º R.P. (Cathode) and Eo = 0.40 V
O. P. (Cd | Cd 2 + )
2+
Here Zn / Zn electrode will act as anode while Because of higher EOo . P. of Cd, Cd | Cd 2+ will act as anode
2+
Cu / Cu as cathode and Ag + | Ag will act as cathode.
Eº 2+ = 0.76V (Q E ºO.P. = -E ºR.P. ) o
ECell = Eo o
O.P.(Zn / Zn ) 2+ + E
O. P. (Cd |Cd ) + R. P. ( Ag | Ag )
. V
E ºR.P. (Cu 2+ / Cu) = 034
= 0.40 + 080
. = 1.2 V
\ E ºcell = 0.76 + 034. = 110 . V According to Nernst equation
According to Nernst equation o 0.059 [O. S. ]Anode
0.0591 [0.5] anode ECell = ECell - log
E cell = E ºcell - log n [O. S. ]Cathode
n [0.5] cathode 0.059 [Cd 2+ ]
but concentrations of both Zn 2+ and Cu 2+ are 1 molar. So, ECell = 1.2 - log
00591
. 1
2 [ Ag + ]2
E cell = 110
. - log =1.10V = 1.2 V (Q [Cd 2+ ] = [ Ag + ] = 1 M )
n 1
Problem 11: Find out the EMF of Zn | Zn(21+M ) || Co(21+M ) | Co Problem 13: The standard molar conductivity Ù° m for
2+ 2+
cell. For this cell E ° for Zn | Zn and Co | Co values are -0.76 NaCl, HCl and CH 3COONa are 126.4 S cm 2 mol -1 , 425.9 S
volt and +028. volt respectively and their concentrations are 1 cm 2 mol -1 and 91.0 S cm 2 mol -1 respectively.
molar. (U.P. 2019)
Solution: Given that, Calculate Ù° for CH 3COOH. (U.P. 2020)
Eº 2+ = -0.76V , Eº 2+ . V
= +028 Solution: LomNaCl = 126.4 S × cm 2 × mol -1
R.P.(Zn |Zn) R.P.(Co |Co)
Because for Co 2+ | Co , E ºR.P. is higher so it will act as LomHCl = 425.9 S × cm 2 × mol -1
cathode and Zn|Zn 2+ as anode.
35

LomCH 3COONa = 91.0 S × cm 2 × mol -1 Here Zn electrode is acting as anode while Ag as cathode.
Given that E Ro . P. ( Zn 2+ / Zn ) = -0.76 V, E Ro . P. ( Ag + | Ag ) = + 0.80 V
LoCH 3COOH = [ LoCH 3COONa + LoHCl ] - [ LoNaCl ] o o
\ ECell = EO.P. ( Zn | Zn 2+ )
+ E Ro . P. ( Ag + | Ag )
= (91.0 + 425.9) - 126.4 = 390.5S × cm 2 × mol -1
Problem 14: Find out the e.m.f. of the cell in the following = +0.76 + 0.80 = 1.56 V
reaction : Now, according to Nernst equation
Zn ( s) + 2Ag + ( aq ) ¾
¾® Zn 2+ ( aq ) + 2Ag ( s) o 0.059 [O. S. ]Anode
ECell = ECell - log
Given : E °Zn 2+ = -0.76 volt and n [O. S. ]Cathode
Zn
E °Ag + = +0.80 volt (U.P. 2020) 0.059 [ Zn 2+ ]
Ag = 156
. - log
n [ Ag + ]2
Solution: Zn ( s) + 2Ag + ( aq ) ¾
¾® Zn 2+ ( aq ) + 2Ag ( s)

Solution of NCERT Text Book Problems


Q.1 Arrange the following metals in the order in which (i) 2Cr( s)+ 3Cd 2+ ( aq) ¾® 2Cr 3+ ( aq)+ 3Cd
they displace each other from the solution of their (ii) Fe 2+ ( aq)+ Ag+ ( aq) ¾® Fe 3+ ( aq)+ Ag( s)
salts. Al, Cu, Fe, Mg and Zn.
Take E ° values from the table in the text.
Ans. Mg, Al, Zn, Fe, Cu, Ag.
Calculate the D rG° and equilibrium constant of the
Q.2 Given the standard electrode potentials, reactions.
K+ / K = –2.93 V , Ag+ / Ag = 0.80 V , Ans. (i) º
E cell º
= E O.P. º
+ E R.P.
Hg 2+ / Hg = 0.79V , Mg 2+ /Mg = –2.37 V , = 0.74 + (– 0.40) = + 0.34 V
º
Cr 3+ / Cr = – 0.74 V D rG ° = – nF E cell
= –6 ´ 96500 ´ 0.34 CV / mol
Arrange these metals in their increasing order of
= – 196860 J/mol
reducing power.
D rG °= - 2.303 RT log K
Ans. Ag < Hg < Cr < Mg < K. Lower the reduction
-196860 = -2.303 ´ 8.314 ´ 298 log K
potential, greater is the reducing power.
or log K = 34.5014
Q.3 Depict the galvanic cell in which the reaction,
or K = antilog 34.5014 = 3.19 ´ 10 34
Zn( s)+ 2Ag+ ( aq) ¾® Zn 2+ ( aq)+ 2Ag( s)
º º º
(ii) E cell = E O.P. + E R.P
takes place. Further show :
= +0.80+(–0.77 V) = 0.03 V
(i) which of the electrode is negatively charged ?
º
D rG °= – nF E cell
(ii) the carriers of the current in the cell
(iii) individual reaction at each electrode. = –1 ´ 96500 ´ 0.03 CV / mol
2+ + = –2895 J / mol
Ans. Zn( s) / Zn ( aq) || Ag ( aq) / Ag( s)
At anode : Zn ¾® Zn 2+ –
+ 2e (Oxidation) D rG °= - 2.303 RT log K
At cathode : Ag + e ¾® Ag+ –
(Reduction) -2895 = - 2.303 ´ 8.314 ´ 298 log K
(i) anode, i.e., Zn electrode is –vely charged. log K = 0.5074
(ii) the current will flow from silver to zinc in the K = antilog 0.5074 = 3.22
external circuit.
Q.4 Calculate the standard cell potentials of galvanic cell
in which the following reactions take place :
36

Q.5 Write the Nernst equation and the emf of the following cells at 298K :
(i) Mg( s)| Mg 2+ (0.001M )|| Cu 2+ (0.0001 M )|Cu( s)
(ii) Fe( s)| Fe 2+ (0.001 M ) || H+ (1M ) | H 2 ( g ) 1 bar | Pt( s)
(iii) Sn( s) | Sn 2+ (0.050 M ) || H+ (0.020 M ) | H 2 ( g ) (1 bar) | Pt( s)
(iv) Pt( s) | Br2 ( l) | Br – (0.010 M ) || H+ (0.030 M ) | H 2 ( g ) (1 bar) | Pt( s)
Take E ° values from the table in the text.
Ans. (i) Cell reaction is Mg + Cu 2+ ¾® Mg 2+ + Cu ( n = 2)
º º º
E cell = E O.P. + E R.P. = 2.37 + 0.34 = 2.71 V
º 0.0591 [Mg 2+ ] 0.0591 0.001
Nernst equation is : E cell = E cell – log 2+
= 2.71 - log
n [Cu ] 2 0.0001
= 2.71 - 0.03 log (10) = 2.71 - 0.03 = 2.68 V
(ii) Cell reaction is Fe + 2H+ ¾® Fe 2+ + H 2 ( n = 2)
º º º
E cell = E O.P. + E R.P. = 0.44+ 0.0 = 0.44 V
0.0591 [Fe 2+ ] 0.001
Nernst equation is : E cell = 0.44 – log + 2 = 0.44 - 0.03 log = 0.44 + 0.03 ´ 3 = 0.53
2 [H ] 1´ 1
(iii) Cell reaction is Sn + 2H+ ¾® Sn 2+ + H 2 ( n = 2)
º º º
E cell = E O.P. + E R.P. = 0 – (–0.14) = +0.14 V
º 0.0591 [Sn 2+ ] 0.05
Nernst equation is : E cell = E cell – log + 2 = 0.14 – 0.03 log
2 [H ] 0.02 ´ 0.02
= 0.14 – 0.03 log (125) = 0.077
(iv) Cell reaction is : 2Br – + 2H+ ¾® Br2 + H 2 ( n = 2)
º º º
E cell = E O.P. + E R.P. = 0 – 1.08 = –1.08 V
º 0.0591 [Product]
Nernst equation is : E cell = E cell – log
2 [Reactant]
º 1 1
E cell = E cell – 0.03 log – 2 + 2
= –1.08 – 0.03 log
[Br ] [H ] 0.01 ´ 0.01 ´ 0.03 ´ 0.03
7
= -1.08 - 0.03 log (1.111 ´ 10 ) = -1.08 - 0.03 ´ 7.0457 = -1.29 V
E cell cannot be –ve. Therefore, cell reaction will be reverse to that of given, i.e.,
Br2 + H 2 ¾® 2H+ + 2Br – ,
E cell will be + 0.129 V.
Q.6 In the button cell, widely used in watches and other devices, the following reaction takes place :
Zn( s)+ Ag 2O( s)+ H 2O( l) ¾® Zn 2+ ( aq) + 2Ag( s) + 2OH – ( aq)
Determine D rGº and E º for the reaction. Take E º values from the table in the text.
Ans. Ag 2O+ H 2O+ 2e – ¾® 2Ag + 2OH – , º
E º = 0.344 V (given in the question); E Zn 2+
/ Zn
= -0.76
º º º
E cell = E cathode - E anode = 0.344 – (–0.76) = +1.104 V
D rG º = – nF E cell= –2 ´ 96500 ´ 1.104 = –2.13 ´ 10 5 J
º

Q.7 Define conductivity and molar conductivity for a solution of an electrolyte. Discuss their variation with concentration.
Ans. Conductivity : Conductance of unit volume of an electrolytic solution refers to its conductivity (or specific
conductance).
l
Mathematically, k =G´
a
Molar Conductivity: It is equal to the conductance of a solution having 1 g mole of electrolyte dissolved in it.
1000
Lm = k ´
M
On dilution, conductivity of a solution decreases while molar conductivity increases.
Q.8 The conductivity of 0.20 M solution of KCl at 298 K is 0.0248 S cm –1 . Calculate its molar conductivity.
k ´ 1000 0.0248 ´ 1000
Ans. l m = = = 124 S cm 2 / mol
molarity 0.2
37

Q.9 The resistance of a conductivity cell containing 0.001 M KCl solution at 298K is 1500 W. What is the cell constant if
conductivity of 0.001M KCl solution at 298 K is 0.146 ´ 10 –3 S cm –1 ?
Conductivity
Ans. Cell constant = = Conductivity ´ Resistance = 0.146 ´ 10 –3 ´ 1500 = 0.219 cm –1
Conductance
Q.10 The conductivity of sodium chloride at 298 K has been determined at different concentrations and the results are given
below :
Concentration (M) 0.001 0.010 0.020 0.050 0.100
Conductivity (10 2 ´ k / S m –1 ) 1.237 11.85 23.15 55.53 106.74
Calculate Ù m for all concentrations and draw a plot between Ù m and c 1/ 2 . Find the value of Ù ºm .
1000 ´ k
Ans. Conc. (c ) c or M k (Sm –1 ) k (S cm –1 ) lm =
Molarity On extrapolation to zero
–4 concentration.
1000 ´ 1.237 ´ 10
10 -3 0.0316 1.237 ´ 10 –2 1.237 ´ 10 –4 -3
= 123.7 124.0
10
1000 ´ 11.85 ´ 10 –4 120.0
10 -2 0.100 11.85 ´ 10 –2 11.85 ´ 10 –4 = 118.5
10 -2 116.0
-2 –2 –4 1000 ´ 23.15 ´ 10 –4
2 ´ 10 0.141 23.15 ´ 10 23.15 ´ 10 = 115.8 lcm 112.0
2 ´ 10 -2
1000 ´ 55.53 ´ 10 –4 108.0
5 ´ 10 -2 0.224 55.53 ´ 10 –2 55.53 ´ 10 –4 -2
= 111.1
5 ´ 10
–4 0.0
-1 –2 –4 1000 ´ 106.74 ´ 10 0.15 0.20 0.25 0.30 0.40
10 0.316 106.74 ´ 10 106.74 ´ 10 -1
= 106.7 c
10
º 2 –1
l m = Intercept on the Ù m axis = 124.0 S cm mol
Q.11 Conductivity of 0.00241 M acetic acid is 7.896 ´ 10 –5 S cm –1 . Calculate its molar conductivity if Ù ºm for acetic acid is
390.5 S cm 2 mol –1 . What is its dissociation constant ?
k ´ 1000 7 .896 ´ 10 –5 ´ 1000
Ans. lm = = = 32.76
Molarity 0.00241
lc 32.76
a = ºm = = 8.39 ´ 10 –2
l m 390.5
c a 2 0.00241 ´ 8.39 ´ 10 –2 ´ 8.39 ´ 10 –2
Ka = = –2
= 1.69 ´ 10 –5
1-a 1 – 8.39 ´ 10
Q.12. How much charge is required for the following reductions ?
(i) 1 mole of Al 3+ to Al,
(ii) 1 mole of Cu 2+ to Cu,
(iii) 1 mole of MnO -4 to Mn 2+
Ans. Deleted for examination 2021.
Q.13 How much electricity in terms of Faraday is required to produce :
(i) 20.0 g of Ca from molten CaCl 2 ,
(ii) 40.0 g of Al from molten Al 2O 3 ?
Ans. Deleted for examination 2021.
Q.14 How much electricity is required in coulomb for the oxidation of :
(i) 1 mole of H 2O to O 2 ,
(ii) 1 mole of FeO to Fe 2O 3 ?
Ans. Deleted for examination 2021.
Q.15 A solution of Ni(NO 3 ) 2 is electrolysed between platinum electrodes using a current of 5 amperes for 20 minutes. What
mass of Ni is deposited at the cathode ?
Ans. Deleted for examination 2021.
Q.16 Three electrolytic cells A, B, C containing solutions of ZnSO 4 , AgNO 3 and CuSO 4 respectively are connected in series.
A steady current of 1.5 ampere was passed through them until 1.45 g of silver deposited at the cathode of cell B. How
long did the current flow ? What mass of copper and zinc were deposited ?
Ans. Deleted for examination 2021.
38

Q.17 Using the standard electrode potentials given in table (iv) Fe 3+ + Ag ¾® Fe 2+ + Ag+
in the text, predict if the reaction between the º
E cell = E O.P. º
- E R.P. º
= E Fe º
- E Ag
2+
following is feasible : / Fe 3+ +
/ Ag
(i) Fe 3+ (aq) and I - (aq), = 0.77 – 0.80 = –0.03 V
(ii) Ag + (aq) and Cu (s), (Not feasible)
(iii) Fe 3+ (aq) and Br - (aq), (v) Br2 + 2Fe 2+ ¾® 2Br – + 2Fe 3+
(iv) Ag (s) and Fe 3+ (aq), º º º º
(v) Br2 (aq) and Fe 2+ (aq). E cell = E O.P. + E R.P. = E Br – - E Fe 3+
/ Fe 2+
2 / Br
Ans. A reaction is feasible if EMF of the cell is +ve. = 1.09 – 0.77 = 0.32 V (Feasible)
Cathode : At which reduction occurs, Q.18 Predict the products of electrolysis in each of the
Anode : At which oxidation occurs. following :
(i) 2Fe 3+ + 2I – ¾® 2Fe 2+ + I 2 (i) An aqueous solution of AgNO 3 with silver
º º º
E cell = E O.P. + E R.P. = E Fe 2+
/ Fe 3+
- E Iº –
electrodes.
2 /I
(ii) An aqueous solution of AgNO 3 with platinum
= 0.77 + (–0.54) = + 0.23 V (Feasible)
+ electrodes.
(ii) 2Ag + Cu ¾® 2Ag + Cu 2+
º º º º
(iii) A dilute solution of H 2SO 4 with platinum
E cell = E O.P. + E R.P. = E Ag / Ag+
– E Cu 2+
/ Cu electrodes.
= 0.80 + (– 0.34) = 0.46 V (Feasible) (iv) An aqueous solution of CuCl 2 with platinum
3+
(iii) 2Fe + 2Br – ¾® 2Fe 2+ + Br2 electrodes.
º
E cell = E O.P. º
+ E R.P. º
= E Fe º
- E Br Ans. Deleted for examination 2021.
2+
/ Fe 3+ 2 / Br

vvv
= 0.77 – 1.09 = –0.32 V
(Not feasible)
39

Chapter

CHEMICAL KINETICS
Syllabus: Rate of a reaction (Average and instantaneous), Factors affecting rate of reaction: concentration, temperature,
catalyst; order and molecularity of a reaction, rate law and specific rate constant, integrated rate equations and half life (only for
zero and first order reactions), *Collision theory of reaction rates (elementary idea, no mathematical treatment),
Activation energy, Arrhenius equation.

* Topics Deleted for Examination 2021

Objective Questions
1. Unit of specific rate constant for first order reaction is: 9. The rate of zero order reaction, A + B ¾® C is:
(U.P. 2017, 20) (U.P. 2018)
(a) mol L-1 sec -1 (b) mol L-1 (a) Rate = K [ A]0 [ B]0 (b) Rate = K [ A]1[ B]0
(c) mol sec -1 (d) sec -1 (c) Rate = K [ A]0 [ B]1 (d) None of these
2. Unit of rate constant for zero order reaction is: 10. Which of the following reaction is pseudo-
(U.P. 2012, 15, 19, 20) unimolecular ? (U.P. 2016)
(a) mol L time -1 (b) L mol -1 time (a) H 2 + Cl 2 ¾® 2HCl
(c) mol L-1 time -1 (d) mol sec -1 (b) CH 3COOC 2 H 5 + NaOH ¾®
3. For the reaction, 2H 2O 2 ¾® 2H 2O + O 2 CH 3COONa + C 2 H 5OH
R = K[ H 2O 2 ]. This reaction is : (U.P. 2018) (c) 2FeCl 3 + SnCl 2 ¾® 2FeCl 2 + SnCl 4
(a) zero order (b) first order (d) CH 3COOCH 3 + H 2O ¾® CH 3COOH + CH 3OH
(c) second order (d) third order 11. In reaction A ® B, the rate of reaction is doubled, on
4. Relation between rate constant and half life for first increasing the concentration of the reactants four
order reaction is: (U.P. 2017) times. The order of the reaction is : (U.P. 2019)
0.693 t1 2 1
(a) t 1 2 = (b) K = (a) zero (b) (c) 2 (d) 4
K 0.693 2
K 12. The approximate time required for the completion of
(c) t 1 2 = 0.693 + K (d) t 1 2 =
0.693 50% of a first order reaction is : (U.P. 2019)
5. Which of the following is correct for first order (a) 1.1 times of t 1 2 (b) 2.2 times of t 1 2
reaction ? (U.P. 2007, 17) (c) 3.3 times of t 1 2 (d) 4.4 times of t 1 2
(a) t 1 2 µ 2 a (b) t 1 2 µ 1 a 13. The half-life period of a first order reaction is 400
(c) t 1 2 µ a 0 (d) t 1 2 µ a 2 seconds. Its velocity constant will be : (U.P. 2019)
6. For a first order reaction, half life period: (a) 1.73 ´ 10 -3 sec -1 (b) 1.44 ´ 10 -3 sec -1
(U.P. 2006, 13, 14, 15, 18) (c) 2.72 ´ 10 -3 sec -1 (d) 2.88 ´ 10 -3 sec -1
(a) depends upon initial concentration 14. The rate constant for the reaction A + 2B ¾® Product
(b) is inversely proportional to initial concentration is expressed by following equation : R = [ A][ B]2 . The
(c) does not depend upon initial concentration order of reaction will be : (U.P. 2019)
(d) depends upon square root of initial concentration (a) 2 (b) 3 (c) 5 (d) 6
dx 15. The half life for a first order reaction is 4 min., the
7. Order of reaction, having rate law µ[ a]0 is : time after which 99.9% reaction gets completed is:
dt
(U.P. 2017) (U.P. 2019)
(a) zero (b) first (c) second (d) none (a) 16 minutes (b) 8 minutes
8. Time required to complete 90% of first order reaction (c) 32 minutes (d) 40 minutes
is approximately : (U.P. 2015, 18) 16. If a reaction obeys the following equation :
2.303 a
(a) 2.2 times of half-life (b) 3.3 times of half-life K = log 10
(c) 1.1 times of half-life (d) 4.4 times of half-life t (a - x )
40

the order of reaction will be : (U.P. 2019) (a) CH 3COOC 2 H 5 + NaOH ¾® CH 3COONa
(a) zero (b) first (c) second (d) third + C 2 H 5OH
17. The unit of velocity constant for first order reaction is: (b) CH 3COOCH 3 + H 2O ¾® CH 3COOH + CH 3OH
(U.P. 2020) (c) 2FeCl 3 + SnCl 2 ¾® 2FeCl 2 + SnCl 4
(a) mol L-1 S -1 (b) S -1 (d) H 2 + Cl 2 ¾® 2HCl
(c) mol -1 LS -1 (d) mol -1 L-1 S 19. The unit of rate and rate constant is same for which
18. Which of the following is an example of zero order order of reaction? (U.P. 2020)
reaction ? (U.P. 2020) (a) Zero (b) First (c) Second (d) Third.

Answers
1. (d) 2. (c) 3. (b) 4. (a) 5. (c) 6. (c) 7. (a) 8. (a) 9. (a) 10. (b)
11. (b) 12. (a) 13. (a) 14. (b) 15. (d) 16. (b) 17. (a) 18. (d) 19. (a)

Very Short Answer Type Questions


Q.1 Write the formula of rate constant for the first order 1 D[ A] 1 D[ B]
Ans. Rate = - × =- ×
reaction. (U.P. 2015) n1 Dt n 2 Dt
Ans. The formula for rate constant of first order reaction is 1 D[C ] 1 D[ D]
2.303 a = × = ×
K = log 10 m1 Dt m 2 Dt
t a-x
where a = initial concentration of reactant Q.4 Explain the following terms: (U.P. 2017, 18)
a - x = concentration of the reactant after time t (i) Temperature coefficient (ii) First order reaction
K = rate constant Ans. (i) Temperature coefficient: The temperature
Q.2 Mention the order of following reactions with reason: coefficient of rate constant is defined as, “the ratio of
(a) H 2 + Cl 2 ¾ light
¾¾® 2HCl; (U.P. 2013, 16, 18) rate constants at two temperatures separated by 10°C,
(b) C 12 H 22O11 + H 2O ¾® C 6 H12O 6 + C 6 H12O 6 usually 25°C and 35°C.”
(c) CH 3COOC 2 H 5 + NaOH ¾® CH 3COONa That is,
+ C 2 H 5OH Temperature coefficient of rate constant
(d) CH 3COOC 2 H 5 + H 2O ¾® CH 3COOH + C 2 H 5OH Value of rate constant at (t +10)° C
Ans. First, it should be clear that order of reaction is an =
experimental value, not a theoretical value. It can not Value of rate constant at t ° C
be determined by the balanced chemical equation. K t + 10 K 35
= = Î ( 2,3)
Thus, the question asked is incorrect. However, above Kt K 25
examples can be considered as standard for the Usually, value of temperature coefficient lies between
corresponding orders. 2-3 [Î ( 2,3)], that is, for every 10°C rise in
hv
(a) H 2 + Cl 2 ¾ ¾ ® 2HCl temperature, the rate of reaction nearly doubles or
Rate = K[H 2 ] [Cl 2 ]1 ; order = 1 + 1 = 2
1
triples up.
But if reaction takes place at very high speed, it will (ii) First order reaction : A first order reaction is
follow zero order kinetics. that in which rate of reaction is directly proportional
(b) C 12 H 22O11 + H 2O ¾® C 6 H12O 6 + C 6 H12O 6 to the first power raised to the concentration term of
Rate = K[C 12O 22O11 ]1[H 2O]0 ; order = 1 + 0 = 1 the reactant.
(c) CH 3COOC 2 H 5 + NaOH ¾® CH 3COONa The rate law of a first order reaction is expressed by
+ C 2 H 5OH either equation (i) or equation (a).
Rate = K[CH 3COOC 2 H 5 ]1[NaOH]1 ; Consider the following of first order reaction:
order = 1 + 1 = 2 A ¾¾® Product
(d) CH 3COOC 2 H 5 + H 2O ¾® CH 3COOH + C 2 H 5OH
Rate = K[CH 3COOC 2 H 5 ]1[H 2O]0 ; At t = 0 : a
order = 1 + 0 = 1 At t = t : a-x
Q.3 Express the average rate of following reaction: Where, a = Initial concentration of A
n1 A + n 2 B ¾® m1C + m 2 D (U.P. 2017) dx
\ Rate = = K (a - x )
dt
41

Short Answer Type Questions


Q.1 What is the rate constant of a reaction ? (U.P. 2013) Experiments prove that rate of the reaction is directly
or What is the rate of a reaction? How does the proportional to first power raised to the
temperature affect the rate of reaction? (U.P. 2018) concentration term of ethyl acetate. Thus,
Ans. Rate constant: Suppose in the following reaction dx
Rate = = K[CH 3COOC 2 H 5 ]
X ¾® Y + Z , the initial concentration of reactant X is dt
C mol/litre at any moment. Thus, the reaction is of first order but its molecularity
Now according to law of mass action, is two for the following facts:
dx dx (a) Since concentration of water does not affect the
µ C or = KC …(i)
dt dt rate of reaction and its concentrations does not
where K is constant and known as rate constant. show any appreciable change, hence,
If C = 1 mol/litre, then, concentration of water is taken as constant. The
dx reaction, therefore, is of first order.
=K …(ii)
dt (b) Since, water takes part in the reaction, thus,
Hence, rate constant is the rate of reaction when the molecularity of the reaction is two.
concentration of reactant is unity at a given In this way, above reaction is of first order and
temperature. bimolecular. Thus, it is a pseudo unimolecular
Rate of reaction increases as the temperature is reaction.
increased. Other examples:
Q.2 Explain with reasons, what will be the order of H SO
2
(i) C 12 H 22O11 + H 2O ¾¾®4
C 6 H12O 6 + C 6 H12O 6
reaction for the following reaction :
Sucrose Glucose Fructose
2FeCl 3 + SnCl 2 ¾® 2FeCl 2 + SnCl 4 (U.P. 2016) dx
Ans. It has been experimentally observed that rate of Rate = = K[C 12 H 22O11 ];
reaction depends upon the concentration of both the dt
reactant. That is \ order = 1, molecularity = 2
Rate = K [ FeCl 3 ]2[SnCl 2 ]1 (ii) (CH 3CO) 2 O + 2NaOH ¾¾® 2CH 3COONa + H 2O
dx
Thus, the order of reaction = 2 + 1 = 3 Rate = = K[[(CH 3CO) 2 O][NaOH]0 ]
Q.3 Explain the first order reaction with an example. dt
(U.P. 2016) \ Order = 1, molecularity = 3
Ans. A reaction in which rate of reaction depends upon the Q.6 What do you understand by average rate and
instantaneous rate ? Explain the effect of temperature
concentration of only one reactant is called first order
on rate of reaction briefly. (U.P. 2017)
of reaction.
Ans. Let a reaction is taking place as follows.
For example: 2N 2O 5 ¾® 4NO 2 + O 2 A ¾® B
In this reaction, concentration of only one molecules at t 1 C1
determines the velocity reaction, hence it is a first at t 2 C2
order reaction. C - C 1 DC
Q.4 Explain zero order reaction with an example. then average rate = 2 =
t 2 - t1 Dt
(U.P. 2016)
i. e., it expresses rate of reaction for a significantly
Ans. A reaction in which rate of reaction is independent of
large time interval.
the concentration of reactant is called zero order If Dt is infinitely small i. e., Dt ® dt
reaction. then DC = dC
hn
For example : H 2 + Cl 2 ¾ ¾ ® 2HCl dC
then instantaneous rate =
In the above reaction, the rate of reaction does not dt
depend on the concentration of reactant. Hence the It expresses the rate of reaction at particular moment.
order of reaction is zero. Q.7 Prove that the half-life of any first order of reaction
does not depend on the initial concentration of the
Q.5 Explain pseudo-unimolecular reaction with example. reactant. (U.P. 2014)
(U.P. 2009, 15) or
Ans. There are several reactions which are of first order Prove that for a first order reaction, time required to
although in reality they are bi- or tri- molecular. complete by 50% does not depend upon initial
These reactions are called pseudo unimolecular concentration. (U.P. 2017)
reactions. Ans. For a first order reaction,
For example: Hydrolysis of ethyl acetate 2.303 a
+ t = log 10 …(i)
CH 3COOC 2 H 5 + H 2O ¾ H¾
¾® CH 3COOH + C 2 H 5OH k a-x
42

Where a = initial concentration of reactant 4. Molec ularity of the while order of reaction gives the
a - x = concentration of reactant after time t reaction gives the number relation between rate and
According to the definition of half-life period, molecules taking part in c o n c e n t r a ti o n t e r m s o f
a an elementary step. reactants.
t = t 1 2 and a - x =
2
a For hydrolysis of ethyl acetate
Substituting t = t 1 2 and a - x = in eqn. (i), we get CH 3COOC 2 H 5 + H 2O ¾® CH 3COOH + C 2 H 5OH
2 ( ex.)
2.303 a Rate = K [CH 3COOC 2 H 5 ]
t1 2 = log 10
k a2 Its molecularity is 2 which order is 1.
2.303 Q.9 Write order of reaction for the following reactions
t1 2 = log 10 2 and explain why?
k
2.303 ´ 0.3010 (i) 2N 2O 5 ¾® 4NO 2 + O 2 (U.P. 2013)
= log 10 2 = 0.3010 (ii) NH 4 CNO ¾® NH 2CONH 2
k
Urea
0.693
t1 2 = …(ii) Ans. (i) 2N 2O 5 ¾® 4NO 2 + O 2
k
The above reaction appears to be of second order
From equation (ii) it is clear that half-life period of a
reaction. But the experimental data show the
first order reaction does not depend on the initial
reaction to be of the first order. The following
concentration of reactant.
mechanism has been suggested for the reaction.
Q.8 What is the difference between order of reaction and
molecularity? Clarify with an example. (U.P. 2016) (i) N 2O 5 ¾ Slow
¾¾® NO 2 + NO 3
Or Define molecularity and order of the reaction and the (ii) N 2O 5 + NO 3 ¾ Fast
¾¾® 3NO 2 + O 2
difference between them (U.P. 2020) 2N 2O 5 ¾¾® 4NO 2 + O 2
Ans. We know that the slow step is rate determining step,
S.No. Molecularity Order therefore, this reaction is first order.
(ii) NH 4 CNO ¾® NH 2CONH 2
1. The molecularity of the while the order of reaction is Urea
reaction is given by the g i v e n b y t h e nu m be r o f The conversion of ammonium cyanate into urea
number of molecules of concentration terms of reactant appears to be of the first order. But the experimental
reactant or reactants or reactants on which rate of data show the reaction to be of second order. The
appearing in the balanced reaction depends. following mechanism has been suggested for the
equation for the reaction.
reaction.
Fast
2. Molecularity of a reaction while order of reaction is NH 4 CNO H — N == C == O + NH 3
can be determined from determined experimentally.
H — N == C == O + NH 3 ¾ Slow¾¾® NH 2CONH 2
the mechanism of the
reaction. The second step, which is slow, determine the rate of
the reaction. This is called the rate determining step.
3. Molecularity can not be while order of the reaction can Since involves two molecules, the reaction as a whole
z e r o , f r a c ti o n a l o r a be zero, whole number or a behaves as a reaction of second order.
negative value. fractional value.

Long Answer Type Questions


Q.1 What is rate of reaction ? (U.P. 2012, 13, 18, 20) Ans. Rate of reaction is defined as “the change in
or concentration of reactants or products with
What do you understand by rate of a reaction? time”.
(U.P. 2013) Decrease in concentration of
or reactant after the reaction
What is meant by rate of reaction. For the reaction Rate of reaction =
Time taken in the reaction
NO 2( g ) + CO( g ) ¾® CO 2 + NO
Increase in concentration of
The mechanism is as given below :
product after the reaction
(i) NO 2 + NO 2 ¾® NO + NO 3 (slow) =
(ii) CO + NO 3 ¾® CO 2 + NO 2 (fast) Time taken in the reaction
What is order of reaction? Consider the reaction,
or A ¾¾® B
What is instantaneous velocity? How will you Concentration at t = 0: a 0
determine it? (U.P. 2018) Concentration at t = t: (a - x ) x
\ Average rate of reaction
43
+
Decrease in concentration of (i) CH 3COOC 2 H 5 + H 2O ¾ H¾ ¾® CH 3COOH
reactant after the reaction x + C 2 H 5OH
= = ...(i)
Time taken in the reaction t The above reaction is the first order of reaction,
The rate given by the above expression (i) can be the because the rate of reaction depends only on the
average rate of reaction, but, it is not the concentration of ethyl acetate and not on that of
instantaneous rate of reaction, because concentration water.
+
of reactant goes on decreasing with time during the (ii) C 12 H 22O11 + H 2O ¾ H¾
¾® C 6 H12O 6 + C 6 H12O 6
reaction as in figure (a). Hence, according to law of Cane sugar Glucose Fructose
mass action, rate of reaction will also regularly The above reaction is also a first order of reaction, as
decrease with time as in figure (b) and it will not be the rate of reaction depends only on the
proper to determine average rate of reaction. Hence, concentration of cane sugar, while the concentration
the instantaneous rate of reaction is expressed as of water remains constant.
follows: Characteristics of First Order Reactions :
Suppose –dC A represents a small decrease in 1. First order of reaction obey the following
concentration of reactant, whereas dC B represents a equations.
small increase in concentration of product in a very 2.303 a
t = log 10
short time interval dt after time t, then, k a-x
dC A dC B 2. The half-life period of a first order reaction is
Instantaneous rate of reaction = - = ...(ii)
dt dt independent of initial concentration of reactants,
dC A that is
where,- = Rate of decomposition of reactant A
dt t 1 2 µ [ a]0
dC B
and, = Rate of formation of product B. 3. The units of rate of reaction, for the reactions
dt of first order rate constant are mol litre -1 sec -1
The negative sign in equation (ii) implies that the and sec -1 respectively. (by mol1- n Ln-1 sec -1 )
concentration of reactant A decrease with time rate
of reaction, in general, is expressed in mol 4. First order reactions take infinite time for
litre -1 sec -1 . completion.
Consider the reaction, 5. In the reactions of first order, if the concentration
of reactant is increased by n times, the rate of
m1 A + m 2 B + ¼¾¾® n1C + n 2 D + ¼ being proceed.
reaction is also increased by n times.
Instantaneous rate of reaction For I order reaction:
1 dC A 1 dC B 0.693 0.693
=- . =- . K = = (Q t 1/ 2 = 60 min)
m1 dt m 2 dt t 1/ 2 60
1 dC C 1 dC D
= . = . For 90% completion
n1 dt n 2 dt 2.303 a
K = log
1 DC A 1 DC B t 90% (a - x )
Average rate of reaction = - =- .
m1 Dt m 2 Dt 0.693
a = 100 ( a - x ) = 10, K =
1 DC C 1 DC D 60
= . =
n1 Dt n 2 Dt 0.693 2.303 100
\ = log
If Dt ® 0, then, instantaneous rate = average rate. 60 t 90% 10
For the reaction, 2.303 ´ 60
t 90% = = 199.39 min
NO 2 + NO 2 ¾® NO + NO 3 (slow) 0.693
CO + NO 3 ¾® CO 2 + NO 2 (fast) Q.3 What is the rate constant of a reaction ?
Q Slow step is rate determining. (U.P. 2017)
\ Rate = K[ NO 2 ]2 Ans. Consider the reaction, A ¾¾® B
Q.2 Write the unit of rate constant of first order reaction Molar concentration at t = 0 : a
and write two characteristics of first order reactions. Molar concentration at t = t : ( a - x )
(U.P. 2018) Molar concentration at t = (t + dt ) : ( a - x - dx )
Or Write four important characterstics of first order dC A dx
And, Rate of reaction =- =
reactions. Half-life of a first order reaction is 60 dt dt
minutes. How much time will it take for 90% According to law of mass action, rate of reaction is
completion? (U.P. 2020) directly proportional to the concentration of reactant
Ans. First order of reaction : A reaction in which rate A; thus
of reaction is directly proportional to concentration of dC dx
- A µ C A or µ (a - x ) ...(i)
one reactant only, is called first order of reaction. dt dt
For example:
44
dC A dx molecularity three or more than three are, in
or, - = K . C A or = K (a - x ) ...(ii)
dt dt general, very rare.
Eqn. (ii) represents the rate law of reaction, where K Order of reaction : Order of a reaction may be
is known as rate constant or velocity constant. defined as; “the number of molecules of
Thus, the rate constant may be defined as, "the rate reactants which determines the rate
of reaction when the molar concentration of expression.”
each reactant is unity." It is also known as For example: Consider the reaction,
specific rate of reaction. aA + bB ¾¾® cC + dD
dC If rate of the reaction is dependent on powers m1 and
if, CA = 1 then - A =K
dt m 2 raised on concentration terms of reactants A and
Unit of Rate Constant : Units of rate constant B respectively, then,
depend upon the order of reaction. Rate = K[ A] m1 [ B] m 2
Rate = K[ A]n , where n = order of reaction. and, order of reaction = m1 + m2
Rate mol L-1 s -1 Thus, order of reaction is also defined as; “the sum
Thus, K = = = mol1- n Ln -1 s -1
[ A] n -1 n
[mol L ] of powers raised on concentration terms in
order to write rate expression.”
Q.4 What do you understand by molecularity and order of
Following facts are important regarding the order of
reaction ? Explain giving example.
reaction:
(U.P. 2004, 13, 14)
1. In the reactions completing in more than one
or
step, the slowest step is the rate determining step.
Differentiate between order of a chemical reaction
e.g., if in a car manufacturing unit, 1 engine, 10
and molecularity by giving an example. (U.P. 2018)
chesis, 20 steerings, 40 wheels etc. are
Ans. Molecularity of reaction : In chemical kinetics,
manufactured, but just one car will be produced
the reactions are studied in terms of either
daily as each car requires one engine which
molecularity or order of reaction.
manufactured daily.
Molecularity of a reaction is defined as, “the
2. Rate of reaction can be determined by
number of reactant molecules taking part in
experiments, thus order of reaction is an
the elementary step of a balanced chemical
experimental value and can not be determined
equation.”
from the balanced equation of the reaction.
For example: Following reactions are simple reactions
3. Order of reaction may be the whole number, zero
going to completion in elementary step itself.
or fractional value. Reactions having order of
Thus,
reaction zero, one, two ... etc. are called zero
NH 4 NO 2 ¾¾® N 2 + 2H 2O; Unimolecular reaction
order, first order, second order reactions etc.
H 2 ( g ) + I 2 ( g ) ¾¾® 2HI( g ); Bimolecular reaction respectively.
2NO( g ) + O 2 ( g ) ¾¾® 2NO 2 ; Trimolecular reaction 4. Order of reaction also depends on experimental
Following facts are important regarding the conditions. e.g., reaction, H 2 + Br2 ¾¾® 2HBr
molecularity of reaction: follows second order rate equation, that is, rate
1. The molecularity of a simple reaction can be = K[ H 2 ][Br2 ]. However, when this reaction is
determined from the stoichiometric equation for carried out in a tube filled with water, it follows
the reaction, however, for a complex reaction, its rate equation of zero order (rate =
molecularity can not be determined from its K[H 2 ]°[Br2 ]°).
balanced equation. In complex reactions, 5. The reaction having order of reaction three, or
molecularity of each step is expressed separately higher than three are very rare.
and total molecularity of complex reaction has no 6. Examples of various order of reactions are:
meaning. (a) NH 4 NO 2 ¾¾® N 2 + 2H 2O;
2. In a complex reaction, molecularity of each Q Rate = K[NH 4 NO 2 ]
elementary step may be different.
\ order = 1
3. Molecularity of a reaction is the theoretical value
(b) CH 3COOH + NaOH ¾¾® CH 3COONa + H 2O;
that is determined from each elementary step
involved in the mechanism of reaction.
4. Molecularity of a reaction is always a positive Q Rate = K[CH 3COOH][NaOH]
integer except zero, that is, its value can not be \ order = 2
zero, negative or fractional. (c) 2NO + O 2 ¾¾® 2NO 2 ; Q Rate = K[NO]2[O 2 ]
5. Molecularity of a reaction means that the number
\ order = 3
of molecules colliding effectively. Since, the
collision between more number of molecules at a (d) CH 3COOR + H 2O ¾¾® CH 3COOH + ROH;
time is not possible, therefore, reactions having
45

Q Rate = K[CH 3COOR][H 2O]0 For example:


+
\ order = 1 (i) CH 3COOC 2 H 5 + H 2O ¾ H¾
¾® CH 3COOH
Q.5 Explain the effect of temperature and pressure on the + C 2 H 5OH
rate of reaction. (U.P. 2018) D
(ii) NH 4 NO 3 ¾¾ ® N 2 + 2H 2O
or How is the rate of reaction affected by : Derivation of rate constant of first order
(i) change in concentration of reactants? reaction:
(ii) change in temperature? Let us consider the following of first order reaction:
(iii) presence of catalyst? (U.P. 2018) A ¾® Product
Ans. Factors affecting rate of a reactions : The rate
a 0 (at t = 0)
of a chemical reaction depends on the following
a- x x (at time = t)
factors.
According to law of mass action
(1) Concentration of reactants : The rate of a dx
chemical reaction is directly proportional to the Rate = µ (a - x )
concentration of the reactants means rate of dt
dx
reaction increases on increasing the or = K (a - x )
concentration of reactants. dt
dx
For A reaction: or = K × dt …(i)
A ¾® Product a-x
Rate of reaction gets doubled if concentration of where K is the rate constant.
reactants is doubled. On integrating equation (ii),
(2) Effect of temperature : The rate of chemical - log e ( a - x ) = Kt + C …(ii)
When t = 0, x = 0;
reaction generally increases on increasing the
C = - log e a …(iii)
temperature. The rate of a reaction becomes On substituting equation (ii) in equation (iii),
almost double or tripled for every 10°C rise in - log e ( a - x ) = Kt - log e a
temperature, but the rate of exothermic reaction a
or, Kt = log e
decrease with rise in temperature. (a - x )
(3) Presence of a catalyst : Catalyst can increase 1 a
or decrease rate of reaction but itself remains or, K = log e
t (a - x )
unchanged. Positive catalyst decreases value of 2.303 a
activation energy for the reaction while negative or, K = log 10 …(iv)
t (a - x )
catalyst increases value of activation energy. The
greater the decrease in activation energy caused Equation (iv) is integrated equation for reactions of
by positive catalyst higher will be the reaction first order.
rate. Half-life period : Half-life period (t 1 2 ) of a first
order reaction or the time taken to complete a certain
(4) Effect of radiations : Some reactions occurs
fraction of the reaction (t 1 n ) is independent of the
rapidly in the presence of light due to the
initial concentration of the reactant, that is, a definite
absorption of photons of different energies. time is taken to complete a definite fraction of the
For example : Formation of HCl by combination of H 2 first order reaction.
and Cl 2 t 1 2 µ [ a]0 or t 1 n µ [ a]0
H 2 + Cl 2 ¾ in
¾dark
¾® 2HCl
¾ (Very slow) or, t 1 2 = constant
hn a
H 2 + Cl 2 ¾ ¾® 2HCl (Very fast) When, t = t1 2 , a - x =
2
Q.6 Derive expression for rate constant and half life time 2.303 a
of a first order reaction. (U.P. 2010, 12, 14) From the equation, t = log 10
K (a - x )
or 2.303 a
Derive the expression of rate constant for first order t1 2 = log 10
K a2
reactions. Write usual terms, explain the first order
reaction with example. Also, prove that half-life 2.303
or, t1 2 = log 10 2
period of a first order reaction is independent of K
initial concentration of reactants. (U.P. 2010, 12) 2.303 ´ 0.3010
or, t1 2 = (Q log 10 2 = 0.3010)
Ans. First order reaction : Those reactions whose rate K
is determined by the change of only one 0.693
or, t1 2 =
concentration of reactant are known as first order K
reaction.
46

Q.7 What is zero order reaction? Drive integrated This is integrated rate law of for a zero order
equation for zero order of reaction and also write its reaction.
main characteristics. (U.P. 2016) Main characteristics of zero order reactions:
Or Explain the zero order reaction with proper example. 1. Zero order reactions obey following equations :
Give its mathematical derivation. (U.P. 2019) x = Kt or [ A] = -Kt + [ A]0
Ans. Zero order reaction : When the rate of a reaction 2. On plotting a graph between [ A] and time for a
does not depend on the concentration of reactants, zero order reaction, a straight line is obtained in
are called zero order reactions. accordance with equation (given in point 1).
For example: Slope (tan q) of the straight line gives the negative
hn
H 2 + Cl 2 ¾ ¾ ® 2HCl value of rate constant ( -K ) and intercept gives the
Expression of rate constant for zero order value of [ A]0 .
reaction: Let us consider the reaction 3. For zero order reactions, rate of reaction remains
A ¾® Product always constant through out the reaction and is
Since, the rate of reaction of zero order reaction does equal to its rate constant, that is, rate of reaction
not depend upon the concentration of reactants, thus, is independent of concentrations of reactants.
dx 4. For zero order reactions, units of rate of reaction
= constant ( K ) and rate constant are always similar, that is, mol
dt
(where K is rate constant) litre -1 sec -1 .
or dx = Kdt …(i) 5. Zero order reactions always go to completion and
a
Integrating above equation, time is taken in their completion, where ‘ a’ is
dx = K dt K
ò ò initial concentration of reactant.
or x = Kt + C …(ii) Q x = Kt
where C is integration constant when t = 0, then If whole of reactant (initial concentration = a mol
x = 0, thus, L-1 ) is converted into product in total time t, then
0 = K ´ 0+C Þ C = 0 x=a
Hence from eq. (ii) we have \ a = K ×t
x = Kt a
x or t =
or K = …(iii) K
t

Numerical Problems
Problem 1: For a reaction R ¾® P, concentration of 2.303 a
\ t = log 10
reactant changes from 0.06 M to 0.02 M in 20 minutes. 1.155 ´ 10 -2 0.1a
Calculate the average rate. (U.P. 2017) 2.303
Change in conc. of reactant = log 10 10
Solution: Average rate = 1.155 ´ 10 -1
Time interval 2.303
0.06 - 0.02 t = = 199 minutes
= = 20 ´ 10 -3 mol. L-1 min -1 1.155 ´ 10 -2
20
Thus, in 199 minutes, the reaction will be 90%
Problem 2: Half-life time of a first order reaction is 60 complete.
minutes. In how much time will it go for 90% completion ?
Problem 3: For a reaction of first order, prove that :
(U.P. 2014, 15)
0.693 (a) The time taken to complete 99.9% reaction is ten times of
Solution: For a first order reaction, K = its half-life time. (U.P. 2015, 16)
t1 2
(b) The time taken to complete 99.9% reaction is three times of
0.693
or, K = = 1.155 ´ 10 -2 sec -1 the time taken to complete 90% of the reaction.
60 (U.P. 2016)
2.303 a
and, K = log 10 (c) Derive an expression for the time required for completion of
t (a - x ) 1/3 of a first order reaction. (U.P. 2018)
Initial concentration (at t = 0) = a (d) Prove that the time required for 99% completion of a first
After time, t; concentration of reactant consumed order reaction is twice the time required for the completion of
= x = 0.9a 90%. (U.P. 2015)
After time, t; concentration of reactant consumed (e) Prove that time required for the completion of three-fourth
= a - x = a - 0.9a = 0.1a (3/4) of a first order reaction is twice the time required for the
completion of half reaction. (U.P. 2015, 20)
47

Solution: (a) For a reaction of first order, 0.693 0.693


K = = = 1.7 ´ 10 -2 sec -1
2.303 a t 1/ 2 40
t = log 10
K (a - x ) Problem 7: A first order reaction is 75% complete in 60
If t = t 99. 9% , ( a - x ) = 100 - 99.9 = 0.1, a = 100 minutes. Find the half-life time of this reaction.
2 .303 100 (log 10 2 = 0.3010) (U.P. 2020)
\ t 99. 9% = log 10
K 0.1 Solution: Q a = 10, a - x = 25
2 .303 2.303 a
= log 10 1000 \ k= log
K t a-x
2.303 100
And if t = t 50% , ( a - x ) = 100 - 50 = 50, a = 100 = log
2 .303 100 2 .303 60 25
\ t 50% = log 10 = log 10 2 2.303
K 50 K = log 4
t log 10 1000 3 60
or, 99. 9% = = = 10 2.303
t 50% log 10 2 0.3010 = ´ 2 ´ 0.3010 = 2.3 ´ 10 -2 min -1
60
or, t 99. 9% = 10 ´ t 50% 0.693 0.693
Now, t 1/ 2 = = = 30 min
(b) Similarly, prove that t 99. 9% = 3 ´ t 90% K 2.3 ´ 10 -2
2.303 3
(c) t 1 3 = log Problem 8: The initial concentration of SO 2Cl2 reduces to
K 2 half in 60 min. Calculate the rate constant if the dissociation of
(d) Solve as above. (t 99% = 2 ´ t 90% ) SO 2Cl2 is first order reaction. (U.P. 2017, 18)
(e) t 3 4 = 2 ´ t 1 2 0.693 0.693
Solution: K = = = 1.155 min -1
Problem 4: For a first order reactions, the concentration t 1/ 2 60
of reactant is reduced to its half in 50 seconds. Calculate the Problem 9: Half-life time of a first order reaction is 60
rate constant. (U.P. 2012, 17) minutes. In what time, 90% of reaction would be completed ?
Solution: For a first order reaction (U.P. 2014, 15)
0.693 0.693 Solution: For the first order reaction,
K = = = 1.38 ´ 10 -2 sec -1
t1 2 50 0.693 0.693
K = = = 1.155 ´ 10 -2 sec -1 …(i)
t1 2 60
(Q t 1 2 = 50 sec. )
Problem 5: For a first order reaction amount of a Now let initial concentration of reactant
substance becomes half in 100 seconds. Calculate a = 100, then a - x = 100 - 90 = 10
(a) The rate constant of reaction. According to first order reaction
(b) The time required to change the concentration from 0.05 M 2.303 a 2.303 100
to 0.0125 M. OR t = log 10 = log 10
K a - x 1.155 ´ 10 -2 10
Calculate the time in which a radioactive substance is reduced 2.303
to 1/4 to its initial amount. (U.P. 2017) = ´ log 10 10
1.155 ´ 10 -2
Solution: (a) For first order reaction 2.303
0.693 0.693 = ´ 1 {Q log 10 10 = 1}
K = = = 6.93 ´ 10 -3 sec -1 1.155 ´ 10 -2
t1 2 100
= 199.3 minutes
2.303 a Problem 10: From the data given below prove that the
(b) K = log
t a-x decomposition of H 2 O is a first order of reaction:
2.303 0.05
\ t = log = 2000 second Time (in minute:) 0 10 20 30
6.93 ´ 10 -3 0.125
Volume used ofKMnO 4 (V ml) 25 16 10.5 7.09
Problem 6: In a first order reaction, initial concentration
of reactants reduces to half in 40 sec. Calculate the rate
V is that volume of KMnO 4 which is used in various intervals of
constant of the reaction. (U.P. 2015)
time against a definite volume of the mixture.
or In a first order reaction, concentration of a substance
(Given : log 10 2 = 0.301, log 10 5 = 0.698 ,
becomes half of its initial concentration in 40 seconds.
log 10 10.5 = 1.021, log7.09 = 0.850) (U.P. 2014)
Calculate the rate constant of the reaction. (U.P. 2018)
Solution: For the first order reaction,
Solution: Given that, t 1/ 2 = 40 seconds
2.303 a
For I order reaction, K = log 10
t a-x
48

Here, a = 25 mL and ( a - x ) at t = 10, 20 and 30 minutes Thus, the value of rate constant ( K ) is nearly same at
is 16, 10.5 and 7.09 mL respectively. different time. Hence it is first order of reaction.
At t = 10 minutes Problem 12: 20% of a first order reaction completes in 10
2.303 25 minutes. How much time will be required to complete 75% of
K1 = log 10
10 16 the reaction? (log 10 2 = 0.31010) (U.P. 2016)
2.303 Or
= [log 10 ( 5) 2 - log 10 ( 2) 4 ]
10 A first order reaction is 20% complete in 10 minutes. Find the
2.303 time required for 75% completion of the reaction.
= ´ 0.192 Þ K 1 = 0.0442 min -1 …(i)
10 (U.P. 2020)
At t = 20 minutes, Solution: For a first order reaction,
2.303 25 2.303 2.303 a
K2 = log 10 = [ 2 log 10 5 - log 10 10.5] t = log 10
20 10.5 20 K a-x
2.303 If a = 100 , a - x = 100 - 20 = 80, thus
= ´ 0.375 Þ K 2 = 0.0431 min -1 …(ii)
20 2.303 100
10 = log
At t = 30 minutes, K 80
2.303 25 2.303 2.303
K3 = log 10 = [ 2 log 5 - log 7.09] K = [log 10 10 - 3 log 10 2]
30 7.09 30 10
2.303 2.303 2.303
= [ 2 ´ 0.698 - 0.850] = ´ 0.546 = [1 - 3 ´ 0.3010]
30 30 10
K 3 = 0.0419 min -1 …(iii) 2.303 2.303
K = [1 - 0.9030] = ´ 0.0970
10 10
Thus, the value of rate constant ( K ) is nearly equal at
different time. Hence decomposition of H 2O 2 is first order of K = 0.2303 ´ 0.0970 …(i)
reaction. and if a = 100 , a - x = 100 - 75 = 25 , t = ?
2.303 100 2.303
Problem 11: In a reaction concentration of a reactant is t = log 10 = log 10 4
reduced by 20% in 20 minutes, while 40% is reduced in 40 K 25 K
minutes. Determine the order of reaction. (U.P. 2015) 2.303 ´ 2
= ´ log 10 2
Solution: For the first order of reaction, K
2.303 a 2.303 ´ 2 ´ 0.3010
K = log 10 t = = 62.0619 min
t a-x 0.2303 ´ 0.097
According to problem, if a = 100, Problem 13: Rate constant for the first order reaction has
then a - x = 100 - 20 = 80, thus at t = 20 been found to be K = 2.54 ´ 10 -3 sec -1 . Calculate its
2.303 100 2.303 three-fourth life. (log 2 = 0.301) (U.P. 2019)
K1 = log = [log 10 10 - log 10 8]
20 80 20 Solution: t 3 4 = 2 ´ t 1 2
2.303 0.693 0.693
= [log 10 10 - 3 log 10 2] t3 4 = 2´ = 2´ = 545.6sec.
20 K 2.54 ´ 10 -3
2.303 OR
= [1 - 3 ´ 0.3010]
20 2.303 a 2.303
2.303 2.303 K = log = log 2
= [1 - 0.9030] = ´ 0.097 t1 2 a-a 2 t1 2
20 20
2.303 a
K 1 = 0.0111 min -1 and K= log
t3 4 3a
At t = 40 minutes a-
4
2.303 100 2.303 100 2.303 2.303
K2 = log = log 10 or log 2 = log 4
40 100 - 40 40 60 t1 2 t3 4
2.303
= [log 10 - log 10 6] 2 log 2 0.693
40 t 3 4 = t1 2 ´ = ´2
2.303 2.303 log 2 K
= [1 - 0.7782] = ´ 0.2218 0.693
40 40 = ´ 2 = 545.6 sec.
K 2 = 0.012 min -1 2.54 ´ 10 -3
49

Problem 14: Prove that the time required is 10 times of Solution: For a reaction, R = K[ A]n
the half-life period for the completion of 99.9% of a first order n n
énù énù
reaction. (U.P. 2019) Given that, R1 = K ê ú ; R2 = K ê ú
Solution: We have to prove, t 99. 9 t 1 2 = 10 ë V1 û ë V2 û
n
2.303 100 R1 é V2 ù
t 99. 9 = log or =
K 100 - 99.9 R 2 êë V1 úû
2.303 100
t1 2 = log 1
K 100 - 50 but R 2 = R1 Þ R1 = 2R 2 and V2 = 2V1
t 99. 9 log 1000 3 2
= = » 10 n
t1 2 log 2 0.3010 2R 2 é 2V1 ù
=ê ú
Problem 15: The initial concentration of N 2O 5 at 318 K R2 ë V1 û
was 0.60 ´ 10 -2 mol L-1 in the first order reaction 2 = ( 2) n Þ n = 1
1
N 2O 5 ( g ) ¾® 2NO 2 ( g ) + O 2 ( g ), which became i. e., reaction is of I order.
2
Problem 19: Prove that time required for the completion
0.20 ´ 10 -2 mol L-1 after 60 minutes. Calculate the velocity
of three-fourth of a first order reaction is twice the time
constant at 318 K (log 3 = 0.4771). (U.P. 2019) required for the completion of half reaction (U.P. 2019)
1
Solution: N 2O 5( g ) ¾® 2NO 2 + O 2 Solution: Time for 50% of the reaction
2 2.303 a 0.693
-2
a = 0.60 ´ 10 mol L -1 k= log = …(1)
t1 a2 t1
a - x = 0.20 ´ 10 -2 mol L–1 ; t = 60 min.
Time for 3 4 of reaction
For I order reaction, 2.303 a 2.303 a
2.303 a 2.303 0.60 ´ 10 -2 k= log = log
K = log = log t2 3a t2 a4
a-
t a-x 60 0.20 ´ 10 -2 4
2.303 2.303 ´ 0.4771 2.303 2 ´ 2.303 ´ 0.3010
= log3 = = 1.83 ´ 10 -2 min -1 = 2
log 2 =
60 60 t2 t2
Problem 16: Find out the unit of rate constant for the first 0.693
order reaction. A first order reaction is completed 50% in 100 k = 2´
t2
seconds. Find out the rate constant of the reaction.
(U.P. 2019) Put the value of k from eq. (1)
Solution: Unit of rate constant 0.693 0.693
= 2´
For I order reaction = time -1 (sec -1 or min -1 ), and t1 t2
t 2 = 2 t1
0.693 0.693 Problem 20: The initial concentration of N2 O5 at 318 K
K = = = 6.93 ´ 10 -3 sec -1 .
t 1/2 100 was 1.20 ´ 10-2 molL-1 in first order reaction
1
Problem 17: Define molecularity and order of the N2 O5(g) ® 2NO2(g) + O2(g) which became 0.60 ´ 10 mol L-1
-2
2
reaction. In a first order reaction, the concentration of
after 60 minutes. Calculate velocity constant at 318 K
substance reduces to half of the initial concentration in 40
(U.P. 2020)
seconds. Calculate velocity constant for the reaction. -2 -1
(U.P. 2019) Solution: [ A]0 = 1.20 ´ 10 mol × L ,
Solution: Molecularity of a reaction is equal to the [ A] = 0.60 ´ 10 -2 mol × L-1 , t = 60 min
number of reactant molecules taking part in the elementary For I order reaction,
step of balanced chemical reaction. Molecularity of a
2.303 | A|0 2.303 1.20 ´ 10 -2
reaction is always integral (never negative, zero or K = log = ´ log
fractional). Order of a reaction expresses the number of t | A| 60 0.60 ´ 10 -2
molecules which determines the rate expression of a 2.303 ´ log 2 2.303 ´ 0.3010
= = = 0.0115 min -1
reaction. It may be zero, fractional or integer. 60 60
For a I order reaction, Problem 21: The velocity of a reaction is K [A]1 2 [B] 3 2 .
0.693 0.693 Write the total order of reaction. (U.P. 2020)
rate constant, K = = = 1.73 ´ 10 -2 s -1
t1 2 40 Solution: Rate law = K [ A]1/ 2 [ B]3/ 2
Problem 18: The rate of gaseous reaction is halved when 1 3
\ Order of reaction = + = 2
the volume of the vessel is doubled. What is the order of 2 2
reaction ? (U.P. 2019)
50

Solution of NCERT Text Book Problems


Q. 1 From the rate expressions for the following reactions, Q.4 The decomposition of dimethyl ether leads to the
determine their order of reaction and the dimensions formation of CH 4 , H 2 and CO and the reaction rate is
of the rate constants : given by Rate = K [ PCH 3OCH 3 ]3/ 2 . If the pressure is
(a) 3NO( g ) ¾® N 2O( g ) + NO 2 ; Rate = K [NO]2 determined in bar and time in minutes, then what are
(b) H 2O 2 ( aq.) + 3I - ( aq. ) + 2H + ® 2H 2O( l) + I -3 ; the units of rate and rate constant ?
Rate = K [H 2O 2 ][I - ] Ans. Rate in terms of pressure : bar min -1
(c) CH 3CHO(g) ® CH 4 (g) + CO(g); Unit of rate constant = (pressure)1- n min -1
Rate = K [CH 3CHO]3/2 = (bar)1-1.5 min -1 = (bar) –0.5 min -1
(d) CHCl 3 ( g ) + Cl 2 ( g ) ® CCl 4 ( g ) + HCl( g ); Q.5 Mention the factors that effect the rate of a chemical
Rate = K [CHCl 3 ][Cl 2 ]1/2 reaction.
(e) C 2 H 5Cl( g ) ® C 2 H 4 ( g ) + HCl( g ); Ans. Following factors affect the rate of a reaction:
Rate = K [C 2 H 5Cl] (1) Concentration of reactants: On increasing
Ans. (a) Order = 2, K = L mol -1 s -1 . the concentration of reactants at a constant
temperature, the rate of reaction also increases
(b) Order = 2, K = L mol –1 s –1
because the number of collisions is increased due to
(c) Order = 3 2, K = (mol / L)1- n s -1 increase in molecular concentration of reactant.
= (mol/L)1-1.5 s -1 = mol -0.5 L0.5 s -1 (2) Temperature: In general, rate of reaction
(d) Order = 1 + 1/2 = 1.5, K = mol -0. 5 . L0.5 s -1 increases with increase in temperature because
(e) Order = 1, K = sec -1 number of effective collisions increases.
Q.2 For the reaction 2 A + B ¾® A 2 B, rate = K [ A][ B]2 (3) Catalyst: A positive catalyst lowers the value of
with K = 2.0 ´ 10 -6 mol -2 L2 s -1 . Calculate the initial activation energy. As a result, number of activated
rate of the reaction when [ A] = 0.1 mol L-1 and molecules is increased with increase in the rate of
reaction.
[ B] = 0.2 mol L-1 . Calculate the rate of reaction after (4) Surface area of reactants: On increasing the
[ A] is reduced to 0.06 mol L-1 . surface area of reactant molecules, the rate of
Ans. Initial rate = K [ A][ B]2 = 2 ´ 10 -6[ 0.1][ 0.2]2 reaction will also increase.
= 8 ´ 10 -9 mol L-1 s -1 (5) Light: In some reactions, absorption of light
Now concentration of [ A] becomes 0.06. causes the attainment of threshold energy level. In
Amount of A reacted = 0.10 – 0.06 = 0.04 such reactions, rate of reaction increases in presence
Amount of B reacted = 0.04/2 = 0.02 of light.
So B left = 0.20 - 0.02 = 0.18 (6) Nature of reactants: In comparison of the
Rate = K [ A][ B]2 = 2 ´ 10 -6 (0.06)(0.18) 2 reaction, 2NO + O 2 ¾¾® 2NO 2 , the reaction,
= 3.89 ´ 10 -9 mol L-1 s -1 CH 4 + 2O 2 ¾¾® CO 2 + 2H 2O is slow at the
Q.3 Decomposition of NH 3 on the surface of Pt is zero conditions of same temperature and concentration. It
order reaction. If value of K is 2.5 ´ 10 –4 mol L–1 sec –1 , is because of the fact that in the first reaction, only
then what will be the rate of formation of N 2 and H 2 ? one N—O bond is to be broken while in the second
Pt 1 3 reaction four C — H bonds are to be broken.
Ans. NH 3 ¾¾ ® N2 + H2
2 2 Q.6 A reaction is second order with respect to a reactant.
d[NH 3 ] 2 d[N 2 ] 2 d[H 2 ] How is the rate of reaction affected if the
Rate = – =+ =+
dt dt 3 dt concentration of the reactant is : (a) doubled,
Rate for zero order reaction = K = 2.5 ´ 10 –4 (b) reduced to 1/2 ?
d[NH 3 ] 2 d[N 2 ] 2 d[H 2 ]
\2.5 ´ 10 –4 = – =+ =+ Ans. Rate µ [ A]2 (for second order reaction)
dt dt 3 dt (a)If [ A] is doubled, rate will become 4 times.
d[N 2 ] 2.5 ´ 10 –4 (b)If [ A] is reduced to 1 2, rate will become 1 4 times.
Rate of formation of N 2 = =
dt 2 Q.7 What is the effect of temperature on the rate constant
= 1.25 ´ 10 –4 mol L–1 sec –1 of a reaction ? How can this temperature affect on the
d[H 2 ] 3 rate constant be represented quantitatively ?
Rate of formation of H 2 = = ´ 2.5 ´ 10 –4
dt 2 Or Write Arrhenius equation. How will you determine the
= 3.75 ´ 10 –4 mol L–1 sec –1 activation energy of a reaction by diagram?
(U.P. 2017)
Ans. Deleted for examination 2021.
51

Q.8 In a pseudo first order hydrolysis of an ester in water, Q.11 Following results were obtained during kinetic study
the following results were obtained: of the following reaction :
2 A + B ¾¾® C + D
t/s 0 30 60 90
Calculate the rate law and rate constant for this
-1
[Ester]/mol L 0.55 0.31 0.17 0.085 reaction.
Experiment [ A] [ B] Rate of
(a) Calculate the average rate of reaction between –1 –1 formation of D in
the time interval 30 to 60 seconds. mol × L mol × L
mol × L–1min –1
(b) Calculate the pseudo first order rate constant for
the hydrolysis of ester. I 0.1 0.1 6. 0 ´ 10–3
Ans. (a) Average rate during 30-60 sec : II 0.3 0.2 7. 2 ´ 10–2
0.17 - 0.31
- = 4.67 ´ 10 -3 mol L-1 sec -1 III 0.3 0.4 2.88 ´10–1
60 - 30
2.303 [A ] 2.40 ´10–2
(b) K = log 0 IV 0.4 0.1
t [ A]
Ans. Considering the experiment I and IV
[ A 0 ] : 0.55 0.55 0.55
Suppose rate law of the reaction is
[ A] : 0.31 0.17 0.085
Rate = K [ A]m[ B]n
t : 30 60 90
From the values of [ A 0 ],[ A] and t from each set, R (I) K [ A]m n
I ´ [ B] I
=
calculate the value of K. The values obtained are R (IV) K [ A]m n
IV ´ [ B] IV
1.91 ´ 10 -2 , 1.96 ´ 10 -2 and 2.07 ´ 10 -2 respectively. 6.0 ´ 10 –3 K [ 0.1]m ´ [ 0.1]n
=
Average of these values is 1.98 ´ 10 -2 sec -1 . 2.40 ´ 10 –2 K [ 0.4]m ´ [ 0.1]n
Q.9 A reaction is first order in A and second order in B. m
1 é1 ù
(a) Write differential rate equation. = Þ m =1
(b) How is the rate affected on increasing the 4 êë 4 úû
concentration of B three times ? Now, considering the experiments II and III
(c) How is the rate affected when concentration of R (II) K [ A]m n
II ´ [ B] II
=
both A and B is doubled ? R (III) K [ A]m n
III ´ [ B] III
Ans. Suppose, a and b are the initial molar concentrations
of A and B respectively. 7.2 ´ 10 –2 K [ 0.3]m ´ [ 0.2]n
=
(a)
dx
= K [ A][ B]2 2.88 ´ 10 –1 K [ 0.3]m ´ [ 0.4]n
dt n
1 æ1ö
(b) If concentration of B is increased 3 times, then =ç ÷ Þ n = 2
4 è 2ø
rate becomes 9 times.
(c) If concentration of both A and B is doubled, then \So rate law of the reaction is
rate becomes 8 times. Rate = K [ A]1[ B]2
Q.10 In a reaction between A and B, the initial rate ( r0 ) was Overall order = 1 + 2 = 3
measured for different initial concentrations of A and Q.12 The reaction between A and B is first order w.r.t. A
B. What is the order with respect to A and B ? and zero order w.r.t. B. Fill in the blanks in the
following table :
Set I Set II Set III Experiment [ A] [ B] Initial rate
0.20 0.20 0.40 mol × L–1 mol × L–1 mol × L-1min -1
[A]
I 0.1 0.1 2. 0 ´ 10–2
[B] 0.30 0.10 0.05
II ... 0.2 4. 0 ´ 10–2
r0 5.07 ´ 10-5 5.07 ´ 10-5 14.3 ´ 10-5
III 0.4 0.4 ...
Ans. Suppose order of reaction with respect to A is m, then IV ... 0.2 2. 0 ´ 10–2
dx
= K [ A]m Ans. Rate = K [ A]
dt
For expt. I : rate = K [ A] or 2 ´ 10 -2 = K [ 0.1]
From the given data,
5.07 ´ 10 -5 = K [ 0.20]m or K = 0.2
For expt. II : rate = K [ A] or 4 ´ 10 -2 = 0.2[ A]
14.3 ´ 10 -5 = K [ 0.40]m
or [ A] = 0.2
14.3 [ 0.40]m
= Þ m = 1.5 For expt. III : rate = K [ A] or rate = 0.2 ´ 0.4 = 0.08
5.07 [ 0.20]m For expt. IV : rate = K [ A] or 2 ´ 10 -2 = 0.2[ A]
or [ A] = 0.1
52

Q.13 Calculate the half-life of a first order reaction from (In graph locate the time against the conc. 0.0081)
their rate constants given below : (c) Graph between log [ N 2O 5 ] and time, t.
(a) 200 s -1 (b) 2 min -1 (c) 4 year -1
0.693 0.693 t (in sec) 102 ´ [N 2O5 ] log[N 2O5 ]
Ans. (a) t 1/ 2 = = = 3.46 ´ 10 -3 sec
K 200 0 1.63 –1.79
0.693 0.693
(b) t 1/ 2 = = = 0.346 min 400 1.36 –1.87
K 2
0.693 0.693 800 1.14 –1.94
(c) t 1/ 2 = = = 0.173 year
K 4 1200 0.93 –2.03
Q.14 The half-life for radioactive decay of 14 C is 5730 year. 1600 0.78 –2.11
An archaeological artifact contained wood that had
only 80% of the 14 C found in living tree. Estimate the 2000 0.64 –2.19
age of the sample. 2400 0.53 –2.28
0.693 0.693 2.303 [ A]0
Ans. K = = year -1 and t = log 2800 0.43 –2.37
t 1/ 2 5730 K [ A]
3200 0.35 –2.46
2.303 ´ 5730 100
t = log = 1845 years
0.693 80 –1.8
Q.15 The experimental data for decompositions of
–1.9
N 2O 5[ 2N 2O 5 ¾® 4NO 2 + O 2 ] in gas phase at 318 K
are given below: –2.1

log [N2O5]
–2.2
Time/seconds 0 400 800 1200 1600
–2.3
10-2 ´[ N 2O 5] M 1.63 1.36 1.14 0.93 0.78
–2.4

Time/seconds 2000 2400 2800 3200 –2.5


-2
10 ´[ N 2O 5] M 0.64 0.53 0.43 0.35 –2.6

0 400 800 1200 1600 2000 2400 2800 3200


(a) Plot [N 2O 5 ] against t. Plot of log [N2O5] vs time
(b) Find the half-life period.
(c) Draw a graph between log [N 2O 5 ] and t.
(d) Rate = K [N 2O 5 ]1
(d) What is rate law ?
(e) Calculate the rate constant. 2.303 [ A ] 2.303 1.63 ´ 10 -2
(e) K = log 0 = log
(f) Calculate the half-life period from K and compare t [ A] 400 1.36 ´ 10 -2
it with (b). -4 -1
= 4.5 ´ 10 sec
Ans. (a) Figure show the plot of [ N 2O 5 ] against time, t in 0.693 0.693
(f) t 1/ 2 = = = 1540 sec
seconds. K 4.5 ´ 10 -4
0.016
Q.16 The rate constant for a first order reaction is 60 sec -1 .
How much time will it take to reduce the initial
0.014
concentration of the reactant to 1/16th ?
0.012 2.303 [A ]
Ans. K = log 0
0.010 t [ A]
0.8 × 10–2 2.303 100
[N2O5]M

0.008 60 = log Þ t = 4.6 ´ 10 -2 sec


0.006
t 100 / 16
Q.17 During nuclear explosion, one of the products is 90 Sr
0.004
with half-life of 28.1 years. If one mg 90Sr was
0.002 t1/2 = 1420 s
absorbed in bones of a newly born baby instead of
0 400 1200 2000 2800 3200 calcium, how much of it will remain after 10 years
time in sec and 60 years if it is not lost metabolically ?
Plot of [N2O5] vs time 0.693 0.693 2.303 [A ]
Ans. K = = year -1 and K = log 0 .
(b) Half-life period, t 1 2 for the given reaction is the t 1/ 2 28.1 t [ A]
time during which initial conc. of [ N 2O 5 ] changes 0.693 2.303 1
= log Þ [ A] = 0.78 m g
from 1.63 ´ 10 -2 M to half this value i. e., to 28.1 10 [ A]
0.815 ´ 10 -2 M and in the graph it has been shown to Similarly, amount left after 60 years is 0.22 m g.
be 1420 seconds.
53

Q.18 Show that the time required for 99% completion of a 2.303 35 2.303 37
K2 = log = log
first order reaction is twice the time required for the 720 2 ´ 35 – 63 720 7
completion of 90%. 2.303 –3 –1
= ´ 0.699 = 2.23 ´ 10 sec
Ans. For a first order reaction, 720
2.303 a K + K2
K = log Average rate constant = 1
t (a - x ) 2
For 99% completion, a = 100, a - x = 100 - 99 = 1 2.17 ´ 10 –3 + 2.23 ´ 10 –3
2.303 100 2.303 = = 2.20 ´ 10 3 sec –1
\ K = log = ´2 …(i) 2
t 99% 1 t 99% Q.21 The following data were obtained during the first
For 90% completion, a = 100, a - x = 100 - 90 = 10 order thermal decomposition of SO 2Cl 2 at a constant
2.303 100 2.303 volume :
K = log = ´1 …(ii)
t 90% 10 t 90% SO 2Cl 2 ( g ) ¾® SO 2 ( g ) + Cl 2 ( g )
From eq. (i) and (ii),
2.303 2.303 Experiment Time/s Total pressure/atm
´2= ´1
t 99% t 90% 1. 0 0.5
or t 99% = 2 ´ t 90% 2. 100 0.6
Q.19 A first order reaction takes 40 min for 30%
decomposition. Calculate t 1/ 2 . Calculate the rate of reaction when the total pressure
2.303 [A ] is 0.65 atm.
Ans. K = log 0
t [ A] Ans. Initial pressure P0 = 0.5 atm and pressure Pt at time t,
2.303 100 2.303 P0
or K = log K = log (QPt = 0.6 atm)
40 100 - 30 t 2P0 – Pt
or K = 8.92 ´ 10 -3 =
2.303
log
2.303
log
0.5
0.693 100 100 2 ´ 0.5 – 0.6
t 1/ 2 = = 77.69 min
8.92 ´ 10 -3 2.303 0.5
= log = 2.23 ´ 10 –3 sec –1
Q.20 For the decomposition of azoisopropane to hexane 100 0.4
and nitrogen at 543 K, the following data are When Pt = 0.65 atm, P0 = 0.50
obtained : P = Pt – P0 = 0.65 – 0.50 = 0.15 atm
When the total pressure is 0.65 atm, then
t (in sec.) P (mm of Hg) PSO2Cl 2 = P0 – P = 0.50 – 0.15 = 0.35
0 35.0
Rate of reaction = K ´ PSO2Cl 2
= 2.23 ´ 10 –3 ´ 0.35
360 54.0 = 78.05 ´ 10 –4 atm sec –1
720 63.0 Q.22 The rate constant for the decomposition of N 2O 5 at
various temperatures is given below :
Calculate the rate constant.
Ans. (CH 3 ) 2 CH ¾ N==N ¾ CH(CH 3 ) 2 ® N 2 + C 6 H14 T/ºC 0 20 40 60 80
Initial pressure P0 0 0 5 -1
Pressure at ‘t’ time P P
10 ´ k /s 0.0787 1.70 25.7 178 2140
P0 - P
Total pressure of mixture
Draw a graph between ln K and 1/T and calculate
Pt = PA + PN 2 + PC6H14
the values of A and E a . Predict the rate constant at
= ( P0 – P ) + P + P = P0 + P
P = Pt – P0 30ºC and 50ºC.
PA = P0 – ( Pt – P0 ) = 2P0 – Pt Ans. Deleted for examination 2021.
Q PA µ a – x and P0 µ a Q.23 The rate constant for the decomposition of a
2.303 P0 hydrocarbon is 2.418 ´ 10 -5 s -1 at 546 K. If the
K1 = log
t 2P0 – Pt energy of activation is 179.9 kJ/mol, what will be the
Ist experiment value of pre-exponential factor ?
2.303 35 2.303 35 Ans. Deleted for examination 2021.
K1 = log = log Q.24 Consider a certain reaction A ¾® Product with
360 2 ´ 35 – 54 360 16
2.303 2.303 K = 2.0 ´ 10 -2 s -1 . Calculate the concentration of A
= log 2.1875 = ´ 0.339
360 360 remaining after 100 s if the initial concentration of A
= 2.17 ´ 10 –3 sec –1 is 1.0 mol L-1 .
IInd experiment Ans. The unit of K show that it is a first order reaction.
54
2.303 [A ] log K = 14.34 - 1.25 ´ 10 4 K / T
K = log 0
t [ A] Calculate E a for this reaction and at what
-2 2.303 1 temperature will its half-period be 256 minutes ?
or 2 ´ 10 = log Þ
100 [ A] Ans. Deleted for examination 2021.
[ A] = 0.135 mol L–1 Q.28 The decomposition of A into product has value of K as
Q.25 Sucrose decomposes in acid solution into glucose and 4.5 ´ 10 3 s -1 at 10ºC and activation energy
fructose according to the first order rate law with t 1/ 2 60 kJ mol -1 . At what temperature would K be
=3.00 hours. What fraction of the sample of sucrose 1.5 ´ 10 4 s –1 ?
remains after 8 hours. Ans. Deleted for examination 2021.
2.303 [ A ] 0.693 0.693 Q.29 The time required for 10% completion of a first order
Ans. K = log 0 , = = = 0.231 reaction at 298 K is equal to that required for its 25%
t [ A] t 1/ 2 3
[A ] completion at 308 K. If the value of A is 4 ´ 1010 s -1 ,
2.303
or 0.231 = log 0 calculate K at 318 K and E a .
8 [ A]
Ans. Deleted for examination 2021.
[ A0] [ A] 1
or = 6.345 or = = 0.158 Q.30 When temperature of a reaction is raised 293 K to
[ A] [ A 0 ] 6.345 313 K, the rate of reaction increases four times.
Q.26 The composition of a hydrocarbon follows the Calculate the activation energy for the reaction
equation K = (4.5 ´ 1011 s -1 )e -28000 K / T . Calculate E a . assuming that it does not vary with temperature.
Ans. Deleted for examination 2021. Ans. Deleted for examination 2021.
Q.27 The rate constant for the first order decomposition of
H 2O 2 is given by the following equation:

vvv
55

Chapter

SURFACE CHEMISTRY
Syllabus: Adsorption : Physisorption and chemisorption (sorption), factors affec ting adsorption of gases on solids.
Catalysis : *Homogeneous and heterogeneous, activity and selectivity; enzyme catalysis. Colloidal state :
Distinction between true solutions and suspension, lyophilic, lyophobic multimolecular and macromolecular colloids; properties of
colloids; Tyndall effect, Brownian movement, electrophoresis, coagulation, *emulsion and types of emulsions.

* Topics Deleted for Examination 2021

Objective Questions
1. Fog is an example of which type of colloid? 7. Foam is a colloid in which: (U.P. 2015, 18)
(U.P. 2012, 14) (a) liquid is dispersed in gas
(a) Liquid dispersed in gas (b) gas in liquid
(b) Gas dispersed in gas (c) liquid in liquid
(c) Solid dispersed in gas (d) solid in gas
(d) Solid dispersed in liquid 8. In a solvent, size of dispersed phase particles ranges
2. On adding dil. HCl or FeCl 3 solution in fresh between 50 Å to 200 Å . It is : (U.P. 2015)
precipitate of Fe (OH) 3 , red coloured colloid is (a) suspension (b) true solution
obtained. This process is called : (U.P. 2011, 12) (c) colloid (d) saturated solution
(a) peptization (b) dialysis 9. Brownian movement is observed due to : (U.P. 2015)
(c) protector (d) solvation (a) variation in temperature of liquid state
3. The process used for the purification of colloids is: (b) size of particles
(U.P. 2017) (c) unequal collisions of particles of dispersion
(a) peptization (b) coagulation medium with dispersed phase particles
(c) dialysis (d) Bredig’s arc method (d) attraction and repulsion between colloid particles
4. Purple of cassius is: (U.P. 2011, 18) due to charge
(a) Fe(OH) 3 sol (b) Au sol 10. Select the most effective solution for the coagulation
(c) sulphur sol (d) As 2S 3 sol of arsenious sulphide colloid. (U.P. 2015, 16)
5. The process of conversion of a precipitate into colloid (a) NaCl (b) Na 3 PO 4 (c) BaCl 2 (d) Na 2SO 4
by adding electrolyte is called : (U.P. 2013) 11. Which of the following is not a natural colloid?
(a) dialysis (b) peptization (U.P. 2016)
(c) electro-osmosis (d) electrophoresis (a) NaCl (b) Blood (c) RCOONa (d) Sugar
6. When the dispersion medium is air, the sol formed is 12. Among the following, hydrophobic colloid is :
called : (U.P. 2014) (U.P. 2018)
(a) alcosol (b) hydrosol (a) starch (b) glue
(c) benzosol (d) aerosol (c) stannic oxide (d) gelatin
Answers
1. (a) 2. (a) 3. (c) 4. (b) 5. (b) 6. (d) 7. (b) 8. (c) 9. (c) 10. (c)
11. (c) 12. (c)

Very Short Answer Type Questions


Q.1 What is peptization? (U.P. 2007, 09, 10, 12) Ans. The process of converting a freshly prepared
Give the proper peptizing agent for precipitation of precipitate into colloidal state by the addition of
Fe(OH) 3 . (U.P. 2010) some suitable electrolyte. The electrolytes used for
this purpose are called peptizing agent or
56

peptide. For the precipitation of Fe(OH) 3 , the FeCl 3 Q.2 Lyophilic colloids are more stable than lyophobic
is used as peptide. colloids, why ? (U.P. 2012, 15)
Fe(OH) 3 + Fe 3+ ¾® [ Fe(OH) 3 ]Fe 3+ Ans. In lyophilic colloids, colloidal particles are highly
(from FeCl 3 )
ppt colloidal particle solvated and the layer of solvent formed round the
colloidal particles acts as a protective layer.

Short Answer Type Questions


Q.1 What is Hardy-Schulze’s law ? Explain. Importance of Dialysis : Blood is a colloidal
(U.P. 2010, 12, 15, 18,19, 20) solution and impurities of blood are removed by
Or What is Hardy Schulze law ? Arrange the following kidneys through dialysis technique.
solutions in order of coagulating power giving reason. Q.3 What is purple of Cassius ? How will you obtain it ?
1 M AlCl 3 ,1 M CaCl 2 ,1 M KCl, 1 M Th(NO 3 ) 4 . (U.P. 2013)
(U.P. 2019) Or Describe Bredig’s Arc method for the preparation of
Ans. According to Hardy-Schulze’s law, “Greater the gold sol. (U.P. 2014)
valency of active ion greater will be its coagulating Or Describe one method of preparation of lyophobic
power”. (The active ion is the ion carrying charge colloid. (U.P. 2010, 14)
opposite to the charge on colloidal particles). Ans. Reduction of auric chloride with stannous chloride
Examples: gives a violet coloured gold sol called purple of
(i) For the coagulation of a positive sol of Fe(OH) 3 , Cassius.
the coagulating power of different anions is found 2AuCl 3 + 3SnCl 2 ¾® 2Au + 3SnCl 4
gold sol
decrease in the order.
[Fe(CN) 6 ]4 - > PO 4 3- > SO 4 2- > Cl - Preparation of gold sol by Bredig’s Arc
(ii) For the coagulation of a negative sol of As 2S 3 , the Method: In this method, two electrodes of the metal
coagulating power of different cations is found to be (gold) whose colloidal solution is to be prepared are
immersed in dispersion medium such as water
in order.
containing NaOH. The dispersion medium is kept
Ti 4 + > Al 3+ > Ca 2+ > K + cooled by surrounding it with ice. On applying high
Q.2 What is meant by dialysis ? What is its importance? voltage electric arc is produced between the
(U.P. 2013) electrodes. As a result, electrode vaporises which get
Or With the help of suitable diagram, explain the condensed immediately in the liquid to give colloidal
purification of colloid by dialysis. (U.P. 2017) solution.
Or Write down a method used for the purification of (+) (–)
colloidal solutions with diagram. (U.P. 2019)
Ans. Purification of colloids by Dialysis : Metal
Separation of ions of electrolyte or molecules of electrodes
impurity from a colloidal solution by diffusion
Arc
through a parchment membrane is called dialysis.
Apparatus used for this process is called dialyser. Ice
In this method, the colloidal sol is taken in a bag of
parchment paper and placed in running water. The
soluble impurities are going in running water and H 2O
filtered and the pure colloidal solutions remains in with KOH
bag. It is shown in figure.
Bredig’s arc method
Parchment Q.4 Write short notes on Brownian movement.
paper (U.P. 2012, 16, 17, 19, 20)
Ans. In 1928, an English botanist Robert Brown observed
Solution that the colloidal particles are moving at random in a
zig-zag motion in all the possible directions. This type
Impure
of motion is called Brownian movement as shown
colloidal sol. in figure.
Water

Process of dialysis for removing impurities from colloidal solution

Brownian movement of colloidal particles


57

Cause of Brownian movement : Brownian and flow out with water. The electrodialysis is shown
movement is observed due to unequal collisions of in Figure.
particles of dispersion medium with dispersed phase
particles.
Uses : (i) It provides evidence in favour of kinetic
theory of gases.
(ii) It helps in one of the method of determination of
Avogadro’s number.
Q.5 How are the colloidal solutions purified by
electro-dialysis ? (U.P. 2019)
Or Explain Electrodialysis by giving diagram.
(U.P. 2020)
Ans. Electro-dialysis : Dialysis is a slow process. But
when impurity is an electrolyte then, dialysis can be
fastened by electolysis. In electrodialysis electrodes Mechanism of Electrodialysis
are fitted outside the parchment bag. Impurity
particles move towards oppositely charged electrodes

Long Answer Type Questions


Q.1 Explain protection or protective colloids with suitable
examples. (U.P. 2003, 05, 06, 08, 13, 16, 17) Protective colloid Gold number
Or Define Gold number.(U.P. 2010,11,14,16,18, 19)
Ans. “Stability of lyophobic colloids increases in presence of Gelatin 0.005-0.01
lyophilic colloids.” This process is called as Haemoglobin 0.03
protection because lyophilic colloid protects
lyophobic colloid from coagulation. The lyophillic Gum 0.15
colloid is termed as protective colloid. Albumin of egg 0.08-0.10
For example: When an electrolyte is added to
lyophobic gold sol, its coagulation occurs. However, Starch 25-40
when lyophilic gelatin sol is added to it, coagulation Sodium oleate 0.4
of gold sol does not occur by that amount of
electrolyte. Here, gelatin is protective colloid Bovine albumin 0.10-0.20
while gold sol is protected colloid. Other Q.2 Write short note on adsorption. (U.P. 2016, 17)
examples are gum, starch, albumin etc.
or
Mechanism of Protection : On adding a lyophilic
Clarify the difference of adsorption and absorption
colloid to a lyophobic colloid; particles of lyophilic
with suitable examples. (U.P. 2014)
colloid, because of their high tendency of solvation,
Ans. Adsorption and Absorption :
form a protective layer round the particles of
lyophobic colloid. As a result, particles of lyophobic S.No. Adsorption Absorption
colloids can not approach one another and process of
1. It is a surface phenomenon. It is a bulk phenomenon.
coagulation slows down. The result is increased
stability of lyophobic colloid. 2. Adsorption is responsible Concentration of absorbed
Protecting Power: Gold Number : The concept for higher concentration of material remains more or less
of gold number given by Zsigmondy is used to adsorbate on the surface of uniform throughout the bulk
determine the protective power of a lyophilic colloid. adsorbent. of solid phase.
Gold number is defined as: 3. It occurs at high rate It occurs at uniform rate and is
"The number of milligrams of the protective (lyophilic) initially but afterwards the slower than rate of adsorption.
colloid which must be added to 10 c.c. of a given gold rate decreases till the
equilibrium is reached.
sol so as to just prevent its possible coagulation by
addition of 1 c.c. of 10% NaCl solution." 4. It is exothermic process. No significant change occurs.
Smaller the gold number, higher the protective power 5. Example: Adsorption of Blue colour of chalk when
of a lyophilic colloid. water molecules on surface dipped in blue ink (due to
Gold Numbers of Some Protective Colloids of CaCl 2 or silica gel. capillarity)
58

Solution of NCERT Text Book Problems


Q.1 Distinguish between the meaning of the terms or Differentiate between physical adsorption and
adsorption and absorption. Give one example of chemical adsorption with suitable example.
each. (U.P. 2018)
Ans. Adsorption is a surface phenomenon while Or Write four main differences in physical adsorption
absorption is a bulk phenomenon. and chemisorption. (U.P. 2019)
Q.2 What is the difference between physisorption and Ans.
chemisorption?

Comparison between physisorption and chemisorption


S.No. Property Physisorption Chemisorption
1. Nature of binding forces between Weak van der Waals’ forces (dipole-dipole Strong valence bond forces, that is why the
adsorbate and adsorbent attraction, polarisations dispersion effects etc.) activation energy of desorption is very high.
2. Reversibility Reversible Irreversible
3. Effect of temperature Occurs appreciably at low temperature and as Occurs at all temperatures. Extent of adsorption
tem per a ture is in creased the ex tent of first increases with increase in temperature and
adsorption decreases. then decreases after a certain temperature.
4. Heat of adsorption (DH ads. ) Due to weak forces, DH ads. values are low Due to stronger attraction forces, DH values are
(4 - 40 kJ mol ) and DH is always -ve . higher and ranges between 40 - 400 kJ mol.
5. Number of layers of adsorbate May be monolayered or multilayered. Always monolayered.
6. Activation energy Does not require activation energy. Requires activation energy.
7. Equilibrium Attains equilibrium state rapidly. Attains equilibrium state after sometime.
8. Specificity Non-directional and non-specific. Always directional and specific.

Q.3 Give reason why a finely divided substance is more


effective as an adsorbent ? Methane 16.2 190
Ans. Due to higher surface area in finely divided state.
Carbon monoxide 9.3 134
Q.4 What are the factors which influence the adsorption
of a gas on a solid ? Oxygen 8.2 154
or Write the factors affecting the adsorption of gases Nitrogen 8.0 126
on solids. (U.P. 2018)
Ans. All the gases adsorb on all the solid surfaces and Hydrogen 4.7 33
extent of adsorption depends upon following factors:
2. Nature of adsorbent : A surface with more
1. Nature of adsorbate : The gases which can be
residual forces will have higher adsorptive capacity.
liquefied more easily (i. e., gases with higher critical
Due to presence of incomplete d-subshells and high
temperature) are adsorbed to greater extent. For
charge/size ratio, d-block metals are good
example, HCl, NH 3 , CO 2 , SO 2 etc., are adsorbed to
adsorbents. Other common surfaces used for
greater extent than permanent gases like, H 2 , N 2 , O 2
adsorption are charcoal, silica gel, aluminium oxide,
etc.
clay etc.
Adsorption of Gases by Charcoal
3. Surface area of adsorbent : Larger is the
Gas Volume sorbed (mL) Critical Temp. surface area of adsorbent, higher is the adsorptive
(Tc in K) capacity of adsorbent, and that is why, finely divided
substances adsorb more than granulated substance.
Sulphur dioxide 380 430
4. Temperature : The effect of temperature on the
Chlorine 235 417 extent of adsorption can be studied with the help of
Ammonia 181 406 adsorption isobars (the graphs plotted extent of
adsorption, x m, vs. T at constant pressure). For
Hydrogen sulphide 99 373 physical and chemical adsorption following type of
Hydrogen chloride 72 324 isobars are obtained :
Nitrous oxide 54 310
Carbon dioxide 48 304
59

The graph obtained gives following informations


about extent of adsorption.
1. At very low pressure : At very low pressures,
Rate

x/m
the graph is nearly straight line and at these
pressures.
Time T
x x
(a) Physical adsorption (b) Chemical adsorption µ P or = K ×P
m m
Adsorption isobars
At very low pressure, adsorption is first order
Because usually the adsorption process is exothermic
reaction.
in nature, so extent of adsorption decreases with
increases in temperature. 2. At intermediate range of pressure : At
5. Effect of pressure : The relation between these pressures graph is curved and
x x
extent of adsorption (i. e., amount of adsorbate µ P 1 n or = K × P1 n
adsorbed per unit mass of adsorbent) can be studied m m
by plotting graphs x m (i. e., extent of adsorption) At intermediate pressures, adsorption is fractional
versus pressure ( P ). These graphs are known as order reaction.
adsorption isotherms. The nature of these 3. At very high pressure : The graph becomes
adsorption isotherms for adsorption of N 2 on the parallel to y-axis which indicates that extent of
surface of charcoal is shown in figure. adsorption is independent to the pressure, i. e.,
40 –190°C x x
µ P ° or =K
N2 adsorbed (x/m)

–78°C m m
At high pressures, adsorption follows zero order
–29°C
20 kinetics.
0°C Above conditions are supported by experimental
results.
Limitations of Freundlich Adsorption
50 100 0
Pressure, cm Hg Isotherm
Nitrogen adsorption on charcoal vs. pressure 1. It is valid only over a limited range of pressure
at fixed temperature (adsorption isotherm) only and it does not explain why at very high
Q.5 What is an adsorption isotherm ? Describe Freundlich pressure, extent of adsorption becomes
adsorption isotherm. independent to pressure.
Ans. Freundlich proposed an empirical relation between 2. The constant K and n vary with nature of
extent of adsorption and pressure. According to it, at adsorbate and temperature.
constant temperature. 3. The relation is purely empirical and has no
x
= K × P1 n mathematical support.
m
Q.6 What do you understand by activation of adsorbent ?
(n > 1 and 1 n can have values between 0 - 1,
How is it achieved ?
probably 0.1 - 0.5)
Where x = amount of adsorbate, m = amount of Ans. It means increasing the adsorbing power of an
adsorbent adsorbent. It is done : (i) by increasing the surface
P = pressure, K and n are constants which depend on area, (ii) by making the surface rough, (iii) by
the nature of the adsorbent and the gas at given removing the gas already adsorbed.
temperature. Q.7 What role does adsorption play in heterogeneous
x 1
x catalysis ?
n
m = KP m= K Ans. A heterogeneous catalyst provides a surface to
reactants so that they come close and react with each
other.
Q.8 Why is adsorption always exothermic ?
(x/m)

Ans. Because adsorption involves forces of attraction


x Saturation leading to decrease in potential energy.
m = KP Pressure Q.9 How are the colloidal solutions classified on the basis
Pressure, P of physical states of the dispersed phase and
The Freundlich adsorption isotherm dispersion medium?
60

Ans.
Types of Colloids on the Basis of Nature of Dispersion Medium and Dispersed Phase.

S. No. Physical state of Physical state of Name Example


dispersed phase dispersion medium
1. Solid (i) Solid (i) Solid sol Alloys, coloured glass, Ruby glass, gems etc.
(ii) Liquid (ii) Sols Mud, Gold sol, Fe(OH)3 sol, sols of agar-agar and gelatin etc.
(iii) Gas (iii) Aerosol of solid Dust, smoke etc.
2. Liquid (i) Solid (i) Gel Gel, jelly, cheese, fruits, vegetable, pearls etc.
(ii) Liquid (ii) Emulsion Butter, milk, oil in water, shampoo, cosmetics etc.
(iii) Gas (iii) Aerosol of liquid Cloud, mist, fog etc.
3. Gas (i) Solid (i) Solid foam Adsorbed gases, styrene, foam, rubber etc.
(ii) Liquid (ii) Foam or Froth Froth, soap, leather etc.

Q.10 Discuss the effect of pressure and temperature on the adsorption of gases on solids.
Ans. On increasing pressure, extent of adsorption increases while on increasing temperature, extent of adsorption
decreases.
Q.11 What are lyophilic and lyophobic sols ? Give one example of each. Why is hydrophobic sol easily coagulated ?
Or What are lyophilic and lyophobic colloids ? Write main difference between lyophilic and lyophobic colloids.
(U.P. 2010, 14, 16)
Or Write the difference between reversible and irreversible colloids ? (U.P. 2006, 17)
Or Clarify the difference between lyophobic and lyophilic sols on the basis of following properties: (U.P. 2019)
(i) Viscosity (ii) Effect of electrolyte (iii) Surface tension (iv) Electric charge
Ans.
Distinction Between Lyophilic and Lyophobic Colloids

S.No. Property Lyophilic colloids Lyophobic colloids


1. Nature Dispersed phase has high affinity for dispersion Dispersed phase has less affinity for dispersion
medium.They are generally of organic nature. medium. They are generally of inorganic nature.
2. Preparation As soon as dispersed phase comes in contact with They are prepared by special methods.
medium, they are prepared.
3. Concentration of dispersed Higher concentration is possible Only low concentration is possible.
phase in sol.
4. Stability Highly stable and no stabilising agent is needed. Less stable, stabilising agents are required to
stabilise them.
5. Effect of electrolytes Small quantities of electrolytes have no effect. Even small quantities of electrolytes cause
Larger quantities, however, cause coagulation coagulation.
6. Size of particles of Size of colloidal particles is in the colloidal range Particles are either aggregates of small particles
dispersed phase and smaller than those of lyophobic colloids. or formed by division of very large particles.
7. Viscosity Higher than that of dispersion medium Almost same as that of dispersion medium
8. Surface tension Lower than that of dispersion medium Almost same as that of dispersion medium.
9. Reversibility Reversible Irreversible
10. Charge Depends on the pH of medium Does not depend on the pH of medium.
11. Solvation Have greater tendency of solvation Have less tendency of solvation
12. Tyndall effect Less distinct More distinct
13. Electrophoresis More difficult to measure Easily measured
14. Gelatinisation May be gelatinised easily and show isoelectric Do not gelatinise easily and coagulate on the
point reversal of charge.
15. Examples Gum, starch, albumin sols etc. Metal sols, metal sulphide sols, metal hydroxide
sols etc.

Hydrophobic colloids are easily coagulated due to low affinity of DP with DM.
61

Q.12 What is the difference between multimolecular and Q.17 What is demulsification ? Name two demulsifiers.
macromolecular colloids ? Give one example of each. Ans. Deleted for examination 2021.
How are associated colloids different from these two Q.18 Action of soap is due to emulsification and micelle
types of colloids ? formation. Comment.
Ans. Multimolecular colloids : When a large number Ans. Following two processes are involved in the action of
of atoms/small molecules (having diameters less washing of soap.
than 1nm) of a substance combine together in a 1. Solubilization of oily particles
dispersion medium to form aggregates having size in 2. Emulsification of oily particles.
the colloidal range, the colloid is called
multimolecular colloid, e. g ., gold sol, sulphur sol etc.
Their molecular masses are not very high and atoms/
molecules forming them are held together by weak
van der Waals’ forces. Usually they are lyophobic in
nature.
Macromolecular colloids : They are made up of
molecules of sufficiently large size (lying in colloidal
range), e. g ., polymers like rubber, starch, proteins
etc. Their molecular masses are very high and due to
long chain, van der Waals’ forces holding them are
comparatively stronger. Usually they are lyophilic in Oily particles are lyophobic in nature, hence when
nature. soap solution is poured on a dirty fabric, oily particles
Associated colloids : They are formed by get adsorbed on central hydrophobic part of micelle
aggregation of larger number of ions in concentrated and its emulsification takes place. When, this fabric is
solution, e. g ., soap sol. soaked in water, due to attraction between
Their constituent ions have both lyophobic and hydrophilic part of micelle and water, micelle along
lyophilic end. with oily particles escapes into water.
Q.13 What are enzymes ? Write in brief the mechanism of Q.19 Give four examples of heterogeneous catalysis.
enzyme catalysis. Ans. Deleted for examination 2021.
Or Explain the enzyme catalysis with suitable examples. Q.20 What do you mean by activity and selectivity of
(U.P. 2001, 06, 07, 10, 16, 17) catalysts ?
Ans. Deleted for examination 2021. Ans. Deleted for examination 2021.
Q.14 How are colloids classified on the basis of : Q.21 Describe some features of catalysis by zeolites.
(i) physical states of components Or Explain the Zeolites and their uses. (U.P. 2016)
(ii) nature of dispersion medium and Ans. Deleted for examination 2021.
(iii) interaction between dispersed phase and
Q.22 What is shape selective catalysis ?
dispersion medium ?
Ans. (i) Solid sols (S/S), Sols (S/L), Aerosol of solid Ans. Deleted for examination 2021.
(S/G), Gel (L/S), Emulsion (L/L), Aerosol of Q.23 Explain the following terms :
liquid (L/G), Solid foam (S/G), Foam (G/L). (i) electrophoresis (U.P. 2010, 13, 17)
(ii) Hydrosol (water), aerosol (air), alcosol (ii) coagulation (U.P. 2010, 13, 16, 19, 20)
(alcohol), benzosol (benzene) etc. (iii) Tyndall effect
(iii) Lyophilic and lyophobic colloid. (U.P. 2011,12,14,16,17,18,19, 20)
Q.15 Explain what is observed : Ans. (i) Electrophoresis : This movement of colloidal
(i) when a beam of light is passed through a particles under influence of electric field is known as
colloidal sol. electrophoresis.
(ii) an electrolyte, NaCl is added to hydrated ferric Electrophoresis occurs due to the presence of charge
oxide sol. on colloidal particles. If colloidal particles migrate
(iii) electric current is passed through a colloidal sol ? towards cathode then there is a positive charge on
Ans. (i) Tyndall effect is observed. colloidal particle while negative charge on particles
(ii) The ferric oxide sol is positively charged. which migrate towards anode.
Colloidal particles get coagulated by Cl – ions.
In electrophoresis,colloidal particles get discharged
(iii) Electrophoresis and electroosmosis are observed.
Q.16 What are emulsions ? What are their different types ? at oppositely charged electrodes and are precipitated
Give example of each type. due to the formation of aggregates.
Ans. Deleted for examination 2021.
62

(5) By electrophoresis: In electrophoresis,


electrically charged colloidal particles migrate
towards oppositely charged electrodes and get
discharged. As a result, coagulation occurs.
(6) Mutual coagulation: When two oppositely
charged colloids are mixed together, opposite
charges neutralise each other and coagulation
takes place. This is known as mutual coagulation
e.g., When positively charged red coloured ferric
hydroxide sol is mixed with negatively charged
yellow coloured arsenic sulphide sol, mutual
coagulation occurs and a colourless solution is
obtained.
(7) By adding electrolytes: This is the important
method of coagulation. Particles of dispersed
Applications of Electrophoresis : phase of colloids carry electric charge. When an
(1) With the help of electrophoresis, the nature of electrolyte is added to the colloidal solution, the
electric charge present on colloidal particles can colloidal particles take up the ions given by
be determined. electrolyte whose charges are opposite to that on
(2) Zeta potential can be calculated with its help. colloidal particles. Consequently the charge on
colloidal particles is neutralised and coagulation
(3) Electrostatic precipitator used to separate carbon
takes place. In order to explain the coagulation
particle from smoke in pollution control acts on caused by electrolyte, Hardy-Schulze rule was
the basis of electrophoresis. proposed. According to this law:
(4) In rubber plating of metals. (a) When an electrolyte is added to a colloidal
(5) In the plating of colloidal dyes on metal surfaces. system, the electrolyte ion carrying opposite
(6) In coagulation of colloids. charge to that of colloidal particle is responsible
(ii) Coagulation : "The process of conversion of for the coagulation. This ion is termed as
colloids into suspension is known as coagulation." effective ion e.g. For positively charged colloids
or (like- hydroxide sols), anions (like- Cl – , Br – , I – ,
"The process by means of which colloidal particles are SO 2– 3–
4 , PO 4 etc.) are effective ions. On the other
precipitated is known as coagulation." hand, cations (like- Na + , Ba 2+ , Al 3+ etc.) are
Coagulation is also known as flocculation. effective ions for negatively charged colloids
Methods of Coagulation (like- sulphide sols).
(1) By boiling: Few sols, like sols of sulphur, silver (b) Greater the valency of effective ion of an
chloride etc., coagulate on heating. This is electrolyte, greater will be the coagulating power
because kinetic energy of colloidal particles of electrolyte. e.g.,
increases on heating and particles overcome Order of Coagulating power of anions:
repulsion force acting between them. As a result KNO 3 < K 2SO 4 < K 3 PO 4 < K 4 [Fe(CN) 6 ]
they come closer to one another forming or NO –3 < SO 2– 3– 4–
4 < PO 4 < Fe(CN) 6
aggregates. For example: On heating the butter,
Order of coagulating power of cations:
oil and water separate out from each other.
KNO 3 < Ba(NO 3 ) 2 < Al(NO 3 ) 2 < Th(NO 3 ) 4
(2) By cooling: Some colloids, especially emulsions,
or K + < Ba 2+ < Al 3+ < Th 4 +
get coagulated on excessive cooling.
(c) Coagulating power of an electrolyte is expressed
For example: on excessive cooling of milk in terms of its flocculation value.
(emulsion), fat separates out above the milk. "The minimum amount of the electrolyte in
(3) By violet stirring or vibratory action: On violet milli-mole required to coagulate completely one
stirring or on vibrating with ultrasound, some litre of a sol is called as its flocculation value."
colloidal systems get coagulated. Lower the flocculation value of an electrolyte ,
For example: Removal of butter from the curd by greater will be its coagulating power.
vigorous stirring. (iii) Tyndall effect : When a beam of light passes
(4) By dialysis: Repeated dialysis of a colloidal through a colloidal solution, it is scattered by
system removes the traces of electrolyte from the colloidal particles in all the directions and the path of
colloid essential for the stability of colloid. light becomes visible from all the angles. This
Consequently, coagulation of colloid occurs. scattering is called Tyndall effect.
63

Tyndall effect successfully explains following


phenomena:
(1) Blue colour of sky
(2) Blue colour of ocean
(3) Twinkling of stars
(4) Illumination of light rays path in a dark room
(5) Illumination of light path of projector
(6) Illumination of light path thrown by circuses
(7) Red colour of sun during sunrise and sunset
Q.24 Give four examples of emulsions.
Ans. (i) Butter (ii) Milk (iii) Creams (iv) Grease
Scattering of light Q.25 What are micelles ? Give an example of a miceller
system.
Microscope Colloidal Or Explain the mechanism of the formation of micelle.
Lens solution (U.P. 2019)
Ans. On dissolving some substances in a solvent, their
molecules associate spontaneously forming colloidal
particles. The colloids in which colloidal particles are
aggregates of small particles are termed as
association colloids. Association colloids are of
Scattered
light following two types:
Illuminated path (a) Colloidal electrolytes.
(c) (Tyndall cone) (b) Colloidal non-electrolytes
Tyndall effect Some substances which behaves like electrolytes in
When the light beam is seen from the direction dilute solutions but begins to behave like colloids
opposite to the incident light, a bright cone of light is with increase in concentration. On increasing the
visible known as Tyndall cone or Tyndall beam concentration, the aggregate formed by association
or Tyndall light. The scattering of light by colloidal of ions is termed as micelle. Size of a micelle is in
solutions is known as Tyndall effect. the range of colloidal particles. The minimum
Factors Affecting Scattering of Light : Tyndall concentration of the solution at which first micelle
effect is affected by following factors. appears is called as critical micelle
(1) Wavelength of light: Intensity of light concentration (CMC). The number of particles
scattered is related to the wavelength of light as: present in a micelle is known as aggregation
1 number. Such type of substances are known as
Intensity of light scattered µ 4
l colloidal electrolytes.
Since wavelength of violet light is smallest, hence its For example:
scattering is highest while scattering of red light is Soaps like-sodium stearate (C 17 H 35COO – Na + ),
lowest due to its larger wavelength.For this reason, sodium laurate (C 11 H 23COO – Na + ) etc., detergents
sky and ocean are seen blue in colour.
(2) Size of particles of dispersed phase: like-sodium dodicyl sulphate (C 12 H 25OSO –3 Na + ),
Larger the size of particles of dispersed phase, greater alkyl ammonium chloride (RNH 4 Cl) etc.
the magnitude of scattering. Due to this fact, Micelle formation : The formation of micelles in
lyophobic sols scatter more light. soap (sodium stearate) can be explained as follows:
(3) Difference between refractive indices of C 17 H 35COO – Na + ¾¾® C 17 H 35COO – + Na +
dispersed phase and dispersion medium: The structure of anion formed is as:
Greater is the difference between refractive indices of O
dispersed phase and dispersion medium greater is the ||
intensity of scattered light. This difference is small in CH 3 (CH 2 )16 ——— C — O –
Hydrocarbon part Polar part
lyophilic sols, therefore, they show weak Tyndall (Hydrophobic) (Hydrophilic)

effect while strong Tyndall effect is shown by


lyophilic colloids.
64

Hydrocarbon part (C 17 H 35 ) forms tail and polar part RNH 3Cl consists of aggregates of RNH +3 cations,
(—COO – ) forms head of the anion. hence, it is positively charged.
Hydrophobic and hydrophilic parts of anion arrange Q.26 Explain the terms with suitable examples :
(i) Alcosol,
themselves according to figure forming a micelle.
(ii) Aerosol,
One micelle of sodium stearate consists of about 70 (iii) Hydrosol.
ions. Ans. On the basis of nature of dispersion medium,
colloidal solutions (systems) are of following types:
(i) Alcosol: In these colloidal solutions, alcohol is
the dispersion medium, e. g ., cellulose nitrate in
ethyl alcohol.
(ii) Aerosol: In these colloidal systems, air is the
dispersion medium, e. g ., dust, smoke etc.
(iii) Hydrosol: In these systems, water is the
dispersion medium, e. g ., starch in water.
Q.27 Comment on the statement that “Colloid is not a
Micelle structure substance but a state of substance”.
The outer surface of micelle obtained from soap Ans. The given statement is true. It is because colloidal is
consists of negative part, thus, it is negatively not the property of a substance. Colloidal is a state in
charged. Due to presence of similar charge, these which all the substances can be brought by suitable
micelles are stable. In the similar manner, micelle of methods. If the size of the particle lies in the range
1 nm to 1000 nm, it is in the colloidal state.

vvv
65

Chapter
PRINCIPLES AND PROCESSES OF
EXTRACTION OF ELEMENTS
(This Chapter has been Deleted from Syllabus for
Session 2020-21 Due to COVID-19 Epidemic)

Chapter

7 p-BLOCK ELEMENTS
(Group 15, 16, 17 and 18)
Syllabus : Elements of group-15 : General introduction, Electronic configuration, Occurrence, Oxidation states, V ariation in physical and chemical
properties, Preparation, Properties and uses of nitrogen, Compounds of nitrogen : Methods of preparation, Properties and uses of NH 3, HNO3, Oxides of
nitrogen (only structure), *Allotropes of phosphorus, Compounds of phosphorus : Phosphine, phosphorus halides (PCl3 and PCl5),
their preparation and properties, oxyacids of phosphorus (only elementary idea)
Elements of gp. 16 : General Introduction, Electronic configurations, oxidation states, Occurrence, variation in physical and chemical properties, Di-
oxygen: Preparation, properties and uses, classification of oxides, Ozone, Allotropes of sulphur, Compounds of sulphur: Methods of preparation, properties
and uses of SO2,H2S, *sulphuric acid: Industrial methods of preparation, properties and uses, Oxyacids of sulphur (only structure).
Elements of group-17 : General introduction, Electronic configuration, Oxidation state, Occurrence, Variation in physical and chemical properties,
Compounds of halogens : Methods of preparation, Properties and uses of Chlorine and HCl, Interhalogen compounds, Oxyacids of halogens (only
structure).
Elements of group 18 : General introduction, electronic configuration, occurrence, variation in physical and chemical properties, uses.

* Topics Deleted for Examination 2021

Objective Questions

1. Most stable hydride is : (U.P. 2017) 5. Most abundant inert gas in atmosphere is :
(a) NH 3 (b) PH 3 (c) AsH 3 (d) BiH 3 (U.P. 2017)
2. In XeF4 , hybridization of Xe atom is: (U.P. 2018) (a) He (b) Ne (c) Ar (d) Kr
6. Which inert gas is most reactive ?
(a) sp 3 (b) sp 3 d (c) sp 3 d 2 (d) dsp 2
(a) Ne (b) Xe (c) He (d) Ar
3. Which of the following liberates phosphine with 7. The strongest oxyacid of chlorine is : (U.P. 2016)
phosphorus ? (U.P. 2017) (a) HClO 2 (b) HClO 4 (c) HOCl (d) HClO 3
(a) HCl (b) NaOH (c) CO 2 (d) Na 2CO 3 8. The order of reactivity towards hydrogen by the
4. Correct order of acidic strength is : (U.P. 2015) elements of group 15 is : (U.P. 2019, 20)
(a) HClO4 < HClO3 < HClO2 < HClO (a) N > P > As > Sb > Bi (b) P > As > N > Sb > Bi
(b) HClO < HClO2 < HClO3 < HClO4 (c) Bi > Sb > As > P > N (d) Bi > As > Sb > N > P
(c) HClO < HClO4 < HClO3 < HClO2 9. The oxidation number of fluorine is : (U.P. 2020)
(d) HClO4 < HClO2 < HClO3 < HClO (a) + 1 (b) -1 (c) -2 (d) +2

Answers
1. (a) 2. (c) 3. (b) 4. (b) 5. (c) 6. (b) 7. (b) 8. (a) 9. (b)
66

Very Short Answer Type Questions


Q.1 PCl 5 is known but NCl 5 is not? (U.P. 2014) Ans. Structure of H 2SO 4 is as given O
Or Phosphorus forms PF5 while nitrogen does not adjacent:
form NF5 . Explain. (U.P. 2018) It contains two —OH groups where S
Ans. In the outermost shell of N, empty d-subshells are H-atom is acidic i. e., why H 2SO 4 is HO OH
absent while they are present in PCl 5 . In P, s-electrons dibasic acid. O
excite to d-orbitals so it shows pentavalency. Q.11 Write two oxidising and two reducing properties of
Q.2 Write one reaction of ozone in which it acts both as SO 2 . Write equations only. (U.P. 2018)
oxidant as well as reductant. (U.P. 2017, 18) Ans. A. It oxidises
Ans. H 2O 2 + O 3 ¾® H 2O + 2O 2 (i) H 2S into S
BaO 2 + O 3 ¾® BaO + 2O 2 SO 2 + 2H 2S ¾® 2H 2O + 3S
Q.3 What is major source of helium? (U.P. 2020) (ii) Mg into MgO
Ans. Clevite mineral SO 2 + 2Mg ¾® 2MgO + S
Q.4 HF is liquid while other hydrogen halides (HCl, HBr B. It reduces
and HI) are gases why ? (U.P. 2014, 18) (i) Fe 3+ into Fe 2+
Or 2Fe 3+ + SO 2 + 2H 2O ¾® 2Fe 2+ + SO 24- + 4H +
Explain HF is a liquid, while HCl is a gas. (U.P. 2018) (ii) MnO -4 into Mn 2+
Ans. Due to hydrogen bonding present between HF
5SO 2 + 2MnO -4 + 2H 2O ¾® 5SO 24- + 2Mn 2+ + 4H +
molecules, HF molecules lie closer to each other.Thus,
HF is liquid. On the other hand, hydrogen bonding is Q.12 H 2SO 4 is an oxidant and dehydrating agent explain
absent in HCl, HBr and HI. Thus, they are gases. with one example of each. (U.P. 2010, 18)
Q.5 Giving conditions of pressure and temperature, write Ans. (i) Dehydration of formic acid :
conc. H SO / D
the chemical equation to prepare ClF3 . (U.P. 2017) HCOOH ¾¾¾¾¾® 2 4
H 2O + CO
Ans. Cl 2 + 3F2 ¾ ¾Ni ¾-tube
¾¾ ¾® 2ClF3 (ii) Oxidation of C into CO 2 :
Excess 300 °C High P
D
Q.6 It is very difficult to study chemistry of radon, why ? C + 2H 2SO 4 ¾® CO 2­ + 2H 2O + 2SO 2­
(U.P. 2017) Q.13 Write the inert gas obtained by disintegration of
Ans. Rn is radioactive with very short half life. 226
88 Ra. Write down its one use. (U.P. 2018)
Q.7 Helium and neon do not react with fluorine like
xenon, why? (U.P. 2017) Ans. He and Rn are obtained.
226 222 4
Ans. Electronic configurations of He and Ne are 1s 2 and 88 Ra ¾® 86 Rn + 2 He
1s 2 , 2s 2 2 p 6 respectively. They do not have unpaired Rn is used in radiotherapy while He is used in
electrons and vacant d-subshells. They, also have high creating inert atmosphere.
ionization energies and can not be excited to higher Q.14 Write the chemical equation used in method of
energy levels. preparation of XeF4 . (U.P. 2018)
Q.8 Among noble gases, maximum compounds of xenon 847 K , 7 bar P
Ans. Xe( g ) + 2F2( g ) ¾¾¾¾¾® XeF4
are known, why? (U.P. 2017) (1 : 5 ratio)
Ans. In all the noble gases, xenon has lowest ionization Q.15 Write two important characterstics of inert gases.
energy. It goes in excited state on supplying relatively (U.P. 2020)
less amount of energy. In excited state, it has Ans. (i) They are chemically inert.
unpaired electrons and forms compounds with other (ii) They are colourless, tasteless and
elements like fluorine and oxygen. non-inflammable.
Q.9 Why is water not added to H 2SO 4 for making dilute (iii) They are monoatomic and have high ionization
solution? (U.P. 2018) potential.
Ans. Due to extensive H-bonding, interaction between Q.16 Write down the main uses of Neon. (U.P. 2020)
H 2SO 4 and H 2O is highly exothermic. This heat is Ans. Neon : (i) On passing electric discharge through
sufficient enough to convert some water into steam. neon tube at low pressure, a red luminescence is
So to avoid bumping of solution it is not added in produced. For this property it is used in neon bulbs,
water. tubes etc. Since neon lights are visible from long
Q.10 Sulphuric acid is dibasic acid. Discuss this statement. distances and even visible through fog and mist, they
(U.P. 2018) are used for advertisement purpose and as signals on
airstrips for aircraft landing.
(ii) Neon bulbs are also used for decorating purposes.
67

Short Answer Type Questions


Q.1 Discuss the bleaching action of H 2O 2 and SO 2 . Brodie’s ozoniser : This ozoniser is also used in
(U.P. 2013) laboratory preparation of ozone. It is shown in figure.
Ans. Bleaching action of H 2O 2 is due to oxidising nature
(permanent).
H 2O 2 ¾® H 2O + [O]
Coloured material + [O] ¾® colourless material
Bleaching action of SO 2 is due to reducing nature
(temporary).
SO 2 + 2H 2O ¾® H 2SO 4 + 2[ H]
Coloured material + [ H] ¾® colourless material
Q.2 Discuss the laboratory method of preparation of ozone
giving suitable diagrams. Write the equations also.
Write the chemical test of ozone also.
(U.P. 2013, 18)
Or Describe with the help of diagram the manufacture of
ozone by Siemens and Halske ozonizer. Write down
the chemical equation of its reactions with potassium
ferrocyanide and stannous chloride. (U.P. 2016)
Or Discuss the industrial method of preparation of O 3
with diagram. How is pure ozone prepared. Write one
reaction in which ozone is reductant and is itself
reduced also. (U.P. 2011, 13, 17)
Or Draw labelled diagram of Brodie ozonizer.
(U.P. 2018) O2 gas is fed into a 'U' shaped tube. One limb of
Or How will you obtain pure ozone? write its reactions U-tube is narrow while other limb is broad. The broad
with SnCl 2 and KI solutions. (U.P. 2014) limb contains a test tube filled with dilute H2 SO4 . The
Ans. Laboratory method of Preparation of Ozone: U-tube placed in another vessel containing dilute
Siemen’s ozoniser : This ozoniser is used in H2 SO4 . Copper or platinum electrodes are dipped in
laboratory preparation of ozone. It consists of two both, test tube and vessel containing dilute H2 SO4 .
concentric tubes sealed together at one end. The Both electrodes are connected to an induction coil.
inner surface of inner tube and outer surface of outer O2 gas is passed slowly through the annular space at
tube are coated with tin foil which act as electrodes. low temperature and subjected to silent electric
These thin tin foils are connected with two terminals discharge. O2 gas is partially converted into ozone.
of an induction coil and a silent electric discharge is The ozonised oxygen thus obtained is found to
passed through them. contain about 10% ozone.
Tests of Ozone :
1. A paper dipped into starch-iodine turns blue in
contact with ozone.
2. The paper dipped in tetra-methyl base turns violet in
contact with ozone.
3. Benzidine paper turns brown in contact with ozone.
4. It oxidises silver into black silver oxide (Ag 2O).
5. It destroys fluidity of mercury. Mercury starts
sticking onto glass or paper in presence of ozone.
Oxygen gas at low temperature is passed slowly Reactions (oxidising properties) :
through the concentric tubes.When the silent electric (i) Reaction with stannous chloride :
discharge is passed, oxygen is partially converted into It oxidizes stannous chloride into stannic chloride
ozone. Ozonised oxygen is formed that contains 3SnCl 2 + 6HCl + O 3 ¾® 3SnCl 4 + 3H 2O
about 10% ozone by volume. The percentage can be (ii) Reaction with KI.
increased upto 15% if temperature is lowered to 0°C. It oxidises KI into iodine
Silent electric 2KI + H 2O + O 3 ¾® 2KOH + I 2 + O 2
3O2 ¾¾¾¾®
discharge
2O3 Purification of O 3 : Impure O 3 gas is passed
through a spiral coil cooled by liquid air ozone
or
68

condensed at -112.4º C into blue liquid and O 2 Ans. Electrolysis of fused sodium chloride yields sodium
remains in gaseous state. Liquid ozone is fractionally metal at cathode and Cl 2 gas at anode. This method is
distilled to get pure ozone. used in the extraction of sodium metal and Cl 2 is
Reactions: obtained as by product. The apparatus used is shown
(i) Reaction with potassium ferrocyanide : It in figure.
oxidises potassium ferrocyanide solution into
ferricyanide.
2K 4 [Fe(CN) 6 ] + H 2O + O 3 ¾® 2K 3[Fe(CN) 6 ] + O 2
+ 2KOH
(ii) Reaction with stannous chloride : It
oxidises acidic SnCl 2 into SnCl 4 .
3SnCl 2 + O 3 + 6HCl ¾® 3SnCl 4 + 3H 2O
Stannous stannic
chloride chloride

(iii) Reaction with KI :


KI + 3O 3 ¾® KIO 3 + 3O 2­
Q.3 Write the chemical reaction for the preparation of
hydrochloric acid. Also write its chemical reaction
with (U.P. 2017, 18)
(i) KMnO 4 (ii) Pb( NO 3 ) 2 (iii) Lead acetate The main components of apparatus used in Down’s
Ans. Laboratory method : In the laboratory, HCl gas is process are as follows :
prepared by action of conc. H 2SO 4 on sodium Cathode Anode Electrolyte Temperature
chloride (common salt). Steel rod Graphite Mixture of NaCl 600°C
D
NaCl + H 2SO 4 ¾® NaHSO 4 + HCl CaCl 2 and NaF etc.
D CaCl 2 and NaF are added to electrolyte in order to
NaHSO 4 + NaCl ¾® Na 2SO 4 + HCl lower the melting point of NaCl (M.pt. 800°C).
2NaCl + H 2SO 4 ¾® Na 2SO 4 + 2HCl Following reactions occur at electrodes :
On heating, HCl gas is produced and is passed in conc. At cathode : Na + + e - ¾® Na
H 2SO 4 as shown in figure. Conc. H 2SO 4 absorbs At anode : 2Cl - ¾® Cl 2 + 2e -
moisture, from where dry HCl is escaped which is Cl 2 evolved at anode is collected.
collected in a flask. Q.5 Write short note on compounds of xenon.
(U.P. 2017)
Ans. Then Bartlett prepared first yellow-orange coloured
crystalline compound, [Xe + ][PtF6 ]- .
Xe + PtF6 ¾® [Xe+ ][PtF6 ]-
Later, various compounds of noble gases with highly
electronegative elements (such as F, O) were
prepared, such as, XeF2 , XeF4 , XeF6 etc, fluorides,
XeO 3 , XeOF2 , XeO 2 F2 , XeOF4 etc. oxy fluorides.
These compounds are stable. Structure of XeF2 (Xe :
sp 3 d hybridization) is linear, XeF4 (Xe : sp 3 d 2
hybridization) has square planar structure, XeF6 (Xe :
sp 3 d 3 hybridization) has pentagonal pyramidal
structure. Similarly, XeOF4 (Xe : sp 3 d 2 hybridization)
has square pyramidal structure, XeOF2 (Xe : sp 3 d
hybridization) has T-shaped structure, XeO3 (Xe :
HCl gas dissolves in water and its aqueous solution is
called hydrochloric acid. sp 3 hybridization) has trigonal pyramidal structure,
Chemical reactions : (i) XeO2 F2 (Xe : sp 3 d 2 hybridization) has trigonal
2KMnO 4 + 16HCl ¾® 2KCl + 2MnCl 2 + 8H 2O + 5Cl 2 bipyramidal structure.
(ii) Pb( NO 3 ) 2 + 2HCl ¾® PbCl 2 + 2HNO 3 Q.6 Write down the chemical equation of the laboratory
(iii) (CH 3COO) 2 Pb + 2HCl ¾® PbCl 2 + 2CH 3COOH method of preparation of ammonia. Write the
Q.4 Giving suitable diagram, explain the electrolytic reaction with CuO. (U.P. 2016)
method of manufacturing of chlorine. (U.P. 2017) Or
69

Describe laboratory method of preparation of Ans.


ammonia. What is the reaction of ammonia on CuSO 4
solution? Explain with chemical equation. SO2 Cl 2
(U.P. 2018) 1. Bleaching action of SO 2 is due Bleaching action of Cl 2 is due
Ans. Laboratory Preparation : Principle : In the to reduction. to oxidation.
laboratory, ammonia is prepared by heating SO 2 + 2H2O ® 2H2SO 4 + 2[H] Cl 2 + H2O ¾® 2HCl + [O]
ammonium chloride (NH 4 Cl) with slaked lime. Coloured Substance + [H] ® Coloured substance + [O] ®
2NH 4 Cl + Ca(OH) 2 ¾® 2NH 3 + CaCl 2 + 2H 2O colourless colourless
Ammonia evolved is directly passed over quicklime
and collected in gas jars by downward displacement 2. Its bleaching is temporary. Its bleaching is permanent.
of air. 3. SO 2 is weak bleaching agent. Cl 2 is strong bleaching agent.
2NH 4 Cl + Ca(OH) 2 ¾® 2NH 3 + CaCl 2 + 2H 2O
4. It bleaching delicate It bleaches vegetable colour,
substances like flowers, wool, paper and cotton etc.
silk and straw.

Q.9 Why noble gases are chemically inert ? Explain on the


basis of their electronic configuration. (U.P. 2015)
Ans. Due to complete electronic configuration of their
valence shell, they are stable and inert in nature.
He (2) : 1s 2
Ne (10) : 1s 2 , 2s 2 2 p 6
Ar (18) : 1s 2 , 2s 2 2 p 6 , 3s 2 3 p 6
Kr (36) : 1s 2 , 2s 2 2 p 6 , 3s 2 3 p 6 3d10 , 4s 2 4 p 6
Xe (54) : 1s 2 , 2s 2 2 p 6 , 3s 2 3 p 6 3d10 , 4s 2 4 p 6 4d10 ,
5s 2 5 p 6
Q.10 Explain the position of noble gases in periodic table.
(U.P. 2015)
Reaction with hot CuO : Nitrogen gas is obtained. Ans. Noble gases : Helium (He), Neon (Ne), Argon (Ar),
3CuO + 2NH 3 ¾® 3Cu + N 2 + 3H 2O Krypton (Kr), Xenon (Xe) and Radon (Rn) are called
Reaction with CuSO 4( aq. ) : inert gases due to chemical inactive but now a days
NH 3 + H 2O ¾® NH 4 OH some compounds of these gases are prepared,
CuSO 4 + 2NH 4 OH ¾® Cu(OH) 2 + ( NH 4 ) 2 SO 4 therefore these gases are also called noble gases.
Cu(OH) 2 + ( NH 4 ) 2 SO 4 + 2NH 4 OH ¾® The noble gases are placed in group of 18 of the
[Cu( NH 3 ) 4 ]SO 4 + 4H 2O
periodic table on the basis of electronic configuration
Q.7 Write notes on the following :
(i) Allotropic forms of Sulphur (U.P. 2016) as given below:
(ii) Oxidising nature of H 2SO 4 (U.P. 2016) He (2) : 1s 2
Ans. (i) Allotropic forms of Sulphur : Ne (10) : 1s 2 , 2s 2 2 p 6
Sulphur exists in many allotropic forms such as : Ar (18) : 1s 2 , 2s 2 2 p 6 , 3s 2 3 p 6
(i) Rhombic or a-Sulphur Kr (36) : 1s 2 , 2s 2 2 p 6 , 3s 2 3 p 6 3d10 , 4s 2 4 p 6
(ii) Monoclinic or b-Sulphur
(iii) Plastic or g-Sulphur Xe(54) : 1s 2 , 2s 2 2 p 6 , 3s 2 3 p 6 3d10 , 4s 2 4 p 6 4d10 ,
(iv) Colloidal or d-Sulphur 5s 2 5 p 6
(v) l-Sulphur etc. Rn(86):1s 2 , 2s 2 2 p 6 , 3s 2 3 p 6 3d10 , 4s 2 4 p 6 4d10 4 f 14 ,
(ii) Oxidising nature of H 2SO 4 :
5s 2 5 p 6 5d10 , 6s 2 6 p 6
On heating, it decomposes to give nascent oxygen
which acts as an oxidising agent From electronic configurations it is clear that all
H 2SO 4 ¾® SO 2 + H 2O + [O] noble gases except He have electronic configuration
For example, it oxidised C to CO 2 , ns 2 np 6 showing completeness of outer valence shell.
C + 2H 2SO 4 ¾® CO 2 + 2SO 2 + 2H 2O In helium, only 1s 2 electrons are present but being
Q.8 Write the difference in bleaching action of SO 2 and first orbit it is also complete. Thus, all the noble gases
Cl 2 . (U.P. 2013, 17) have similar electronic configuration. Due to the
similar electronic configuration their physical and
70

chemical properties are also similar which justified presence of vacant d-orbitals, other elements of the
the position of noble gases to put in the same group, group can expand their octet and their covalence
i. e., zero group. exceeds 4.
Q.11 Write one important use of Helium and Argon. 6. Multiple bonds : Oxygen forms pp-pp bonds.
(U.P. 2014) Other elements of this group do not show this
Or Write four important uses of noble gas, helium. tendency. Other elements of this group form pp-dp
(U.P. 2020) bonds.
Ans. Uses of Helium : (a) This is second lightest gas 7. O 2 is paramagnetic whereas S 8 is diamagnetic.
after hydrogen. Being non-inflammable, its mixture Q.14 Explain the reactivity of elements of group 15 of the
with hydrogen (85% He and 15% H2 ) is used to periodic table towards hydrogen. (U.P. 2019)
inflate balloons and tyres of aircrafts. Ans. Reactivity of group 15 elements towards hydrogen :
(b) Mixture of O2 and He is used by divers in (i) All the elements of this group form covalent
respiration. hydrides of MH3 type.
(c) It liquefies at very low temperature (–269°C), NH3 PH3 AsH3 SbH3 BiH3
therefore, it is used to produce low temperature. Ammonia Phosphine Arsine Stibene Bismuthine
(d) After loss of two electrons from He-atom, (ii) Thermal stability decreases from NH3 to BiH3 .
a - particle (He-nucleus) is obtained which is used in The decomposition temperature decreases from NH3
artificial radioactive disintegration. to BiH3 .
(e) Mixture of He and O2 is used to assist breathing in Hydride : NH3 PH3 AsH3 SbH3 BiH3
asthama and other respiratory diseases.
(f) It is used in producing inert atmosphere. Decomposition 1300 440 280 150 Room
(g) It is used for food preservation. temperature : ºC ºC ºC ºC temperature
Uses of Argon : (a) Argon-oxygen (15%) mixture is (iii) NH3 is a stable hydride which does not show
used for producing inert atmosphere in electric bulbs. reducing nature. PH3 is less stable than NH3 and is a
It helps in increasing the life of filament. reducing agent. Reducing nature of hydrides of this
(b) It is used in producing inert atmosphere during group increases from top to bottom. Thus, NH3 is a
welding and extraction of various metals. very weak reducing agent while BiH3 is a strong
(c) It is also used for filling radio-valves. reducing agent. On passing PH3 through aqueous
Q.12 Write the structures of fluorides of xenon. solutions of AgNO3 and CuSO4 , precipitates of silver
(U.P. 2018) phosphide and cupric phosphide are obtained which
Ans. are reduced into Ag and Cu.
F
F 6AgNO3 + 2PH3 ¾® 2Ag 3 P + 6HNO3
F F F F
Xe Xe Xe 2Ag 3 P + 3H2 O ¾® 6Ag + H3 PO3 + PH3
¾¾¾¾¾¾¾¾¾¾¾¾¾¾¾¾¾¾¾¾¾¾¾¾¾¾¾¾
F F F F 6AgNO3 + 3H2 O + PH3 ¾® 6Ag + 6HNO3 + H3 PO3
F F ¾¾¾¾¾¾¾¾¾¾¾¾¾¾¾¾¾¾¾¾¾¾¾¾¾¾¾¾
XeF2 XeF4 XeF6 3CuSO4 + 3H2 O + PH3 ¾® 3Cu + 3H2 SO4 + H3 PO3
(Linear) (Square planar) (Distorted octahedral) Q.15 Write the cause of inertness of noble gases and write
Q.13 Explain with reason the anomalous behaviour of the the use of the noble gas which is present in the mineral
elements of oxygen family of the periodic table. cleveite. (U.P. 2019)
(U.P. 2019) Or Why are the ionization potentials of noble gases high?
Ans. Anomalous Behaviour of Oxygen : Explain. (U.P. 2020)
Oxygen differs from other elements of the group. It is Ans. Noble gases are chemically inert because their octet
due to : (i) its small size, (ii) its high ionisation of valence shell is complete and that is why they have
enthalpy, (iii) its high electronegativity, (iv) absence no tendency to either accept or loose electrons. and
of vacant d-orbitals. that is why their ionization potential is very high.
1. Physical state : Dioxygen is a gas. The other In mineral cleveite, (impure radioactive variety of
elements of the group are solids. uraninite), noble gas helium (He) is found. Its uses
2. Atomicity : Oxygen is diatomic (O 2 ) while other are :
elements of this group are octaatomic. (a) production of inert atmosphere
3. Hydrides : H 2O is liquid whereas H 2S is gas. H 2O (b) filling of air balloons.
Q.16 Write down short notes on the allotropes of sulphur.
is more stable than H 2S.
(U.P. 2019)
4. Hydrogen bond : H 2O is associated through
Ans. Sulphur :
intermolecular hydrogen bonding and is liquid. Due 1. Rhombic or octahedral (a) sulphur
to the absence of intermolecular hydrogen bonds, 2. Monoclinic or prismatic (b) sulphur
H 2S is gas. 3. Plastic sulphur 4. Milk of sulphur
5. Valence : Oxygen shows covalence of 2 and rarely 5. Monoclinic sulphur
exceeds 2 to show the covalence of 4. Due to the 6. Engel’s sulphur
71

Long Answer Type Questions


Q.1 Describe Birkeland-Eyde method for the manufacture delivery tube. Conc. HCl is added by thistle funnel
of nitric acid with diagram. Write the reaction of Sn whose lower end of remains dipped in HCl. The
with conc. HNO 3 . (U.P. 2018) contents are heated gently, yellowish-green Cl 2 gas is
Or Describe the manufacture of nitric acid by Arc method
with diagram. Write chemical equations for the
reactions which take place on heating of tin metal and
iodine with concentrated nitric acid. (U.P. 2019)
Ans. Birkeland-Eyde or Arc proess: Principle : In
this process, nitric acid is prepared by using N2 and
O2 present in the air according to following reaction.
N2 + O2 º2NO ; DH = 43.2kcal

evolved which is contaminated with water vapours


and HCl gas. The gas evolved are first passed through
water to remove HCl gas and then through conc.
This reaction is reversible and endothermic. So, the H2 SO4 to remove water vapours. The dry and pure
temperature, is maintained at about 3000ºC by using Cl 2 gas is finally collected in a jar by upward
an electric arc. displacement of air.
2NO + O2 ¾® 2NO2 Precautions : (a) Round bottom flask should be
2NO2 + H2 O ¾® 2HNO3 + NO heated slowly otherwise explosion may occur.
Reaction with Sn: Sn is oxidised into metastannic (b) The lower end of thistle funnel must remain
acid. dipped in HCl otherwise the gas formed will enter
3Sn + 4HNO 3 + H 2O ¾® 3H 2SnO 3 + 4NO thistle funnel.
( Conc.) 2. Laboratory method : When KMnO4 or
Note: Owing to high production cost, this process is
K 2 Cr2 O7 is treated with conc. HCl then HCl is
not used now.
oxidised to give Cl 2 .
(i) Reaction with tin metal :
2KMnO4 + 6HCl ¾® 2KCl + 2MnCl 2 + 3H2 O + 5[O]
Sn + 4HNO 3 ¾® 4NO 2 + H 2O + H 2SnO 3
Conc. Nitric oxide Metastannic acid [2HCl + [O] ¾® H2 O + Cl 2 ] ´ 5
(ii) Reaction with I 2 : 2KMnO4 + 16HCl ¾® 2KCl + 2MnCl 2 + 8H2 O + 5Cl 2­
I 2 + 10HNO 3 ¾® 10NO 2 + 4H 2O + 2HIO 3 K 2Cr2O7 + 8HCl ¾® 2KCl + 2CrCl 3 + 4H 2O + 3 (O)
Iodic acid
( 2HCl + O ¾® H 2O + Cl 2 ) ´ 3
Q.2 Explain the laboratory method of preparation of
chlorine giving suitable diagram. Write the related K 2Cr2O7 + 14HCl ¾® 2KCl + 2CrCl 3 + 7 H 2O + 3 Cl 2­
equations also. Also give two chemical properties.
(U.P. 2017)
Or Write down the chemical equation for the preparation
of chlorine. Write down the chemical equation of the
reaction of chlorine on dry slaked lime. Also write the
name of main product in the reaction. (U.P. 2019)
Ans. 1. Laboratory method : Cl 2 gas is obtained by
heating manganese dioxide with conc. HCl.
MnO2 + 2HCl ¾® MnCl 2 + H2 O + [O]
2HCl + O ¾® H 2O + Cl 2
MnO2 + 4HCl ¾® MnCl 2 + 2H2 O + Cl 2
Apparatus : Small quantity of MnO2 is taken in a
round bottom flask fitted with a thistle funnel and a
72

Precautions : (a) HCl is added slowly otherwise White precipitate of AgCl dissolves in excess of NH 3
explosion may occur. to form diammine silver (I) chloride.
(b) A brisk reaction occurs even at normal AgCl + 2NH 3 ¾® [Ag(NH 3 ) 2 ] Cl
temperature thus, reagents should not be heated. (ii) Reaction with Cl 2 :
(i) Reaction with water : (i) In excess of ammonia, nitrogen gas is obtained.
Cl 2 + H 2O ¾® HCl + HOCl 8NH 3 + 3Cl 2 ¾® 6NH 4 Cl + N 2
(ii) Reaction with sodium :
(ii) In excess of Cl 2 , nitrogen trichloride (NCl 3 ) is
2Na + Cl 2 ¾® 2NaCl obtained.
Decon’s process:
NH 3 + 3Cl 2 ¾® NCl 3 + 3HCl
4HCl + O 2 ¾ CuCl
¾¾2 /¾
450°C
¾® 2H 2O + 2Cl 2
(iii) Reaction with CuO : Nitrogen gas is
Reaction of chlorine on dry slaked lime: obtained.
Cl 2 + Ca(OH) 2 ¾® CaOCl 2 + H 2O
Bleaching powder
3CuO + 2NH 3 ¾® N 2 + 3Cu + 3H 2O
Q.4 Describe Ostwald’s process for manufacturing of nitric
Q.3 Describe with diagram the industrial method of
acid giving labelled diagram. Give one equation which
manufacture of ammonia by Haber process. What are
shows its oxidising nature. What is fuming nitric acid ?
the necessary conditions to obtain maximum yield of
ammonia ? Give its reaction on silver chloride and Cl 2 (U.P. 2011, 13, 14, 15, 17)
also. (U.P. 2010, 14, 15) Or Explain one industrial method for manufacturing of
Or Write down Haber process for the industrial nitric acid. Give one equation which shows its
manufacture of Ammonia. Mention the conditions oxidising nature.
required to maximise the yield of Ammonia. Give also Or Explain the preparation method of HNO 3 with
the labelled diagram in the industrial manufacture of diagram. How it reacts with (i) carbon, (ii) H 2S and
Ammonia. (U.P. 2019) (iii) iodine? Give also the equation of related
Or Explain Haber process for manufacture of ammonia reactions. (U.P. 2019)
with the help of flow chart and chemical equations. Or Describe the manufacture of Nitric acid by Ostwald
Write two important uses of ammonia. (U.P. 2020) method with labelled diagram giving chemical
Ans. Manufacturing of Ammonia by Haber’s equations. Give chemical equations for reaction of hot
Process : Principle : When a mixture of nitrogen and concentrated nitric acid with iodine and cold and
and hydrogen in the ratio of 1:3 is heated at dilute nitric acid with Zn. (U.P. 2019)
450 – 500º C under high pressure 200 atmosphere in Or Describe the manufacture of HNO3 by Ostwald's
the presence of Fe as catalyst and molybdenum (Mo) method giving chemical reactions and write chemical
as promoter, ammonia is obtained. equation for the reaction of HNO3 with the following :
N 2 + 3H 2 v 2NH 3 + 22400 cal. (i) Reaction of copper with hot and conc. HNO3
Labelled diagram : (ii) Reaction of phosphorus with conc. HNO3 .
H2 + N2 N2 + H2 (U.P. 2020)
Ans. Industrial preparation of nitric acid by
Ostwald’s process : Principle : For the
H2 + N 2

N2 + H2

manufacture of nitric acid, mixture of NH 3 and air in


the ratio of 1:8 is passed over platinum gauze at
H2+N2+NH3
Ammonia
converter

N2 H2
800º C, when 90% of ammonia gets oxidised to nitric
Compressor oxide.
100-1000
atm
4NH 3 + 5O 2 ¾ Pt ¾gauze
¾¾® 4NO + 6H 2O
800ºC

Nitric oxide is oxidised into NO 2 which is absorbed in


water to form nitric acid
Liquid Recirculating (i) 2NO + O 2 ¾® 2NO 2
ammonia pump nitrogen peroxide
Heber’s process for the manufacture of ammonia
(ii) 3NO 2 + H 2O ¾® 2HNO 3 + NO
Conditions for the Maximum yield of
Ammonia. Method and labelled diagram : The mixture of
200 atm, 450-500°C NH 3 and air in the ratio of 1:8 is passed over platinum
N 2 + 3H 2 2NH 3 + 22400 cal gauze at 800º C when 90% of ammonia gets oxidised
the reaction is reversible, exothermic and to nitric oxide (NO).
proceeds with a decrease in volume. According to Now the mixture of gases are passed into oxidising
Le-chatelier principle, the favourable conditions tower. Here, nitric oxide is oxidised into NO 2 by O 2 .
are low temperature and high pressure. The gaseous mixture thus obtained is passed into
Reactions: absorption tower where water is sprayed the tower,
(i) Reaction with silver chloride :
73

NO 2 is absorbed in water and dilute solution of HNO 3 oxidation of HCl gas with air at 450º C in the presence
is obtained. of cupric chloride catalyst.
4HCl + O 2 ¾ CuCl
¾¾ 2 ® 2H O + 2Cl ­
2 2
450ºC

Methods and labelled diagram : The apparatus


used in this process shown in figure. In this method
the mixture of HCl gas and air is heated at 220º C and
then sent to oxidising chamber containing pumce
stone dipped in solution of CuCl 2 in HCl. In this
chamber HCl gas oxidised to Cl 2 gas.
4HCl + O 2 ¾ CuCl
¾¾ 2 ® 2H O + Cl
2 2
450ºC

Oxidising properties : HNO 3 is a strong oxidising


agent. For examples:
(i) It oxidised H 2S to sulphur
H 2S + 2HNO 3 ¾® S + 2NO 2 + 2H 2O
(ii) Iodine is oxidised to iodic acid
I 2 + 10HNO 3 ¾® 2HIO 3 + 10NO 2 + 4H 2
(iii) C is oxidised to CO 2 :
C + 4HNO 3 ¾® 4NO 2 + 2H 2O+CO 2
Deacon process for preparation of Cl2
Fuming nitric acid : Fuming nitric acid is 98%
pure nitric acid. When NO 2 and N 2O 3 gases are The gaseous mixture (Cl 2 , HCl and water vapours) is
dissolved in nitric acid is called fuming nitric acid. It is cooled in cooler and then passed in washing tower
obtained by distilling conc. HNO 3 with starch, HNO 3 and then dried with conc. H 2SO 4 in drying tower and
is reduced by starch into NO 2 which dissolves in the dry mixture of Cl 2 and air is collected.
remaining acid to form fuming nitric acid. By action of HCl on KMnO 4
Reaction with hot conc. HNO 3 and I 2 : 2KMnO 4 + 16HCl ¾® 2KCl + 2MnCl 2 + 8H 2O + 5Cl 2 ­
I 2 + 10HNO 3 ¾® 10NO 2 + 4H 2O +2HIO 3 (a) Reaction with ammonia:
Nitrogen Iodic acid
dioxide (i) In excess of ammonia : N 2 gas is obtained
8NH 3 + 3Cl 2 ¾® 6NH 4 Cl + N 2
Reaction of Zn with cold and dilute HNO 3 : (ii) In excess of Cl 2 : NCl 3 is obtained
4Zn + 10HNO 3 ¾® 4Zn(NO 3 ) 2 + N 2O + 5H 2O NH 3 + 3Cl 2 ¾® NCl 3 + 3HCl
Nitrous oxide
(b) Reaction with SO -2 :
Reaction of Cu with hot/conc. HNO 3 : Nitrogen
dioxide is formed SO 2 + Cl 2 ¾® SO 2Cl 2
Cu + 4HNO 3 ¾® Cu( NO 3 ) 2 + 2NO 2 + 2H 2O (c) Reaction with H 2O :
Nitrogen dioxide Cl 2 + H 2O ¾® HCl + HOCl
Reaction of phosphorus with conc. HNO 3 : Q.6 What are interhalogen compounds ? Explain with
Phosphoric acid is formed examples. Write chemical equation for the formation
P + 5HNO 3 ¾® H 3 PO 4 + 5NO 2 + H 2O of AB 3 type interhalogen between fluoride and
Q.5 Describe with the help of diagram the manufacture of chloride ? (U.P. 2014)
chlorine by Deacon’s process. Write down the Or
chemical equation of its reaction with ammonia, SO 2 Write down the chemical equation for the preparation
and H 2O. (U.P. 2013, 15, 16, 18) of any four interhalogen compounds of chlorine,
Or Write two methods of preparation of chlorine and bromine and iodine with fluorine. (U.P. 2016)
chemical equation of three chemical properties. Or
(U.P. 2020) What are interhalogen compounds ? Explain with
Ans. Preparation of Cl 2 by Deacon’s process : examples. Discuss the geometry of AB 3 type of inter
Principle : Chlorine is manufactured by the halogen compounds of chlorine and fluorine.
(U.P. 2016)
74

Ans. Interhalogen compounds : Due to difference in Q.7 Write the chemical equation for the preparation of
electronegativity, two different halogens combine to SO2 in laboratory and write the chemical equation of
each other to form covalent compounds which are its reaction with acidic potassium permanganate
called interhalogen compounds. These are four types. solution. (U.P. 2020)
AB : ClF, BrF, BrCl, ICl etc. Or Write the method of preparation of sulphur dioxide.
AB 3 : ClF3 , BrF3 , ICl 3 Write chemical equation also.
AB 5 : BrF5 , IF5 Write chemical equations of reaction when sulphur
dioxide reacts with : (U.P. 2020)
AB7 : IF7
Formation of AB 3 type interhalogen (i) Sodium hydroxide solution
Compounds (ii) Chlorine gas
(i) Cl 2 + 3F2 ¾ 300ºC ¾¾® 2ClF3 (iii) Oxygen gas
(in excess) Ans. Preparation of SO 2 in the laboratory: In the
(ii) Br2 + 3F2 ¾® 2BrF3 laboratory SO 2 is prepared by
(in excess) (i) Action of conc. H 2SO 4 on Cu
(iii) I 2 + 3Cl 2 ¾® 2ICl 3 Cu + 2H 2SO 4 ¾® CuSO 4 + SO 2 ­ +2H 2O
Geometry of AB 3 type compounds : The (ii) Action of dil. HCl on Na 2SO 3
structure of AB 3 type interhalogen compounds is very Na 2SO 3 + 2HCl ¾® 2NaCl + H 2O + SO 2 ­
interesting e. g . in ClF3 , the central metal atom Cl in Reaction with acidic potassium
excited state has following configuration: permangnate:
3s 3p 3d 5 SO 2 + 2KMnO 4 + 2H 2O ¾®
K 2SO 4 + 2MnSO 4 + 2H 2SO 4
Potassium sulphate Mangnese sulphate Sulphuric acid
3
Thus, sp d hybridisation is found in AB 3 type Reaction with sodium hydroxide: Na 2SO 3 is
compound. Thus the structure is trigonal bipyramidal formed
in which 2 positions are occupied by lone pairs of SO 2 + 2NaOH ¾® Na 2SO 3 + H 2O
electrons. But due to lone electron repulsion, they are Reaction with O 2 : SO 3 is formed
T-shaped. Pt/D
B 2SO 2 + O 2 2SO 3
B
Reaction with Cl 2 : SO 2Cl 2 is formed
SO 2 + Cl 2 ¾® SO 2Cl 2
A Sulphuryl chloride

B
Structure of AB3 type interhalogen compounds

Question B ased o n Chemical R eactions


Q.1 Mention with the help of chemical equations what (h) NH 4 OH solution is added in ZnCl 2 solution.
happens when : (U.P. 2015)
(a) An excess of NH4 OH is added to solution of (i) NH 4 OH reacts with AgNO 3 . (U.P. 2018)
copper sulphate ? (U.P. 2015) Ans. (a) Complex coppertetraammine is formed.
(b) An excess of NH4 OH is added to precipitate of CuSO 4 + 4NH 4 OH ¾® [Cu( NH 3 ) 4 ]SO 4 + 4H 2O
AgCl ? (U.P. 2015) (b) Soluble complex diaammine silver chloride is
(c) HNO3 is added to aqueous solution of KI ? obtained.
(U.P. 2013) AgCl + 2NH 4 OH ¾® [ Ag( NH 3 ) 2 ]Cl + 2H 2O
(d) Copper metal reacts with dilute and conc. HNO3 ? (c) KI is oxidised to I 2 .
(U.P. 2011, 20) 6KI + 8HNO 3 ¾® 2NO + 6KNO 3 + 4H 2O + 3I 2
(e) Sn is heated with conc. HNO3 ? (d) (i) With dilute HNO 3 , NO is obtained.
(f) Ammonia reacts with Cl 2 or I 2 ? (U.P. 2014) 3Cu + 8HNO 3 ¾® 3Cu( NO 3 ) 2 + 2NO + 4H 2O
(g) At high temperature and pressure, CO 2 reacts (ii) With conc. HNO 3 , NO 2 is obtained.
with liquid NH 3 . (U.P. 2014) Cu + 4HNO 3 ¾® Cu( NO 3 ) 2 + 2NO 2 + 2H 2O
75

(e) NO 2 is obtained. Q.4 How will you obtain following ?


Sn + 4HNO 3 ¾® H 2SnO 3 + 4NO 2 + H 2O (a) Cl 2 from KClO3 (U.P. 2014)
(f) (i) With excess of NH 3 , NH 4 Cl is obtained. (b) H 2SO 4 from S
8NH 3 + 3Cl 2 ¾® 6NH 4 Cl + N 2 Ans. (a) 2KClO 3 + I 2 ¾® 2KIO 3 + Cl 2
(ii) Solid I 2 reacts with NH 3( l ) to form nitrogen
(b) S + O 2 ¾® SO 2
2SO 2 + O 2 s 2SO 3
tri-iodide.
2NH 3 + 3I 2 ¾® NI 3 × NH 3 + 3HI
(g) Urea is formed. SO 3 + H 2SO 4 ¾® H 2S 2O7
200 atm 150°C/ 35 atm
H 2S 2O7 + H 2O ¾® 2H 2SO 4
2NH 3 + CO 2 ¾¾® NH 2COONH 4 ¾¾¾¾® Q.5 What happens when (U.P. 2015)
NH 2CONH 2 + H 2O (i) Ammonium hydroxide is added in precipitate of
(h) Zinc hydroxide is formed. silver chloride. (U.P. 2020)
ZnCl 2 + 2NH 4 OH ¾® Zn(OH) 2 + 2NH 4 Cl (ii) H 2S gas is passed in an acidic solution of KMnO 4 .
(i) 2NH 4 OH + AgNO 3 ¾® [ Ag( NH 3 ) 2 ] NO 3 + 2H 2O
(iii) Phosphine gas is passed in acidic copper sulphate
Q.2 What happens when following reactions occur ?
(a) Ozone is passed into aqueous solution of KI. solution.
(U.P. 2014) Or
(b) PbS reacts with O3 . (U.P. 2011, 20) (i) Iodine reacts with sodium sulphite solution ?
(c) O 3 is passed through KMnO 4 solution. (ii) SO 2 gas is passed in the acidic solution of
(U.P. 2018) K 2Cr2O7 ?
(d) Ozone with stannous chloride. (U.P. 2014, 16) (iii) Excess of potassium iodide solution is added in
(e) SO 2 is passed through ferric chloride solution. excess in copper sulphate soultion.
(U.P. 2014) Ans. (i) Complex compound diammine silver (I) chloride
(f) Reaction of ozone with mercury. (U.P. 2016)
is obtained
Ans. (a) O 3 + 2KI + H 2O ¾® O 2 + I 2 + 2KOH
(b) PbS + 4O 3 ¾® PbSO 4 + 4O 2 AgCl + 2NH 4 OH ¾® [Ag(NH 3 ) 2 ] Cl + 2H 2O
(c) 2K 2 MnO 4 + O 3 + H 2O ¾® 2KMnO 4 + 2KOH (ii) H 2S oxidesed into sulphur
+ O2 2KMnO 4 + 3H 2SO 4 + 5H 2S ¾® K 2SO 4
(d) 3SnCl 2 + 6HCl + O 3 ¾® 3SnCl 4 + 3H 2O + 2MnSO 4 + 8H 2O + 5S
(e) 2FeCl 3 + SO 2 + 2H 2O ® 2FeCl 2 + 2HCl + H 2SO 4 (iii) A black ppt. of copper phosphide is obtained
(f) 2Hg + O 3 ¾® Hg 2O + O 2 CuSO 4 + 2PH 3 ¾® Cu 3 P2 + 3H 2SO 4
Black ppt
Q.3 What happens when :
Or
(a) Cl 2 is passed through hot and conc. NaOH
(i) Iodine reacts with sodium sulphite to form sodium
solution? (U.P. 2013, 20)
sulphate.
(b) Cl 2 is passed through cold and dilute caustic
Na 2SO 3 + H 2O + I 2 ¾® Na 2SO 4 + 2HI
potash solution?
(ii) K 2Cr2O7 oxideses SO 2 as
(c) NH 3 reacts with excess of Cl 2 . (U.P. 2014, 16)
K 2SO 4 + H 2SO 4 + 3SO 2 ¾® K 2SO 4 + Cr2 (SO 4 ) 3
(d) K 2Cr2O7 is heated with sodium chloride and
+ H 2O
conc. H 2SO 4 .
(iii) Cuprous iodide is obtained
(e) KMnO 4 is heated with conc. HCl (U.P. 2015)
CuSO 4 + 2KI ¾® [CuI 2 + K 2SO 4 ] ´ 2
(f) I 2 reacts with hot and conc. NaOH solution.
(U.P. 2015) 2CuI 2 ¾® Cu 2 I 2 + I 2
Ans. (a) Chloride and chlorate are formed. 2CuSO 4 + 4KI ¾® 2Cu 2 I 2 + 2K 2SO 4 + I 2
Q.6 Complete and balance the following reactions :
Cl 2 + 6NaOH ¾® 5NaCl + NaClO 3 + 3H 2O
(U.P. 2019)
(b) Chloride and hypochlorite are formed. (i) NaOH + Br2 ¾ Cold
¾¾®
Cl 2 + 2KOH ¾® KCl + KOCl + H 2O
(ii) XeF6 + H 2O ¾®
(c) NCl 3 is formed. Cold
NH 3 + 3Cl 2 ¾® NCl 3 + 3HCl Ans. (i) 2NaOH + Br2 ¾¾® NaBr + NaOBr + H 2O
Sodium
(d) K 2Cr2O7 + 4NaCl + 3H 2SO 4 ¾® 2CrO 2Cl 2 hypobromite
+ 2Na 2SO 4 + K 2SO 4 + 3H 2O (ii) XeF6 + 3H 2O ¾® XeO 3 + 6HF
Xenon oxide
(e) 2KMnO 4 + 16HCl ¾® 2KCl + 2MnCl 2 + 8H 2O
+ 5Cl 2 Q.7 What happens when: (U.P. 2019)
(i) Ammonium dichromate is heated ?
(f) I 2 + 6NaOH ¾® 5NaI + NaI + 3H 2O
(ii) Sodium azide is heated ?
76

(iii) The aqueous solution of ammonia is added to the (iv) Barium from barium azide
solution of cupric ion ? (U.P. 2020) (v) Nitric acid from ammonia.
(iv) Sulphur is heated with concentrated nitric acid? Ans. (i) Ferric sulphate from ferrous sulphate :
(v) Carbon is heated with sulphuric acid ? 2FeSO 4 + O 3 + H 2SO 4 ¾® O 2 + Fe 2 (SO 4 ) 3
Ans. (i) ammonium dichromate is heated: N 2 is +H 2O
formed. (ii) XeO 3 from XeF4
D 6XeF4 + 12H 2O ¾® 2XeO 3 + 24HF + 4Xe + 3O 2
(NH 4 ) 2 Cr2O7 ¾® N 2 + Cr2O 3 + 4H 2O
(iii) Copper phosphide from copper sulphate:
(ii) sodium azide is heated: N 2 is formed. 3CuSO 4 + 2PH 3 ¾® Cu 3 P2 + 3H 2SO 4
D (iv) Barium from barium azide :
2NaN 3 ¾® 3N 2 + 2Na
(iii) aqueous solution of ammonia is added to D
Ba(N 3 ) 2 ¾® Ba + 3N 2
solution of cupric ion : deep blue complex, copper (v) Nitric acid from ammonia :
tetra ammine is formed. Pt
Cu 2+ + 4NH 4 OH ¾® [Cu(NH 3 ) 4 ]2+ + 4H 2O 4NH 3 + 5O 2 ¾¾® 4NO + 6H 2O;
800ºC
Copper tetra ammine
2NO + O 2 ¾® 2NO 2
(iv) sulphur is heated with conc. nitric acid: NO 2 is 2NO 2 + H 2O ¾® HNO 3 + HNO 2 ;
formed. 3HNO 2 ¾® HNO 3 + 2NO + H 2O
S + 6HNO 3 ¾® 6NO 2 + H 2SO 4 + 2H 2O Q.10 Write chemical equation of the reaction of ozone with
nitrogen
dioxide acidic stannous chloride and mercury. (U.P. 2019)
(v) carbon is heated with sulphuric acid: CO 2 is Ans. Reaction of ozone with stannous chloride :
formed. O 3 + 3SnCl 2 + 6HCl ¾® 3SnCl 4 + 3H 2O
D
C + 2H 2SO 4 (Conc. ) ¾® CO 2 + 2SO 2 + 2H 2O Q.11 What happens when (Write only chemical equations):
(U.P. 2019)
Q.8 What happens when (write chemical equations only): (i) Sodium thiosulphate reacts with iodine.
(U.P. 2019) (ii) Chlorine gas is passed in hot solution of sodium
(i) Sulphur trioxide is passed into conc. H 2SO 4 ? hydroxide.
(ii) Conc. H 2SO 4 is added to calcium fluoride ? (iii) Ammonium chloride is heated with slaked lime.
(iii) Chlorine is heated with fluorine at 437 K ? (U.P. 2020)
(iv) Chlorine is heated with fluorine at 573 K ? Ans. (i) Sodium thiosulphate reacts with iodine :
(v) XeF4 and O 2 F2 react at 143 K ? 2Na 2S 2O 3 + I 2 ¾® Na 2S 4 O 6 + 2NaI
Ans. (i) Sulphur trioxide is passed into Conc. Sodium
H 2SO 4 : Oleum is formed. tetrathionate
H 2SO 4 + SO 3 ¾® H 2S 2O7 (ii) Chlorine gas is passed into hot NaOH
oleum
solution :
(ii) Conc. H 2SO 4 is added to calcium fluoride: Cl 2 + 6NaOH ¾® 5NaCl + NaClO 3 + 3H 2O
D hot
CaF2 + H 2SO 4 ¾® CaSO 4 + 2HF
(iii) Chlorine is heated with fluorine at 473K: (iii) Ammonium chloride is heated with
473K slaked lime :
Cl 2 + F2 ¾¾® 2ClF 2NH 4 Cl + Ca(OH) 2 ¾® 2NH 3 + CaCl 2 + 2H 2O
(iv) Chlorine is heated with fluorine at 573K: Q.12 Write chemical equation of reaction of chlorine with
573K
Cl 2 + 3F2 ¾¾® 2ClF3 ferrous sulphate and terpentine oil. (U.P. 2020)
Ans. (i) Cl 2 oxidises ferrous sulphate into ferric sulphate
(v) XeF4 and O 2 F2 react at 143K : 2FeSO 4 + H 2SO 4 + Cl 2 ¾® Fe 2 (SO 4 ) 3 + 2HCl
143K
XeF4 + O 2 F2 ¾¾® XeF6 + O 2 (ii) Terpentine oil reacts explosively with chlorine
Q.9 How will you obtain the following ? (U.P. 2019) and burns to form carbon and HCl.
(i) Ferric sulphate from ferrous sulphate C 10 H16 ( l) + 8Cl 2 ( g ) ¾® 10C( s) + 16HCl ( g )
Turpentine oil
(ii) XeO 3 from XeF4
(iii) Copper phosphide from copper sulphate

Solution of NCERT Text Book Problems

Q.1 Discuss the general characteristics of group 15 Or Discuss the position of N, P and As in periodic table on
elements with reference to their electronic the basis of electronic configuration.
configuration, oxidation state, atomic size, ionisation (U.P. 2011, 13, 15)
enthalpy and electronegativity.
77

Ans. All the elements of nitrogen family are placed in (iii) Atomic Radii : Atomic radius increases with
group 15 (or VA) in periods second to seventh. increase in atomic number in the group.
(i) Electronic Configuration : The electronic
Element : N P As Sb Bi
configuration of valence shell of all these elements is
ns 2 np 3 where n represents outermost shell. All these Atomic radius (Å) : 0.74 1.10 1.21 1.41 1.52
elements are called p-block elements because (iv) Ionization Energy : (a) Due to increase in
differentiating electron enters p-shell. distance between nucleus and outermost shell, value
7N : [He] 2s 2 2 p 3 of ionization energy decreases from top to bottom.
2 3
15 P : [Ne] 3s 3 p Element : N P As Sb Bi
10 2 3
33 As : [Ar] 3d , 4s 4 p I.E. [kcal/mol] : 336.0 253.9 231.0 199.1 184.9
10 2 3
51 Sb : [Kr] 4d , 5s 5 p
14 (b) First ionization energy of nitrogen is higher than
83 Bi : [Xe] 4 f ,5d ,6s 2 6 p 3
10
those of other elements of this group. This is because
(ii) Valency and Oxidation state: (a) The of small size of N-atom. The difference between
elements of this group have two paired electrons in ionisation energies of N and P is comparatively high
their outer s-subshell and three unpaired electrons in but the difference afterwards between two consecutive
outer p-shell. Thus, all the elements show valency of elements is small. This is due to less shielding effect of
three. d-electrons in As and Sb and f-electrons in Bi.
2s 2p (c) First ionization energy of nitrogen is higher than
7N : that of oxygen (1s 2 , 2s 2 2 p 4 ) because half-filled
3s 3p p-subshell in nitrogen (1s 2 , 2s 2 2 p 3 ) is a stable
configuration. Thus, more energy is required to
15P :
remove electron from 2p-subshell in N-atom.
(b) Except nitrogen, other elements have vacant Similarly, it can be explained that second ionization
d-orbitals in their outer shell. One of s-electrons gets energy of oxygen ( O+ ion : 1s 2 , 2s 2 2 p 3 ) is higher than
excited by absorbing energy and moves to d-subshell. second ionization energy of nitrogen
Now, these elements (except nitrogen) have five ( N+ : 1s 2 , 2s 2 2 p 2 ).
unpaired electrons in valence shell. Thus, they show
pentavalency in excited state. (v) Electronegativity : (a) Electronegativity
decreases from top to bottom in the group.
ns np nd
Ground State : Element : N P As Sb Bi
Electronegativity : 3.0 2.1 2.0 1.9 1.8
ns np nd
(b) Nitrogen has the highest electronegativity in the
Excited State :
group, hence, it has highest tendency to form N3- ion
(c) Due to presence of three unpaired electrons, in the group.
N-atom is capable of forming three covalent bonds. (c) Electronegativity of Bi is lowest in the group, thus,
Simultaneously, it can form a co-ordinate bond by it combines with highly electronegative elements to
using electron pair of 2s-subshell. Thus, maximum form ionic compounds.
valency of nitrogen may be four. Due to absence of Q.2 Why does the reactivity of nitrogen differ from
d-subshell, it does not show pentavalency like other phosphorus or phosphorus is more reactive than
elements of this group. For this reason, phosphorous nitrogen?
forms PCl 3 and PCl 5 while nitrogen forms only NCl 3 . Ans. Bond dissociation energy of N 2 (N ºº N) is very high,
(d) The various oxidation states shown by elements of so it is less reactive. On the other hand, in P4 all P — P
this group are as: bonds are single and bond dissociation energy of
N = -3, - 2, - 1, + 1, + 2, + 3, + 4, + 5 phosphorus is low. so it is more reactive.
P = -3, + 3, + 4, + 5 Q.3 Discuss the trends in chemical reactivity of group 15
As = -3, + 3, + 5 elements.
Sb = -3, + 3, + 5 Ans. (i) Nitrogen is quite inert under ordinary conditions
Bi = -3, + 3 due to strong N ºº N bond between N-atoms.
(e) The tendency to show +3 ionic state increases
from top to bottom in the group. This is due to inert (ii) Phosphorus (especially white phosphorus) is
pair effect, i.e., extremely reactive because P—P bond is not too
As 3+ < Sb 3+ < Bi 3+ strong. White P readily burns in oxygen to form
(f) The tendency to show +5 oxidation state decreases oxides.
from top to bottom in the group, i.e., (iii) Heavier elements are less reactive as they react
P > As > Sb > Bi with oxygen only on heating.
78

Q.4 Why does NH 3 form hydrogen bond but PH 3 does lone pair of electron. For details read structure of H 2S
not? in group 16 elements.
Ans. Due to high electronegativity and small size of N. Q.10 Why does R 3 P == O exist but R 3 N == O does not
Q.5 How is nitrogen prepared in the laboratory? Write (R = alkyl group)?
the chemical equations of the reactions involved. Ans. N does not show pentavalency due to non-availability
Ans. In the laboratory, it is prepared by action of NaNO 2 of d-orbitals.
solution on ammonium chloride solution. Q.11 Explain why NH 3 is basic while BiH 3 is only feebly
D
NH 4 Cl( aq) + NaNO 2( aq) ¾® N 2( g ) + NaCl( aq) basic?
Ans. The size of N-atom is smaller than that of Bi-atom.
+ 2H 2O( l ) Hence electron density on N-atom is higher and so it
In this reaction, small amounts of HNO 3 and NO are has higher tendency to donate electron pair.
also formed which are removed by passing them Q.12 Nitrogen exists as diatomic molecule and phosphorus
through H 2SO 4 solution containing some K 2Cr2O7 . as P4 . Why?
Q.6 How is ammonia manufactured industrially? Ans. Nitrogen atom has small size and high
Ans. Haber’s process : See Long answer type Q.4. electronegativity and may form pp-pp bonds,
Q.7 Illustrate how copper metal can give different therefore, it exists as diatomic molecule (N ºº N).
products on reaction with HNO 3 . The discrete N 2 molecules are held together by weak
Ans. (i) With dil. HNO 3 van der Waals’ forces. Phosphorus atom has large size
3Cu + 8HNO 3 ¾® 3Cu( NO 3 ) 2 + 2NO + 4H 2O and small electronegativity. Usually, it does not form
(ii) With conc. HNO 3 pp-pp bonds with itself. It prefers to form P—P single
Cu + 4HNO 3 ¾® Cu( NO 3 ) 2 + 2NO 2 + 2H 2O bonds and exists as P4 molecule.
Q.8 Give the resonating structures of NO 2 and N 2O 5 . Q.13 Write main difference between the properties of
Ans. (i) NO 2 white phosphorus and red phosphorus.
N N Ans. Deleted for examination 2021.
O 134° O O O Q.14 Why does nitrogen show catenation properties less
(ii) N 2O 5 than phosphorus?
O O O O Ans. Because N—N bond is weaker than P—P bond.
N—O—N N — O —N Q.15 Give the disproportionation reaction of H 3 PO 3 .
O O O O Ans. 4H 3 PO 3 ¾¾® 3H 3 PO 4 + PH 3
Q.9 The HNH angle value is higher than HPH, HAsH and +3 +5 -3
HSbH angles. Why? Q.16 Can PCl 5 act as an oxidising as well as a reducing
Ans. In all the hydrides of group 15 elements, the central agent? Justify?
atom is sp 3 -hybridised. The central element (E) forms Ans. Because P is in its highest oxidation state (+5), so it
three E—H s bonds. The fourth orbital contains the may act as oxidising agent only.
lone pair of electrons. Since lone pair-bond pair Q.17 Justify the placement of O, S, Se, Te and Po in the
repulsions are larger than bond pair-bond pair same group of the periodic table in terms of
repulsions, therefore the bond angle in all the electronic configuration, oxidation state and hydride
hydrides of group 15 elements is less than tetrahedral formation.
angle 109° 28'. Ans. All the members of this group have general electronic
The electronegativity of the group 15 elements configuration [ ] ns 2 , np 4 , so they are placed in gp.
(central atom) decrease from N to Sb. Therefore, the 16 of periodic table.
bond pair of electrons lie away and away from the (i) Electronic configuration: The general
central atom. Bond pair-bond pair repulsion is electronic configuration of all the elements of this
maximum in NH 3 and minimum in SbH 3 . Therefore, group is ns 2 , np 4 . Thus, they all p-block elements
bond angle decreases in the order: because differentiating electron enters p- subshell of
NH 3 > PH 3 > AsH 3 > SbH 3 . Read in reactivity of their valence shell.
group 15 elements towards hydrogen. Oxygen ( 8 O) : [He] 2 s 2 2 p 4
According to modern views, only N-atom in NH 3 is Sulphur ( 16 S) : [Ne] 3 s 2 3 p 4
sp 3 -hybridised. There is no hybridisation on the
Selenium ( 34 Se): [ Ar] 3d 10 ,4 s 2 4 p 4
central atom of PH 3 , AsH 3 , SbH 3 etc. Their
half-filled p-orbitals overlap with the half-filled Tellurium ( 52 Te): [Kr] 4d 10 , 5 s 2 5 p 4
s-orbitals of H-atoms and form three s E — H bonds. Polonium ( 84 Po): [Xe] 4 f 14 , 5d 10 ,6 s 2 6 p 4
This is confirmed by the fact that bond angle in (ii) Oxidation state and valency: (a) Oxygen is
PH 3 , AsH 3 etc is about 90°. The difference in the bivalent only because it has only two unpaired
bond angles of PH 3 , AsH 3 etc. is due to the electrons in its valence p-subshell. It does not have
difference in the electronegativity of P, As, Sb etc. and vacant d-subshell, thus, it can not expand its octet. In
repulsion between the bond pair of electrons and general, oxygen exhibits –2 oxidation state, however,
79

it also exhibits +2 oxidation state in OF2 where Q.22 How is SO 2 , an air pollutant ?
F-atom is more electronegative than O-atom. Ans. (i) SO 2 irritates the respiratory tract. It causes eye
(b) Except oxygen, all the other elements of this and throat irritation and breathlessness.
group have vacant d-subshells in which electrons of (ii) Even at low concentration, SO 2 has damaging
s-and p-subshells get unpaired in excited state. effect on plants. If plants are exposed to SO 2 gas for a
Therefore except oxygen, all the other elements of long time, it slows down the formation of
this family show oxidation state from +4 to +6. chlorophyll. There is loss of green colour. It is called
(iii) Hydrides : All the elements of this group form chlorosis. (iii) SO 2 dissolves in the moisture
the hydrides of type H2 M where M = O, S, Se, Te or present in air to form H 2SO 3 which is an acid.
Po; Example : H2 O , H2 S, H2 Se, H2 Te and H2 Po. SO 2 + H 2O ¾¾® H 2SO 3
(a) Acidic nature : All the hydrides of this group The acid corrodes iron and steel. The colour of the
are weak acids. Their acidic nature increases from fabrics, paper, leather, paints etc. gets faded.
H2 O to H2 Te. Q.23 Why are halogens strong oxidising agents ?
(b) Reducing nature : Except H2 O, all the Ans. Due to low bond dissociation energy, large -ve
hydrides of this group act as reducing agent. electron gain enthalpy and high electronegativity,
Reducing nature increases from H2 O to H2 Te with the halogens have strong tendency to accept the electron.
increases in atomic size from O to Te. Due to this X + e - ¾¾® X -
reason, overlapping is less effective and weaker M-H Therefore, halogens are strong oxidants.
bond is formed. Q.24 Explain why fluorine forms only one oxoacid, HOF ?
Q.18 Why is dioxygen a gas but sulphur a solid ? Ans. Fluorine shows only one oxidation state (–1). The
or Sulphur has greater tendency for catenation than other halogens show different oxidation states such
oxygen. as -1, + 1, + 3, + 5 and +7. It is due to the presence of
Ans. Due to small size and high electronegativity, oxygen vacant d-orbitals in halogen atoms except F-atom.
atom form pp-pp bonds forming diatomic (O 2 ). There Thus fluorine forms only one oxoacid HOF (F = -1).
are weak van der Waals’ forces among O 2 molecules Q.25 Explain why inspite of nearly the same
and hence dioxygen is gas. electronegativity, oxygen forms hydrogen bonding
Sulphur atoms have large size and lower while chlorine does not?
electronegativity so they cannot form pp-pp multiple Ans. The atomic size of Cl-atom is much longer than
bonds among sulphur atoms. It forms S—S single O-atom. Therefore , electron density at Cl-atom is
bonds. Also the S—S bond is stronger than O—O much lower than that of O-atom. Hence, chlorine
bond, therefore, sulphur has strong tendency for does not form hydrogen bond.
catenation. Sulphur forms octaatomic (S 8 ) molecules Q.26 Write two uses of ClO 2 .
and is solid. Ans. It is a powerful oxidising agent and chlorinating
Q.19 Knowing the electron gain enthalpy values of agent. It is used for bleaching wood pulp and
O ¾® O - and O ¾® O 2- as -141 and 702 kJ mol -1 cellulose and purifying drinking water.
respectively, how can you account for the formation Q.27 Why are halogens coloured?
of a large number of oxides having O 2- species and Ans. Because their molecules absorb in visible region and
not O - ? so complementary colour appears.
Ans. The formation of O - from O is exothermic whereas Q.28 Write the reactions of F2 and Cl 2 with water.
the formation of O 2- from O is endothermic. Yet Ans. F2 oxidises H 2O to O 2 or O 3 .
oxides having O 2- ions are large. It is due to the 2F2 + 2H 2O ¾¾® 4H + + 4F - + O 2 ,
greater lattice energy. 3F2 + 3H 2O ¾¾® 6H + + 6F - + O 3
Q.20 Which aerosols deplete ozone? Cl 2 gives hydrochloric acid and hypochlorous acid.
Ans. Aerosols such as chlorofluoro carbons (CFC’ s), i. e.,
Cl 2 + H 2O ¾¾® HCl + HOCl
freon (CCl 2 F2 ), oxides of nitrogen, oxides of sulphur
etc. deplete ozone layer. It is due to the formation of Q.29 How can you prepare Cl 2 from HCl and HCl from
Cl free radical. It converts O 3 to O 2 . Cl 2 ? Write reactions only.
· · Ans. MnO 2 + 4HCl ¾ Heat ¾¾® MnCl 2 + 2H 2O + Cl 2 ,
CCl 2 F2 ¾¾® Cl + CClF2
· ·
H 2 + Cl 2 ¾ Diffused
¾¾¾ light
¾® 2HCl
Cl + O 3 ¾¾® ClO + O 2 Q.30 What inspired N. Bartlett for carrying out reaction
· · ·
between Xe and PtF6 ?
Cl O ¾¾® Cl + O
· ·
Ans. He observed O 2 gives O +2 [PtF6 ]– with PtF6 , because
ClO + O ¾¾® Cl + O 2 I.E. of O 2 is nearly equal to that of Xe, so like O +2 , Xe+
Q.21 Describe the manufacture of H 2SO 4 by contact must also exist.
process. Bartlett thought Xe could also be oxidised to Xe + by
Ans. This is deleted from 2021 examination. PtF6 . When Xe and PtF6 were mixed together, a rapid
80

reaction took place and a red ionic solid Xe + PtF6- in the p-orbitals of F-atom and the bond pair of F—F
was formed. bond.
Xe + PtF6 ¾ 278K
¾¾® Xe + [PtF6 ]- (ii) Read in the hydrides of group 17 elements.
(iii) Read in the hydrides of group 15 elements.
Q.31 What are the oxidation states of phosphorus in the
Q.37 Which one of the following does not exist ?
following ?
(i) XeOF4 (ii) NeF2 (iii) XeF2 (iv) XeF6
(i) H 3 PO 3 (ii) PCl 3 (iii) Ca 3 P2 (iv) Na 3 PO 4
Ans. NeF2 , due to very high I.E of Ne.
(v) POF3
Q.38 Give the formula and describe the structure of a
Ans. +3, + 3, - 3, + 5 and +5 respectively.
noble gas species which is isostructural with:
Q.32 Write balanced equations for the following:
(i) ICl -4 (ii) IBr2- (iii) BrO -3
(i) NaCl is heated with sulphuric acid in the presence
of MnO 2 . Ans. ICl -4 :
(ii) Chlorine gas is passed into a solution of NaI in ICl -4 and XeF4 both have 36 electrons in their valence
water. shell and both are square planar.
Ans. (i) 4NaCl + MnO 2 + 4H 2SO 4 ¾¾® 4NaHSO 4 Valence electrons in ICl -4 = 7 + 1 + ( 4 ´ 7 ) = 36
+ MnCl 2 + Cl 2 + 2H 2O Valence electrons in XeF4 = 8+(4 ´ 7) = 36
(ii) 2NaI + Cl 2 ¾¾® 2NaCl + I 2 –

Q.33 How are xenon fluorides XeF2 , XeF4 and XeF6 Cl Cl


obtained?
I
Ans. (i) XeF2 is prepared by action of F2 on Xe or action of
O 2 F2 on Xe. Cl Cl
Ni / 400°C or
Xe + F2 ¾¾¾¾®
Hg arc light
XeF2

Square planar (ICl4 )
Xe + O 2 F2 ¾® XeF2 + O 2­
IBr2- :
(ii) XeF4 is obtained by action of excess of F2 on Xe
( Xe : F2 : : 1 : 5). Valence electrons in IBr2- = 7 + 1 + ( 2 ´ 7 ) = 22
Ni / 400°C
Xe + 2F2 ¾¾¾¾® XeF4 Valence electrons in XeF2 = 8 + (2 ´ 7) = 22
Both are isoelectronic and linear.
(iii) Mixture of Xe + F2 (1 : 20 ratio) on heating at Br –
400°C through Ni-tube gives XeF6 .
Ni / 400°C
Xe + 3F2 ¾¾¾¾® XeF6
I
D
or XeF4 + F2O 2 ¾® XeF6 + O 2­
Q.34 With what neutral molecule is ClO - isoelectronic? Is
that molecule a Lewis base? Br
Linear (IBr 2– )
Ans. ClO - has 17 + 8 + 1 = 26 electrons.
ClF also has 26 electrons. It is a Lewis base because it BrO -3 :
reacts with F2 and forms ClF3 . Valence electrons in BrO -3 = 7 + 1+(3 ´ 6) = 26
ClF + F2 ¾¾® ClF3 Valence electrons in XeO 3 = 8 + (3 ´ 6) = 26
Q.35 How are XeO 3 and XeOF4 formed ? Both are isoelectronic and pyramidal.
Ans. (i) XeO 3 is obtained by hydrolysis of xenon fluorides.
XeF6 + 3H 2O ¾® XeO 3 + 6HF
(ii) XeOF4 is obtained by partial hydrolysis of XeF6 . Br
XeF6 + H 2O ¾® XeOF4 + 2HF
Q.36 Arrange the following in the order of property O –
O
indicated for each set: O
(i) F2 , Cl 2 , Br2 , I 2 —increasing bond dissociation Pyramidal (BrO3– )
enthalpy. Q.39 Why do noble gases have comparatively large atomic
(ii) HF, HCl, HBr, HI—increasing acid strength. size ?
(iii) NH 3 , PH 3 , AsH 3 ,SbH 3 , BiH 3 —increasing base Ans. Because their atomic radii are van der Waals’ radii,
strength. which are larger than covalent/ionic/ metallic radii.
Ans. (i) I 2 < F2 < Br2 < Cl 2 Q.40 List the uses of neon and argon gases.
Bond dissociation energy increases from bottom to Ans. See short answer type question 11 and very short
top in a group due to the decrease in the atomic size. answer type question 16.
But F2 is an exception. Bond dissociation energy of
F—F is unexpectedly low. It is due to the greater vvv
repulsion among the non-bonding electrons present
81

Chapter

8 d-AND f-BLOCK
ELEMENTS
Syllabus : General Introduction, Electronic configuration, Occurrence and properties of transition metals, General properties of
elements of first series of (3d series) transition metals: metallic character, ionization enthalpy, oxidation states, ionic radii, colour,
catalytic property, magnetic property, interstitial compounds, alloy formation, *methods of preparation and properties of
K2Cr2O7 and KMnO4. Lanthanides: Electronic configuration, oxidation states, *chemical reactivity, contraction and its
consequences. *Actinides : Electronic configuration, oxidation states and comparison with lanthanides.

* Topics Deleted for Examination 2021

Objective Questions
1. Which of the following ion is colourless? (U.P. 2013) 6. The number of unpaired electrons in Co 2+ ( Z = 27) is:
(a) Cu + (b) Cu 2+ (U.P. 2016)
2+
(c) Ni (d) Fe 3+ (a) 1 (b) 2 (c) 3 (d) 4
2. Which of the following element show catalytic 7. Among the following ions which ion has maximum
behaviour ? (U.P. 2014) number of unpaired electrons ? (U.P. 2016)
(a) Ca (b) Fe (a) Mn 2+ ( Z = 25) (b) Ni 2+ ( Z = 28)
(c) Pb (d) all of these (c) Cr 3+ ( Z = 24) (d) Ti 2+ ( Z = 22)
3. Which transition metal does not show variable
8. Element having electronic configuration 3d 4 4s 2 is :
oxidation state ? (U.P. 2014)
(a) Semi-metal (b) Non-metal
(a) Ti (b) V (c) Fe (d) Zn
(c) Transition metal (d) Inert gas
4. Which of the following is an element of 4d transition
9. Among the following, which one is paramagnetic ?
series ? (U.P. 2015)
(U.P. 2017)
(a) 37 A (b) 47 B (c) 57 C (d) 30 D
(a) CuCl (b) AgNO 3 (c) FeSO 4 (d) ZnCl 2
5. Which of the following is lanthanide ? (U.P. 2015)
(a) Ra (b) Ce (c) Ac (d) Zr

Answers
1. (a) 2. (b) 3. (d) 4. (b) 5. (b) 6. (c) 7. (a) 8. (c) 9. (c)

Very Short Answer Type Questions


Q.1 Most of the compounds of transition metals are Ans. With increase in atomic number through series,
coloured, why ? (U.P. 2017) effective nuclear charge increases and it leach to
Ans. Because most of ions of transition elements have decrease in size.
unpaired electrons. Q.4 Transition elements show different oxidation states,
Q.2 Ions of transition metals, generally are paramagnetic, why ? (U.P. 2017, 20)
why ? (U.P. 2017) Ans. In transition metals, both ns and ( n - 1) d electrons
Ans. Because majority of ions of transition metals have take part in bonding when they show lower oxidation
unpaired electrons. state, ns electrons take part in bonding while in
Q.3 Atomic radii of elements does not change much higher oxidation states, both ns and ( n - 1) d electrons
significantly as the atomic number increases in a are used.
transition series, why ? or Q.5 Mn 2+ is more stable than Mn 3+ , why ? (U.P. 2017)
How does the atomic radii of transition metals vary Ans. Mn 2+ = 1s 2 , 2s 2 , 2 p 6 , 3s 2 , 3 p 6 , 3d 5 , 4s 0
across a series ? (U.P. 2014)
82

Mn 3+ = 1s 2 , 2s 2 , 2 p 6 , 3s 2 , 3 p 6 , 3d 4 , 4s 0 Cu 2+ , configuration is 3d 9 and owing to d - d


3d 5 (half filled) configuration is more stable than transition, it is blue.
3d 4 (incomplete), i. e. why Mn 2+ is more stable. Q.7 In volumetric analysis, to acidify KMnO 4 solution,
Q.6 Zn 2+ salts are colourless while Cu 2+ salts are blue, HNO 3 can not be used in place of dil H 2SO 4 , why ?
why ? (U.P. 2017) (U.P. 2015)
Ans. Due to 3d10 configuration, d - d transition is not Ans. Because HNO 3 itself is an oxidising agent.
possible in it, i. e. why Zn 2+ salts are colourless. In

Short Answer Type Questions

Q.1 What are lanthanides ? Explain their oxidation state. called transitional elements. These are also called
(U.P. 2014) d-block elements.
or Write the electronic configuration of lanthanide For examples :
elements. (U.P. 2018) Sc (21) : 1s 2 , 2s 2 2 p 6 , 3s 2 3 p 6 3d1 , 4s 2
Ans. The elements with atomic number 58 to 71, i. e., Ti (22) : 1s 2 , 2s 2 2 p 6 , 3s 2 3 p 6 3d 2 , 4s 2
cerium to lutetium [which come immediately after
The transition elements are classified into four series
lanthanum ( Z = 57)] are called lanthanides or
i. e.,
rare earth’s element. These elements involve the
(i) 3d series (ii) 4d series
filling of 4 f-orbitals. Their general electronic
(iii) 5d series (iv) 6d series
configuration is [Xe] 4 f 1-14 5d 0-10 6s 2 .
Characteristics of transition elements :
Oxidation states : Most stable oxidation state of (1) Transition elements are metals and good
lanthanides is +3. They also show +2 and +4 conductors of heat and electricity.
oxidation state. (2) Transition elements show the variable valencies
Q.2 Write general electronic configuration of inner due to participation of ( n - 1) and ns electrons.
transition elements. (U.P. 2014) (3) Transition metals are less reactive than s-block
Ans. The elements in which the last electron enters in elements due to high ionisation potential.
( n - 2) f-orbitals are called inner transition elements. (4) Transition metal and their compounds acts as
The general electronic configuration of inner catalyst due to presence of unpaired d-electrons.
transition elements is ( n - 2) s 2 p 6 d10 f 1-14 , Q.6 Discuss the following properties of transition
( n - 1)s 2 p 6 d 0-1 , ns 2 . elements.
Q.3 Why do +3 and +4 oxidation states of Cerium (i) Metallic nature
( Z = 58) remain stable? (U.P. 2018) or
Ans. Ce = [ Xe ] 4 f 1
, 5d1
, 6s 2 Why most of the transition metals are paramagnetic?
58
(U.P. 2018, 20)
+3 oxidation state is stable due to low sublimation
Ans. (i) Metallic nature : All the transition elements
and ionization energy and highly exothermic
are metals. Metallic character of transition elements
hydration energy. +4 oxidation state is stable due to
is due to presence of one or two electrons in
completely filled outermost shells (i. e., inert gas
outermost orbit.
configuration).
Being, metals, these elements are good conductors of
Q.4 Variation in ionisation enthalpies (first and second)
electricity and heat. They also form alloys by mixing
in first series of transition metals is irregular, explain.
together.
(U.P. 2017)
(ii) Paramagnetic character : Most of the
Ans. In 3d-series, effective nuclear charge increases with
transition elements or ions having at least one
increase in atomic number. As a result, a rise in
unpaired electron are paramagnetic in nature.
ionisation enthalpy is expected but stabilities of d 0 ,
Greater is the number of unpaired electrons, greater
d 5 and d10 configurations is more than other so rise is the paramagnetic character.
in I.E. is not continuous but irregular. For example: Cu 2+ ,Co 2+ , Mn 2+ etc. are
Q.5 Which elements are called transition elements ?
paramagnetic nature.
Explain. Write their two main characteristics.
Q.7 Explain why most of the transition metals form
(U.P. 2014, 18)
coordination compounds and exhibit variable
or
valence? (U.P. 2018)
Describe the special characteristics of transition
Ans. (i) Due to small size, high effective nuclear charge
elements. (U.P. 2014, 16)
and availability of vacant d-orbitals, transition
Ans. The elements having incomplete ( n - 1) d orbital in
metals form coordination compounds easily.
the ground state or in any of its excited states are
83

(ii) Due to involvement of ( n - 1)d and ns electrons in series, although the increase is not very regular. The
bond formation, they show variable valence. first ionisation energy of first transition series
Q.8 Write four properties of lanthanides. (U.P. 2015) (3d-series) are given below.
Ans. Properties of lanthanides :
(1) General electronic configuration of lanthanides is Element Sc Ti V Cr Mn Fe Co Ni Cu Zn
[Xe] 4 f 0-14 , 5d 0-1 , 6s 2 . Ionisation 631 656 650 652 717 762 758 736 745 905
(2) Most stable common oxidation state of potential
lanthanides is +3. Some elements also show +2 (kJ mol -1)
and +4 oxidation states.
(3) Most of the lanthanide ions are coloured due to Q.11 Write name and E.C. of lanthanides. (U.P. 2018)
partially filled f-orbitals. Ans.
(4) Due to lanthanide contraction all elements of 4 f-
block have similar size and thus almost similar Element Electronic configuration Oxidation states
properties also. Cerium [ Xe]4 f1, 5d1, 6s2 +3, + 4
Q.9 What are transition elements ? How do atomic radii
of them change in any series ? Explain. Praseodymium [ Xe]4 f 3, 5d0, 6s2 +3, + 4
(U.P. 2014, 20)
Ans. Elements in which last electron enters in ( n - 1) d- Neodymium [ Xe]4 f 4, 5d0, 6s2 +2, + 3, + 4
orbitals are called d-block elements. Since the
properties of these elements are midway between Promethium [ Xe]4 f 5, 5d0, 6s2 +3
those of s-block and p-block elements, they are called Samarium [ Xe]4 f 6, 5d0, 6s2 +2, + 3
as transition elements.
In a transition series, atomic radii decrease till the Europium [ Xe]4 f 7 , 5d0, 6s2 +2, + 3
middle. It becomes constant for next 3-4 elements
and then increases with the increase in atomic Gadolinium [ Xe]4 f 7 , 5d1, 6s2 +3
number. In the first transition series, the atomic radii
gradually decrease from scandium to chromium but Terbium [ Xe]4 f 9, 5d0, 6s2 +3, + 4
from chromium to copper, it is nearly the same.
Dysprosium [ Xe]4 f10, 5d0, 6s2 +3, + 4
3d series Sc Ti V Cr Mn Fe Co Ni Cu Zn
Holmium [ Xe]4 f11, 5d0, 6s2 +3
Radii 144 132 122 117 117 117 116 115 117 125
(pm) Erbium [ Xe]4 f12, 5d0, 6s2 +3

Q.10 Write down general electronic configuration of Thulium [ Xe]4 f13, 5d0, 6s2 +2, + 3
d-block elements. How do their ionisation potential
values change in any d-block series. (U.P. 2016) Ytterbium [ Xe]4 f14, 5d0, 6s2 +2, + 3
Ans. The elements in which the differentiating electron
enters into ( n - 1) d subshell are called transition
Lutetium [ Xe]4 f14, 5d1, 6s2 +3
elements. These elements belong to d-block of the
Q.12 Give two examples of lanthanoid elements. Also write
periodic table. Therefore, they are also known as
their two main uses. (U.P. 2019)
d-block elements. The general electronic
Ans. Lanthanoid Elements : 14 elements of 4f-series
configuration of d-block elements is (n - 1)
are called lanthanoids. Two examples are cerium
d1 -10 ns1 - 2 .
( 58 Ce) and lutetium ( 71 Lu).
Variation of Ionisation energy of d-block
Two uses of lanthanoids :
element : The ionisation energies of d-block
(i) In making Misch metal (pyrophoric alloy)
elements lie between those of s-block and p-block
(ii) As catalyst in petroleum industry (Cerium
elements. The first ionisation energy increases with
phosphate)
increase in atomic number across a given transition

Long Answer Type Questions

Q.1 Why the transition elements are also called as d-block Ans. The elements having incomplete (n–1)d sub-shell or
elements ? Explain on the basis of their electronic in which differentiating electron enters in (n–1)d
configuration. How do their ionization potential sub-shell (ns–subshell may also be incomplete in
values change in any d-block series ? (U.P. 2016) these elements) are called as d-block elements. In the
long form of periodic table, these elements are placed
84

in between s-block and p-block elements and that is Ionic radii : For the same oxidation state, the ionic
why, they are also known as transition elements. radii generally decrease from left to right in a
All the transition elements are metals, so, they are particular transition series. In the group, ionic radii
increase from top to bottom.
also called transition metals.
Ionization energy (or enthalpy) : The
On the basis of entry of differentiating electron in ionization energy most of the transition elements lie
d-subshell, transition metals are classified into four in between those of s- and p-block elements. Since,
series : these elements have smaller ionic radii, their
1. 3d-series : In this series, differentiating ionization energy values are high.
electron enters in 3d-subshell. Q.2 Explain the following properties of transition
2. 4d-series : In this series, differentiating elements. (U.P. 2014, 20)
electron enters in 4d - sub shell. (a) Metallic nature
(b) Paramagnetic character (U.P. 2018, 20)
3. 5d-series : In this series, differentiating
(c) Coloured ion formation (U.P. 2017)
electron enters in 5d-subshell. (d) Catalytic property (U.P. 2017, 20)
4. 6d-series : In this series, differentiating (e) M.Pt. and B.pt. (U.P. 2017)
electron enters in 6d-subshell. (f) Ionisation enthalpy (U.P. 2017)
Electronic configuration : General electronic Or What do you understand by transition elements?
configuration of transition elements is Explain the formation of interstitial compounds and
( n - 1)d1-10 ns1- 2 . ( 4d10 , 5s 0 in Pd) coloured ions by them. (U.P. 2019)
Electronic configuration of transition elements Ans. (a) Metallic character : All the transition
elements are metals having one or two electrons in
3d-series their valence shell. Except mercury (liquid), all other
transition elements are solid, rigid and ductile
Atomic number Element Electronic
metals. In their crystal lattices, atoms are strongly
configuration held together by metallic bonding.
21 Sc [Ar] 3d 1 , 4s 2 (b) Magnetic character : On the basis of
magnetic character, compounds of transition
22 Ti [Ar] 3d 2 , 4s 2 elements are divided into following three classes :
(i) Diamagnetic : The compounds which are
23 V [Ar] 3d 3 , 4s 2 repelled by magnetic field are called diamagnetic.
24 *Cr [Ar] 3d 5 , 4s1 These compounds have zero magnetic moment. The
atoms and ions containing only paired electrons are
25 Mn [Ar] 3d 5 , 4s 2 diamagnetic in nature.
(ii) Paramagnetic : The compounds which are
26 Fe [Ar] 3d 6 , 4s 2 attracted by magnetic field are called paramagnetic.
27 Co [Ar] 3d 7 , 4s 2 The atoms or ions having atleast one unpaired
electron are paramagnetic in nature. Greater the
28 Ni [Ar] 3d 8 , 4s 2 number of unpaired electrons, greater is the
paramagnetic character.
29 *Cu [Ar] 3d 10 , 4s1
(iii) Ferromagnetic : Those strongly paramagnetic
30 Zn [Ar] 3d 10 , 4s 2 substances which retain their magnetic character even
after the removal of external magnetic field are called
According to aufbau principles, electron first enters ferromagnetic. For example : Iron.
the sub-shell for which value of (n + l) is lower. Thus, Magnetic character of paramagnetic substances is
electron, first, enters in ns- subshell instead of ( n - 1)d expressed in terms of Bohr - magneton (B.M.)
subshell. Magnetic moment, m = n( n + 2) B.M.
For example : For 3d -subshell, value of (n+ l) is where n = Number of unpaired electrons.
3+2 = 5, while it is 4 + 0 = 4 for 4s-subshell. Thus Except Zn, Cd and Hg, all the transition elements
electron, first enters 4s-subshell instead of have half filled subshells and unpaired electrons,
3d-subshell. Similarly, electron first enters in thus, they show para magnetic property.
5s-subshell instead of 4d-subshell. (c) Formation of coloured compounds:
Atomic radii : Atomic radii of transition elements Generally, transition elements form coloured ions in
are lesser than those of s-block elements but greater aqueous solution.
than those of p-block elements.

*Abnormal configuration.
85

Reason of colour of ions : Colour of ions is due to covalent bonds is not possible between such kind of
d - d transition and ions the having d1 -d 9 atoms, Examples of such transition metals are Zn,
configuration are colourd ions with d 0 od d10 Cd, and Hg.
configuration are colourless because d-d transition is Melting points increase with increase in number of
not present. electrons in d- subshell and tungsten has the highest
(d) Catalytic properties : Due to their small melting point. When pairing of electrons begins in d-
atomic size and high effective nuclear charge, subshell, the value of melting point begins to fall. It
transition elements exhibit catalytic properties. means, greater the number of unpaired electrons in
For example : d-subshell, higher is the value of melting point.
(i) V 2O 5 catalyst is used to convert SO 2 to SO 3 . Q.3 Write general electronic configuration of transitional
(ii) For the adsorption of reactant, transition element metals and explain shielding effect. (U.P. 2020)
provides large surface area making the reaction Ans. The attraction exerted by nucleus on outermost
easier. electron (valency electron) is reduced due to
Other examples are, use of Ni in hydrogenation of repulsion between valency electron and inner
oils, use of Fe in manufacturing of NH 3 by Haber electrons. This effect is called shielding effect.
process. Inner electron
(e) Melting and boiling points : Due to strong – Repulsion
metallic forces, melting and boiling points of
transition metals are higher than those of s- block + –
elements. Melting points of transition metals having Attraction
Nucleus Outer electron
complete d- subshell are lower than other transition
metals. This is because of the fact that formation of
Shielding effect

Solution of NCERT Text Book Problems


Q.1 Write down the electronic configuration of : configurations in the ground state of their atoms :
(i) Cr 3+ (ii) Pm 3+ (iii) Cu + (iv) Ce 4+ (v) Co 2+ 3d 3 , 3d 5 , 3d 8 and 3d 4 ?
(vi) Lu 2+ (vii) Mn 2+ (viii) Th 4+ Ans. 3d 3 , 4s 2 = +5
Ans. (i) Cr 3+ = [Ar]3d 3 (ii) Pm 3+ = [Xe]4 f 4 3d 5 , 4s 2 = +7, 3d 8 4s 2 = +2
+ 10
(iii) Cu = [Ar]3d (iv) Ce 4+ = [Xe]54 3d 4 , 4s 2 = 3d 5 4s1 = +6 and + 3
2+ 7
(v) Co = [Ar]3d (vi) Lu 2+ = [Xe]4 f 14 5d1
Q.6 Name the oxometal anions of the first series of the
(vii) Mn 2+ = [Ar]3d 5 (viii) Th 4+ = [Rn]86 transition metals in which the metal exhibits the
Q.2 Why are Mn 2+ compounds more stable than Fe 2+ oxidation state equal to its group number.
towards oxidation to the +3 state ? Ans. Cr2O72– and CrO 2– 4 (Cr = +6 = group number).
Ans. Electronic configuration of Mn 2+ is 3d 5 . The d-orbitals –
MnO 4 (Mn = +7 = group number).
are half-filled and hence more stable. It is difficult to
Q.7 What is lanthanoid contraction ? What are the
remove one more electron from Mn 2+ due to high 3rd
consequences of lanthanoid contraction ?
ionisation enthalpy. Electronic configuration of Fe 2+
or Describe lanthanide contraction and its effects.
is 3d 6 . It can loose one electron to form Fe 3+ having
(U.P. 2016, 18, 20)
stable 3d 5 configuration.
or Write short notes on lanthanide contraction.
Q.3 Explain briefly how +2 state becomes more and more
stable in the first half of the first row transition (U.P. 2016)
elements with increasing atomic number. or What is lanthanide contraction? Explain its reason.
Ans. The E Ro .P. values for M 2+ / M couple are negative, (U.P. 2016, 17)
or Explain why size of lanthanides decreases from La
M 2+ ion has little tendency to accept electron and to Lu? (U.P. 2017)
form M. Mn 2+ is stable due to stable 3d 5 Ans. The ionic radii of M 3+ ions in lanthanides series show
configuration. Ni 2+ is stable due to more negative a regular decreases in the size of ions with increase in
enthalpy of hydration. atomic nature. This regular decrease in the size of
Q.4 To what extent do the electronic configurations
lanthanide ions ( M 3+ ) with the increase in atomic
decide the stability of oxidation states in the first
series of the transition elements? Illustrate your number is known as lanthanide contraction.
answer with examples. Cause of lanthanide contraction : The
Ans. The oxidation states which give half-filled or lanthanide contraction along the lanthanide series
completely filled 3d-orbitals are more stable, e.g., has been explained in terms of poor shielding or
Mn 2+ , Fe 3+ , Zn 2+ etc. screening effect of f-subshell of pre-penultimate
subshell. On moving along the lanthanoid series, the
Q.5 What may be the stable oxidation state of the nuclear charge increases due to increase in number of
transition element with following d electron
86

protons as well as one electron increases in f-subshell Q.9 In what way the electronic configuration of the
of pre-penultimate shell. Recall that shielding effect is transition elements differ from that of the non-
poor in case of pre-penultimate shell as well as it transition elements?
becomes very less effective if shielding is by Ans. Transition elements contain incompletely filled
f-subshell as they are more diffused. The shielding d-subshells.
order is s > p > d > f . Q.10 What are the different oxidation states exhibited by
Evidently, the increase in nuclear charge due to the lanthanoids ?
increase in number of proton with increase in atomic Ans. +2, +3 and +4 (+3 being most common).
number predominates over decrease in nuclear Q.11 Explain giving reasons :
charge due to shielding effect and thus valence shells (i) Transition metals and many of their compounds
are pulled more effectively towards nucleus to show a show paramagnetic behaviour. (U.P. 2018)
decrease in atomic size. (ii) The enthalpies of atomisation of the transition
Salient Features of Lanthanides Contraction metals are high.
(i) On moving from La 3+ to Lu 3+ , the ionic radii (iii) The transition metals generally form coloured
change from 106 pm to 85 pm in steps showing compounds.
an average decrease of about 1.4 pm (21 pm for (iv) Transition metals and their many compounds act
15 elements). as good catalyst.
(ii) This small average decrease in atomic size is Ans. (i) Most of the transition elements and their
responsible for a small decrease in compounds show paramagnetic behaviour due to
electronegativity and standard oxidation presence of unpaired electrons in atoms or ions
potential along the lanthanides series. or molecules.
M ( g ) ¾® M (3aq
+
.) + 3e
For example: Cu ++ : 1s 2 , 2s 2 2 p 6 , 3s 2 3 p 6 3d 9
(iii) The basic character of oxides and hydroxides or
decreases from La(OH) 3 to Lu(OH) 3 . Due to 1s 2 , 2s 2 2 p 6 , 3s 2 3 p 6
smaller size of Lu, the Lu—OH bond acquires Cu ++ have one unpaired electron, therefore it is
more covalent character. paramagnetic in nature.
(iv) It is the influence of Lanthanides contraction (ii) They have strong interatomic interaction and
which is responsible for the closer values of strong bonding between the atoms. It is due to
atomic radii for Zr—Hf, Nb—Ta and Mo—W the presence of large number of unpaired
pairs of elements, the members of 4th-5th electrons in their atoms. Therefore, transition
d-series of a subsequent group. Here the usual elements have high enthalpies of atomisation.
increase in size on moving down the group from
(iii) The transition metal ions or their compounds are
4d-series to 5d-series is cancelled by the decrease
mostly coloured in solid state as well as in
in size due to lanthanide contraction. This is the
reason that members of 4d and 5d-series of a solution. The colour of transition metal ions or
group resemble more closely in comparison to their compounds is due to the partially filled
3d-members. ( n - 1) d-orbitals, i. e., the unpaired electrons in
(v) Lanthanides have similar chemical properties ( n - 1) d-orbitals.
and it is, therefore, difficult to separate them For examples: Cu 2+ (blue), Co 2+ (blue), Ni 2+
from each other. However, due to lanthanide (green) etc. are coloured ions.
contraction, their ability to form complexes, (iv) Transition metals and their compounds
varies slightly. This slight variation is utilised to particularly oxide have good catalytic properties.
separate lanthanides by ion-exchange Catalytic properties of these metal are due to
method. presence of vacant d-orbitals and tendency to
Q.8 What are the characteristics of the transition exhibit variable oxidation state.
elements and why are they called transition Q.12 What are interstitial compounds ? Why are such
elements? Which of the d-block elements may not be compounds well known for transition metals ?
regarded as the transition elements ? Ans. Transition metals have tendency to form interstitial
Ans. (i) Their general E.C. is ( n - 1)d1-10 ns1- 2 . compounds. In the crystal lattice of transition metals,
(ii) They all are metals which are hard. small non-metal atoms (like H, C, N, B etc.) occupy
(iii) They are called transition elements because in their positions and form interstitial compounds.
periodic table. They are placed between s and p- In these compounds, there is no true bond between
block elements. metal and non-metal atoms, so they are neither ionic
(iv) Zn, Cd and Hg do not fit in definition of nor covalent. These non-metal atoms do not affect
transition elements but due to similarities in the chemical properties of metal atoms. They
some properties, they are studied with transition decrease their ductility and malleability but increases
elements. their tensile strength. For example : Steel and pit
(v) They show variable oxidation states. iron are hard due to presence of carbon atom in the
lattice of iron.
87

Mn 4 N, Fe 3 N, TiC, TiH 2 etc. do not have formula Mn 3+ and (ii) the ease with which iron can be
according to their oxidation state and so they are oxidised as compared to a similar process for either
non-stoichiometric compounds, e. g ., TiH17
. , VH 0. 56 . chromium or manganese metal.
Their physical and chemical properties are as given º
Ans. (i) By comparing E R.P. values for Cr 3+ Cr 2+ ,
below. 3+ 2+ 3+
(a) Their m.pts. are higher than pure metals. Fe Fe and Mn Mn 2+ , it is observed that
(b) They are very hard. Some borides are as hard as Cr 3+ is reduced with maximum difficulty while
diamond. among these Mn 3+ is most easily reduced. So
(c) They are electrical conductors order of stability is
(d) They are chemically inert. Mn 3+ < Fe 3+ < Cr 3+
º
Note: Small non-metal atom holds position in lattice of metal atom (ii) On the basis of E O.P. values for Fe / Fe 2+ ,
2+ 2+
due presence of vacant space is lattice. These spaces arises due Cr / Cr and Mn / Mn .
to variable oxidation state and defects in metal lattice. The order of ease of oxidation is
Q.13 How is the variability in oxidation states of transition Mn > Cr > Fe
metals different from that of the non-transition Q.18 Predict which of the following will be coloured in
metals? Illustrate with examples. aqueous solution? Ti 3+ , V 3+ , Cu+ , Sc 3+ , Mn 2+ , Fe 3+ ,
Ans. Non-transition elements show variable valence due
Co 2+ and MnO –4 . Give reasons for each.
to involvement of ns and np electrons. On the
otherhand, transition elements show variable valence Ans. Colourless : Sc 3+ ( 3d 0 ), Cu + ( 3d10 )
due to involvement of ( n - 1)d and ns electrons. Coloured : Ti 3+ , V 3+ , Mn 2+ , Fe 3+ ,Co 2+ , MnO –4
e. g ., P shows +3 and +5 oxidation states. Q.19 Compare the stability of +2 oxidation state for the
2 2 6 2 3
15 P( III) = 1s , 2s , 2 p , 3s , 3 p elements of the first transition series.
(+3 ox. state) Ans. For the first transition series i. e., 3d-series, the most
= 1s , 2s , 2 p , 3s 3 p 3 3d1
2 2 6 1 common oxidation state is +2 (except Sc). Zn 2+ and
15 P( V )
Cu 2+ ions are highly stable.
Q.20 Compare the chemistry of lanthanoids with that of
On the otherhand, 21 Sc, shows +1 and +3 oxidation the actinoids with special reference to :
state. (i) electronic configuration
2 2 6 2 6 1 2
21 Sc = 1s , 2s , 2 p , 3s , 3 p , 3d , 4s (ii) atomic and ionic sizes and
21 Sc
+
= 1s 2 , 2s 2 , 2 p 6 , 3s 2 , 3 p 6 , 3d1 , 4s1 (iii) oxidation state, or why do actinoids exhibit
(1s electron is removed) larger number of oxidation states than the
Sc 3+
= 1s 2
, 2s 2
, 2 p 6
, 3s 2
, 3 p 6 , 3d 0 , 4s 0 corresponding lanthanoid ?
21
(iv) chemical reactivity.
(2s and 1d electron are removed)
Q.14 Describe the preparation of potassium dichromate Ans. Deleted for examination 2021.
from iron chromite ore. What is the effect of increa- Q.21 How would you account for the following ?
sing pH on a solution of potassium dichromate? (i) Of the d 4 species, Cr 2+ is strongly reducing while
manganese (III) is strongly oxidising.
Ans. Deleted for examination 2021. (ii) Cobalt (II) is stable in aqueous solution but in the
Q.15 Describe the oxidising action of potassium presence of complexing reagents it is easily
dichromate and write the ionic equations for its oxidised.
reaction with : (i) iodide (ii) iron (II) solution and (iii) The d1 configuration is very unstable in ions.
(iii) H 2S. Ans. (i) E ° value for Cr 3+ /Cr 2+ is –0.41 V. Thus Cr 2+ has
Ans. Deleted for examination 2021. the tendency to get oxidised into Cr 3+ . Therefore,
Q.16 Describe the preparation of potassium perma-
Cr 2+ is a strong reducing agent. E ° for
nganate. How does the acidified permanganate
Mn 3+ / Mn 2+ is +1.5 V. Mn 3+ has the tendency to
solution react with : (i) iron(II) (ii) SO 2 and (iii) oxalic
acid (iv) S 2O 2– 3 ion ?
get reduced into Mn 2+ . Thus Mn 3+ is an
Ans. Deleted for examination 2021. oxidising agent.
Q.17 For M 2+ / M and M 3+ / M 2+ systems the E ° values for (ii) Cobalt (III) has greater tendency to form
coordination compounds than cobalt (II).
some metals are as follows :
Therefore, in the presence of ligands, cobalt (II)
Cr 2+ /Cr –0.9 V , Cr 3+ / Cr 2+ –0.4 V
2+ 3+ 2+
is oxidised to cobalt (III) to form a more stable
Mn / Mn –1.2 V , Mn / Mn +1.5 V coordination compound.
Fe 2+ / Fe –0.4 V , Fe 3+ / Fe 2+ +0.8 V (iii) The ions with d1 configuration have the tendency
Use this data to comment upon : (i) the stability of to acquire stable d 0 configuration by losing one
Fe 3+ in acid solution as compared to that of Cr 3+ or electron.
88

Q.22 What is meant by ‘disproportionation’ ? Give two Q.30 Which is the last element in the series of the
examples of disproportionation reaction in aqueous actinoids? Write the electronic configuration of this
solution. element. Comment on the possible oxidation state of
Ans. The reactions in which same atom/ion/molecule is this element.
oxidised and reduced simultaneously. Ans. Lr (Z = 103). Lr = +3
e. g ., 2Cu + ¾® Cu 2+ + Cu Q.31 Use Hund’s rule to derive the electronic configuration
Q.23 Which metal in the first series of transition metals of Ce 3+ ion, and calculate its magnetic moment on
exhibits +1 oxidation state most frequently and why? the basis of ‘spin-only’ formula.
Ans. Cu : 3d10 4s1 , Cu + : 3d10 Ans. 58 Ce : [Xe] 4 f 1 5d1 6s 2 , Ce 3+ : [Xe] 5 f 1 .
Cu loses 4s1 electron readily and forms Cu + . n = 1.
Q.24 Calculate the number of unpaired electrons in the m = n ( n + 2) = (1 + 2) = 3 = 1.73 BM
following gaseous ions : Mn 3+ , Cr 3+ , V 3+ and Ti 3+ . Q.32 Name the members of the lanthanoid series which
Which one of these is the most stable in aqueous exhibit +4 oxidation state and those which exhibit
solution? +2 oxidation states. Try to correlate this type of
Ans. Mn 3+ : [ ] 3d 4 (No. of unpaired electrons = 4) behaviour with the electronic configurations of these
elements.
Cr 3+ : [ ] 3d 3 (No. of unpaired electrons = 3)
Ans. +4 = Ce, Pr, Nd, Tb and Dy (Z = 58, 59, 60, 65 and
V 3+ : [ ] 3d 2 (No. of unpaired electrons = 2) 66 respectively).
Ti 3+ : [ ] 3d1 (No. of unpaired electrons = 1) +2 = Nd, Sm, Eu, Tm and Yb ( Z = 60, 62, 63, 69 and
In aqueous solution Cr 3+ is most stable. 70 respectively).
Q.25 Give examples and suggest reasons for the following Lanthanoids having 5d 0 6s 2 configuration show +2
features of the transition metal chemistry : oxidation state. It is easy to lose 2 electrons in
(i) The lowest oxide of transition metal is basic, the s-subshells.
highest is amphoteric/acidic. Lanthanoids show +4 oxidation state when
(ii) A transition metal exhibits highest oxidation configuration left is close to 4 f 0 (i.e., 4 f 0 , 4 f 1 , 4 f 2 )
state in oxides and fluorides. or close to 4 f 7 (i.e., 4 f 7 or 4 f 8 ).
(iii) The highest oxidation state is exhibited in Q.33 Compare the chemistry of the actinoids with that of
oxoanions of a metal. lanthanoids with reference to : (i) electronic
Ans. (i) MnO (basic, ionic), Mn 2O7 (acidic, covalent) configuration, (ii) oxidation states and (iii) chemical
(ii) O & F are highly electronegative elements. reactivity.
(iii) Due to high electronegativity of oxygen. Ans. See Q. 20.
Q.26 Indicate the steps in the preparation of : Q.34 Write the electronic configurations of the elements
(i) K 2Cr2O7 from chromite ore. with the atomic numbers 61, 91, 101, and 109.
(ii) KMnO 4 from pyrolusite ore. Ans. 61 Pm = [ Xe] 7 f 4 , 6s 2 , 91 Pa = [ Rn] 5 f 2 , 6d1 , 7 s 2
Ans. Deleted for examination 2021. 13 2 14
101 Md = [ Rn] 5 f , 7 s , 109 Ds = [ Rn] 5 f , 6d7 , 7 s 2
Q.27 What are alloys ? Name an important alloy which
contains some of the lanthanoid metals. Mention its Q.35 Compare the general characteristics of the first series
uses. of the transition metals with those of the second and
Ans. Alloys are mixtures of metals with metals or with third series metals in the respective vertical columns.
non-metals. Misch metal is an important alloys of Give special emphasis on the following points:
lanthanides. Containing about 40.5% Ce, 77%% La & (i) electronic configurations,
Nd and rest Ca, C & Si. It is used in making bomb (ii) oxidation states,
shells. (iii) ionisation enthalpies and
Q.28 What are inner transition elements? Decide which of (iv) atomic sizes
the following atomic numbers are the atomic Ans. (i) Electronic configuration : The elements in
numbers of the inner transition elements : 29, 59, 74, the same vertical column generally have similar
95, 102, 104. electronic configuration of the valence shell. But
Ans. The f-block elements are called inner transition there are few exceptions also, e.g., in group 5 the
elements, so elements with atomic numbers 59, 95 electronic configuration of valence shell of
and 102 are inner transition elements. vanadium (Z = 23) is 3d 3 4s 2 whereas that of Nb
Q.29 The chemistry of the actinoid elements is not so (Z = 41) is 4d 4 5s1 . In group 6, the electronic
smooth as that of the lanthanoids. Justify this configuration of valence shell of Cr is 3d 5 4s1
statement by giving some examples from the whereas that of W (Z = 74) is 5d 4 6s 2 , in group 8
oxidation state of these elements. the electronic configuration of valence shell of Fe
Ans. Because actinides show a variety of oxidation. States is 3d 6 4s 2 whereas that of Ru (Z = 44) is 4d7 5s1 ,
and being radioactive in nature. Their chemistry has in group 10 electronic configuration of valence
not been studied to much extent.
89

shell of Ni is 3d 8 4s 2 whereas that of Pd (Z = 46) (iii) Ionisation enthalpies of 5d series are higher than
is 4d10 5s 0 . the corresponding elements of 3d and 4d series.
In first series, there are two exceptions (Cr and (iv) Enthalpies of atomisation of 5d series are higher
Cu) in electronic configuration. The second series than the corresponding elements of 3d and 4d
has more exceptions in electronic configuration, series.
e.g., Mo (42), Ru (43), Rh (44), Pd (46) and Ag Q.38 What can be inferred from the magnetic moment
(47). In third series exceptions are W (74), Pt values of the following complex species?
(78) and Au (79). Example Magnetic Moment (BM)
(ii) Oxidation states : The elements in same K 4 [Mn(CN) 6 ] 2.2
vertical column generally show similar oxidation [Fe(H 2O) 6 ]2+ 5.3
states. K 2[MnCl 4 ] 5.9
(iii) Ionisation enthalpies : The ionisation
enthalpies generally decrease in a vertical Ans. m = n( n + 2) BM
column with increase in atomic number. But the If n = 1, m = 1 ( 1 + 2 ) = 3 = 1.73
ionisation enthalpies of the elements of 5d series
are higher than the elements of 3d and 4d series. n = 2, m = 2 (2 + 2) = 8 = 2.83
(iv) Atomic size : The atomic size generally
increases with the increase in atomic number in a n = 4, m = 4 (4 + 2) = 24 = 4.89
vertical column. The size of the elements of 4d
n = 5, m = 5(5 + 2) = 35 = 5.92
and 5d series are larger than the corresponding
elements of 3d series. The atomic size of elements K 4[Mn(CN)6 ] : Mn is present in +2 oxidation state
of 4d series is nearly equal to the corresponding i.e., Mn 2+ which has 5 unpaired electrons. But m =
elements of 5d series. 2.2, this shows that it has only one unpaired electron.
Q.36 Write down the number of 3d electrons in each of the Due to the strong ligand effect of CN - ions, the
following ions : electrons in the d-orbital get paired.
Ti 2+ , V 2+ , Cr 3+ , Mn 2+ , Fe 2+ , Co 2+ , Ni 2+ and Cu 2+ . 3d 4s 4p
Indicate how would you expect the five 3d-orbitals to
be occupied for these hydrated ions (octahedral).
Ans. Electrons are filled according to Hund’s rule. Water is d2sp3-hybridisation
a weak ligand and it cannot pair up the electrons. 2 3
d sp -hybridisation explains the formation of inner
eg
orbital octahedral complex ion which has only one
unpaired electron.
t2g [Fe(H 2O)6 ]2+ : Fe is present in +2 oxidation state,
Ti2+(3d2) V2+(3d3) Cr3+(3d3) i.e., Fe 2+ which has 4 unpaired electrons.
m = 5.3 BM shows the presence of 4 unpaired
eg electrons. H 2O has weak ligand effect. The electrons
in the d-orbital do not get paired.
t2g 3d 4s 4p 4d

Mn2+(3d5) Fe2+(3d6) Co2+(3d7)


sp3d2-hybridisation
eg 3 2
sp d -hybridisation explains the formation of outer
t2g orbital octahedral complex ion which has 4 unpaired
electrons.
Ni2+(3d8) Cu2+(3d9)
K 2[MnCl 4 ] : Mn is present in +2 oxidation state,
i.e., Mn 2+ which has 5 unpaired electrons.
Q.37 Comment on the statement that elements of the first
m = 5.9 shows that it has 5 unpaired electrons. The
transition series possess many properties different
complex ion is tetrahedral and hybridisation is sp 3 .
from those of heavier transition elements. 3d 4s 4p
Ans. The given statement is true.
(i) The atomic size of 4d and 5d series are larger than
the corresponding elements of 3d series. But the
atomic size of 4d and 5d series are nearly equal. sp3-hybridisation
(ii) The m.pt. and b.pt. of heavier transition elements
are generally greater than those of first transition
series.
vvv
90

Chapter

9 CO-ORDINATION
COMPOUNDS
Syllabus : Co-ordination Compounds: Introduction, ligands, coordination number, colour, magnetic properties and
shapes, IUPAC nomenclature of mononuclear coordination compounds, Bondin g: Werner’s theory, VBT and CFT, *structural and
stereo isomerism, uses in extraction of metals qualitative analysis and biological systems.
* Topics Deleted for Examination 2021

Objective Questions
1. Oxidation number of Ni in Ni(CO) 4 is : (U.P. 2014) 6. Valency of Cr in ion [Cr ( H 2O) 4 Cl 2 ]+ is : (U.P. 2017)
(a) 0 (b) 4 (c) 8 (d) 2 (a) 3 (b) 1 (c) 6 (d) 5
2. IUPAC name of complex K 3 [ Al(C 2O 4 ) 3 ] is : 7. Coordination number of Co in [Co(en ) 2 Cl 2 ] is :
(U.P. 2014) (U.P. 2017)
(a) Potassium alumino-oxalate (a) 3 (b) 4 (c) 5 (d) 6
(b) Potassium trioxalatoaluminate (III) 8. Which of the following cation does not form complex
(c) Potassium aluminium (III) oxalate with amines ? (U.P. 2017)
(d) Potassium trioxalatoaluminate (VI) + 3+ 2+ 2+
(a) Ag (b) Al (c) Cd (d) Cu
3. The number of stereoisomers of [ Pt ( NH 3 ) 2 Cl 2 ] is : 9. Number of unpaired electrons in complex of Fe,
(U.P. 2014) [ Fe ( H 2O) 5 NO]SO 4 is : (U.P. 2017)
(a) 1 (b) 2 (c) 4 (d) 3 (a) 2 (b) 3
4. Heteroleptic complex is : (U.P. 2015) (c) 4 (d) none of these
(a) [ Fe (CN ) 4 ]4- (b) [Co( NH 3 ) 5 SO 4 ]+ 10. Which of the following ion does not form complex
(c) [ HgI 4 ]2- (d) [Co( NH 3 ) 6 ]3+ compound ? (U.P. 2017)
5. Oxidation number of Co in [Co( NH 3 ) 5 Cl]Cl 2 is : (a) Na + (b) Cr 2+ (c) Cr 3+ (d) Co 2+
(U.P. 2017) 11. Colourless complex ion in the following is : (U.P. 2017)
(a) +1 (b) +2 (c) +3 (d) +4 (a) [Cu ( NH 3 ) 4 ]2+ (b) [ Zn ( NH 3 ) 4 ]2+
(c) [ Fe(H 2O) 6 ]3+ (d) [ Fe (CN ) 6 ]3–
Answers
1. (a) 2. (b) 3. (b) 4. (b) 5. (c) 6. (a) 7. (d) 8. (b) 9. (c) 10. (a)
11. (d)

Very Short Answer Type Questions


Q.1 In [Fe(C 2O 4 ) 3 ]3- , what is the co-ordination number of (c) [Mn(H 2O) 6 ]2+ (U.P. 2017)
central metal ion ? (U.P. 2017) (d) [Ni(CO) 4 ] (U.P. 2016, 20)
Ans. 6 (e) K 3 [Cr(C 2O 4 ) 3 ] (U.P. 2016, 17)
Q.2 Specify the oxidation numbers of the metals in the
(f) K 3 [Co(NO 2 ) 6 ] (U.P. 2016, 19, 20)
following co-ordination entity :
[Cu(NH 3 ) 4 ] SO 4 (U.P. 2014) (g) [Co(CO 3 )(NH 3 ) 5 ]Cl (U.P. 2016, 17)
(h) K 3 [Fe(CN) 5 NO] 2H 2O (U.P. 2016)
Ans. +2
Q.3 Using IUPAC forms write the systematic names of the (i) Na 2 [Fe(CN) 6 ] (U.P. 2016)
following : (j) [Fe(H 2O) 6 ] Cl 3 (U.P. 2015, 16, 20)
(a) [Cr(Cl)(H 2O)5 ]Cl 2 (U.P. 2016) (k) [Ag(NH 3 ) 2 ] Cl (U.P. 2014, 15)
(b) [Ni(NH 3 )6 ]Cl 2 (U.P. 2017) (l) K 3 [Cr(CN) 6 ] (U.P. 2015, 16, 17)
91

(m) [Co(NH 3 ) 5 Cl] SO 4 (U.P. 2015, 16) (xii) K 3 [AlF6 ] (U.P. 2015)
(n) [Pt(NH 3 ) 6 ] Cl 4 (U.P. 2015, 16) (xiii) [Fe(H 2O) 6 ]Cl 3 (U.P. 2014, 16)
(o) Na 2 [Fe(CN) 5 ]NO (U.P. 2015) (xiv) K 4 [Ni(CN) 4 ] (U.P. 2017)
(p) [Pt(NH 3 ) 4 Br 2 ] Cl 2 (U.P. 2015) Ans. (i) Potassium-tris-oxalato ferrate (III)
(q) [Fe(H 2O)6 ]Cl 3 (U.P. 2016) (ii) Potassium hexacyanoferrate (II)
(iii) Tetraamminezinc (II) chloride
(r) [Pt(NH 3 )6 ]Cl 4 (U.P. 2016)
(iv) Potassium tetraiodomarcurate (II)
(s) K 2[Ni(CN)4 ] (U.P. 2016, 19) (v) Sodium dicyanoargentate (I)
(t) [Co(NH 3 )4 ]ClSO 4 (U.P. 2016) (vi) Tetraamminecopper (II) sulphate
(u) pentaamminenitrito-N-cobalt (III) ion (U.P. 2014) (vii) Hexamminecobalt (III) chloride
(v) Fe 4 [ Fe (CN ) 6 ]3 (U.P. 2019) (viii) Diammineargentem dicyanoargentate (II)
(ix) Potassium ammine tribromidoplatinate (II)
(w) K 2 [ Zn (OH) 4 ] (U.P. 2019)
(x) Potassium trioxalatocobaltate (III)
(x) [Co(NH 3 )5 (H 2O)]Cl 3 (U.P. 2019) (xi) Dichlorido-bis-(ethylenediamine)cobalt (III)
Ans. (a) pentaaquachloridochromium (III)chloride chloride
(b) hexaamminenickel (II) chloride. (xii) Potassium hexafluoridoaluminate (III)
(c) hexaaquamanganese (II) ion (xiii) Hexaaquairon (III) chloride
(d) tetracarbonylnickel (o) (xiv) Potassium tetracyanonickelate(o)
(e) potassium trioxalatochromate (III) Q.5 Cu + is colourless while Cu 2+ is coloured, why?
(f) potassium hexanitrocobaltet (III) (U.P. 2017)
(g) pentaammine carbonatocobalt (III) chloride Ans. Electronic configuration of penultimate d- subshell of
(h) potassium pentacyanonitrosylferrate(II)dihydrate Cu + is 3d 10 , hence, d-d transition does not take place.
(i) Sodium hexacyanoferrate (II) Cu 2+ ion has an unpaired electron, thus, d-d transition
(j) hexaquairon (III) chloride occurs in this ion.
(k) diamminesilver (I) chloride Q.6 Find out the effective atomic number of iron in
(l) potassium hexacyanochromate (III) [ Fe(CN) 6 ]3– . (Atomic number of Fe = 26)
(m) pentaammine chloridocobalt (III) sulphate (U.P. 2018)
(n) hexammineplatinum (IV) chloride
Ans. EAN of Fe = Atomic number of metal ( Z )
(o) sodium pentacyanonitrosylferrate (III)
+ (2 ´ Number of ligands) ± (Charge on complex)
(p) tetraammine dibromidoplatinum (IV) chloride
= Z + 2L ± x
(q) hexaaqua ferrum (III) chloride
= 26 + 2 ´ 6 - 3 = 35
(r) hexaammineplatinum (IV) chloride
Q.7 Write the formulae of following compounds :
(s) potassiumtetracyanonickelate (II)
(i) Triamminetrichlorochromium(III)
(t) diammine cobalt (III) chloride sulphate
(iii) Potassium hexacyanoferrate(IIII) (U.P. 2018)
(u) [Co( NO 2 )( NH 3 ) 5 ]2+ Ans. (i) [Cr(NH 3 ) 3 Cl 3 ] (ii) K 3 [ Fe(CN) 6 ]
(v) Iron(III)hexacyanidoferrate (II) Q.8 Write the IUPAC names of following : (U.P. 2018)
(w) Potassium tetrahydroxidozincate (II) (i) [CoCl 2 ( en ) 2 ]SO 4 (ii) Na 3 [Cr(NO 2 ) 6 ]
(x) Tetra ammine copper (II) sulphate (iii) [ Ni(NH 3 ) 6 Cl 2 ] (iv) [ Mn(H 2O) 6 ]2+
Q.4 Write the IUPAC name of the following compounds : (v) [ Fe(H 2O) 6 ]Cl 3 (vi) [Co(NH 3 ) 4 × SO 4 ]NO 3
(i) K 3 [ Fe (C 2O 4 ) 3 ] (U.P. 2014, 19, 20) (vii) Fe 4 [ Fe(CN) 6 ]3
(ii) K 4 [Fe(CN) 6 ] (U.P. 2014, 15, 16, 20) Ans. (i) diochloridobisethylenediammine cobalt(IV)
(iii) [Zn(NH 3 ) 4 ]Cl 2 (U.P. 2014) sulphate
(ii) sodium hexanitritochromate(III)
(iv) K 2 [HgI 4 ] (U.P. 2014, 15) (iii) hexaamminedichloridonickel(II)
(v) Na[Ag(CN) 2 ] (U.P. 2014, 19) (iv) hexaaqua manganese(II)
(vi) [Cu(NH 3 ) 4 ]SO 4 (U.P. 2014, 15, 19, 20) (v) hexaaquuairon (III) chloride
(vi) tetraamminesulphatocobalt(III) nitrate
(vii) [Co(NH 3 ) 6 ]Cl 3 (U.P. 2014, 15, 20) (vii) iron(III) hexacyanoferrate(II)
(viii) [Ag(NH 3 ) 2 ][Ag(CN) 2 ] (U.P. 2014, 16) Q.9 How many ions are present in aqueous solution of
(ix) K[PtBr 3 (NH 3 )] (U.P. 2014, 16) complex CoCl 3 × 4NH 3 ? (U.P. 2018)
Ans. Two,
(x) K 3 [Co(C 2O 4 ) 3 ] (U.P. 2014, 16)
CoCl 3 × 4NH 3 Þ [CoCl 2 ( NH 3 ) 4 ]Cl
(xi) [Co(en) 2 Cl 2 ]Cl (U.P. 2014, 17) a [CoCl 2 ( NH 3 ) 4 ]+ Cl –
92

Short Answer Type Questions


Q.1 Explain Sidgwick’s law of effective atomic number Q.4 What are ligands ? How can they be classified on the
(EAN). (U.P. 2017) basis of charges ? (U.P. 2014, 20)
Ans. The total number of electrons possessed by the central Ans. Ligands : The atoms, molecules or ions which form
atom ion in complex including those donated by the co-ordinate bond with central atom by donating a pair
ligands is called effective atomic number of the central of electrons are called ligands.
atom. Classification of ligands on the basis of
EAN = Atomic number of metal - Electron donated by charges : Ligands are classified into following
metal + electron gained from ligand. classes.
For example: EAN of Fe in [Fe(CN) 6 ]3- . (i) Cationic ligands : Ligands which carry positive
In this ion oxidation number of Fe is +3 thus Fe atom charge are called cationic ligands.
donates 3 electron and there are six ligands which For example : NO + .
donate 12 electrons. Thus, (ii) Anionic ligands : The ligands which carry
EAN = 26 - 3 + 6 ´ 2 negative charge are called anionic ligands.
= 38 - 3 = 35 For examples : F - ,Cl - ,CO 23- ,SO 24- ,CN - etc.
Q.2 Explain on the basis of valence bond theory that
(iii) Neutral ligands : The ligands which neither
[Ni(CN) 4 ]2 ion with square planar structure is
carry any positive charge nor negative charge are
diamagnetic while tetrahedral geometry is called neutral ligands.
paramagnetic. (U.P. 2019) For example : H 2O, NH 3 , CO, RNH 2 etc.
Ans. Square planar geometry Q.5 What are co-ordination compounds ? Give difference
[Ni(CN) 4 ]2- between co-ordination compounds and double salts.
x - 4 = -2 Þ x = +2 (U.P. 2014)
Ni = ..... 3s 2 3 p 6 3d 8 4s 2 Ans. Co-ordination compounds : Co-ordination
Ni 2+ = ..... 3d 8 4s 0 compounds or complex salts are those addition
3d 4s 4p compounds which retain their identity even when
2+
Ni = dissolved in water and their properties are completely
different from those of the constituents.
For example : K 4 [Fe(CN) 6 ]
d s p2 Differences between double salts and
LLLL co-ordination compounds.
Square planar structure is dsp 2 hybridised and
Double salts Co-ordination compounds
electrons are paired so this structure is diamagnetic.
(complex compounds)
Tetrahedral geometry
Ni2+ 3d8 4s0 4p0 (1) Double salts are stable in Complex compounds are stable
solid state but lose their in solid as well as in solution.
identity in solution. They
L L L L dissociate into ions.
sp3 (2) In double salts, metals Metals may show higher valency
In this geometry hybridisation is sp 3 and there are two show their normal valency. in complex compounds.
unpaired electrons so it is paramagnetic. (3) The physical and chemical Their properties are completely
Q.3 On the basis of VBT, explain the structure and properties of double salts different from their constituent
magnetic nature of [FeF6 ]3- . (U.P. 2017) a r e si m i la r to t he i r compounds.
3- constituent compounds.
Ans. In [ FeF6 ] , Fe is in +3 oxidation state.
(4) In double salt, the nature of In complex compound, the bond
3d 4s 4p 4d bond is ionic. between the central metal ion
3+ and ligand is always co-ordinate.
Fe =

Q.6 Write IUPAC name of [Co (NH3 ) 4 Cl 2 ] + and with the


[FeF6]3+ =
help of its structural formulae, explain what type of
isomerism does it exhibit. (U.P. 2020)
F– F– F– F– F– F– Ans. Tetraamminedichloridocobalt (II) ion
sp3d2 hybridization II part is deleted for examination 2021 (Isomerism)
Structure — Octahedral Magnetic nature — Paramagnetic
93

Solution of NCERT Text Book Problems


Q.1 Explain the bonding in coordination compounds in Cl Cl
terms of Werner’s postulate. NH3 NH3 Cl NH3
Explain Werner’s theory. If [Co(NH 3 ) 5 Cl]Cl 2 is Co Cl Co
hydrolysed how many ions will be produced?
NH3 NH3 NH3 NH3
(U.P. 2016, 20)
Cl Cl
Ans. Werner proposed following postulates in his theory :
(i) In complexes, metal atom exhibit two types of (c) CoCl3·4NH3 (d) CoCl3·3NH3
valencies : Fig. Structure of Werner’s complexes
(a) primary or ionisable valency 3. CoCl 3 . 4 NH 3 : It is green complex and its one
(b) secondary or non-ionisable valency mole yields one mole AgCl with AgNO 3 .
(ii) Primary valencies are satisfied by anions only and
primary valency depends upon oxidation number
CoCl 3 × 4NH 3 º [CoCl 2 × ( NH 3 ) 4 ]+ + Cl -
[Fig. (c)]
of central metal atom. Metal atom may have
4. CoCl 3 . 3 NH 3 : It is violet coloured complex and
different primary valencies in different
gives no precipitate with AgNO 3 . It does not ionise
compounds. Primary valencies are represented by
in solution.
dotted lines.
(iii) Secondary valencies are satisfied by ligands and CoCl 3 × 3NH 3 º No ionisation
Limitations of Werner’s Theory
[Fig. (d)]
they represent co-ordination number of metal
atom. They are represented by thick lines and This theory left following questions unanswered :
their value depends upon size and charge of metal 1. Complexes are formed by some metals only. Why?
atom. 2. Geometry of complex is decided by secondary
(iv) When complex is dissolved in water then anions valencies. How?
attached by primary valencies are ionised while 3. This theory could not explain magnetic and optical
secondary valencies remain un-ionised. properties of complexes.
(v) Primary valencies are non-directional while Q.2 FeSO 4 solution mixed with (NH 4 ) 2 SO 4 solution in
secondary are directional. Geometry of 1 : 1 molar ratio gives the test of Fe 2+ ion but CuSO 4
complex is decided by secondary valencies. solution mixed with aqueous ammonia in 1 : 4 molar
Stereoisomerism is also due to different ratio does not give the test of Cu 2+ ion. Explain why ?
arrangements of secondary valencies. Ans. In the first case, a double salt
Structure of following complexes have been explained FeSO 4 × (NH 4 ) 2 SO 4 × 6 H 2O (Mohr’s salt) is formed.
on the basis of Werner’s theory : It gives Fe 2+ ions in solution. In the second case, the
1. CoCl 3 . 6 NH 3 : It is yellow coloured complex. Its coordination compound [Cu(NH 3 ) 4 ]SO 4 is formed.
one mole yields three moles of AgCl with excess of This complex does not give Cu 2+ ions in solution.
AgNO 3 which indicates that three Cl - are Q.3 Explain with two examples each of the following :
ionisable. Coordination entity, ligand, coordination number,
CoCl 3 × 6NH 3 º [Co(NH 3 ) 6 ]3 + +3Cl - coordination
heteroleptic.
polyhedron, homoleptic and
[Fig. (a)]
2. CoCl 3 × 5 NH 3 : It is purple coloured complex and Or Define the following : (U.P. 2019)
its one mole yields two mole AgCl with excess of (a) Complex ion (b) Co-ordination number
AgNO 3 . Ans. Co-ordination Sphere or Complex Sphere:
CoCl 3 × 5NH 3 º [CoCl × ( NH 3 ) 5 ]2+ + 2 Cl -
[Fig. (b)]
(i) Co-ordination sphere is written in bracket [ ]. It is
called complex ion also. In complex ion, there is a
Cl NH central metal atom ( M ), which is attached to ligands
3 Cl (electron donors) through co-ordinate bonds. Here
NH3 NH3 NH3 NH3 ligand is electron pair donor while central metal atom
Co Cl Cl Co Cl is electron pair accepter (Lewis acid).
NH3 NH3 NH3 NH3 ¬
¾ M ¬¾ L ¬¾ Ligand
Cl NH3 NH3
(a) CoCl3·6NH3 (b) CoCl3·5NH3 Central metal atom
e. g ., [CoCl 3 ( NH 3 ) 3 ] is a complex species in which
one Co(III) ion is co-ordinated to three Cl - ions and
three NH 3 molecules. Other examples are
[ Fe(CN ) 6 ]4- ,[ Ni( NH 3 ) 6 ]2+ ,[ Ni(CO) 4 ],[ PtCl 4 ]2- etc.
94

(ii) Atoms /ions present in co-ordination sphere are Ans. Denticity of a ligand represents the number of
not ionised or feebly ionised. co-ordinate bonds formed by it. In Latin, dentate =
(iii) Atoms /ions which are not part of complex ion are tooth or dentis which symbolize that ligands
kept outside the co-ordination sphere. These ions are surround metal atom in some way as teeth surround
called counter ions. e. g ., in K 4 [ Fe(CN ) 6 ], 4K + are tongue. On the basis of denticity, ligands are classified
counter ions and [ Fe(CN ) 6 ]4- is complex ion. Other into following classes.
(i) Monodentate legands : They contain only one
examples are [Cu( NH 3 ) 4 ], SO 4 , [Co( NH 3 ) 4 Cl 2 ] Cl,
donor atom that is they donate only one electron pair
[Co( NH 3 ) 5 Cl]Cl 2 , [Co( NH 3 ) 6 ]Cl 3 etc.
to central metal atom/ion. The may be neutral or
(iv) In solution, counter ion is ionised.
anionic,
Ligand : Atom/molecule or/ion, which form
e. g ., F - , Cl - , Br - , I - , NH3 , H2 O, CN- , OH- , O2- , NO -2 ,
co-ordinate bond with central metal atom by donating
its electron pair is called ligand. Ligands are electron CO, etc.
pair donors so they are Lewis bases. Important monodentate ligands and their donor
Thus complex ion is generally formed as: atoms are given below :
M a+ + xLb- ¾¾®[ MLx ][( a-( xb)] Important monodentate ligands
A complex compound may have more than one type of Donor
Name Formula Charge Name of ligand
ligands. e. g ., [Co( NH 3 ) 5 Cl]2+ contains two types of atom
ligands NH 3 and Cl - . 1. Water ·· O 0 Aqua or aquo
H2 O ··
Coordination number : The number of
co-ordinate bonds formed by central metal atom refers 2. Ammonia ·· N 0 Ammine
N H3
to its co-ordination number. With monodentate
ligands it is equal to the number of ligands. 3. Halide ion X - (X =F, X -1 Halido
Example : Cl, Br, I) (flourido,
[Ni(NH 3 ) 4 ]Cl 2 —Co-ordination number of Ni is four. chlorido,
bromido,
[Fe(H 2O) 6 ]Cl 3 —Co-ordination number of Fe is six. iodido) or
Co-ordination number 4 and 6 are most common. halide
Other co-ordination numbers are less common. A brief 4. Hydride OH- O -1 Hydroxo or
discussion of different co-ordination numbers is given ion (hydroxide)
below.
5. Carbon CO O 0 Carbonyle
(i) Co-ordination number 1, 2 and 3 : These are monoxide
rare co-ordination numbers. Some organometallic
compounds of Tl (I) and In (I) have co-ordination 6. Nitrogen NO N 0 Nitrosyle
oxide
number equal to one. In [CuCl 2 ]– , [ Ag( NH 3 ) 2 ]+ and
[Hg(CN) 2 ] co-ordination number is two. In
7. Cynide ion CN- N or C -1 Cyno (or
cynido)
[Zn(OH) 3 ]- and [HgI 3 ]- co-ordination number is
8. Nitrite ion NO-2 N -1 Nitro or nitrito
three.
-
(ii) Co-ordination number 4, 5 and 6 : Six is the (ONO )
most common co-ordination number followed by four. 9. Sulphate SO2- O -2 Sulphato
4
Five is less common. ion
Example : [Ni(NH 3 ) 4 ]2+ , [Cu(NH 3 ) 4 ]2+ ,
(ii) Bidentate or didentate ligands : These
[Fe(CO) 5 ], [Ni(CN) 5 ]3- , [Cr(H 2O) 6 ]3+ , [Ti(H 2O) 6 ]3+
ligands contain two donor atoms per ligand and
etc. central metal atom co-ordinates with ligands at two
Q.4 What is meant by unidentate (monodentate), sites. For example :
didentate and ambidentate ligands ? Give two
:

examples for each. O== C—O– CH2 — NH2


Or Explain ambidentate ligands with examples.
:

O== C—O– CH2 — NH2


(U.P. 2017) Oxalato (ox) Ethylediamine (en)
Or What are monodentate ligands? Explain with
(iii) Tridentate ligand : They contain three donor
example. (U.P. 2018, 20)
atoms per ligand, which form three bonds with central
Or What are ligand? Define monodentate and bidentate
metal atom.
ligands with one example. Write IUPAC names of ••
following compounds : (U.P. 2020) •• CH2CH2NH2
NH ••
(i) [ Fe( H 2O) 6 ]Cl 3 CH2CH2NH2
(ii) K 4 [ Fe(CN ) 6 ] Diethylene triammine (dien)
Here, all the three donor atoms are N-atoms.
95

(iv) Tetra dentate ligands : They contain four (c) K 2[PdCl 4 ] (d) K 3 [Cr(C 2O 4 ) 3 ]
donor atoms per ligand which co-ordinate with central (e) [Pt(NH 3 ) 2 Cl 2 ] (f) [Pt(NH 3 ) 6 ]4+
metal atom. For example : (g) K 2[Ni(CN) 4 ] (h) [CuBr4 ]2–
••
CH2—NHCH2CH2NH2 (i) [Co(NH 3 ) 5 (ONO)]2+ (j) [Co(NH 3 ) 5 (NO 2 )]2+
•• Q.7 Using IUPAC norms, write the systematic names of
CH2—NHCH2CH2NH2
Triethylene tetrammine
the following :
(a) [Co(NH 3 ) 6 ]Cl 3 (b) [Co(NH 3 ) 4 Cl(NO 2 )]Cl
(v) Pentadentate ligands : They contain five
(c) [Ni(NH 3 ) 6 ]Cl 2
donor atoms per ligand which co-ordinate with central
(d) [Pt(NH 3 ) 2 Cl(NH 2CH 3 )]Cl
metal atom. For example :
– (e) [Mn(H 2O) 6 ]2+ (f) [Co(en) 3 ]3+
• • CH2COO 3+
CH2—N (g) [Ti(H 2O) 6 ] (h) [NiCl 4 ]2–
CH2COO– (i) [Ni(CO) 4 ]
•• Ans. (a) Hexaamminecobalt(III) chloride
CH2—NH—CH2COO–
Ethylene diammine acetate ion
(b) Tetraamminechloridonitrito-N-cobalt(III)
chloride
(vi) Hexa-dentate ligands : They contain six
(c) Hexaamminenickel(II) chloride
donor atoms per ligand which co-ordinate with central
(d) Diamminechlorido (methyl amine) platinum(II)
metal atom.
– chloride
• • CH2COO

OOCH2C • • (e) Hexaaquamanganese(II) ion
N—CH2—CH2—N

OOCH2C CH2COO– (f) tris(ethane-1, 2-diamine) cobalt(III) ion
Ethylen diammine tetra acetatoin (g) Hexaaquatitanium(III) ion
Two N and four O-atoms act as donor. (h) Tetrachloridonickelate(II) ion
Ambidentate ligands : Some ligands have more than (i) Tetracarbonyl nickel(0)
one donor atoms but actually only one of them ligates Q.8 List the various types of isomerism possible for
with central metal atom. Such ligands are called coordination compounds, giving an example of each.
ambidentate ligands. Or Explain structural isomerism in coordination
O compounds with the help of suitable example.
(a) M ¬ N Nitro group (N-donor atom) (U.P. 2017)
O Or Explain with an example the ionisation isomerism
(b) M ¬ O — N == O Nitrito group (O-donor atom) in complex compounds. (U.P. 2018, 19)
(c) M ¬ CN Cyno (C-donor atom) Ans. Deleted for examination 2021.
(d) M ¬ NC Isocyno (N-donor atom) Q.9 How many geometrical isomers are possible in the
(e) M ¬ SCN S-thiocynute (S-donor atom) following coordination entities ?
(f) M ¬ NCS N-thiocynute (N-donor atom) (a) [Cr(C 2O 4 ) 3 ]3– (b) [CoCl 3 (NH 3 ) 3 ]
Q.5 Specify the oxidation numbers of the metals in the Ans. Deleted for examination 2021.
following coordination entities : Q.10 Draw the structures of optical isomers of :
(a) [Co(H 2O)(CN)(en) 2 ]2+ (b) [PtCl 4 ]2– (a) [Cr(C 2O 4 ) 3 ]3– (b) [PtCl 2 (en) 2 ]2+
(c) [Cr(NH 3 ) 3 Cl 3 ] (d) [CoBr2 (en) 2 ]+ (c) [Cr(NH 3 ) 2 Cl 2 (en)]+
(e) K 3 [Fe(CN) 6 ] Ans. Deleted for examination 2021.
Ans. (a) x = +3 (b) x = +2 Q.11 Draw all the isomers (geometrical and optical) of :
(c) x = +3 (d) x = +3 (e) x = +3 (a) [CoCl 2 (en) 2 ]+
Q.6 Using IUPAC norms, write the formulae of the (b) [Co(NH 3 )Cl(en) 2 ]2+
following :
(c) [Co(NH 3 ) 2 Cl 2 (en)]+ .
(a) Tetrahydroxozincate(II)
(b) Hexaamminecobalt(III) sulphate Ans. Deleted for examination 2021.
(c) Potassium tetrachloridopalladate(II) Q.12 Write all the geometrical isomers of
(d) Potassiumtrioxalatochromate(III) [Pt(NH 3 )(Br)(Cl)(py)] and how many of these will
(e) Diamminedichloridoplatinum(II) exhibit optical isomerism ?
(f) Hexaammineplatinum(IV) Ans. Deleted for examination 2021.
(g) Potassium tetracyanonickelate(II) Q.13 Aqueous copper sulphate solution (blue in colour)
(h) Tetrabromidocuprate(II) gives (a) a green precipitate with aqueous potassium
(i) Pentaamminenitrito-O-cobalt(III) fluoride and (b) a bright green solution with aqueous
(j) Pentaamminenitrito-N-cobalt(III) potassium chloride. Explain these experimental
Ans. (a) [Zn(OH) 4 ]2– (b) [Co(NH 3 ) 6 ]2 (SO 4 ) 3 results.
96

Ans. (a) Cu 2+ + 4F – ¾®[CuF4 ]2– , series. It is an experimentally determined series.


Green ppt The order is difficult to explain due to involvement of
(b) Cu 2+ + 4Cl – ¾® [CuCl 4 ]2– both s and p-bonding. Some ligands in
Green solution spectrochemical series are given below :
Q.14 What is the coordination entity formed when excess Weak field ligands
of aqueous KCN is added to an aqueous solution of I - < Br - < S 2- < SCN - < Cl - < N -3 , F - <
copper sulphalte ? Why is that no precipitate of
copper sulphide is obtained when H 2S(g) is passed Urea, OH - < Oxalate,
2- -
through this solution ? O < H 2O < NCS < EDTA < py , NH 3 <
Ans. CuSO 4 + 4KCN ¾® K 2[Cu(CN) 4 ] + K 2SO 4 - SO 23- < bipy,
en ~
because the solution does not contain free Cu 2+ ions phen < NO -2 < CH -3 < C 6 H 5 < CN - < CO
instead they are present in coordination entity
[Cu(CN) 4 ]2– . strong field ligands
Generally the order depends upon the donor atom
Q.15 Discuss the nature of bonding in the following
and is in the following order :
coordination entities on the basis of valence bond
C-donor > N-donor > O-donor > Halogen donor
theory :
(a) [Fe(CN) 6 ]4– (b) [FeF6 ]3– Q.18 What is crystal field splitting energy ? How does the
magnitude of D o decide the actual configuration of
(c) [Co(C 2O 4 ) 3 ]3– (d) [NiCl 4 ]2–
d-orbitals in a coordination entity ?
Ans. (a) [Fe(CN)6 ] 4– : d 2 sp 3 -hybridisation, octahedral,
Ans. See Long answer type question 1.
diamagnetic Q.19 [Cr(NH 3 ) 6 ]3+ is paramagnetic while [Ni(CN) 4 ]2– is
(b) [FeF6 ]3– : sp 3 d 2 -hybridisation, octahedral, para diamagnetic. Explain why ?
-magnetic. Ans. [Cr(NH 3 ) 6 ]3+ complex ion is octahedral,
(c) [Co(C 2O 4 )3 ] 3– : d 2 sp 3 -hybridisation, octa-
hybridisation on the metal ion is d 2 sp 3 and complex
hedral, diamagnetic
ion is paramagnetic.
(d) [NiCl 4 ] 2– : sp 3 -hybridisation, tetrahedral, para-
[Ni(CN) 4 ]2– complex ion is square planar ,
magnetic. 2
hybridisation on the metal ion is dsp and complex
Q.16 Draw figure to show splitting of degenerated orbitals
in an octahedral crystal field. ion is diamagnetic.
Ans. Q.20 A solution of [Ni(H 2O) 6 ]2+ is green but a solution of
[Ni(CN) 4 ]2– is colourless. Explain.
e g , ( d z2 , d x 2 - y 2 )
Ans. Due to higher CFSE.
+6 Dq. CFSE (Do)= 10 Dq. Q.21 [Fe(CN) 6 ]4– and [Fe(H 2O) 6 ]2+ are of different
Energy

–4 Dq. Bari centre colours in dilute solutions. Why ?


Ans. Because for different ligands value of CFSE is
t 2 g ( d xy , d yz , d xz ) different.
Degenerate d-orbitals
in free ion (M n+) Q.22 Discuss the nature of bonding in metal carbonyls.
Ans. Bonding in Metal Carbonyls : The metal-carbon
Splitting of d-orbitals in a octahedral crystal field bond in metal carbonyls has s as well as p character.
Q.17 What is spectrochemical series ? Explain the (i) s-overlap : In a sigma bonded complex, the lone
difference between weak field ligand and a strong pair of electrons is present on the bonding orbital of
field ligand. carbon monoxide. This bonding orbital containing
Ans. The value of D depends upon nature of ligands. lone pair interacts with the empty d-orbital of the
Ligands which causes only a small degree of crystal metal to form a metal-carbon bond as shown figure:
field splitting are termed as weak field ligands
while those ligands which cause large splitting are +
called strong field ligands. The common ligands
can be arranged in ascending order of CFSE caused
by them. This series remains practically constant for
(ii) p-overlap : In addition to this, the antibonding
different metals and is known as spectrochemical
orbitals of CO can also overlap with the filled
97

d-orbitals of the metal resulting in back bonding as It is important to note here that the s-bond is in the
explained earlier. Thus metal carbonyls become much nodal plane of the s-electrons whereas p-overlap is
more stable compounds due to multiple bonding in perpendicular to the nodal plane.
them.

Q.23 Give the oxidation state, d-orbital occupation and (c) cis-[Cr(en) 2 Cl 2 ]Cl (d) [Mn(H 2O) 6 ]SO 4
coordination number of central metal ion in the
Ans. Except the complex (b), all other complexes are
following complexes :
octahedral.
(a) K 3[Co(C 2O 4 ) 3 ] (b) (NH 4 ) 2[CoF4 ]

Complex Oxidation state of metal in Coordination no. d-orbital occupation


complex ion

(a) K 3 [Co(C 2O 4 )3 ] x - 6 + 3 = 0, x = +3 6 (as C 2O 2–


4 is didentate) Co 3+ = 3d 6 = t 26g eg0

(b) (NH 4 ) 2[CoF 4 ] x - 4 + 2 = 0, x = +2 4 Co 2+ = 3d 7 = eg4 t 23g

(c) cis-[Cr(en)2 Cl 2 ]Cl x + 0 - 2 - 1 = 0, x = +3 6 (en is didentate) Cr 3+ = 3d 3 = t 23g eg0

(d) [Mn(H 2O)6 ]SO 4 x - 2 = 0, x = +2 6 Mn 2+ = 3d 5 = t 23g eg2

Q.24 Write down the IUPAC name for each of the following Shape = Octahedral
complexes and indicate the oxidation state, electronic Oxidation state of Cr : x - 3 + 0 = 0, x = +3
configuration and coordination number. Also give
E.C. of Cr 3+ = 3d 3 = t 23g e g0 , n = 3
stereochemistry and magnetic moment of the
complex : Magnetic moment (m ) = n( n + 2)
(a) K[Cr(H 2O) 2 (C 2O 4 ) 2 ] × 3H 2O = 15 = 3.87 BM
(b) Cr[Cl 3 (py) 3 ] (c) K 4 [Mn(CN) 6 ] (c) Potassium hexacyanomanganate(II).
(d) [Co(NH 3 ) 5 Cl]Cl 2 (e) Cs[FeCl 4 ] Coordination number of Mn = 6.
Ans. (a) Potassium diaquadioxalatochromate(III) Shape = Octahedral
hydrate. Oxidation state of Mn : x - 6 + 4 = 0, x = +2
Coordination no. = 6. Shape = octahedral. E.C. of Mn 2+ = 3d 5 = t 25g e g0 , n = 1
Oxidation state of Cr : Magnetic moment (m ) = n( n + 2)
= 3 = 1.73 BM
x + 0 + 2 ( -2) + 1 = 0, x = +3 (d) Pentaamminechloridocobalt(III) chloride.
E.C. of Cr 3+ = 3d 3 = t 23g e g0 , Coordination number of Co = 6
Shape = Octahedral
Unpaired electrons ( n ) = 3 Oxidation state of Co : x + 0 - 1 - 2 = 0
or x = +3
Magnetic moment (m ) = n( n + 2) = 3 ´ 5
E.C. of Co 3+ = 3d 6 = t 26g e g0 , n = 0
= 15 BM = 3.87 BM
(b) Trichloridotripyridinechromium(III). Magnetic moment (m ) = 0

Coordination number of Cr = 6
98

(e) Caesium tetrachloridoferrate(III). Thus, three ions are produced.


Coordination number of Fe = 4 Q.29 Amongst the following ions, which one has the
Shape = tetrahedral highest magnetic moment ?
Oxidation state of Fe : x - 4 + 1 = 0 or x = +3 (a) [Cr(H 2O) 6 ]3+ (b) [Fe(H 2O) 6 ]2+
E.C. of Fe 3+ = 3d 5 = e g2t 23g , n = 5 (c) [Zn(H 2O) 6 ]2+
Ans. The oxidation states are : Cr (III), Fe (II) and Zn (II)
Magnetic moment (m ) = n( n + 2)
E.C. of Cr 3+ = 3d 3 , unpaired electrons = 3
= 35 = 5.92 BM E.C. of Fe 2+ = 3d 6 , unpaired electrons = 4
Q.25 What is meant by stability of a coordination E.C. of Zn 2+ = 3d10 , unpaired electrons = 0
compound in solution ? State the factors which Hence, (b) has highest magnetic moment.
govern stability of complexes. Q.30 The oxidation number of cobalt in K[Co(CO) 4 ] is :
Ans. See Long answer type question 2. (a) +1 (b) +3 (c) –1 (d) –3
Q.26 What is meant by chelate effect ? Give an example.
Ans. –1
Ans. Chelation is possible in didentate and polydendate
ligands. The donor atoms coordinate with the central Q.31 Amongst the following, the most stable complex is :
metal ion in such a way that a 5 or 6 membered ring (a) [Fe(H 2O) 6 ]3+ (b) [Fe(NH 3 ) 6 ]3+
is formed. It leads to greater stability of the complex (c) [Fe(C 2O 4 ) 3 ]3– (d) [FeCl 6 ]3–
compounds. It is called chelate effect. Ans. C, it is chelate complex.
Q.27 Discuss briefly giving an example in each case the
Q.32 What will be the correct order for the wavelength of
role of coordination compounds in : (U.P. 2018)
absorption in the visible region of the following?
A. biological systems
[Ni(NO 2 ) 6 ]4– , [Ni(NH 3 ) 6 ]2+ , [Ni(H 2O) 6 ]2+
B. analytical chemistry
C. medicinal chemistry Ans. The metal ion is same, the increasing field strength of
D. extraction/metallurgy of metals the ligands from the spectrochemical series are in the
order : H 2O < NH 3 < NO –2 .
Ans. Deleted for question 2021.
Q.28 How many ions are produced from the complex, Thus, the energies absorbed for excitation will be in
Co(NH 3 ) 6 Cl 2 in solution ? the order :
(a) 6 (b) 4 (c) 3 (d) 2
[Ni(H 2O) 6 ]2+ < [Ni(NH 3 ) 6 ]2+ <[Ni(NO 2 ) 6 ]4–
Ans. Coordination number of cobalt = 6. Hence, the
complex is [Co(NH 3 ) 6 ]Cl 2 . It ionizes in the solution hc
Since, E = , the wavelength absorbed will be in the
as: l
aq opposite order.
[Co(NH 3 ) 6 ]Cl 2 ¾¾® [Co(NH 3 ) 6 ]2+ + 2Cl –

vvv
99

Chapter

10 HALOALKANES AND
HALOARENES
Syllabus : Haloalkanes: Nomenclature, Nature of C—X bond, Physical and Chemical properties, Mechanism of substitution
reactions, Polarized rotation (Walden inversion).
Haloarenes: Nature of C—X bond, Substitution reactions, (Directive influence of halogen atom in monosubstituted compounds
only,) *Uses and effects on environment of Dichloromethane, Trichloromethane, Tetrachloromethane,
Iodoform, freons and DDT.

* Topics Deleted for Examination 2021

Objective Questions
1. The number of possible isomers of the compound (a) phenol (b) aniline
C 3 H 6Cl 2 are : (U.P. 2013) (c) benzene (d) benzoic acid
(a) 2 (b) 4 (c) 6 (d) 8 7. When chloroform is exposed to air and sunlight, it is
2. The product obtained when n-propyl bromide reacts slowly oxidised and gives : (U.P. 2018)
with KOH(alc.) is: (U.P. 2017) (a) phosgene (b) formic acid
(a) propane (b) propyne (c) propene(d) propanol (c) formyl chloride (d) methyl chloride
3. Phosgene is: (U.P. 2017) 8. The product in the following reaction is :
(a) PH 3 (b) POCl 3 (c) CS 2 (d) COCl 2 C 6 H 6 + Cl 2 ¾ Sunlight
¾ ¾¾® Product (U.P. 2016)
4. What is the product obtained when chloroform reacts (a) C 6 H 6Cl 6 (b) o - C 6 H 4 Cl 2
with conc. HNO 3 ? (U.P. 2017) (c) p - C 6 H 4 Cl 2 (d) C 6 H 5Cl
(a) CHCl 3 × NO 2 (b) C( NO 2 )Cl 3 9. Haloforms are trihalogen derivatives of : (U.P. 2018)
(c) CHCl 3 × HNO 3 (d) CHCl 2 × NO 2 (a) methane (b) ethane
5. On oxidation, CHCl 3 gives: (U.P. 2017, 18) (c) propane (d) benzene
(a) phosgene (b) formic acid 10. In the presence of light, chloroform oxidises to
(c) carbon tetrachloride (d) chloropicrin produce : (U.P. 2020)
6. Product obtained by reaction of NH 3 on (a) Carbon tetrachloride (b) Carbonyl chloride
chlorobenzene in the presence of cuprous oxide is : (c) Dichloromethane (d) None of these
(U.P. 2017)
Answers
1. (b) 2. (c) 3. (d) 4. (b) 5. (a) 6. (b) 7. (a) 8. (a) 9. (a) 10. (b)

Very Short Answer Type Questions


Q.1 Sodium and two mole of compound ( A ) react to form Ans. CH 3CH 2CH 2 Br + 2Na + BrCH 2CH 2CH 3
ethane. What is compound ( A )? (U.P. 2014, 16) ¾ Dry
¾ ¾¾ether
® CH 3CH 2CH 2CH 2CH 2CH 3 + 2NaBr
Or Sodium and two moles of a compound A react in Hexane
presence of dry ether to form ethane. What is It is Wurtz reaction.
compound A? Write chemical equation also. Q.3 In S N reactions, halogen atom of aryl halides, is less
(U.P. 2018) reactive than halogen atom of alkyl halides, why ?
Ans. Compound ( A ) is methyl halide. It is Wurtz reaction. OR Alkyl halides give alcohol with NaOH( aq.) while
CH 3 X + 2Na + CH 3 X ¾ Dry ether
¾ ¾¾ ® CH 3 . CH 3 + 2NaX aryl halides do not why ?
Q.2 Write the structural formula and IUPAC name of OR Aryl halides show S N reactions less readily than
alkane obtained by reaction between
alkyl halides, why ?
1-bromopropane and sodium in presence of ether.
OR Chlorine atom of chlorobenzene is less reactive
What is the name of reaction ? (U.P. 2016)
than chlorine atom of ethyl chloride, why ?
(U.P. 2015)
100

Ans. In alkyl halides C — X bond is single s-bond while in Q.5 Write one ring substitution reaction of chlorobenzene
aryl halides C — X bond acquires some partial double giving chemical equation. (U.P. 2017)
bond character due to resonance. Or How is chlorobenzene halogenated in presence of
Q.4 (i) Write the structure of all possible isomers of halogen carrier? Write related equation.(U.P. 2018)
C 2 H 2Cl 2 . (U.P. 2014) Cl Cl Cl
(ii) Arrange the following in increasing order of Cl
activity during nucleophilic reactions. Cl2/FeCl3
CH 3 F, CH 3 Br, CH 3 I, CH 3Cl (U.P. 2014) Ans. ¾¾¾® +
or AlCl3
Ans. (i) The possible isomers of C 2 H 2Cl 2 are :
(a) H—C—Cl (b) H—C—Cl
Cl
o/p dichlorobenzene
H—C—Cl Cl—C—H
Cis-1,2-dichloro ethane Trans-1,2-dichloro ethane Here FeCl 3 or AlCl 3 acts as halogen carrier. It is E +
(c) H—C==C—Cl substitution reaction.
Q.6 Write equation of any of nucleophilic substitution
reaction of ethyl bromide. (U.P. 2018)
H Cl
1,1-dichloro ethane
Ans. C 2 H 5 Br + NaOH aq. ¾® C 2 H 5OH+ NaBr
Here OH – is nucleophile (OH – ).
(ii) CH 3 F < CH 3Cl < CH 3 Br < CH 3 I

Short Answer Type Questions


Q.1 What are haloarenes ? Write their two main corresponding halogen acid, the diazo group is
substitution reactions. Write formula and uses of replaced by halogen atom. Such type of reactions are
Freon. (U.P. 2019) called Sandmeyer reactions.
Cu Cl 2
Ans. Haloarenes are halogen derivatives of aromatic C 6 H 5 N +2 Cl - ¾ ¾2¾¾® C 6 H 5Cl + N 2
HCl
hydrocarbons Cu Br
Cl CH2Br C 6 H 5 N +2 Cl - ¾ ¾2¾
¾2
® C 6 H 5 Br + N 2
HBr
e.g., etc. (ii) Dehydrohalogenation : When alkyl halides
heated with alcoholic KOH, molecules of hydrogen
Chlorobenzene Benzyl bromide halides are eliminated and alkene is obtained and
The aryl halides in which halogen atom is directly reaction is called dehydrohalogenation.
attached to benzene ring do not give Nu - substitution For example :
D
reactions easily. On the other hand in which halogen (i) R — CH 2 — CH 2 — X + KOH(alc) ¾¾ ®
atoms is attached to side chain, Nu - substitution R — CH == CH 2 + KX + H 2O
reaction takes place easily. Q.3 Write the structural formula of 2-methyl-propene.
Cl OH
Write its addition reaction of HBr in the presence of
Very high T / P peroxide and write the name of this law.(U.P. 2016)
+ NaOH( aq.) ¾¾¾¾¾® Or What is peroxide effect? Explain with an example.
- NaCl
(U.P. 2016)
CH2Cl CH2OH Ans. The addition of halogen halide (such as HBr) on an
asymmetrical alkene in the presence of peroxide,
takes place opposite to Markownikoff’s rule. This
+ NaOH( aq.) ¾¾¾¾®
effect is called peroxide effect or Kharasch
effect.
They also undergo E +- substitution at benzene ring. For example :
Cl Cl Cl The addition reaction of HBr to 2-methyl propene in
NO2 the presence of peroxide takes place according to the
Conc.HNO /H SO
3 2 4
¾¾¾¾¾¾¾® + peroxide effect.
Peroxide
CH3—C==CH2+HBr ¾¾¾® CH3—CH—CH2Br
NO2
CH3 CH3
Q.2 Write short notes on the following : 2-methyl propene 1-bromo 2-methyl
(i) Sandmeyer’s reaction (U.P. 2016) propane
(ii) Dehydrohalogenation (U.P. 2015) Q.4 Write the structural formula of but-2 ene and show
Ans. (i) Sandmeyer’s reaction : When a diazonium addition reaction of HBr in presence and in absence of
salt solution is heated with cuprous halide and peroxide. (U.P. 2016)
101

Ans. The structural formula of but-2- ene is (ii) Reimer-Tiemann reaction : On heating
4 3 2 1
CH 3 — CH == CH— CH 3 chloroform with conc. alkali and phenol at 60º C,
But - 2- ene or butene - 2 o-hydroxybenzaldehyde (salicylaldehyde) is formed.
(i) The addition reaction of HBr to but-2-ene in the Such types of reaction is called Reimer-Tiemann
absence of peroxide takes place according to the reaction.
OH OH
Markownikoff’s rule.
d+ d+ CHO
CH 3 — CH == CH — CH 3 + H Br ¾® 60ºC
+3NaCl+2H2O
+CHCl3+3NaOH ¾¾®
CH3 — CH— CH 2 — CH 3
| Phenol Salicylaldehyde

Br
(ii) In the presence of peroxide the addition takes Q.7 Write short notes on the following :
place opposite to Markownikoff’s rule (peroxide (1) Carbylamine reaction (U.P. 2014, 15)
effect). (2) Darzen reaction (U.P. 2016)
CH 3 — CH == CH — CH 3 + HBr ¾(C ¾6H¾5COO)2
¾¾® Ans. (1) Carbylamine reaction : When chloroform is
heated with primary amine in the presence of
CH 3 — CH 2 — CHBr — CH 3
alcoholic KOH solution, isocyanide or carbylamine is
Q.5 Write short notes on the following : formed which has very bad smell.
(i) Wurtz reaction (U.P. 2016)
For examples :
(ii) Wurtz-Fittig reaction (U.P. 2015, 16, 18)
(i) C 2 H 5 NH 2 + CHCl 3 + 3KOH(alc. ) ¾®
Ans. (i) Wurtz reaction : When an alkyl halide is heated Ethylamine
with sodium metal in the presence of dry ether, an C 2 H 5 NC + 3KCl + 3H 2O
alkane is obtained and reaction is called Wurtz (ii) C 6 H 5 NH 2 + CHCl 3 + 3KOH(alc. ) ¾®
reaction. Aniline
C 6 H 5 NC + 3KCl + 3H 2O
For example : Phenylisocyanide
(a) CH 3 I +2Na + ICH 3 ¾ Dry ether
¾ ¾¾ ® CH 3 × CH 3 + 2NaI
Methyl Ethane (2) Darzen reaction : When primary alcohol
iodide distilled with thionyl chloride (SOCl 2 ) in the
(b) C 2 H 5 Br + 2Na + BrC 2 H 5 ¾ Dry
¾ ¾¾ether
® presence of small amount of pyridine, alkyl chloride
Ethyl bormide
is formed.
C 2 H 5 × C 2 H 5 + 2NaBr D
ROH + SOCl 2 ¾¾ ® RCl + SO 2 + HCl
Butane
D
(ii) When aryl halide and alkyl halide are heated CH 3CH 2OH + SOCl 2 ¾¾ ® CH 3CH 2Cl + SO 2 + HCl
Ethanol Thionyl
with sodium metal in the presence of dry ether, an chloride
aromatic hydrocarbon is obtained and the Q.8 Write notes on the following :
reaction is called Wurtz-Fittig reaction. (i) Hunsdiecker reaction (U.P. 2015)
For example : (ii) Saytzeff’s rule (U.P. 2016, 18)
Cl CH3
(iii) Haloform or Iodoform reaction.
Dry ether (U.P. 2014, 18, 20)
+2Na+CH3Cl ¾¾¾® + 2NaCl
Ans. (i) Hunsdiecker reaction : When salt of a
Chlorobenzene Toluene carboxylic acid is heated with bromine in the
Q.6 Write notes on the following : presence of an inert solvent like CCl 4 bromoalkane is
(i) Markownikoff’s rule (U.P. 2014) formed and reaction is called Hunsdiecker
(ii) Reimer-Tiemann reaction (U.P. 2014, 16) reaction.
Ans. (i) Markownikoff’s rule : The addition of RCOOAg + Br2 ¾ CCl 4
¾¾ ® RBr + CO 2 + AgBr
hydrogen halide to alkenes takes place according to 70ºC Bromoalkane
the Markownikoff’s rule. According to this rule, C 2 H 5COOAg + Br2 ¾ CCl
¾¾ 4
® C 2 H 5 Br + AgBr + CO 2
‘‘when addition take place in an asymmetrical alkene, 70ºC

the negative part of halogen halide link with the (ii) Saytzeff’s rule : According to Saytzeff’s rule,
carbon atom containing the least number of hydrogen ‘‘If an alkyl halide can eliminate the hydrogen in two
atoms.’’ different ways, that alkene will be formed in excess
For example : in which carbon atoms joined by double bond are
d+ d-
more alkylated.’’
CH 3 — CH == CH 2 + H Br ¾® CH3 — CH— CH 3
Propene -1 |
Br
2-bromopropane
102

CH 3 — CH 2 — CH— CH 3 CH 3CO — (or —COCH 3 ) group is formed during the


| reaction (e. g ., C 2 H 5OH etc.) react with halogens
Br (Cl 2 , Br2 , I 2 ) in the presence of alkali to form
CH3—CH==CH—CH 3 haloform and reaction is called haloform
Butene-2 (major)
AlC. KOH reaction.
D
RCOCH 3 + 3 X 2 + 4NaOH ¾¾ ®
CH3CH2—CH==CH 2 CHX 3 + RCOONa + 3NaX + 3H 2O
Butene-1(minor) When iodine is used as halogen, then reaction is
(iii) Haloform reaction : The compounds called Iodoform reaction.
containing CH 3CO —group (e. g ., acetone, CH 3CHO + 3I 2 + 4NaOH ¾®
acetaldehyde) or those compounds in which CHI 3 + HCOONa + 3NaI + 3H 2O
Iodoform

Long Answer Type Questions


Q.1 What are electrophilic substitution reactions? Explain o/p positions i.e., why the incoming E + binds at o/p
its mechanism with an example of Aryl halide. positions.
(U.P. 2019) +d
Ans. Electrophilic Substitution Reactions : Those X X+ X+ X+ X
reactions in which a part of substrate is substituted by
–d –d
electrophile ( E + ) are known as electrophilic
substitution reactions. These reactions are
characteristic reactions of aromatic ring because the –d
p-e - cloud present in both sides of aromatic ring X X X +E+
attracts the E + .
E E E –H+

e.g., + E + ¾® + H+
E
o– p– o–
In aryl halides, halogen atom shows two electronic
effects on benzene ring : Q.2 Explain the mechanism of monomolecular
(a) –I effect, which reduces the electron density on nucleophilic substitution ( S N 1 ) and bimolecular
benzene ring and so aryl halides are less reactive in nucleophilic substitution ( S N 2 ) reactions in
these reactions than benzene ring. haloalkane by examples. (U.P. 2019)
(b) +M effect, which increases the electron density Ans. Mechanisms of S N 1 and S N 2 reactions :
on benzene ring and increased e - density appears at

SN1 reactions SN 2 reactions


1. It is a bistep process in which first step is slow and so rate deter- 1. It is monostep reaction which proceeds via transition state. Nu -
mining. In this step X - goes out leaving behind R + . In second always attacks from backside. In transition state attachment of
step, which is fast Nu - attacks on R + to form product. Nu - and removal of X - occur simultaneously and carbon atom
-
R — X ¾ Slow ¾¾- ® R
+
¾ Nu
¾¾® R — Nu seems to be pseudo-pentavalent in T.S.
-X Fast Nu - ¾® R — X ¾® [ Nu - - - R - - - X ]- ¾® Nu — R + X -
2. In this mechanism there may be retention or inversion of 2. Configuration of product is always inverted and it is called
configuration, because Nu - may attack on R + from both front Walden inversion.
or backside. Nu–
I
Nu–—C
C—X ¾®Nu --- C --- X
Inversion
C—X ¾® Cl
X– –
¾® Nu --- C +X

I C—Nu
II II
Nu– Retention
103

3. Reactivity of halides in SN1 reactions is 3. Reactivity order of halides is


{t > s > p > CH3 X {t < s < p < CH3 X
S
144244 3 S
144244 3
N1 S N1 S
N2 N2
Halides with less steric hindrance one more reactive.
4. Weak electrophiles favours SN1 reactions and wait to be invited 4. Strong nucleophiles favours SN 2 mechanism and they replace
by carbocation. X -.
5. Polar solvents favour SN1 mechanism. 5. Non-polar solvents favour SN 2 mechanism.

Q.3 Write chemical equation of two general methods of cuprous chloride and HCl.This reaction is known as
preparation of haloalkanes. Write their reactions Sandmeyer’s reaction.
with: (U.P. 2020) (a) NaNO 2 + HCl ¾® NaCl + HNO 2
(i) KOH (alcoholic) (ii) AgCN + –
Ans. General methods of preparation of (b) C 6 H 5 NH 2 + HNO 2 + HCl ¾ 0¾¾
- 5°C
® C 6 H 5 N 2Cl
haloalkanes: Aniline Benzene
diazonium
(1) By action of PCl 5 on alcohols: chloride
ROH + PCl 5 ¾® RCl + POCl 3 + HCl + 2H 2O
e. g ., C 2 H 5OH + PCl 5 ¾® C 2 H 5Cl + POCl 3 + HCl Cu 2Cl 2 /HCl
(2) By addition of HX on alkenes: (c) C 6 H 5 N 2Cl ¾ ¾ ¾ ¾¾® C 6 H 5Cl + N 2
60°C
| | | |
(i) Reaction with conc. HNO 3 /conc. H 2SO 4 :
— C == C — + HX ¾® — C — C —
| | Cl Cl Cl
H X NO2
e. g ., CH 2 == CH 2 + HCl ¾® CH 3CH 2Cl Conc. HNO /H SO
3 2 4
¾¾¾¾¾® +
Ethyl chloride
Reactions of alkyl halide with :
(i) KOH alc.: alkenes are formed NO2
D o/p nitrochlorobenzene
CH 3CH 2Cl + KOH Alc. ¾¾ ® CH 2 == CH 2 + KCl + H 2O
Ethene (ii) CH 3Cl /AlCl 3 :
(ii) AgCN: ethyl isocyanide is formed Cl Cl Cl
D
CH 3CH 2Cl + AgCN ¾¾ ® CH 3CH 2 NC + AgCl CH3
Ethyl isocyanide CH Cl/AlC
3 3
¾¾¾® +
Q.4 Write chemical equation of the laboratory method for
the preparation of chlorobenzene and its chemical
reaction with the following : (U.P. 2020) CH3
(i) Concentrated HNO3 in the presence of H2 SO4 o/p chlorotoluene
(ii) CH3 Cl in the presence of AlCl 3 (iii) NH 3 /Cu 2O :
(iii) NH3 in the presence of Cu 2 O Cl NH2
Ans. Laboratory method of preparation of
chlorobenzene: It is the best method to prepare all Cu2O
the nuclear substituted halogen derivatives. In this + NH3 ¾¾® + HCl
reaction , benzene diazonium chloride is heated with
aniline

Question Based on Chemical Reactions


KOH C H OH
Q.1 Complete the following reactions. (iii) (CH 3 ) 3 CCH 2 Br ¾ ¾ ¾2¾5¾
¾®
D
(i) CCl 4 + SbF3 ¾ Heat
¾¾® ..... + SbCl 3 (U.P. 2014)
(CH 3 ) 2 C == CHCH 3
KOH
aq DD
(ii)CH 3CH 2CHCl 2 ¾ ¾ ¾¾ ® A ¾¾® B B
Q.2 What happens when : (Give chemical equations only)
(U.P. 2015, 18) (U.P. 2019)
KOH C H OH D
2 5
(iii) (CH 3 ) 3 C CH 2 Br ¾¾¾¾¾® B (U.P. 2015) (i) Chloroform reacts with aq. KOH solution
Heat (ii) Ethyl chloride is heated with aq. NaOH solution.
Ans. (i) 3CCl 4 + 2SbF3 ¾ ¾¾® 3CCl 2 F2 + 2SbCl 3
KOH
aq
(iii) Iodobenzene is heated with copper.
(ii)CH 3CH 2CHCl 2 ¾ ¾ ¾
¾ ® CH 3CH 2CH(OH) 2 (iv) Grignard reagent reacts with RCOOR¢.
¾ -¾¾
D
® CH 3CH 2CHO Ans. (i) Chloroform reacts with aq. KOH solution:
H O 2
A
104
Cl HOK OH Q.4 How will you convert,
H ¾C Cl + HOK ¾® H ¾ C OH+3KCl Chlorobenzene into benzene, aniline, phenol, toluene
Cl HOK OH (U.P. 2019)
¯ - H 2O Ans.
OH Cl
KOH
H 2O+ HCOOK ¬¾¾ HCOOH
Potassium NaOH/D Ni-Al/NaOH
formate
High P & t
¾¾¾®
(ii) C2H5 Cl+Na OH(aq.) ¾® C 2 H 5OH + NaCl –NaCl CH
Ethyl alcohol 3 3C
NH l/
2N
(iii) Ullmann biaryl synthesis : NH2 NH 2/ a CH3
Na
D
C6H5 I+Cu+I C6H5 ¾¾® + 2CuI
Sealed
tube
Diphenyl
Q.5 How will you convert?
(iv)Grignard reagent reacts with RCOOR ¢ :
O OMgBr (i) Acetone into chloroform. (U.P. 2015)
|| | (ii) 1-bromopropane into 2-bromopropane.
R ¾ C ¾ OR¢ + CH 3 MgBr ¾® R ¾ C ¾ OR¢ (U.P. 2014)
| (iii) Ethyl alcohol into chloroform. (U.P. 2012)
CH 3 (iv) Acetaldehyde into iodoform. (U.P. 2016)
+
(v) Chloroform into acetylene. (U.P. 2013)
- R ¢ OMgBr ¯ H 2 O/ H Ans. (i) CaOCl 2 + H 2O ¾® Ca(OH) 2 + Cl 2
OMgBr O CH 3COCH 3 + 3Cl 2 ¾® CCl 3COCH 3 + 3HCl
| || Acetone
R ¾ C ¾ CH 3 ¬¾¾¾
CH 3 MgBr
R ¾ C ¾ CH 3 2CCl 3COCH 3 + Ca(OH) 2 ¾®
| Ketone
2CHCl 3 + (CH 3COO) 2 Ca
CH 3 Chloroform

OH (ii) CH 3CH 2CH 2 Br ¾ KOH(alc.)


¾ ¾ ¾®
- H 2O, - KBr
|
Br
R ¾ C ¾ CH 3 + Mg Br |
H+/H 2O OH
|
CH 3CH == CH 2 ¾ HBr
¾¾® CH 3 — CH — CH 3
CH 3 2- Bromopropane
3º alcohol
(iii) C 2 H 5OH + 4Cl 2 + 6NaOH ¾®
Q.3 How will you obtain ? Ethanol
(a) Chlorobenzene from benzene, nitrobenzene, CHCl 3 + HCOONa + 5NaCl + 5H 2O
phenol, aniline (U.P. 2005, 07) Chloroform
(b) Chlorobenzene from diazobenzonium chloride (iv) CH 3CHO + 3I 2 + 4NaOH ¾®
(U.P. 2015) Acetaldehyde
Ans. CHI 3 + HCOONa + 3NaI + 3H 2O
Iodoform
Cl
(v) 2CHCl 3 + 6Ag ¾® CH ºº CH + 6AgCl
Chloroform Acetylene
2 Cl /Fe
¾¾® Q.6 Complete the following reaction.
Cu (i) ... + Cl 2 ¾ AlCl
¾¾ 3
® ... +S 2Cl 2
Cl Heat
NO2 NH2 / HC N+
2Cl

KCN [H]
l (ii) RX ¾¾® A ¾¾® B (U.P. 2018)
Sn/HCl NaNO /HCl
2
Ans. (i) CS 2 + 3Cl 2 ¾ AlCl
¾¾ 3
® CCl 4 + S 2Cl 2
¾¾® ¾¾¾® Heat
KCN [H]
Nitrobenzene Aniline (ii) RX ¾¾® ACN ¾¾® BCH 2 NH 2
Q.7 (i) Write the possible isomers of C 3 H7 Cl.
OH OH Cl (U.P. 2014, 16)
(ii) Sodium and compound A react with each other to
Cl /FeCl Zn dust/D
form ethane. Write the structural formula of
2 3
¾¾® Cl ¾¾® compound A and equation of the reaction.
(U.P. 2014, 16, 18)
(b) See long answer type question 4.
105

Ans. (i) The possible isomers of C 3 H7 Cl are (iii) Phenyl isocyanide is obtained.
D
(a) CH 3 .CH 2 .CH 2Cl (b) CH3 — CH— CH 3 C 6 H 5 NH 2 + CHCl 3 + 3KOH ¾¾ ®
1- chloropropane | Aniline
Cl C 6 H 5 NC + 3KCl + 3H 2O
2-chloropropane
Q.9 Write chemical equations for Halogenation,
(ii) Compound ( A ) is methyl halide. Nitration, Sulphonation and Friedel Crafts reaction of
Chlorobenzene. (U.P. 2020)
CH 3 X + 2Na + CH 3 X ¾ Dry
¾ ¾¾ether
® CH 3 .CH 3 + 2NaX Ans. Halogenation:
Methyl halide Ethane
( X = Cl, Br,I) Cl Cl
NO2
Q.8 What happens when (write chemical equations only):
(i) Chloroform reacts with acetone in presence of an +
alkali. (U.P. 2014)
(ii) Chloroform reacts with phenol and NaOH or NO2
o/p chloronitrobenzene
KOH. (U.P. 2011)
(iii) Aniline is heated with chloroform and alcoholic Nitration HNO3/H2SO4
solution of KOH. Cl Cl Cl Cl Cl
Ans. (i) Chloretone is obtained. CH3 Cl
CH3 NaOH
CH3 O CH3Cl/AlCl3 Cl /Fe
2
C==O+CHCl 3 ¾¾® C + ¾¾¾®
Halogenation +
CH3 Chloroform CH3 CCl3
Acetone Chloretone
CH3 H2SO4/D Cl
(ii) On heating chloroform with phenol in the
o/p chlorotoluene
presence of NaOH salicylaldehyde is formed.
OH Cl Cl
SO3H
60ºC
+3NaOH+CHCl3 ¾¾® +
Phenol
OH SO3H
o/p chlorobenzene sulphonic acid
CHO
+3NaCl+2H2O
Salicylaldehyde

Solution of NCERT Text Book Problems


Q.1 Name the following halides according to IUPAC (iv) 1-bromo-3, 3-dimethyl-1-phenylbutane, 2° alkyl
system and classify them as alkyl, allyl, benzyl halide
(primary, secondary, tertiary), vinyl or aryl halides. (v) 2-bromo-3-methylbutane, 2° alkyl halide
(i) (CH 3 ) 2 CHCH(Cl)CH 3 (vi) 1-bromo-2-ethyl-2-methylbutane, 1° alkyl halide
(ii) CH 3CH 2CH(CH 3 )CH(C 2 H 5 )Cl (vii) 3-chloro-3-methylpentane, 3° alkyl halide
(iii) CH 3CH 2C(CH 3 ) 2 CH 2 I (viii) 3-chloro-5-methylhex-2-ene, vinylic halide
(iv) (CH 3 ) 3 CCH 2CH(Br)C 6 H 5 (ix) 4-bromo-4-methylpent-2-ene, allylic halide
(v) CH 3CH(CH 3 )CH(Br)CH 3 (x) 1-chloro-4-(2-methyl propyl) benzene, aryl
(vi) CH 3C(C 2 H 5 ) 2 CH 2 Br halide
(vii) CH 3C(Cl)(C 2 H 5 )CH 2CH 3 (xi) 3-(2, 2-dimethylpropyl) phenyl chloromethane,
1° benzylic halide
(viii) CH 3CH == C(Cl)CH 2CH(CH 3 ) 2
(xii) 1-bromo-2-(1-methylpropyl) benzene, aryl halide.
(ix) CH 3CH == CHC(Br)(CH 3 ) 2 Q.2 Give the IUPAC name of the following compounds :
(x) p-ClC 6 H 4 CH 2CH(CH 3 ) 2 (i) CH 3CH(Cl)CH(Br)CH 3
(xi) m- ClCH 2C 6 H 4 CH 2C(CH 3 ) 3 (ii) CHF2CBrClF
(xii) o- Br ¾ C 6 H 4 CH(CH 3 )CH 2CH 3 (iii) ClCH 2C ºº CCH 2 Br
Ans. (i) 2-chloro-3-methylbutane, 2° alkyl halide (iv) (CCl 3 ) 3 CCl
(ii) 3-chloro-4-methylhexane, 2° alkyl halide
(v) CH 3C( p-ClC 6 H 4 ) 2 CH(Br)CH 3
(iii) 1-iodo-2, 2-dimethylbutane, 1° alkyl halide
106

(vi) (CH 3 ) 3 CCH == CClC 6 H 4 I- p Q.5 A hydrocarbon C 5 H10 does not react with chlorine
Ans. (i) 2-bromo-3-chlorobutane, but gives a single monochloro compound, C 5 H 9Cl in
bright sunlight. Identify the hydrocarbon.
(ii) 1-bromo-1-chloro-1, 2, 2-trifluoroethane
Ans. (i) It does not give addition reaction with Cl 2 , so it is
(iii) 1-bromo-4-chlorobut-2-yne a cycloalkane.
(iv) 2-(trichloromethyl)-1, 1, 1, 2, 3, 3, 3-heptachlo- (ii) With Cl 2 /sun-light, it gives only one mono
ropropane substituted product. All the H-atoms are equivalent.
CCl 3
Cl Cl
½ + HCl
Cl 3C ¾ C ¾ CCl 3 light
Chloro cyclopentane
½
Cl Q.6 Write the isomers of the compound having formula
(v) 2-bromo-3, 3-bis (4-chlorophenyl) butane C 4 H 9 Br.
(vi) 1-chloro-1-(4-iodophenyl)-3,3-dimethyl-but-1-ene Ans. 1-bromobutane, 2-bromobutane, 1-bromo-2-methyl
Q.3 Write the structures of the following organic propane and 2-bromo-2-methyl propane are four
halogen compounds : isomers.
(a) 2-chloro-3-methylpentane Q.7 Write the equations for the preparation of
(b) p-bromochlorobenzene 1-iodobutane from :
(c) 1-chloro-4-ethylcyclohexane (a) 1-butanol, (b) 1-chlorobutane, (c) but-1-ene
(d) 2-(2-chlorophenyl)-1-iodooctane Ans. (a) CH 3CH 2CH 2CH 2OH ¾ P¾I2
¾ ® CH 3CH 2CH 2CH 2 I
1-butanol 1-iodobutane
(e) perfluorobenzene
NaI / Acetone
(f) 4-tert. butyl-3-iodoheptane (b) CH 3CH 2CH 2CH 2Cl ¾¾¾¾¾¾®
Finkelstein reaction
1-chlorobutane
(g) 1-bromo-4-sec. butyl-2-methylbenzene
CH 3CH 2CH 2CH 2 I
(h) 1, 4-dibromobut-2-ene 1-iodobutane
HBr/peroxide
Ans. (a) CH 3 ¾ CH ¾ CH ¾ CH 2CH 3 (c) CH 3CH 2CH== CH 2 ¾¾¾¾¾¾®
Anti -Mark addition
½ ½ But-1-ene
Cl CH3 NaI / Acetone
Cl CH 3CH 2CH 2CH 2 Br ¾¾¾¾¾¾®
Finkelstein reaction
CH 3CH 2CH 2CH 2 I
1-bromobutane 1-iodobutane
(b) Br Cl (c) Q.8 What are ambident nucleophiles ? Explain with an
H5C2 example.
Ans. Those nucleophiles which have two attacking centre
F but actually one forms bond. e. g .,
I CH2 CH (CH2)5CH3 —CN and —NC, —CNS and —NCS
F F
Cl (e) Q.9 Which compound in each of the following pairs will
(d) react faster in S N 2 reaction with OH - ?
F F
(a) CH 3 Br or CH 3 I
F
(b) (CH 3 ) 3 CCl or CH 3Cl
(f) CH 3 ¾ CH 2 ¾ CH ¾ CH ¾ CH 2 CH 2CH 3
½ ½ Ans. (a) CH 3 I, because C—I bond is weaker than C—Br
I C(CH3 )2 bond.
CH3 (b) CH 3Cl because for S N 2 reaction, 1° alkyl halides
are more reactive due to less steric hindrance.
(g) CH3 CH2 CH Br Q.10 Predict all the alkenes that would be formed by
CH3 dehydrohalogenation of the following halides with
sodium ethoxide in ethanol and identify the major
(h) BrCH 2CH == CHCH 2 Br
alkene.
Q.4 Which of the following has highest dipole moment :
(i) CH 2Cl 2 (ii) CHCl 3 (iii) CCl 4 ? (a) 1-bromo-1-methylcyclohexane
Ans. Order of dipole moment (m ) is in the order (b) 2-chloro-2-methylpropane
CH 2Cl 2 > CHCl 3 > CCl 4 (m = 0) (c) 2, 2, 3-trimethyl-3-bromopentane (Correct name
Order is explained on the basis of vector sum of is 3-bromo-2, 2, 3-trimethyl pentane)
m C— Cl bond.
107
b
CH2 CH 3 CH 3
CH3 CH3 ½
b ½ b C2 H 5 ONa/C2 H 5 OH
Ans. (a) Br + HBr (b) CH ¾ C ¾ CH ¾¾¾¾¾¾® CH 3 ¾ C==CH 2
3 3 –HCl
CH2 2-methylpropene
1-methyl cyclohexene (only product)
½ (only product)
Cl
2-chloro-2-methylpropane
b
CH 3 CH 3 CH 3 CH 2 CH 3 CH 3
½ ½ b C2 H 5 ONa/C2 H 5 OH ½ ½
½ ½ ½
(c) CH 3 ¾ C ¾ C ¾ CH 2 CH 3 ¾¾¾ ¾¾¾®
–HBr
CH 3 ¾ C ¾ C ¾ CH 2 ¾ CH 3 + CH 3 ¾ C ¾ C== CH ¾ CH 3
½ ½ ½ ½
CH 3 Br CH 3 CH 3
(Minor product, less substituted) (Major product more substituted)
Q.11 How will you bring the following conversions ?
(a) Ethanol to but-1-yne, (b) Ethane to bromoethane, (c) Propene to 1-nitropropane
(d) Toluene to benzyl alcohol, (e) Propene to propyne, (f) Ethanol to ethyl fluoride
(g) Bromomethane to propanone, (h) But-1-ene to but-2-ene (i) 1-chlorobutane to n-octane
(j) Benzene to biphenyl
2 SOCl , Pyridine
Ans. (a) CH 3CH 2OH ¾¾¾¾¾¾® CH 3CH 2 ¾ Cl
–SO2 , –HCl
Ethanol Chloroethane ( A )
liq. NH 3 , 196 K
HC ºº CH + NaNH 2 ¾¾¾¾¾® HC ºº C – Na+ + NH 3
Acetylene Sod. acetylide ( B )
A + B ¾¾¾® CH 3CH 2 ¾ C ºº CH + NaCl
But-1-yne
hn, 520-670 K
(b) CH 3 ¾ CH 3 + Br2 ¾¾¾¾¾® CH 3CH 2 ¾ Br + HBr
Ethane (excess) Bromoethane
HBr, peroxide 2 2 5 2 AgNO , C H OH /H O
(c) CH 3 ¾ CH== CH 2 ¾¾¾¾¾®
Peroxide effect
CH 3 ¾ CH 2 ¾ CH 2 Br ¾¾¾¾¾¾¾®
–AgBr
CH 3CH 2CH 2 NO 2
1-bromopropane 1-nitropropane

2 Cl /heat Aq. KOH /heat


(d) C 6 H 5 ¾ CH 3 ¾¾¾¾®
–HCl
C 6 H 5CH 2Cl ¾¾¾¾® C 6 H 5CH 2OH
–KCl
Benzyl chloride Benzyl alcohol
Br2 / CCl 4 KOH (alc.), D
(e) CH 3 ¾ CH== CH 2 ¾¾¾¾¾® CH 3 ¾ CH ¾ CH 2 ¾¾¾¾¾® CH 3 ¾ C ºº CH
–2KBr, – 2H 2 O
Propene ½ ½ Propyne
Br Br
1, 2-dibromopropane
SOCl 2 , Pyridine
2 2 Hg F
(f ) CH 3CH 2OH ¾¾¾¾¾®
–SO , –HCl
CH 3CH 2Cl ¾¾¾®
–Hg Cl
CH 3CH 2 F
2 2 2
Ethanol Ethyl chloride Ethyl fluoride
KCN /(alc.) 2 Hydrolysis Ca(OH) Distil.
(g) CH 3 Br ¾¾¾® CH 3CN ¾¾¾¾®
+ CH 3COOH ¾¾¾® (CH 3COO) 2 Ca ¾¾¾® CH 3COCH 3+CaCO 3
H 3O
Ca-acetate Propanone
HBr KOH (alc.), D
(h) CH 3CH 2CH== CH 2 ¾¾¾¾® CH 3 ¾ CH 2 ¾ CH ¾ CH 3 ¾¾¾¾¾®
–HBr
CH 3 ¾ CH== CH ¾ CH 3
Mark. addn.
But-1-ene ½ But-2-ene (Major product)
Br
2-bromobutane
Dry ether
(i) 2CH 3CH 2CH 2CH 2Cl+ 2Na ¾¾¾¾¾®
Wurtz reaction
CH 3CH 2CH 2CH 2 ¾ CH 2CH 2CH 2CH 3 + 2NaCl
1-chlorobutane n-Octane
Br2 / FeBr3 Na/ether
(j) C 6 H 6 ¾¾¾¾® C 6 H 5 Br ¾¾¾® C 6 H 5 ¾ C 6 H 5
Biphenyl
Q.12 Explain why : (a) the dipole moment of chlorobenzene is lower than that of cyclohexyl chloride.
(b) alkyl halides, though polar, are immiscible in water.
(c) Grignard reagents should be prepared under anhydrous conditions ?
Ans. (a) Because C — Cl bond is shorter in chlorobenzene due to partial double bond character.
(b) They cannot form H-bond with H 2O.
108

(c) Because it reacts with water (moisture), CO 2 , alcohol etc., that is why GR is prepared in anhydrous conditions.
RMgX + H 2O ¾¾® RH + Mg(OH) X
Alkane
Q.13 Give the uses of Freon-12, DDT, carbon tetrachloride and iodoform.
Ans. Deleted for examination 2021.
Q.14 Write the structures of the major organic product in each of the following reactions :
Acetone Ethanol, heat
(a) CH 3CH 2CH 2Cl + NaI ¾¾¾®
Heat
(b) (CH 3 ) 3 CBr + KOH ¾¾¾¾¾®

Water Aq. ethanol


(c) CH 3CH(Br)CH 2CH 3 + NaOH ¾¾¾® (d) CH 3CH 2 Br + KCN ¾¾¾¾®
(e) C 6 H 5ONa + C 2 H 5Cl ¾¾¾® (f ) CH 3CH 2CH 2OH + SOCl 2 ¾¾®
Peroxide
(g) CH 3CH 2CH== CH 2 + HBr ¾¾¾® (h) CH 3CH== C(CH 3 ) 2 + HBr ¾¾®
Acetone
Ans. (a) CH 3CH 2CH 2Cl + NaI ¾¾¾® CH 3CH 2CH 2 I+ NaCl
Heat
CH 3
Ethanol, heat
½
(b) (CH 3 ) 3 CBr + KOH ¾¾¾¾¾® CH 3 ¾ C==CH 2 + KBr + H 2O
2-bromo-2-methyl propane 2-methylpropene
Water
(c) CH 3 ¾ CH ¾ CH 2CH 3 + NaOH ¾¾¾® CH 3 ¾ CH ¾ CH 2CH 3 + NaBr + H 2O
Hydrolysis
½ ½
Br OH
2-bromobutane Butan-2-ol
Aq. ethanol
(d) CH 3CH 2 Br + KCN ¾¾¾¾® CH 3CH 2CN + KBr
Bromoethane Propane nitrile
Williamson’s
(e) C 6 H 5O – Na+ + C 2 H 5Cl ¾¾¾¾®
Synthesis
C 6 H 5 ¾ O ¾ C 2 H 5 + NaCl
Sod. phenoxide Ethyl chloride Phenetole
(f ) CH 3CH 2CH 2OH + SOCl 2 ¾¾® CH 3CH 2CH 2Cl + HCl + SO 2
Peroxide
(g) CH 3CH 2CH== CH 2 + HBr ¾¾¾® CH 3CH 2CH 2CH 2 Br
But-1-ene 1-bromobutane
CH 3 CH 3
½ ½
Mark. addn.
(h) CH 3 ¾ CH==C ¾ CH 3 + HBr ¾¾¾¾® CH 3 ¾ CH 2 ¾ C ¾ CH 3
2-methylbut-2-ene
½
Br
2-bromo-2-methylbutane
Q.15 Explain the following reaction :
2 EtOH -H O
n - BuBr + KCN ¾¾¾¾® n - BuCN
+ -
Ans. KCN ¾¾® K +CN
n - BuBr + CN - ¾¾® n BuCN + Br –
Q.16 Arrange the compounds of each set in order of reactivity towards S N 2 displacement :
(a) 2-bromo-2-methylbutane, 1-bromo pentane, 2-bromopentane.
(b) 1-bromo-3-methylbutane, 2-bromo-2-methylbutane, 2-bromo-3-methylbutane
(c) 1-bromobutane,1-bromo-2, 2-dimethylpropane,1-bromo-2-methylbutane, 1-bromo-3-methylbutane
Ans. (a) 1-bromopentane > 2-bromopentane > 2-bromo-2-methylbutane
(b) 1-bromo-3-methylbutane > 2-bromo-3-methylbutane > 2-bromo-2-methylbutane
(c) 1-bromobutane > 1-bromo-3-methylbutane >1-bromo-2-methylbutane > 1-bromo-2, 2-dimethylpropane
Q.17 Out of C 6 H 5CH 2Cl and C 6 H 5CHClC 6 H 5 , which is more easily hydrolysed by aq. KOH ?
Ans. In S N1 hydrolysis C 6 H 5CHClC 6 H 5 is more easily hydrolysed than C 6 H 5CH 2Cl.
In S N 2 hydrolysis C 6 H 5CH 2Cl is more easily hydrolysed due to less steric hindrance at the carbon atom bearing
Cl-atom.
Q.18 p-Dichlorobenzene has higher melting point and lower solubility than o- and m-isomers. Discuss.
109

Ans. Due to symmetrical structure, its molecules are more closely packed in crystal lattice. So its melting point is high and it
is less soluble than the o- and m-dichlorobenzenes.
Q.19 How the following conversions can be carried out ?
(1) Propene to propan-1-ol (2) Ethanol to But-2-yne
(3) 1-bromopropane to 2-bromopropane (4) Toluene to benzyl alcohol
(5) Benzene to 4-bromonitrobenzene (6) Benzyl alcohol to 2-phenylethanoic acid
(7) Ethanol to propanenitrile (8) Aniline to chlorobenzene
(9) 2-chlorobutane to 3, 4-dimethylhexane (10) 2-methyl-1-propene to 2-chloro-2-methylpropane
(11) Ethyl chloride to propanoic acid (12) But-1-ene to n-butyl iodide
(13) 2-chloropropane to 1-propanol (14) Isopropyl alcohol to iodoform
(15) Chlorobenzene to p-nitrophenol (16) 2-bromopropane to 1-bromopropane
(17) Chloroethane to butane (18) Benzene to diphenyl
(19) Tert.-butyl bromide to isobutyl bromide (20) Aniline to phenylisocyanide.
HBr/Peroxide Aq. KOH, D
Ans. (1) CH 3CH== CH 2 ¾¾¾¾® CH 3CH 2CH 2 Br ¾¾¾¾® CH 2CH 2CH 2OH
2 P/I , D 2 4 KOH (alc.), D Br / CCl KOH (alc.), D
(2) CH 3CH 2OH ¾¾¾® CH 3CH 2 I ¾¾¾¾® CH 2 == CH 2 ¾¾¾¾® CH 2 ¾ CH 2 ¾¾¾¾¾¾¾®
Dehydrohalogenation
Ethanol Ethene ½ ½
Br Br
NaNH 2 , liq. NH 3 CH I (Excess)
HC ºº CH ¾¾¾¾¾® Na+C – ºº C – Na+ ¾¾¾¾®
3
CH 3 ¾ C ºº C ¾ CH 3
196 K Disodium acetylide But-2-yne
KOH (alc.), D HBr
(3) CH 3CH 2CH 2 Br ¾¾¾¾® CH 3CH == CH 2 ¾¾® CH 2 ¾ CHBr ¾ CH 3
Cl
hv aq KOH .
(4) C 6 H 5CH 3 ¾ ¾2 ¾
¾ ® C 6 H 5CH 2Cl ¾ ¾ ¾
¾® C 6 H 5CH 2OH
Br2 / FeBr3 3 2 4 Conc. HNO + Conc. H SO
(5) C 6 H 6 ¾¾¾® C 6 H 5 Br ¾¾¾¾¾¾¾¾¾® p- NO 2C 6 H 4 Br
4-bromo nitro benzene
SOCl 2 KCN /aq.-alc. H 3 O+ 2 1
(6) C 6 H 5CH 2OH ¾¾® C 6 H 4 CH 2Cl ¾¾¾¾® C 6 H 5CH 2CN ¾¾® C 6 H 5CH 2COOH
2-phenyl ethanoic acid
P/I 2 , D KCN. EtOH – H 2 O
(7) CH 3CH 2OH ¾¾® CH 3CH 2 I ¾¾¾¾¾¾¾¾® CH 3CH 2CN
Ethanol 1-iodoethane Propanenitrile
NaNO / HCl + – CuCl /HCl
2
(8) C 6 H 5 NH 2 ¾¾¾¾® C 6 H 5 N 2 Cl ¾¾¾® C 6 H 5Cl
273-278 K
Dry ether
(9) 2CH 3 ¾ CH 2 ¾ CHCH 3 + 2Na ¾¾¾¾¾® CH 3CH 2 ¾ CH ¾ CH ¾ CH 2CH 3 + 2NaCl
Wurtz reaction
½ ½ ½
Cl CH3 CH3
2-chlorobutane 3, 4-dimethylhexane
CH 3 CH 3
½ HCl
½
(10) CH 3 ¾ C==CH 2 ¾¾® CH 3 ¾ C ¾ CH 3
2-methyl -1-propene
½
Cl
2-chloro-2-methylpropane

2 KCN. EtOH – H O
2 H+ /H O
(11) CH 3CH 2Cl ¾¾¾¾¾¾¾¾® CH 3CH 2CN ¾¾¾® CH 3CH 2COOH
Hydrolysis
Ethyl chloride Propanenitrite Propanoic acid
HBr/Peroxide NaI, acetone
(12) CH 3CH 2CH== CH 2 ¾¾¾¾® CH 3CH 2CH 2CH 2 Br ¾¾¾¾® CH 3CH 2CH 2CH 2 I
KOH (alc.), D HBr, Peroxide Aq. KOH
(13) CH 3 ¾CH ¾ CH 3 ¾¾¾¾® CH 3 ¾ CH== CH 2 ¾¾¾¾® CH 3CH 2CH 2 Br ¾¾¾® CH 3CH 2CH 2OH
D
½
Cl
110

Heat
(14) CH 3 ¾ CH ¾ CH 3 + 4I 2 + 6NaOH ¾¾¾¾¾¾® CHI 3 +CH 3COONa+ 5NaI+ 5H 2O
(Iodoform reaction)
½ Iodoform
OH
Isopropyl alcohol

Conc. HNO3 + H2SO4 (i) 15% NaOH, 433 K


(15) Cl O2N Cl O2N OH
Nitration (ii) dil. HCl
Chlorobenzene p-Nitrochlorobenzene p-Nitrophenol
(Major isomer)
KOH (alc.), D HBr, Peroxide
(16) CH 3 ¾ CH ¾ CH 3 ¾¾¾¾® CH 3 ¾ CH== CH 2 ¾¾¾¾® CH 3 ¾ CH 2 ¾ CH 2 ¾ Br
½ 1-bromopropane
Br
2-bromopropane
Dry ether, D
(17) 2CH 3CH 2 ¾ Cl+ 2Na ¾¾¾¾¾®
Wurtz reaction
CH 3CH 2 ¾ CH 2CH 3+ 2NaCl
Chloroethane Butane

(18) Benzene to diphenyl : Refer to Ans. 11 (j).


CH 3 CH 3 CH 3
½ KOH (alc.), D
½ HBr, Peroxide
½
(19) CH 3 ¾ C ¾ CH 3 ¾¾¾¾® CH 3 ¾ C==CH 2 ¾¾¾¾® CH 3 ¾ CHCH 2 Br
2-Methyl -1-propene Isobutyl bromide
½
Br
Warm
(20) C 6 H 5 ¾ NH 2 + CHCl 3 + 3KOH ¾¾® C 6 H 5 NC + 3KCl + 3H 2O
Q.20 The treatment of alkyl chloride with aqueous KOH leads to the formation of alcohols but in presence of alcoholic KOH,
alkene are the major products. Explain, why ?
Ans. Because in alcoholic medium, very strong base RO - C stronger than OH – is formed which favours elimination reaction
over substitution.
Q.21 Primary alkyl halide (A) C 4 H 9 Br reacted with alcoholic KOH to give compound (B). Compound (B) is reacted with HBr
to give (C) which is an isomer of (A). When (A) reacted with Na metal, it gave a compound (D) C 8 H18 that was
different than the compound when n-butyl bromide was reacted with sodium. Give the structural formula of (A) and
write the equation for all the reactions.

Ans. (i) Two primary alkyl halides having molecular formula, C 4 H 9 Br are :
CH 3
½
CH 3CH 2CH 2CH 2 Br and CH 3 ¾ CH ¾ CH 2 Br
n-butyl bromide Isobutyl bromide

(ii) Compound (A) reacts with Na metal to give (D) C 8 H18 which was different from the compound obtained when
n-butyl bromide reacts with Na metal, therefore, (A) must be isobutyl bromide and compound (D) must be 2,
5-dimethylhexane.
Wurtz reaction
CH 3CH 2CH 2CH 2 Br + 2Na ¾¾¾¾¾® CH 3CH 2CH 2CH 2CH 2CH 2CH 2CH 3
n-butyl bromide n-octane
CH 3 CH 3 CH 3
½ Wurtz reaction
½ ½
CH 3 ¾ CH ¾ CH 2 Br + 2Na ¾¾¾¾¾® CH 3 ¾ CH ¾ CH 2 ¾ CH 2 ¾ CH ¾ CH 3
Isobutyl bromide ( A ) (1°) 2, 5-dimethylhexane ( D )

(iii) If compound (A) is isobutylbromide, then the compound (B) which it gives on treatment with alcoholic KOH must
be 2-methyl-prop-1-ene.
111

CH 3 CH 3
½ KOH (alc.), D
½
CH 3 ¾ CH ¾ CH 2 Br ¾¾¾¾® CH 3 ¾ CH== CH 2
Isobutyl bromide ( A ) 2-methyl -prop-1-ene ( B )

(iv) The compound (B) on treatment with HBr gives compound (C) in accordance with Markownikoff’s rule.
CH 3 CH 3
½ ½
HBr
CH 3 ¾ CH== CH 2 ¾¾® CH 3 ¾ C ¾ CH 3
2-methyl -prop-1-ene
½
Br
Tert.-butyl bromide ( C )
[An isomer of compound ( A )]

Thus, (A) is isobutyl bromide, (B) is 2-methyl prop-1-ene, (C) is Tert.–butyl bromide and (D) is 2,
5-dimethylhexane.
Q.22 What happens when ?
(i) n-butyl chloride is treated with alcoholic KOH.
(ii) bromobenzene is treated with Mg in the presence of dry ether.
(iii) chlorobenzene is subjected to hydrolysis.
(iv) ethyl chloride is treated with aqueous KOH.
(v) methyl bromide is treated with sodium in presence of dry ether.
(vi) methyl chloride is treated with KCN.
Ans. (i) But-1-ene is formed. (ii) Phenyl magnesium bromide (C 6 H 5 MgBr) is formed. (iii) There is no reaction with aq.
NaOH under ordinary conditions. But under drastic conditions, sodium phenoxide (C 6 H 5ONa) is formed. (iv)
Ethyl alcohol is formed. (v) It is Wurtz reaction. Ethane is formed. (vi) CH 3CN is formed.

vvv
112

Chapter

11 ALCOHOLS, PHENOLS
AND ETHERS
Syllabus : Alcohols : Nomenclature, Methods of preparation, Physical and chemical properties (only mono hydric alcohols),
Identification of p,s and t-alcohols, Mechanism of dehydration, *Uses of methanol and ethanol.
Ethers: Nomenclature, Methods of preparation, Physical and chemic al properties, Uses.
Phenols : Nomenclature, Methods of preparation, Physical and chemic al properties Acidic nature of phenols, E +-substitution
reaction, Use of phenols
* Topics Deleted for Examination 2021

Objective Questions
1. Alcoholic fermentation is done by: (U.P. 2016) (c) diethyl ether and methoxy methane
(a) invertase (b) diastase (d) diethyl ether and methoxy ethane
(c) zymase (d) maltase 7. Among the following picric acid is : (U.P. 2017)
2. Lucas reagent is used to distinguish : (U.P. 2016, 17) (a) o-hydroxy benzoic acid
(a) p, s and t-alcohols (b) m-nitrobenzoic acid
(b) p, s and t-alkyl halides (c) 2,4,6-trinitrophenol
(c) p, s and t-aliphatic amines (d) o-aminobenzoic acid
(d) p, s and t-aromatic amines 8. Prolonged nitration of phenol by conc. HNO 3 /H 2SO 4
3. Conc. HCl and anhydrous ZnCl 2 (or Lucas reagent) are gives : (U.P. 2017)
used in testing of: (U.P. 2013, 18) (a) o-nitrophenol (b) p-nitrophenol
(a) nitroalkanes (b) alkyl halide (c) picric acid (d) nitrobenzene
(c) alkyl amine (d) alcohols 9. Per cent of alcohol in rectified spirit is : (U.P. 2018)
4. Among the following, which do not give iodoform (a) 75% (b) 95.5% (c) 5.6% (d) 100%
with I 2 NaOH ? (U.P. 2013) 10. Glucose is converted into ethyl alcohol in the presence
(a) CH 3OH (b) C 2 H 5OH of enzyme : (U.P. 2016)
(c) CH 3CHO (d) CH 3COCH 3 (a) invertase (b) zymase
5. In the following possible isomers of butanol, the (c) diastase (d) all of these
compound exhibiting optical isomerism is: 11. Lucas reagent is : (U.P. 2018)
(U.P. 2014) (a) Conc. HCl + anhydrous ZnCl 2
(a) CH 3CHOHCH 2CH 3 (b) CH 3CH 2CH 2CH 2OH (b) Pd / BaSO 4
(c) (CH 3 ) 2 CHCH 3OH (d) (CH 3 ) 2 COH (c) dil. HCl + anhydrous ZnCl 2
6. Common and IUPAC names of CH 3 — O — CH 3 are : (d) None of these
(U.P. 2013) 12. Phenol is heated with Nitric acid to give: (U.P. 2020)
(a) dimethyl ether and methoxy methane (a) Chloretone (b) Picric Acid
(b) dimethyl ether and methoxy ethane (c) Methoxy Benzene (d) Benzene

Answers
1. (c) 2. (a) 3. (d) 4. (a) 5. (a) 6. (a) 7. (c) 8. (c) 9. (b) 10. (b)
11. (b) 12. (b)

Very Short Answer Type Questions


Q.1 Ether is stored in dark coloured bottles, why ? Q.2 Lower alcohols are miscible with water, why ?
(U.P. 2015) (U.P. 2015)
Ans. Ether forms peroxide in presence of light and air. Ans. It is because they form intermolecular hydrogen
Peroxides of ethers are poisonous compounds. bondings with water molecules.
113
–d +d Q.8 Why phenol is also called carbolic acid ? Explain its
R O – – –H — O acidic nature. (U.P. 2017)
H H Ans. Word Carbo has been derived from carbon or coal and
Q.3 How will you distinguish between : ol = oleum = oil. Its literal meaning is carbon oil.
(a) methanol and ethanol ? (U.P. 2015, 16) Because phenol is obtained from coal tar and is acidic
(b) CH 3OH and C 2 H 5OH (U.P. 2014, 16) in nature, so alternative term carbolic acid was also
Ans. (a) and (b) can be distinguished by iodoform test as used for it by J. Lister.
methanol, propanol-1 and hexanol-3 do not respond It is acidic in nature because phenoxide ion is highly
to iodoform test. stabilised by resonance.
Q.4 Phenols are acidic in nature while alcohols are Q.9 Aqueous solutions of alcohols are bad conductors of
amphoteric, why? (U.P. 2014, 15, 18) electricity, why? (U.P. 2015)
Or Phenol and ethanol, both have —OH group. Explain Ans. Aqueous solution of alcohols are bad conductors of
why phenol is acidic while ethanol is neutral. electricity because these solutions do not have ions,
(U.P. 2014, 15) i. e., alcohols do not ionised.
Ans. Phenoxide ion formed by elimination of H + ion from Q.10 Write down IUPAC name of the following :
phenol is stabilised by resonance while resonance (U.P. 2019)
does not take place in alkoxide ion ( RO - ). (a) CH 3 ¾ CH ¾ CH ¾ CH ¾ CH 3
| | |
C 6 H 5OH ¾® C 6 H 5O - + H +
CH 3 OH OH
Q.5 With the help of chemical equation, show how will
you prepare p-alcohol from Grignard reagent? Cl
|
(U.P. 2018)
O OMgX (b) CH 3 ¾ CH ¾ C ¾ CH 2OH
| |
½½ ½
Ans. H ¾ C ¾ H+ RMgX ¾® H ¾ C ¾ H Cl CH 3
½ 5 4 3 2 1
R Ans. (a) CH 3 ¾ CH ¾ CH ¾ CH ¾ CH 3
| | |
2H O
¾¾¾® RCH 2OH CH 3 OH OH
–MgX(OH)
4-methyl pentan-2,3 diol
Q.6 Write two reactions, which proves acidic nature of Cl
phenol. (U.P. 2014, 17) 4 3 | 1
Ans. (a) Phenol liberates H 2 gas with active metals like Na. 2
(b) CH 3 ¾ CH ¾ C ¾ CH 2 OH
2C 6 H 5OH + 2Na ¾® 2C 6 H 5O - Na + + H 2 | |
(b) With NaOH, it gives sodium phenoxide Cl CH 3
2,3-Dichloro-2-methyl butan-1-ol
C 6 H 5OH + NaOH ¾® C 6 H 5O - Na + + H 2O
Q.7 Arrange o, p and m-nitrophenol in increasing order of
Q.11 Why ortho nitrophenol is more acidic than ortho
acidic strength. (U.P. 2017)
OH OH OH methoxy phenol ? (U.P. 2019)
Ans. Ortho nitrophenol is more acidic than ortho methoxy
NO2 phenol because ¾ NO 2 group has -I effect which
Ans. > > increase the acidic nature while o-methoxy phenol
NO2 contains ¾OCH 3 methoxy group which show +I
o-nitrophenol m-nitrophenol effect and decrease acidic nature.
NO2
p-nitrophenol

Short Answer Type Questions


Q.1 C 2 H 5OH and CH 3OCH 3 , both have same molecular absent in CH 3OCH 3 , hence, it is a gas and has lower
mass ; however, at room temperature, C 2 H 5OH is a boiling point.
liquid while CH 3OCH 3 is a gas, and also boiling point Q.2 Solubility of alcohols in water decreases with increase
of C 2 H 5OH is higher than that of CH 3OCH 3 , why ? in their molecular weight, why ? (U.P. 2010, 18)
(U.P. 2014, 15) Ans. Solubility of alcohols in water is due to presence of
Ans. Due to presence of intermolecular hydrogen bonding, hydrogen bondings between alcohol molecules and
C 2 H 5OH is liquid and has higher boiling point. On the water molecules. As the size of alkyl group in alcohols
other hand, intermolecular hydrogen bonding is increases, strength of hydrogen bondings decreases
and hydrophobic part of alcohol becomes
114

predominant. For this fact solubility of alcohols in strength is, o-cresol (ii) < p-cresol (iv) < m-cresol <
water decreases with increase in their molecular phenol (i).
weight and higher alcohols are insolube in water. Q.5 Write short note on the following :
Q.3 What is Lucas test? Which types of compounds are (i) Kolbe-Schmidt reaction (U.P. 2014, 18, 19)
identified by this method. (U.P. 2010, 12, 17) (ii) Coupling reaction of phenol (U.P. 2015)
Ans. A mixture of anhydrous ZnCl 2 and conc. HCl is called Ans. (i) Kolbe-Schmidt reaction : When sodium salt
Lucas reagent. Lucas test is the simplest method of of a phenol is heated with CO 2 at 130º C and high
distinction between primary, secondary and tertiary pressure (4-8 atm), sodium salicylate is formed which
alcohols. When an alcohol is mixed with Lucas on acidification gives salicylic acid. This reaction is
reagent, at room temperature a tertiary alcohol gives called Kolbe-Schmidt reaction.
the turbidity immediately, a secondary alcohol gives ONa OH
OH
within 5 minutes while primary alcohol does not give COONa
any turbidity at room temperature. Thus, alcohols are 130ºC COOH
+CO2 ¾¾® H+
identified by Lucas test.
4-8 atm ¾¾®
Q.4 Arrange the following compounds in increasing order Sodium Sodium
phenoxide Salicylate Salicylic acid
of acidic strength.
OH OH (ii) Coupling reaction of phenol : Phenol reacts
with diazonium salt to form coloured compounds
OH
called azo-dyes and reaction is called coupling
(i) (ii)
reaction of phenol.
Phenol o-cresol NaOH
N==N—Cl+H— OH ¾¾®
30ºC
OH OH
Benzene diazonium
Phenol
chloride
(iii) (iv) (U.P. 2015)
OH N==N OH
m-cresol OH p-hydroxy azo benzene
p-cresol
Q.6 Write down three tests for differentiating methyl
Ans. Presence of electron releasing group which produce alcohol from ethyl alcohol. (U.P. 2014, 16)
+I effect decreases the acidic strength of phenol on all Or How will you differentiate between ethyl alcohol
the positions. Thus, cresols are less acidic than phenol and methyl alcohol? Write two tests. (U.P. 2018)
and m > p > o. Thus, the increasing order of acidic Ans.

S.No. Test Ethyl alcohol (C2H5OH) Methyl alcohol (CH3OH)


1. Reaction with bleaching powder Chloroform is formed No reaction
2. Reaction with I2 + NaOH Yellow crystals of iodoform are obtained Does not form iodoform
C 2H5OH + 4I2 + 6NaOH ¾®
CHI3 + 5NaI + HCOONa + 5H2O
3. Reaction with salicylic acid +conc. No odour of oil of winter green Methyl salicylate is formed with odour of
H2SO 4 oil of winter green.

Q.7 Phenol is more acidic than cyclohexanol, why? Ans. A. Lucas Test : (Lucas reagent — anhydrous ZnCl 2
(U.P. 2014, 15) + Conc. HCl)
Ans. The acidic nature of phenol is due to the formation of (i) t-alcohols give turbidity with Lucas reagent
stable phenoxide ion in solution which is stable due to immediately at R.T.
resonance, but cyclohexanol does not show 2ZnCl + Conc. HCl
R 3C ¾ OH ¾¾¾¾¾¾® R 3C ¾ Cl
resonance. Hence phenol is more acid than
cyclohexanol. (ii) s-alcohols give turbidity after 5 minutes at R.T.
C 6 H 5OH + H 2O l C 6 H 5O - + H 3O + R 2CH ¾ OH ¾¾¾¾¾¾¾®
Anhy. ZnCl + Conc. HCl
2
R 3CH ¾ Cl
Phenoxide ion
(iii) p-alcohols give turbidity only after heating.
Q.8 Differentiate between p, s and t-alcohols. (U.P. 2018)
Anhy. ZnCl + Conc. HCl
2
Or Write a dehydrogenation test for differentiating p, s RCH 2OH ¾¾¾¾¾¾¾® RCH 2Cl
and t-alcohols. Give equations also. (U.P. 2018) B. Dehydrogenation Test : On dehydrogenation
Or How are primary, secondary and tertiary alcohols (i. e., on heating alcohol with Cu at 573 K), p, s and t-
differentiated with each other ? Give only method for alcohols give different products.
it. (U.P. 2019) (i) p-alcohols give aldehydes
115

Cu/573 K (3) Halogenation :


RCH 2OH ¾¾¾® RCHO OH OH OH
–H 2

(ii) s-alcohols give ketones Cl


3 FeCl
Cu/573 K + Cl 2 ¾¾® +
R 2CHOH ¾¾¾® R 2C==O - HCl
–H 2

(iii) t-alcohols give alkenes Cl


R Q.10 Write only chemical equations of the dehydration
Cu/573 K
½ reactions of primary, secondary and tertiary alcohols
R 3C ¾ H ¾¾¾® alkene ( R ¾ C == R )
–H 2 (U.P. 2019)
Q.9 Explain the reactivity of phenol towards the Or Write chemical equation of one example of the
electrophilic substituion. Write chemical equations of formation of Ether by the dehydration of alcohols.
three examples of it. (U.P. 2019) (U.P. 2020)
Or Explain the mechanism of the formation of alcohol by Ans. Dehydration reactions of p,s & t-alcohols:
acid catalysed hydration. (U.P. 2020) On dehydration, alcohols give alkenes.
Ans. Reactivity of phenol in Electrophilic 2 4 Conc.H SO /D
CH 3CH 2CH 2CH 2CH 2OH ¾¾¾¾¾¾®
Substitution Reactions : - H 2O
p- alcohol
In electrophilic substitution reactions, benzene ring is CH 3CH 2CH 2CH == CH 2
more reactive than benzene. ¾OH group attached to Pentene–1
phenol. (Major product)
1. –I effect, which is due to high electronegativity of OH
O-atom. |
Conc.H 2 SO4 / D
2. +M effect, which is due to interaction of lone pair CH 3CH 2 CH ¾ CH 2CH 3 ¾¾¾¾¾¾®
of electron on O-atom with p-electron systems of ring. s- alcohol
+M effect dominates over –I effect and that is why CH 3CH 2CH == CHCH 3
Pentene– 2
overall electron density on ring and the increased (Major product)
electron density appears on o/p positions that is why CH3
the incoming E + is attached at o/p positions. OH
CH3 C CHCH3
O H
+
O H O
+
H
+
O H O H C CH2CH3 Conc.H
¾¾¾¾¾® 2 SO4 /D CH3
CH3 2-methyl butene-2
t-alcohols (Major product)

Order of reactivity ® t > s > p


Q.11 Explain acidic nature of phenol and describe the reason
O H
for the stability of phenoxide ion (U.P. 2019)
–d –d Ans. Acidic nature of phenol :
Phenol turns blue litmus red, so its nature is acidic.
With NaOH, it forms phenoxide, which proves its
E+ –d
acidic nature.
Examples : C 6 H 5OH + NaOH ¾® C 6 H 5O - Na + + H 2O
(1) Friedel-Crafts alkylation : Sodium
OH OH OH phenoxide
CH3 Phenoxide is very stable because it is stabilised by
CH 3 Cl/AlCl 3 + resonance.
¾¾¾¾¾®
O O O O O
CH3
(2) Nitration :
OH OH OH
NO2 O
dil.HNO3 +
¾¾¾¾® –d –d
NO2
–d
116

Long Answer Type Questions


Q.1 Write two general methods for the preparation of (c) Reaction of p-alcohols with acetic acid:
monohydric alcohol and also write the reaction of Ester are formed
concentrated sulphuric acid with ethyl alcohol at RCH 2OH + CH 3COOH ¾® RCH 2OCOCH 3 + H 2O
different temperatures. Also give all equations related Ester
to the above reactions. (U.P. 2016) (d) Reaction of p-alcohols with HX:
Or Write two general methods of preparation of primary RCH 2OH + HX ¾® RCH 2 X + H 2O
alcohol. How will you obtain ethene and ethoxy Alkyl halide
ethane from ethanol? Also write two main uses of Preparation of ethane from ethyl alcohol:
ethanol. Give equations of all the related reactions. Red P/HI
CH 3CH 2OH ¾¾¾¾® CH 3CH 3 + H 2O
(U.P. 2019) Ethyl alcohol Ethane
Or Write chemical equation of preparation of primary
alcohols from p-amines and alkenes. Write chemical Preparation of ethoxy ethane from ethanol :
D /Al O
2 3
equations of reaction of p-alcohols with acetyl 2C 2 H 5OH ¾¾¾¾® C 2 H 5OC 2 H 5 + H 2O
250º C
chloride and acetic acid. (U.P. 2020) Ethyl alcohol Ethoxy ethane
Or Write down the methods of preparation of primary Ethanol is used :
alcohol from primary amines and alkenes and write (i) in pharma industry as solvent.
the reactions of primary alcohol with the following : (ii) as fuel in the form of rectified spirit.
(i) RCOOH (ii) HX (U.P. 2020) Q.2 What is functional group of alcohols? How many types
Or Write two methods of preparation and three chemical of these alcohols are there? Write the names and
properties of alcohol and give chemical equations formulae of all possible isomeric alcohols by taking
also. (U.P. 2020) the example of molecular formula C 4 H10O. How are
Ans. General methods of preparation of these identified? Confirm by giving one test.
monohydric alcohol. (U.P. 2015)
(1) By hydrolysis of alkyl halides : Alkyl halides Ans. The functional group of alcohols is —OH (hydroxyl
are hydrolysed by alkali solution or moist silver oxide group)
to give corresponding monohydric alcohols. Types of alcohols : On the basis of number of
RX + NaOH ¾® ROH + NaX hydroxyl groups alcohols are classified as.
Alkyl Alcohol
halide (1) Monohydric alcohols : Those alcohols, which
or RX + AgOH ¾® ROH + AgX contain only one hydroxyl group (—OH) are called
Alkyl Alcohol
halide
Moist silver
oxide
monohydric alcohols, e. g ., CH 3OH,C 2 H 5OH.
(2) By the action of HNO 2 on primary (2) Dihydric alcohols : Those alcohols which
amines: Primary alcohols are formed when contain two hydroxyl groups are called dihydric
primary amines are treated with nitrous acid (HNO 2 ). alcohols.
RNH 2 + HNO 2 ¾® ROH + N 2 + H 2O For example : CH 2OH (glycol)
Primary |
amine
(3) By hydroboration: CH 2OH
(3) Trihydric alcohols : Those alcohols which
R — CH == CH 2 ¾(i)
¾B¾2H 6
¾¾¾- ® RCH 2CH 2OH
( ii ) H 2O2 / OH contain three hydroxyl groups, e. g ., CH 2OH × CHOH ×
CH 2OH (glycerol).
(a) Reaction of ethyl alcohol with conc. (4) Polyhydric alcohols : Those alcohols which
H 2SO 4 : On the warming ethyl alcohol with
contain more than three hydroxyl groups are called
concentrated H 2SO 4 at different temperatures, the
polyhydric alcohols e. g ., CH 2OH × (CHOH) 4 × CH 2OH
following reaction take place.
(sorbitol).
(i) C 2 H 5OH + H 2SO 4 ¾100ºC
¾¾® C 2 H 5 HSO 4 + H 2O
Ethyl alcohol Ethyl hydrogen
Isomeric of C 4 H10O : The possible isomeric of
sulphate C 4 H10O are given below.
140ºC
(ii) C 2 H 5 HSO 4 + C 2 H 5OH ¾ ¾¾® (i) CH 3CH 2CH 2CH 2OH (ii) CH 3 —CH — CH 2OH
C 2 H 5 — O — C 2 H 5 + H 2SO 4 Butanol -1 |
Diethyl ether (Primary alcohol)
CH 3
(iii) C 2 H 5 HSO 4 ¾160–170ºC
¾¾¾ ¾® C 2 H 4 + H 2SO 4 2-methyl propanol -1
(Primary alcohol)
Ethylene
(b) Reaction of p-alcohols with acetyl
chloride: Ester are formed
RCH 2OH + CH 3COCl ¾® RCH 2OCOCH 3 + HCl
Ester
117
CH 3 Water gas is mixed with half its volume of hydrogen
| and then the mixture is heated to 300º C in the
(iii) CH 3 — CH 2CHOH — CH 3 (iv) CH 3 —C — OH presence of catalyst (ZnO + Cr2O 3 ), vapour of methyl
Butanol - 2 |
(Secondary alcohol) alcohol formed.
CH 3 ZnO+Cr O
2 3
2-methyl propanol -2 CO+H2+H2 ¾¾¾¾®
300ºC
CH3OH
(Tertiary butyl alcohol)

ì
ê
í
ê
î
Methyl
Water gas
Identification of primary, secondary and alcohol
tertiary alcohols : These alcohols are identified by (2) By hydrolysis of methyl halides : Methyl
Lucas test. halides on hydrolysis with dilute aq. solution of NaOH
At room temperature a tertiary alcohol reacts with or KOH, give methyl alcohol.
Lucas reagent (conc HCl + anhydrous ZnCl 2 ) give the CH 3 X + NaOH ¾® CH 3OH + NaX
white turbidity immediately, a secondary alcohol gives Methyl
alcohol
with 5 minutes while primary alcohol does not give
Dimethyl ether from methyl alcohol :
any turbidity at room temperature.
Q.3 Write chemical equations of any two methods for 2CH 3OH ¾ Al
¾¾ 2O3
® CH 3 — O — CH 3 + H 2O
Methyl 250ºC Dimethyl ether
preparation of ethyl alcohol. How will you convert any alcohol
primary alcohol to secondary alcohol? (U.P. 2014) Reaction with Grignard reagent : Methane gas
Ans. Preparation of ethyl alcohol : is obtained.
(1) By hydrolysis of diethyl ether : On I
hydrolysis of diethyl ether with dil H 2SO 4 gives ethyl CH 3OH + CH 3 MgI ¾® CH4+Mg
Grignard Methane
OCH3
alcohol. reagent
Methoxy magnesium
C 2 H 5 — O — C 2 H 5 + H 2O ¾ dil
¾H¾ 2SO4
¾® 2C 2 H 5OH
¾ iodide
(2) By Grignard reagent : Grignard reagent on Ethyl alcohol from methyl alcohol :
reaction with formaldehyde followed by hydrolysis CH 3OH ¾ PCl
¾¾ 5
¾¾® CH 3CN ¾ LiAlH
® CH 3Cl ¾ KCN ¾¾ ¾4
®
Methyl [H]
gives ethyl alcohol. alcohol
+
CH 3 MgI + HCHO ¾® CH 3CH 2OMgI ¾ H¾¾ 3O
® 2
CH 3CH 2 NH 2 ¾ ¾¾
HNO
® CH 3CH 2OH
Grignard Formaldehyde
Ethyl amine Ethyl alcohol
reagent
OH Q.5 Write the chemical equation for the laboratory method
CH3CH2OH+Mg of preparation of phenol. Also write the equation of
Ethyl alcohol
I two chemical reactions showing the presence of —OH
Conversion of primary alcohol into group in phenol. (U.P. 2016)
secondary alcohol : Or Write chemical equations of any two methods for
RCH 2OH ¾ K¾
2Cr2O7 H 2SO4
¾ ¾ ¾¾® RCHO ¾ CH MgBr
¾3¾¾ ® the preparation of phenol. Also write two chemical
Primary [O] reaction to show the presence of —OH group in
alcohol phenol. (U.P. 2016)
CH 3 CH 3 Or Write two chemical equations of any two methods
| +
| of preparation of phenol. Write chemical equation for
R — C — O MgBr ¾ H¾¾
3O
® R — C — OH the Reimer-Tiemann reaction of phenol. (U.P. 2016)
| | Or Describe any one method for the preparation of
H H phenol. Write related chemical equation of the
Secondary
alcohol reaction. How is its chemical test performed?
Q.4 Discuss the method of preparation of methanol. How (U.P. 2018)
will you obtain ethanol from methanol ? Or Describe with chemical equation, the laboratory
(U.P. 2015, 16) method of preparation of phenol and write equation
Or Write two methods for preparation of methyl of following reactions : (U.P. 2018)
alcohol. Also write the necessary chemical equations. (i) Reimer-Tiemann reaction (U.P. 2020)
How will be obtain dimethyl ether from methyl (ii) Kolbe’s reaction (U.P. 2020)
alcohol? (U.P. 2016) Or Write two methods of preparation of Phenol. How
Or Giving suitable diagram, write two methods of does it react with the following? (U.P. 2019)
preparation of methyl alcohol. Write one test for (i) Conc. HNO 3
methyl alcohol which is not shown by ethyl alcohol. (ii) Br2 water
Give chemical reactions also. (U.P. 2017) (iii) Zinc dust (U.P. 2020)
Ans. Methods of preparation of methyl alcohol : Or Describe the methods for preparation of phenol
(1) From water gas : This method is mainly used from
now-a-days for the manufacture of methyl alcohol. (i) Diazonium salts and (ii) Cumene.
118

Write chemical equations for Kolbe reaction and (b) Reaction with NH 3 : Aniline is obtained.
Riemer-Tiemann reactions of phenol. (U.P. 2020) OH NH2
Ans. Methods of preparation of phenol
ZnCl2
1. From diazonium salt (Laboratery method): +NH3 ¾¾® +H2O
300ºC
Phenol is prepared when aqueous solution of benzene
Phenol Aniline
diazonium chloride is boiling with water.
NH2
+
N 2 Cl– OH (c) Reaction with Zn dust : Benzene is formed.
OH
NaNO /HCl
2 D
¾¾¾¾® + H2O ¾¾®
30ºC +N2+HCl D
+Zn(dust) ¾¾® +ZnO
Benzene Phenol
diazonium chloride Phenol Benzene
2. From salicylic acid : Phenol is formed when (2) Reimer-Tiemann reaction : In the presence of
salicylic acid is heated with soda lime (NaOH + CaO). NaOH, when phenol is heated with chloroform,
OH OH salicylaldeyde is obtained and reaction is called
COOH Reimer-Tiemann reaction.
CaO OH OH
+2NaOH ¾¾® D +Na2CO3+H2O
CHO
60ºC
Salicylic acid Phenol +CHCl3+3NaOH ¾¾® +3NaCl+2H2O
3. From chlorobenzene:
Phenol Salicylaldehyde
Cl OH
Kolbe’s reaction :
(i) 10% NaOH, 200 atm . 300°C OH OH
¾¾¾¾¾¾¾¾¾¾®
+
(ii) HOH/H
COOH
Phenol (i) CO2/alkali medium
¾¾¾¾¾¾¾®
4. From cumene: +
(ii) H2O/H
O — OH
| Reactions of Phenol With :
HOH / H +
C 6 H 5 — CH(CH 3 ) 2 ¾ O¾
2
® C 6 H 5C(CH 3 ) 2 ¾ ¾¾® (i) Conc. HNO 3 – picric acid is formed :
Cumene OH
OH
C 6 H 5OH + CH 3COCH 3 NO2 NO2
Phenol Conc. HNO
3
¾¾¾¾¾®
Acidic nature of phenol : Phenol is a weak acid.
The acidic nature of phenol is due to the formation of NO2
stable phenoxide ion in solution. 2,4,6-trinitrophenol
(picric acid)
OH O–
(ii) Br2 water–2,4,6–tribromophenol is formed
+ H2O + H3 O+ OH OH
Br Br
Phenoxide ion Br2 water
The phenoxide ion is stable due to resonance. ¾¾¾¾®
O– O O
– Br
2,4,6-tribromophenol
(iii) Zn–benzene is formed
– OH
O O– Zn dust /distil
¾¾¾¾¾®

Phenol Benzene

Reactions : (iv) NaOH- Sodium phenoxide is formed


1. Reactions of —OH group : OH O–Na+
(a) Reaction with PCl 5 : Chlorobenzene is formed.
NaOH
OH Cl ¾¾¾¾® + H2O

+PCl5 ¾¾® +POCl3+HCl Test of phenol : Phenol on heating with NaNO 2 /


conc. H 2SO 4 gives brown colour which on dilution
Chlorobenzene
119

with water gives blue solution (Liebermann nitroso Thistle funnel


test). Ethyl alcohol
Q.6 Describe the laboratory method for the preparation of Thermometer
diethyl ether and write the equation of its reaction
with H 2O and cold HI. (U.P. 2016) Water
Or Describe Williamson’s synthesis method for the
preparation of diethyl ether and write its reaction with
the following: (i) PCl 5 (ii) HI (iii) Na NH 3( p)
(iv) P/HI (U.P. 2014)
Or Describe laboratory method of preparation of pure Distilling
flask
diethyl ether. Give equations and diagram of
apparatus also. Mention its important properties and Ethyl alcohol Water Ice
+ Conc. H2SO4 To sink
uses. Why does it called as continuous etherification
method ? (U.P. 2010, 12, 14, 15, 16) Ether
Or Describe with diagram, the laboratory method of
preparation of diethyl ether. Write equation of its
Fig. Laboratory method of preparation of diethyl ether
chemical reaction with the following :
(i) Cold HI (ii) Acetyl chloride (U.P. 2018) In the reaction, H 2SO 4 used in first step is liberated in
Or Write chemical equations for the preparations of second step. Since, sulphuric acid is not used up in the
ether by continuous Etherification and Williamson’s reaction, extra amount of sulphuric acid is not
synthesis. Write chemical equations of reactions of required. This method, therefore, is termed as
ether with acetic anhydride and hydrogen iodide. Also continuous etherification process. However, in
write one test of ether. (U.P. 2019) actual practice, the water eliminated during the
Or Write the method and chemical equation for the dehydration of alcohol, dilutes the acid and so acid is
preparation of Diethyl Ether by Williamson’s added at frequent intervals. If benzene sulphonic acid
synthesis. (U.P. 2019) (C 6 H 5SO 3 H) is used in place of H 2SO 4 , the reaction
Or Describe two methods for the preparation of ether occurs continuous. The ether thus obtained has
and write equations of its chemical reactions with the impurities of alcohol, water, H 2SO 4 , H 2SO 3 etc.
following : Williamson’s synthesis: When ethyl bromide or
(i) CH3 COCl (ii) PCl 5 (iii) Hot HI. (U.P. 2020) ethyl iodide is heated with sodium ethoxide, diethyl
Or Write chemical reaction of preparation of ethers by ether is obtained. This reaction also confirms the
dehydration of alcohols. (U.P. 2020) structure of diethyl ether. Potassium ethoxide can also
Ans. Methods of preparation of diethyl ether: be used in place of sodium ethoxide. Reaction takes
1. From ethyl alcohol (Laboratory method) : place S N 2 reaction.
Principle : When ethyl alcohol is heated with conc.
H 2SO 4 at 140º C, diethyl ether or simply ether is R'—X + Na—O
R ¾® R'OR + Na
X
obtained. CH3CH2 I + Na OCH2CH3 CH3CH2OCH2CH3 + NaI
C 2 H 5OH + H 2SO 4 ¾100ºC
¾¾® C 2 H 5 HSO 4 + H 2O Sodium ethoxide
Ethyl alcohol Ethyl hydrogen
sulphate Reactions :
140ºC (1) Reaction with H 2O : Ethyl alcohol is obtained.
C2H5HSO4+HOC2H5 ¾¾® C 2 H 5OC 2 H 5 + H 2SO 4
Ether C 2 H 5 — O — C 2 H 5 + H 2O ¾ dil
¾H¾ 2SO4
¾® 2C 2 H 5OH
¾
Method and labelled diagram : Ethyl alcohol (2 Ether Ethanol

volume) and conc. H 2SO 4 (1 volume) are taken in a (2) Reaction with HI : Ethyl alcohol and ethyl
distillation flask and heated on sand bath at 140ºC, iodide is formed.
ether distils over and is collected in a receiver cooled C 2 H 5 — O — C 2 H 5 + HI ¾¾® C 2 H 5OH + C 2 H 5 I
in ice-cold water. Ether Ethyl alcohol Ethyl
iodide
Ether so obtained possess the impurities of water,
alcohol and sulphurous acid. It is purified by first (3) Reaction with PCl 5 : Ethyl chloride is formed.
D
shaking with dil NaOH which removes sulphurous C 2 H 5 — O — C 2 H 5 + PCl 5 ¾¾ ® 2C 2 H 5Cl+ POCl 3
Ether Ethyl
acid, then separating the etheral layer and treating chloride
with 50% CaCl 2 solution which removes ethyl alcohol.
(4) Reaction with Na/NH 3 (l ):
The etheral layer is then separated again, dried over
Na/NH ( l )
3
anhydrous CaCl 2 which absorbs water and ether C 2 H 5OC 2 H 5 ¾¾¾¾¾® C 2 H 6 + C 2 H 5OH
Reduction 2[H]
remains in the liquid.
120

(5) Reaction with Red P/HI: Differentiation among p,s and t-alcohols
P/HI
C 2 H 5OC 2 H 5 ¾¾¾® 2C 2 H 6 + H 2O (Lucas test) : Mixture of anhydrous ZnCl 2 and conc.
4[H] HCl is called Lucas reagent.
(6) Acetic anhydride : (i) p-alcohols give turbidity with Lucas reagent only
C 2 H 5OC 2 H 5 + (CH 3CO) 2 O ¾® 2CH 3COOC 2 H 5 on heating
Ethyl acetate 2 ZnCl
RCH 2OH + HCl ¾¾® RCH 2Cl + H 2O
Test of ether : Zeisel test D
Ether is first heated with HI / CH 3COOH, where alkyl (very slow reaction)
iodide is formed. These vapours are passed over (ii) s-alcohols give turbidity with Lucas reagent at
Hg(NO 3 ) 2 paper. Formation of red crystals confirms room temperature but after some time.
ester. 2 ZnCl
R 2CHOH + HCl ¾¾¾®
R.T.
R 2CHCl + H 2O
3 CH COOH
R ¾ O ¾ R + HI ¾¾¾¾® RI + ROH (slow reaction)
RI + Hg 2+ ¾® HgI 2 ¯ (iii) t–alcohols give turbidity with Lucas reagent
Red
immediately.
Williamson’s synthesis : In this reaction alkyl 2 ZnCl
halide is treated with a suitable sodium alkoxide. R 3C ¾ OH + HCl ¾¾¾®
R.T.
R 3C ¾ Cl + H 2O
CH 3CH 2O - Na + + CH 3CH 2 Br ¾® CH 3CH 2OCH 2CH 3 (fast reaction)
Diethyl ether
Q.8 Write down the chemical equations for the
+ NaBr preparation of phenol from (i) salicylic acid, (ii)
This method is suitable for the preparation of chlorobenzene and (iii) toluene and write the
symmetrical and unsymmetrical ethers. chemical equation of the reactions of phenol with (i)
Note : If tertiary halide is treated with sodium CHCl 3 and KOH and (ii) with conc. HNO 3 (in
ethoxide then it will form alkene. presence of conc. H 2SO 4 ). Also write the names of
CH 3 main products formed in these reactions. (U.P. 2019)
| Ans. A. Preparation of phenol from :
D
CH 3 ¾ C ¾ Br + C 2 H 5O - Na + ¾® CH 2 == C ¾ CH 3 OH OH
| | COOH
CH 3 CH 3 Soda lime/D
(i) ¾¾¾®
t-butyl bromide Act as a base Salicylic acid Phenol
+ C 2 H 5OH + NaBr
Cl OH
Q.7 Write down the methods of preparation of ethyl
alcohol from (i) ethylamine and (ii) acetylene and NaOH(aq.)
also write the chemical equation for the reaction of (ii) ¾¾¾®
High P&t
alkaline solution of iodine with ethyl alcohol. Write in Chlorobenzene Phenol
brief the differentiation of primary, secondary and CH3
tertiary alcohol by Lucas test (U.P. 2019)
Ans. (a) Preparation of ethyl alcohol from O2 /MnCl 2 + CuCl 2
(i) Ethylamine : (iii) ¾¾¾¾¾¾®D
C 6 H 5OH+ CO 2 + H 2O
Phenol
C 2 H 5 NH 2 + HNO 2 ¾® C 2 H 5OH + N 2 + H 2O Toluene
Ethylamine Nitrous Ethyl alcohol
acid B. Reaction of phenol with :
(ii) Acetylene :
H SO /HgSO
(i) CHCl 3 / KOH ¾Reimmer-Tiemann reaction
2 4 4
HC ºº CH+ H 2O ¾¾¾¾¾® CH 3CHO OH OH
Acetylene Acetaldehyde CHO
H /Ni CHCl 3 /KOH
¾¾¾¾®2
CH 3CH 2OH ¾¾¾¾¾®
60º -70º C
120º -140ºC
Ethyl alcohol
Salicylaldehyde
(b) Reaction of ethyl alcohol with I 2 /alkali
(Iodoform reaction) (ii) Conc. HNO 3 ¾ Nitration
OH
CH 3CH 2OH + 4I 2 + 6NaOH ¾® CHI 3 ¯ + 5NaI NO2 NO2
Iodoform
+ HCOONa + 5H 2O OH
Conc.HNO
3 NO2
¾¾¾¾¾®
D /Conc.H 2 SO4 2,4,6–trinitrophenol
(Picric acid)
121

Q.9 Write preparation methods of methyl alcohol from (b) Reaction of methyl alcohol with Grignard
formaldehyde and methyl acetate. Write chemical reagent and oxalic acid :
equations for the reactions of methyl alcohol with (i) CH 3OH + CH 3 MgI ¾® CH 4 + Mg(I)OH
Grignard reagent and oxalic acid. Also write its main G.R. Methane
uses. (U.P. 2019) COOH COOCH 3
Conc.H 2 SO4
Ans. (a) Preparation of methyl alcohol from (ii) | + 2CH 3OH ¾¾¾¾® | + 2H 2O
formaldehyde and methyl acetate : COOH COOCH 3
Reduction Dimethyl oxalate
(i) HCHO ¾¾¾¾¾® CH 3OH
Formaldehyde LiAlH 4 /Ether Methyl alcohol
Methyl alcohol is used as industrial solvent and as
fuel.
2dil.H SO
4
(ii) CH 3COOCH 3 + H 2O ¾¾¾® CH 3COOH+ CH 3OH
Methyl
alcohol

Question Based on Chemical Reactions


Q.1 How will you obtain : OH
(i) methyl amine from ethyl alcohol ? |
(U.P. 2014) R — C — R ¢ + MgX (OH)
(ii) ethyl alcohol from acetylene ? (U.P. 2013) |
(iii) ethylene from ethyl alcohol ? (U.P. 2017) R ¢¢
(iv) ethyl alcohol from Grignard reagent ? 2Cl /hv KCN
(vi) CH 4 ¾¾¾®
–HCl
CH 3Cl ¾¾® CH 3CN
(U.P. 2016)
Or p-alcohol from Grignard reagent? (U.P. 2018) Reduction 2 HNO
¾¾¾® CH 3CH 2 NH 2 ¾¾® CH 3CH 2OH
(v) tertiary alcohols from Grignard reagent ?
(U.P. 2011) [O ] 2 Ca( OH )
(vii) CH 3CH 2OH ¾® CH 3COOH ¾¾¾®
(vi) ethyl alcohol from methane? (U.P. 2015, 17) (CH3COO)2Ca
(vii) acetone from ethyl alcohol ? (U.P. 2017)
(viii) acetic acid from methyl alcohol. D/distillation
(U.P. 2014, 15, 17)
(ix) ethyl acetate from ethyl alcohol. (U.P. 2017) CH3COCH3
(x) ethyl amine from methyl alcohol. (U.P. 2017) PCl 3 KCN
(xi) ethyl alcohol from diethyl ether (U.P. 2018) (viii) CH 3OH ¾¾® CH 3Cl ¾¾® CH 3CN
(xii) diethyl ether from ethyl iodide (U.P. 2018) HOH /H+
(xiii) ethyl acetate from diethyl ether (U.P. 2018) ¾¾¾¾® CH 3COOH
(xiv) methyl alcohol from ethyl alcohol (U.P. 2020) D/ H +
(xv) acetic acid from methyl alcohol (U.P. 2020) (ix) C 2 H 5OH + CH 3COOH ¾¾® CH 3COOC 2 H 5
[O] NH3/D + H 2O
Ans. (i) C2H5OH CH3COOH CH3CONH2
K2Cr2O7/H+ 3 PCl KCN
(x) CH 3OH ¾¾® CH 3Cl ¾¾® CH 3CN
Br2/KOH
4[H]
¾¾¾® CH 3CH 2 NH 2
CH3NH2
2 dil. H SO
4
2 4H SO / HgSO
4 (xi) C 2 H 5OC 2 H 5 + H 2O ¾¾¾¾® 2C 2 H 5OH
(ii) CH ºº CH + H 2O ¾¾¾¾¾® CH 3CHO
(xii) C 2 H 5 I+ NaOC 2 H 5 ¾¾¾® C 2 H 5OC 2 H 5
2 H / Ni –NaCl
¾¾¾¾® CH 3CH 2OH
120°-140°C
(xiii) C 2 H 5OC 2 H 5 + CH 3COCl ¾® CH 3COOC 2 H 5
Conc. H 2SO4 + C 2 H 5Cl
(iii) C 2 H 5OH ¾¾¾¾®
180°C
CH 2 == CH 2 + H 2O
[O ] Sodalime
(xiv) CH 3CH 2OH ¾ ¾ ¾¾® + CH 3COOH ¾ ¾ ¾¾®
+ K 2Cr2O7 / H D
HOH /H Ethyl alcohol
(iv) HCHO+CH 3 MgX ¾¾¾® CH 3CH 2OH KOH
Cl 2 / hv aq.
+ MgX (OH) CH 4 ¾ ¾ ¾ ¾® CH 3Cl ¾¾¾¾® CH 3OH
Methyl alcohol
PCl 5 +
O OMgX ¾® CH 3CN ¾ H¾2O¾¾
NaCN
(xv) CH 3OH ¾¾® CH 3Cl ¾ /H
®
|| |
/H+ CH 3COOH
(v) R — C — R ¢ + R ¢¢ MgX ¾® R — C — R ¢ HOH
¾¾®
Acetic acid
|
R ¢¢ Q.2 What happens when: (give chemical equations also)
(i) ethyl alcohol is heated with iodine and
concentrated NaOH solution ? (U.P. 2013)
122

(ii) vapours of ethyl alcohol are passed over heated OH NH2


alumina at 360°C ? (U.P. 2012)
(iii) ethyl alcohol reacts with anhydrous acetic acid in 3NH /ZnCl
2
(ii) ¾¾¾®
presence of sulphuric acid? (U.P 2016) –H2O
(iv) ethyl alcohol reacts with conc. H 2SO 4 at OH OH
different temperatures. (U.P. 2018)
(v) methyl alcohol reacts with acetyl chloride. NO2 NO2
Conc. HNO3/H2SO2
(U.P. 2018) (iii) ¾¾¾¾¾®
D
Ans. (i) CH 3CH 2OH+ 4I 2 + 6NaOH ¾®
CHI 3 + 5NaI+ HCOONa+ 5H 2O NO2
Al 2O3 /D
(ii) 2C 2 H 5OH ¾¾¾¾®
250°C
C 2 H 5OC 2 H 5 + H 2O OH OH COOH
2 4 dil. H SO COOH
(iii) C 2 H 5OH+CH 3COOH ¾¾¾® CH 3COOC 2 H 5 (i) CO2/NaOH Zn dust
(iv) ¾¾¾® ¾¾®
+ H 2O (ii) HOH
100°C
(iv) CH 3CH 2OH + H 2SO 4 ¾¾® CH 3CH 2 HSO 4
+ H 2O OH CH3
140°C
CH 3CH 2 HSO 4+ C 2 H 5OH ¾¾® CH 3CH 2OCH 2CH 3 Zn dust CH Cl/AlCl
3 3
+ H 2SO 4 (v) ¾¾® ¾¾¾®
160° -170°C
CH 3CH 2 HSO 4 ¾¾¾¾® CH 2==CH 2 + H 2SO 4 OH OH
(v) CH 3OH+CH 3COCl ¾¾® CH 3COOCH 3 CHO
- HCl
CHCl /KOH
3
Q.3 Write the name and formula of A, B and C in the (vi) ¾¾¾® (R.T. Reaction)
60-70 °C
following reactions. (U.P. 2014)
OH OH
O
Anhydrous ZnCl 2
(i) + NH3 ¾¾¾¾¾¾® A 2O /air
D (vii) ¾¾®
NH2 N==N—Cl Cl
O
B C
Q.5 What happens when:
(ii) ¾¾¾® ¾¾® (i) phenol reacts with NaOH? (U.P. 2016)
0–5°C
(ii) Phenol is heated with mixture of conc. HNO 3 and
NH2 conc. H 2SO 4 ? (U.P. 2014)
(iii) phenol is acylated in presence of pyridine?
Ans. (i) A = aniline (U.P. 2016)
OH O–Na+
(ii)B = NaNO 2 HCl or HNO 2 HCl
(iii) C = Cu 2Cl 2 (Cuprous chloride)/HCl Ans. (i) NaOH
¾¾®
Q.4 How will you convert phenol into:
(i) benzene (U.P. 2016, 17, 19, 20) OH OH
(ii)aniline (U.P. 2017)
NO2 NO2
(iii) picric acid (U.P. 2010, 17, 19) Conc. HNO3/H2SO2
(iv) benzoic acid (U.P. 2014) (ii) ¾¾¾¾¾®
(v) toluene (U.P. 2017)
(vi) Salicylaldehyde (U.P. 2019) NO2
(vii) Benzoquinone (U.P. 2019) OH OCOCH3
OH
CH3COCl/P4
Zn dust (iii) ¾¾¾®
Ans. (i) ¾¾¾®
Q.6 What happens when (write chemical equations only)
(i) Diethyl ether reacts with PCl 5
(ii) Ethyl alcohol is heated with red phosphorus and
HI
123

(iii) Ethyl alcohol is heated with bleaching powder (b) Br2 reacts with phenol in presence of CS 2 .
and water. (U.P. 2016, 18) (c) Why the B.P. of ethanol is more than methoxy
Ans. (i) Ethyl chloride is formed. methane ?
C 2 H 5 — O — C 2 H 5 + PCl 5 ¾® 2C 2 H 5Cl + POCl 3 Ans.
Diethyl ether Ethyl OH
Chloride |
(ii) Ethane is formed alk.
(a) CH 3 ¾ CH ¾ CH 3 ¾¾¾® CH 3COCH 3
C 2 H 5OH +2HI ¾ Red
¾¾ P
® C 2 H 6 + H 2O + I 2 KMnO 4
Ethanol Ethane OH
(iii) Chloroform is obtained. OH
OH
CaOCl 2 + H 2O ¾® Ca(OH) 2 + Cl 2 Br
2 CS
Cl 2 + H 2O ¾® 2HCl + [O] (b) + Br2 ¾® +
C 2 H 5OH ¾[O]
¾® CH 3CHO + H 2O
o-Bromophenol Br
CH 3CHO + 3Cl 2 ¾® CCl 3CHO+ 3HCl p-Bromophenol
Chloral
(c) In ethanol intermolecular H 2 bonding is present
2CCl 3CHO + Ca(OH) 2 ¾® 2CHCl 3 + (HCOO) 2 Ca so its b.p. is more than methoxy methanone.
Chloroform
Q.7 How will you obtain (write the chemical equations O ¾ H- - - O ¾ H - - -
only)? | |
(i) Ethanol from methanol C 2H 5 C 2H 5
(ii) Phenolphthalein from phenol (U.P. 2016, 20) Q.9 Write only chemical equations for the following :
Or (i) Benzene from phenol (U.P. 2020) (U.P. 2019)
(ii) Ethylamine from methyl alcohol (i) Two general methods for the preparation of
(iii) Ethylene from ethyl alcohol (U.P. 2016) monohydric alcohol
Ans. (i) CH 3OH ¾ PCl
¾¾ 5
® CH 3Cl ¾ KCN
¾¾® (ii) Ethyl alcohol from Methyl alcohol
Methanol (iii) Ethylene from Ethyl alcohol
CH 3CN ¾ LiAlH
¾¾ ¾4
® (iv) Methyl amine from Ethyl alcohol
[H]
Ans. (i) Two general methods of preparation
CH 3CH 2 NH 2 ¾ HNO
¾¾ 2
® CH 3CH 2OH of monohydric alcohol :
Ethyl amine Ethanol
(1) From haloalkanes :
OH OH
CH 3CH 2 Br + KOH( aq.) ¾® CH 3CH 2OH + KBr
OH OH (2) From hydration of alkenes :
(ii) (Phenol)
CH 2 == CH 2 + HOSO 2OH ¾® CH 3CH 2OSO 2OH
2 H O
O ¾¾® CH 3CH 2OH + H 2SO 4
H H
C (ii) Ethyl alcohol from methyl alcohol :
C Conc H SO
2 4
O ¾¾¾¾® O P + Br
2 KCN 2 H+ H O
–H O 2 CH 3OH ¾¾¾® CH 3 Br ¾¾® CH 3CN ¾¾¾®
C C or PBr3
Pd/BaSO4
O O CH3COOH CH3COCl CH3CHO
PCl5 S, [H]
Phthalic anhydride Phenolphthalein
LiAlH4 [H]
OH
CH3CH2OH
D CH3CH2OH
Or (i) +Zn (dust) ¾¾® +ZnO
(iii) Ethylene into ethyl alcohol :
Phenol Benzene CH 2 == CH 2 + HOSO 2OH ¾® CH 3CH 2OSO 2OH
(ii) CH 3OH ¾ PCl
¾¾ 5
® CH 3Cl ¾ KCN
¾¾® CH 3CN ¯ H 2O
Methyl CH 3 CH 2OH + H 2SO 4
alcohol (iv) Methyl amine from ethyl alcohol :
¾ LiAlH
¾¾ ¾4
® CH 3CH 2 NH 2 K Cr O
2 2 7 3 NH
[H] Ethylamine
C 2 H 5OH ¾¾¾®
+ H SO
CH 3COOH ¾¾® CH 3COONH 4
2 4
(iii) C 2 H 5OH ¾160–170ºC
¾¾¾ ¾® CH 2 D 2 Br
Ethyl –H 2O
|| ¾¾®
-H O
CH 3CONH 2 ¾¾®
+ KOH
CH 3 NH 2
alcohol 2
CH 2 (Hofmann Bromamide
Ethylene reaction)
Q.8 What happens when : (Give equations only)
(U.P. 2019)
(a) 1-propanol is oxidised with alkaline KMnO 4 .
124

Q.1 Write the IUPAC names of the following compounds : (iv) 2, 3-diethylphenol
CH 3 (v) 1-ethoxypropane
|
(vi) 2-ethoxy-3-methylpentane
(i) CH 3 ¾ CH ¾ CH ¾ C ¾ CH 3
| | | (vii) cyclohexylmethanol
CH 3 OH CH 3 (viii) 3-cyclohexylpentan-3-ol
(ii) CH 3 ¾ CH ¾ CH 2 ¾ CH ¾¾ CH — CH 2CH 3 (ix) cyclopent-3-en-1-ol
| | | (x) 3-chloromethylpentan-1-ol
OH OH C 2H 5 CH 3
(iii) CH 3 ¾ CH ¾¾ CH ¾ CH 3 |
| | Ans. (i) CH 3 ¾ C ¾ CH 2 ¾ CH 3
OH OH |
OH
(iv) HOCH 2 ¾ CHOH ¾ CH 2OH
CH3 CH3 (ii) CH2—CH — CH3
OH
(v) (vi) OH
CH 3 CH 3
| |
OH (iii) CH 2 ¾ CH 2 ¾ C ¾ CH 2 ¾ C ¾ CH 3
CH3 | | |
CH3
OH OH OH
OH OH
(vii) (viii)
OH C2H5
CH3
CH3 (iv) (v) CH 3CH 2O ¾ CH 2CH 2CH 3
(ix) CH 3 ¾ O ¾ CH 2 ¾ CH ¾ CH 3 C2H5
| (vi) CH 3 ¾ CH ¾ CH ¾ CH 2CH 3
(x) C H ¾ O ¾ C H CH 3 | |
6 5 2 5
(xi) C 6 H 5 ¾ O ¾ C 7 H15 (n-) CH 3CH 2O CH 3
(xii) CH 3CH 2 ¾ O ¾ CH ¾ CH 2CH 3 CH2OH
| (vii)
CH 3
Ans. (i) 2, 2, 4-trimethylpentan-3-ol, OH
(ii) 5-ethylheptane-2,4-diol,
(iii) butane-2, 3-diol, CH3—CH2—C —CH2— CH3
(viii)
(iv) propane-1, 2, 3-triol,
(v) 2-methylphenol,
(vi) 4-methylphenol,
(vii) 2, 5-dimethylphenol, OH
(viii) 2, 6-dimethylphenol
(ix) 1-methoxy-2-methylpropane, (ix) (x) HO ¾ CH 2 ¾ CH 2 ¾ CH ¾ CH 2 ¾ CH 3
|
(x) ethoxybenzene,
CH 2Cl
(xi) 1-phenoxyheptane,
Q.3 (a) Draw the structures of all isomeric alcohols of
(xii) 2-ethoxybutane molecular formula C 5 H12O and give their IUPAC
Q.2 Write the structures of the compounds whose IUPAC names.
names are as follows : (b) Classify the isomers as primary, secondary and
(i) 2-methylbutan-2-ol tertiary alcohols.
(ii) 1-phenylpropan-2-ol Ans. Eight isomers possible are
(iii) 3, 5-dimethylhexane-1, 3, 5-triol (i) CH 3CH 2CH 2CH 2CH 2OH
Pentan-1-ol (1º)
125

OH Q.8 While separating a mixture of ortho- and para-


|
nitrophenols by steam distillation, name the isomer
(ii) CH 3CH 2CH 2 ¾ CH ¾ CH 3
Pentan-2-ol (2º) which is steam volatile. Give reasons.
OH Ans. o-nitrophenol, due to intramolecular H-bonding in it.
| Q.9 Give the equations of reactions of preparation of
(iii) CH 3CH 2 ¾ CH ¾ CH 2CH 3 phenol from cumene.
Pentan-3-ol (2º)
Ans.
CH 3 O O H
|
(iv) CH 3CH 2 ¾ CH ¾ CH 2OH H3C CH3 H3C CH3
2-methylbutan-1-ol (1º)
CH C
CH 3
O2
|
(v) CH 3 ¾ CH ¾ CH 2CH 2 ¾ OH
3-methylbutan-1-ol (1º)
CH 3 OH
|
(vi) CH 3 ¾ C ¾ CH 2CH 3 H2O + H+
+CH3COCH3
|
OH
2-methylbutan-2-ol (3º) Q.10 Write chemical reaction for the preparation of phenol
CH 3 OH from chlorobenzene.
| | Ans.
(vii) CH 3 ¾ CH ¾ CH ¾ CH 3 Cl O–Na+ OH
3-methylbutan-2-ol (2º)
CH 3 10% NaOHaq. H2O/H+
|
200 atm/300°C
(viii) CH 3 ¾ C ¾ CH 2OH
| Sodium Phenol
phenoxide
CH 3
2, 2-dimethylpropan-1-ol (1º) Q.11 Write the mechanism of hydration of ethene to form
Q.4 Explain why propanol has higher boiling point than ethanol.
that of the hydrocarbon, butane ? Ans. HO ¾ SO 2 ¾ OH ¾® H + + O - SO 2OH
(H 2SO4 )
Ans. Due to intermolecular H-bonding in propanal.
Q.5 Alcohols are comparatively more soluble in water 2
+ O- SO OH
CH 2 == CH 2 + H + ¾® CH 3CH 2 ¾¾¾®
than hydrocarbons of comparable molecular masses.
CH 3CH 2OSO 2OH
Explain this fact. Ethyl hydrogen sulphate
Ans. Due to ability of alcohol molecules to form H-bond
CH 3 ¾ CH 2OSO 2OH + H 2O ¾® H 2SO 4
with water.
+ CH 3CH 2OH
Q.6 What is meant by hydroboration-oxidation reaction?
Illustrate with an example. Q.12 You are given benzene, conc. H 2SO 4 and NaOH.
Ans. Hydroboration-oxidation : Write the equations for the preparation of phenol
6( R — CH == CH 2 ) + B 2 H 6 ¾¾® 2( RCH 2CH 2 ) 3 B using these reagents.
Conc. H 2SO4 NaOH
¾ 6H
¾2¾O2
¾® 6RCH 2CH 2OH + 2H 3 BO 3 Ans. C 6 H 6 ¾¾¾¾® C 6 H 5SO 3 H ¾¾®
OH - Heat Benzene sulphonic acid
Fuse

Q.7 Give the structure and IUPAC names of monohydric HCl


phenols of molecular formula C 7 H 8O. C 6 H 5ONa ¾¾® C 6 H 5OH
Sodium phenoxide
OH OH OH Q.13 Show how will you synthesise ?
CH3 (i) 1-phenylethanol from a suitable alkene.
Ans. (ii) Cyclohexylmethanol using an alkyl halide by an
CH3 S 2 reaction.
O-cresol N
m-cresol (iii) Pentan-1-ol using a suitable alkyl halide.
CH3
p-cresol
126

Ans. Q.18 Explain the following with an example :


CH==CH2 (i) Kolbe’s reaction (U.P. 2018, 19)
Dil. H
2SO4
(i) + H—OH (ii) Reimer-Tiemann reaction (U.P. 2018)
Ethenylbenzene
(iii) Williamson’s ether synthesis (U.P. 2018, 19)
CHOH—CH3 (iv) Unsymmetrical ether (U.P. 2018)
Ans. (i) Kolbe’s reaction : When phenoxide (or phenol
in NaOH) is heated with CO 2 followed by hydrolysis,
1-phenylethanol salicyclic acid is formed. This reaction is called
CH2Br Kolbe’s reaction.
D
(ii) + NaOH OH O–Na+ OH
SN2 Hydrolysis
Cyclohexylmethyl COOH
bromide NaOH (i) CO2
CH2OH –H2O (ii) H+
+ NaBr
(ii) Reimer-Tiemann reaction : When phenol
Cyclohexylmethanol reacts with CHCl 3 / NaOH aq. , followed by hydrolysis,
salicylaldehyde is formed. This reaction is called
(iii) CH 3CH 2CH 2CH 2CH 2 Br + aq. KOH ¾® Reimer-Tiemann reaction.
CH 3CH 2CH 2CH 2CH 2OH OH O–Na+
Q.14 Give two reactions that show the acidic nature of CHCl2
phenol. Compare its acidity with that of ethanol. CHCl3/aq. NaOH NaOH
Ans. C 6 H 5OH + Na ¾® C 6 H 5ONa + 1/2 H 2
C 6 H 5OH + NaOH ¾® C 6 H 5ONa + H 2O
Q.15 Explain why ortho-nitrophenol is more acidic than O–Na+ O–Na+
OH
ortho-methoxyphenol ? CH CHO
OH
Ans. ¾ NO 2 gp. is electron attracting while ¾OCH 3 is
–H2O
electron releasing.
O H O H Unstable
OH
NO OCH3
2 CHO
HOH/H+

Q.16 Explain how does —OH group attached to a carbon (iii) Williamson’s ether synthesis : Halides
of benzene ring activates it towards electrophilic react with alkoxides to form ethers. This reaction is
substitution? called Williamson’s synthesis.
Ans. Because —OH group increases the electron density RX + R ¢O – ¾® ROR ¢ + X -
on benzen ring due to +M effect and hence its abilly
CH 3Cl+CH 3ONa ¾® CH 3OCH 3 + NaCl
to attract electrophile increases.
(iv) Those ethers in which the alkyl/aryl groups
Q.17 Give equations of the following reactions :
attached to ¾ O ¾ group are different are called
(i) Oxidation of propan-1-ol with alkaline KMnO 4
unsymmetrical ethers.
solution.
e. g ., CH 3OCH 2CH 3 , C 2 H 5OC 6 H 5 , C 2 H 5OC 3 H7
(ii) Bromine in CS 2 with phenol.
Q.19 Write the mechanism of acid catalysed dehydration of
(iii) Dilute nitric acid with phenol.
ethanol to ethene.
(iv) Treating phenol with chloroform in presence of Ans. Mechanism of Dehydration of Alcohols :
aqueous NaOH. Dehydration of monohydric alcohols is expressed by
4 Alk. KMnO
Ans. (i) CH 3CH 2CH 2OH + 2[O] ¾¾¾¾® following reaction.
Oxidation
Propan-1-ol C n H 2n+1OH ¾ Dehydrating
¾ ¾ ¾ ¾agent¾® C n H 2n + H 2O
¾
CH 3CH 2COOH + H 2O Conc. H 2SO 4 and conc. H 3 PO 4 are commonly used
Propanoic acid
dehydrating agents. Besides this vapours of alcohols
For (ii), (iii) and (iv) read chemical properties of on passing over heated alumina ( Al 2O 3 ) give
phenol.
127

alkenes. The reactivity of alcohols follow the order +H+ +


RCH2CH2CH2OH R—CH2—CH — CH2 —OH2
given below.
|
t-alcohol > s-alcohol > p-alcohol. H
Generally, p-alcohols follow E 2 mechanism
H H
(bimolecular elimination) while s- and t-alcohols
| + | +
chiefly follow E 1 mechanism. ¾¾® RCH — CH—CH3 + RCH2CHCH2
(A) Dehydration by E1 Mechanism : –H2O
In this mechanism of dehydration, rate of elimination Major (More stable) Minor (Less stable)
depends upon the concentration of substrate only –H+
and is independent to concentration of attacking
reagent, that is why they are designated as E 1 RCH == CH — CH3 RCH2CH == CH2
reactions. Major product Minor product
Rate = K [Alcohol]
Note: (1) In dehydration reactions, tertiary alcohols are most
Generally, s- and t-alcohols follow E 1 mechanism. reactive and they mainly give alkenes.
Example : Mechanism of a s-alcohol, (2) Dehydration of butanol-1 with conc. H 2SO 4 gives
RCH 2CH(OH)R by conc. H 2SO 4 is given below. mixture of trans-2-butene (56%), cis-2-butene (32%)
OH and butene-1 (12%). Formation of this mixture of
| different products take place due to rearrangement in
R —CH — CH 2 R ¾ Conc.¾ ¾H¾2SO
¾¾4 D ® R — CH == CH — R intermediate carbocation formed in acid catalysed
dehydration.
+ H 2O
(3) Dehydration of secondary and tertiary alcohols follows
This reaction is completed in following steps. Saytzeff's rule.
(i) Protonation of alcohol by H + : CH 3CHOHCH 2CH 3 ¾ ¾¾
-H 2 O
® CH 3CH == CHCH 3 +
H 2SO 4 ¾® 2H + + SO 24- Major product
+
OH OH 2
CH 2 == CHCH 2CH 3
| + D
| Minor product
R —CH — CH 2 R ¾ H¾¾ ® R —CH — CH 2 R According to Saytzeff's rule, in dehydration —OH group
(ii) Formation of carbocation by elimination eliminates along with the H-atom of carbon atom
having least number of H-atoms or more substituted
of H 2O:
+ alkene is the major product.
OH 2
Q.20 How are the following conversions carried out ?
| +
R —CH — CH 2 R ¾ -¾¾ ® R — CH — CH 2 R (i) Propene ¾® propan-2-ol
H O 2 Carbocation intermediate (ii) Benzyl chloride ¾® Benzyl alcohol
(iii) Formation of alkenes by elimination of (iii) Ethylmagnesium chloride ¾® Propan-1-ol
H+ : (iv) Methylmagnesium bromide ¾®
H 2-methylpropan-2-ol
+ | (i) Conc. H SO
H+ 2 4
Ans. (i) CH 3 ¾ CH == CH 2 ¾¾¾¾¾® CH 3CHOHCH 3
R — CH — CH — R ¾ -¾ ¾® R — CH == CH — R (ii)H 2O
(B) Dehydration by E 2 Mechanism : aq. NaOH
Generally, this mechanism is followed by p-alcohols. (ii) C 6 H 5CH 2Cl ¾¾¾® C 6 H 5CH 2OH + NaCl
+
RCH 2CH 2CH 2OH ¾ H¾
¾® RCH == CH — CH 3 (iii) CH 3CH 2 MgCl + CH 2O ¾® CH 3CH 2CH 2OMgCl
D Methanal
Major product
H 3O+
+ RCH 2CH == CH 2 ¾¾® CH 3CH 2CH 2OH
Minor product
(iv) CH 3 ¾ C ¾ CH 3 + CH 3 MgBr ¾®
This mechanism is known as E 2 because the rate of ||
elimination depends upon the concentration of both O
alcohol and attacking reagent. CH 3 CH 3
Rate = K [Alcohol] [ H + ] | H 3O + |
The mechanism of this reaction may be shown in the CH 3 ¾ C ¾ CH 3 ¾¾® CH 3 ¾ C ¾ CH 3
| |
following way. OMgBr OH
2-methyl -propan-2-ol
128

Q.21 Name the reagents in the following reactions : CH 3CH 2CH 2 ¾ O ¾ CH 2CH 2CH 3 + NaBr
(i) Oxidation of a primary alcohol to a carboxylic acid 1- propoxypropane

(ii) Oxidation of a primary alcohol to an aldehyde (ii) C 6 H 5O - Na + + C 2 H 5 Br ¾¾®


Heat

(iii) Bromination of phenol to 2, 4, 6-tribromophenol Sod. phenoxide


(iv) Benzyl alcohol to benzoic acid C 6 H 5OC 2 H 5 + NaBr
(v) Dehydration of propan-2-ol to propene Ethoxy benzene

(vi) Butan-2-one to butan-2-ol CH 3


Ans. (i) K 2Cr2O7 /H 2SO 4 or KMnO 4 /H 2SO 4 , |
Heat
(ii) Pyridinium chlorochromate(PCC), (iii) CH 3 ¾ C ¾ O – Na++CH 3 Br ¾¾®
+ |
C 5 H 5 NHCl - CrO 3 in CH 2Cl 2 , CH 3
Sod. 2-methyl -2-propoxide
(iii) Br2 /H 2O,
CH 3
(iv) KMnO 4 /H 2SO 4 ,
|
(v) conc. H 2SO 4 at 433-443 K, CH 3 ¾ C ¾ O¾ CH 3
(vi) NaBH 4 or LiAlH 4 or H 2 /Ni |
Q.22 Give reasons for the higher boiling point of ethanol in CH 3
comparison to methoxymethane. 2-methyl -2-methoxypropane

Ans. Boiling point of alcohols are higher than isomeric - + Heat


(iv) CH 3CH 2O Na + CH 3 ¾ Br ¾¾®
ethers. Sod. ethoxide Bromoethane
Q.23 Give the IUPAC names of the following ethers : CH 3CH 2 ¾ O ¾ CH 3 + NaBr
(i) CH 3OCH 2 ¾ CH ¾ CH 3 Methoxyethane

| Q.25 Illustrate with examples the limitations of


CH 3 Williamson’s synthesis for the preparation of certain
(ii) CH 3 ¾ O ¾ CH 2CH 2Cl type of ethers.
(iii) O 2 N ¾ C 6 H 4 ¾ OCH 3 (p) Ans. If t-alkyl halide is taken then alkene is major product.
(iv) CH 3 ¾ CH 2 ¾ CH 2 ¾ OCH 3 Q.26 How is 1-propoxypropane synthesised from
H3C CH3 propane-1-ol ?
Ans. It can be prepared by Williamson’s synthesis.
(v) (vi) (i) 3CH 3CH 2CH 2OH + PBr3 ¾®
Propan-1-ol

OC2H5 OC2H5 3CH 3CH 2CH 2 Br + H 3 PO 3


1-bromopropane
(vii) CH 3CH 2 ¾ O ¾ CH ¾ CH 2CH 3 (ii) 2CH 3CH 2CH 2OH + 2Na ¾®
| Propan-1-ol
CH 3 2CH 3CH 2CH 2O – Na + + H 2
Ans. (i) 1-methoxy-2-methylpropane, Sod. propoxide

(ii) 2-chloro-1-methoxyethane, (iii) CH 3CH 2CH 2O - Na + + CH 3CH 2CH 2 ¾ Br


(iii) 4-nitroanisole, Dry ether
¾¾¾® CH 3CH 2CH 2 ¾ O ¾ CH 2CH 2CH 3 + NaBr
(iv) 1-methoxypropane, Heat
1-propoxypropane
(v) 4-ethoxy-1, 1-dimethylcyclohexane,
It can also be prepared by the direct dehydration of
(vi) ethoxybenzene,
propan-1-ol.
(vii) 2-ethoxybutane. H 2SO4
Q.24 Write the names of reagents and equations for the 2CH 3CH 2CH 2OH ¾¾®
413 K
preparation of the following ethers by Williamson’s
CH 3CH 2CH 2 ¾ O ¾ CH 2CH 2CH 3 + H 2O
synthesis :
Q.27 Preparation of ethers by acid-catalysed dehydration
(i) 1-propoxypropane,
of secondary and tertiary alcohols is not a suitable
(ii) ethoxybenzene,
method. Give reason.
(iii) 2-methyl-2-methoxypropane,
Ans. Because is such reactions elimination product is
(iv) methoxyethane
major product.
Heat
Ans. (i) CH 3CH 2CH 2O - Na + + CH 3CH 2CH 2 ¾ Br ¾¾® Q.28 Write the equation for the reaction of hydrogen
Sodium propoxide 1- bromopropane
iodide with :
129

(i) 1-propoxypropane, (ii) methoxybenzene, OCH3 OCH3 OCH3


(iii) benzyl ethyl ether CH3
HI CH3Cl/AlCl3
Ans. (i) CH 3CH 2CH 2OCH 2CH 2CH 3 ¾® +
(i)
CH 3CH 2CH 2OH + CH 3CH 2CH 2 I Anisole Minor
HI
(ii) C 6 H 5OCH 3 ¾® C 6 H 5OH + CH 3 I CH3
Major
HI
(iii) C 6 H 5CH 2OC 2 H 5 ¾® C 6 H 5CH 2 I+C 2 H 5OH
OCH3 OCH3
Q.29 Explain the fact that in aryl ethers : (i) the alkoxy
NO2
group activates the benzene ring towards H2SO4/HNO3
electrophilic substitution and (ii) it directs the +
(ii)
incoming substituents to ortho- and para-positions in Minor
benzene ring. NO2
Ans. (i) Due to +M effect of ¾OR group. The ability of Major
benzene ring to attract E + increases.
OCH3 OCH3
(ii) This increased e - density on benzene ring due to
+M effect of ¾OR group appears at o/p positions, Br
Br2/AcOH
that is why E + is attached at o/p position. (iii)
+
+ +
OR OR OR Minor
Br
– Major
OCH3 OCH3

COCH3
CH3COCl/AlCl3
+d +
+OR OR (iv)
Minor
– –d –d COCH3
Major
–d Q.32 Show how would you synthesise the following
Q.30 Write mechanism of the reaction of HI with alcohols from appropriate alkenes?
methoxymethane.
Ans. CH3 OH
H (i) OH (ii)
+
CH3 O CH3 + H—I CH3 O CH3 + I– OH

(iii) (iv)
OH
CH3OH+CH3I Ans. (i)
Q.31 Write equations of the following reactions : CH3 CH2
–H2O CH3
(i) Friedel-Crafts reaction-alkylation in anisole, OH +
(ii) Nitration of anisole
(iii) Bromination of anisole in ethanoic acid medium, Methylene 1-methyl
cyclohexane cyclohexane
(iv) Friedel-Crafts acetylation of anisole.
Ans.
CH2 CH3
CH3 H2O/H+
or OH
130

OH –H2O Ans. (i)


(ii)
H+
CH 3 ¾ CH ¾ CH ¾ CH 3 ¾® CH 3 ¾ CH ¾ CH ¾ CH 3
4-methyl heptan-4-ol | | | |
CH 3 OH CH 3 + OH 2
+ –H O +
2
4-methyl hept-3-ene 2-propyl pent-1-ene
¾¾® CH 3 ¾ CH ¾ CH ¾ CH 3
|
OH CH 3
–H2O 2º-carbocation
(iii) +
(ii)
Pentan-2-ol Pent-1-ene Pent-2-ene + +
CH 3 ¾ CH ¾ CH ¾ CH 3 ¾® CH 3 ¾ C ¾ CH 2 ¾ CH 3
| |
–H2O
(iv) OH CH 3 CH 3
3º carbocation
2-methyl cyclohexyl 2-methyl cyclohexyl
butan-2-ol but-1-ene Br –

Br
+ |
CH 3 ¾ C ¾ CH 2 ¾ CH 3
2-methyl cyclohexyl |
but-2-ene CH 3
Q.33 When 3-methylbutan-2-ol is treated with HBr, 2-bromo-2-methyl butane

following reaction takes place :


Br
|
HBr
CH 3 ¾ CH ¾ CH ¾ CH 3 ¾¾® CH 3 ¾ C ¾ CH 2 ¾ CH 3
| | |
CH 3 OH CH 3
vvv
Give a mechanism for this reaction.
131

Chapter

12
ALDEHYDES, KETONES
AND CARBOXYLIC
ACIDS
Syllabus : Aldehyde and ketones: Nomenclature, nature of carbonyl group, methods of preparation, physical and chemical
properties, mechanism of nucleophilic addition reactions, reactivity of a-H in aldehydes, uses.
Carboxylic acids: Nomenclature, acidic nature, methods of preparation, ph ysical and chemical properties, uses.

Objective Questions
1. Cannizzaro reaction is not given by: (UP 2009) 6. Product obtained when acetic acid reacts with
(a) CH 3CHO (b) (CH 3 ) 3 C × CHO hydrazoic acid/conc. H 2SO 4 at 0°C is : (U.P. 2017)
(c) C 6 H 5CHO (d) HCHO (a) methane (b) methyl amine
2. Which of the following reagent reacts with both (c) methyl cyanide (d) ethyl amine
aldehydes (acetaldehyde) and ketones (acetone) 7. Compound obtained when acetic acid reacts with
readily ? (U.P. 2013) diazomethane is : (U.P. 2017)
(a) Fehling’s solution (b) Grignard reagent (a) methyl acetate (b) ethyl acetate
(c) methane (d) methyl amine
(c) Schiff’s reagent (d) Tollen’s reagent
8. On oxidation of toluene by KMnO 4 gives: (U.P. 2013)
3. Which of the following does not reduce Fehling’s
(a) benzyl alcohol (b) benzaldehyde
solution ? (U.P. 2010, 17, 18)
(c) benzoic acid (d) none of these
(a) HCOOH (b) CH 3COOH 9. Formalin is aqueous solution of : (U.P. 2018)
(c) HCHO (d) CH 3CHO (a) formaldehyde (b) formic acid
4. Which of the following in strongest acid? (c) fluorscein (d) acetaldehyde
(U.P. 2010, 18) 10. Which does not form iodoform on heating with I 2 and
(a) CH 3COOH (b) ClCH 2COOH base ? (U.P. 2019)
(a) Acetone (b) Ethanol
(c) Cl 2CHCOOH (d) Cl 3C ¾COOH
(c) Methanol (d) Acetaldehyde
5. Iodoform test is not given by : (U.P. 2014, 17) 11. Rosenmund reduction gives : (U.P. 2020)
(a) ethanol (b) ethanal (a) Aldehyde (b) Ether
(c) benzophenone (d) acetophenone (c) Carboxylic acid (d) Hydrocarbon

Answers
1. (a) 2. (b) 3. (b) 4. (d) 5. (c) 6. (b) 7. (a) 8. (c) 9. (a) 10. (c)
11. (a)

Very Short Answer Type Questions


Q.1 What is Hell-Volhard-Zelinsky (HVZ) reaction ? Q.3 Carboxylic acids are stronger acids than alcohols,
(U.P. 2017) why? (U.P. 2015)
Ans. Chlorine or bromine, in presence of red phosphorous, Ans. Carboxylate ion obtained from release of proton is
reacts with carboxylic acid to form a-halo acid. This more stable than alkoxide ion because of resonance.
reaction is called as Hell Volhard- Zelinsky reaction. Therefore, carboxylic acids release proton more
2 Cl Red P 2 Cl Red P readily than alcohols.
CH 3COOH ¾¾¾® ClCH 2COOH ¾ ¾ ¾® CCl 3COOH
¾¾
—2HCl
—HCl Q.4 Identify (A) and (B) in the following reaction:
Q.2 100% pure acetic acid is known as glacial acetic acid, P2O5 2 H O
CONH2 ¾ ¾¾®( A ) ¾ ¾¾ ®( B)
why ? (U.P. 2013, 15) +
H
Ans. When 100% pure acetic acid is cooled, it crystallises (U.P. 2014, 16)
out in icy form, hence, referred to as glacial acetic
acid. Ans. ( A ): CN ; ( B): C 6 H 5COOH
132

Q.5 Arrange o, m and p-nitrobenzoic acid in decreasing Q.6 What is formalin? Write its uses.(U.P. 2005,11, 18)
order of acidic strength. (U.P. 2017) Ans. The 40% aqueous solution of formaldehyde is known
Ans. o-nitrobenzoic acid > p-nitrobenzoic acid > as formalin.
m-nitrobenzoic acid. Uses : It is used for the preservation of biological
specimens.

Short Answer Type Questions


Q.1 How will you differentiate between following with (b) Difference between acetaldehyde and acetone
the help of their analytical tests ?
(a) Formaldehyde and acetaldehyde S. Property Acetaldehyde Acetone
(U.P. 2010, 15, 18) No.
(b) Acetaldehyde and acetone 1. With Na + C 2H5OH Ethyl alcohol is Isopropyl
(c) Ethyl alcohol and acetone. (U.P. 2013) formed alcohol is
(d) Polymerisation and condensation. (U.P. 2015) formed
(e) Formic acid and acetic acid (U.P. 2009)
2. Oxidation with Acetic acid is No oxidation
Ans. (a) Difference between formaldehyde and acidified formed at at ordinary
acetaldehyde. K 2Cr2O7 + H2SO 4 (dil. ) ordinary temperature.
temperature
S.No Test Formaldehyde Acetaldehyde
3. Action with Tollen's Silver mirror is No reaction.
1. Reaction w i t h Forms urotropine F o r m s ad d i ti on reagent formed
ammonia compound
3. Action with Fehling Red precipitate of No reaction.
2. R e a c t i o n w i t h Forms primary Forms secondary solution cuprous oxide is
Grignard reagent alcohol alcohol obtained.
3. Heat with I2 and No reaction Yellow crystals of 5. Action with Grignard Isopropyl alcohol Tert. butyl
NaOH Iodoform is formed. reagent and hydrolysis is obtained. alcohol is
formed.

(c) Difference between ethyl alcohol and acetone

S.No. Property Ethyl alcohol Acetone


1. Action with alkaline solution of sodium No reaction A red colour is obtained which
nitro-prusside changes to yellow on standing.
2. Action with acidified K 2Cr2O7 + dil. H2SO 4 Acetaldehyde having No reaction
characteristic odour is obtained.
3. Action with acetic acid in presence of conc. Ethyl acetate having fruity odour No reaction
H2SO 4 is formed.
4. Action with NH3 1° , 2° and 3° amines are formed. Diacetone amine and triacetone
amine are formed.

(d) Difference between polymerisation and condensation

S.No. Polymerisation Condensation


1. Generally, similar types of molecules take part. Similar or different types of molecules take part.
2. Molecular mass of polymer is an integral multiple of Molecular mass of condensation product may or may not
molecular mass of monomer. be an integral multiple of molecular mass of reacting
molecules. (e. g. , aldol aceto-oxime)
3. Polymerisation is usually reversible and polymer can be This is an irreversible reaction.
changed back to monomer.
4. New C—C or C—N bonds are not formed. New C—C or C—N bonds are formed.
5. Simple molecules like water molecule are not eliminated. Simple molecule may or may not be eliminated.
133

(e) Difference between formic acid and acetic acid

S.No. Test Formic acid Acetic acid


1. Neutral solution of acid is heated Red precipitate of cuprous oxide is formed. No reaction
with Fehling solution.
2. Neutral solution of acid heated Silver mirror or black precipitate of silver is No reaction
with Tollen's reagent (ammoniacal obtained.
solution of silver nitrate)
3. Mercuric chloride is added in White precipitate of mercurous chloride is No reaction
solution of acid. obtained which is reduced to grey coloured
precipitate of mercury on standing.
4. On heating with conc. H2SO 4. Carbon monoxide gas is evolved which burns No reaction
with blue flame.
5. Neutral solution of acid is treated Solution turns greenish blue. No reaction.
with sodium bisulphite and sodium
nitroprusside solution.

Q.2 How will you obtain benzoic acid from benzene by 2 4 Conc. H SO
(i) CH 3COOH + N 3 H ¾¾¾¾® CH 3 NH 2 + N 2 + CO 2
using Friedal-Craft’s reaction? Write its two Acetic acid
60ºC
Methylamine
properties also. (U.P. 2017) COOH NH2
CH3 COOH
Conc. H2SO4
+N3H ¾¾¾¾® +N2+CO2
AlCl3 [O]/K2Cr2O7/H + (ii) 60ºC
Ans. +CH3Cl Benzoic acid Aniline
(ii) Aldol condensation : When two molecules of
(i) It is acidic in nature and reacts with NaOH to give same or different carbonyl compounds containing
sodium benzoate. a-hydrogen combine together in the presence of a
COOH COONa dilute base such as NaOH, Na 2CO 3 , Ba(OH 2 ) etc. to
form compounds called aldols and reaction is called
aldol condensation. e. g .,
+ NaOH + H2O
CH 3CHO + H × CH 2CHO ¾ dil ¾NaOH
¾¾® CH 3 — CH — CH 2CHO
Acetaldehyde |
(ii) Reaction with PCl 5
OH
COOH COCl Aldol
(iii) Cannizzaro reaction : When two molecules
of aldehyde do not contain a-hydrogen are treated
+ PCl
5
+HCl + POCl
3 with conc. NaOH solution, one molecule is reduced
Benzoyl chloride alcohol while the other molecule is oxidised to sodium
Q.3 What is Rosenmund reduction? salt of carboxylic acid. This reaction is called
(U.P. 2010,14,16,18, 19) cannizzaro reaction.
Ans. Acyl chlorides on reduction with hydrogen in For example :
presence of palladium (Pb) and barium sulphate 2HCHO + NaOH ¾® CH 3OH + HCOONa
Formaldehyde Methyl Sod. formate
gives aldehydes and reaction is called Rosenmund alcohol
reduction.
RCOCl + H 2 ¾ Pd,
¾ BaSO
¾¾¾4
® RCHO + HCl 2C 6 H 5CHO + NaOH ¾® C 6 H 5CH 2OH+ C 6 H 5COONa
Acyl chloride Aldehyde Benzaldehyde Benzyl alcohol

Q.4 Write short notes on the following : Etard Reaction : First, chromyl chloride is dissolved
(i) Schmidt reaction (U.P. 2013, 15, 18, 19) in CCl 4 or CS 2 and the solution is added to toluene,
(ii) Aldol condensation (U.P. 2014, 15, 16, 20) when a brown coloured complex is formed which on
(iii) Cannizzaro reaction (U.P. 2014, 16, 19, 20) hydrolysis converts into benzaldehyde.
(iv) Etard Reaction (U.P. 2019) C 6 H 5CH 3 + 2CrO 2Cl 2 ¾ CCl
¾¾ 4
® C 6 H 5CH 3 × 2CrO 2Cl 2
Ans. (i) Schmidt reaction : When carboxylic acid is Complex

dissolved in conc. H 2SO 4 and treated with hydrazoic


acid solution in CHCl 3 at 60º C primary amine is ¾ 4¾¾
H 2O
® C 6 H 5CHO + 2H 2CrO 4 + 4HCl
formed and reaction is called Schmidt reaction.
134

Q.5 Write short notes on the following : Ans. (i) Methods of preparation of benzaldehyde :
(i) Urotropine (U.P. 2014, 18) 1. By Gattermann-kotch synthesis : When
(ii) Silverisation (U.P. 2014) mixture of CO and HCl is passed through a solution of
Ans. (i) Urotropine : Hexamethylene tetramine is benzene in ether in the presence of anhydrous AlCl 3 ,
known as urotropine. It is a solid compound and benzaldehyde is formed.
prepared by reaction of HCHO with conc. NH 3 . CHO
6HCHO + 4NH 3 ¾® (CH 2 ) 6 N 4 + 6H 2O
Urotropine Anhyd. AlCl 3
+CO+HCl ¾¾¾¾® +HCl
It is used as a medicine in the treatment of gout and
diseases of urinary tract. Benzene Benzaldehyde

(ii) Silverisation : On warming with Tollen’s 2. Laboratory method : Benzyl chloride when
reagent, aldehydes form silver mirror. Tollen’s boiled with aqueous solution of copper nitrate in the
reagent is reduced to silver and reaction is called current of CO 2 , forms benzaldehyde.
CO2
silverisation. 2C 6 H 5Cl+ Cu(NO 3 ) 2 ¾ ¾¾®
D
RCHO + Ag 2O ¾® RCOOH + 2Ag Benzyl
chloride
Aldehyde Tollen’s Silver
reagent mirror 2C 6 H 5CHO+ CuCl 2 + 2HNO 3
Q.6 Write short notes on following : Benzaldehyde
(i) Condensation (U.P. 2015)
Reactions
(ii) Polymerisation (U.P. 2010,15)
(i) Reaction with alcoholic KCN : When
Ans. (i) Condensation : Condensation is the process in
benzaldehyde is heated with alcoholic KCN, two
which two or more molecules of aldehydes or ketones
molecules of benzaldehyde condense to form benzoin
combine with each other forming a new
carbon-carbon bond with the elimination or may not and reaction is called benzoin condensation.
be elimination of simple molecules like, H 2O, NH 3 , C 6 H 5CHO + H — C — C 6 H 5 ¾®
||
HCl etc.
O
(ii) Polymerisation : Combination of two or more
molecules of aldehydes, without the elimination of a C 6 H 5 — CH(OH) — C — C 6 H 5
||
small molecules like H 2O, NH 3 , HCl etc. resulting in
the formation of a new compound is known as O
Benzoin
polymerisation. (ii) Reaction with Tollen’s reagent : It reduces
For example : Tollen’s reagent into silver mirror.
(i) When acetaldehyde is treated with conc. H 2SO 4 at C 6 H 5CHO+ Ag 2O ¾® C 6 H 5COOH+ 2Ag
0º C, metaldehyde is obtained. Benzaldehyde Benzoic Silver
Conc .H 2SO4 acid mirror
3CH 3CHO ¾ ¾ ¾ ¾¾ ®(CH 3CHO) 3
Q.9 Write two chemical equations for the preparation of
(ii)When formaldehyde gas is allowed to stand at benzoic acid. What happens when benzoic acid react
room temperature metaformaldehyde is obtained. with (i) PCl 5 and (ii) NH 3?
Room temp.
3HCHO (HCHO)3 Ans. Methods of preparation of benzoic acid:
1. Laboratory method : By the hydrolysis of
Q.7 Distinguish between the following :
benzoyl chloride obtained by Friedel-Craft’s reaction
(i) Aldehyde and ketone (U.P. 2014)
between benzene and carbonyl chloride, benzoic acid
(ii) Benzaldehyde and acetophenone (U.P. 2014)
is obtained.
(Give one chemical test only) COCl COOH
Ans. (i) Aldehyde reduces Fehling’s solution into red
precipitate of Cu 2O while ketone does not reduces AlCl3 2 H O
Fehling solution. +COCl2 ¾¾¾®
–HCl
¾¾¾®
–HCl
RCHO + 2CuO ¾® RCOOH + Cu 2O Benzene Benzoyl chloride
Red ppt
2. By hydrolysis of Phenyl cyanide : On
(ii) Benzaldehyde undergoes Cannizzaro reaction hydrolysis of phenyl cyanide by mineral acid, benzoic
while acetophenone does not undergo Cannizzaro acid is formed.
reaction. CN COOH
2C 6 H 5CHO + NaOH ¾®
C 6 H 5CH 2OH + C 6 H 5COONa dil. H2SO4
+2H2O ¾¾¾¾® +NH3
Q.8 Write chemical equations of any two methods for the
preparation of benzaldehyde. What happens when Phenyl Benzoic acid
benzaldehyde reacts with (i) alcoholic KCN and (ii) cyanide

Tollen’s reagent? (U.P. 2014)


135

Reactions : (ii) Benzoin condensation: Benzoin is obtained


(i) Reactions with PCl 5 : Benzoyl chloride is when benzaldehyde is boiled with alcoholic solution
formed. of KCN for two hours.
COOH COCl 2C 6 H 5CHO ¾ KCN,
¾ ¾alcohol
¾¾® C 6 H 5CH(OH)COC 6 H 5
D Benzoin
Q.11 Write chemical equation for the formation of
+PCl5 ¾¾¾® +POCl3+HCl aldehyde by Stephen's reaction. (U.P. 2020)
Benzoic Benzoyl Ans. Formation of aldehyde by Stephen’s
acid chloride reaction: When etheral solution of an alkyl cyanide
(ii) Reaction with ammonia : Ammonium is reduced with SnCl 2 /HCl followed by the steum
benzoate is obtained which on heating converts into distillation of product obtain, aldehydes are formed.
benzamide. In first step, stannous chloride react with conc. HCl to
COOH COONH4 CONH2 form nasant hydrogen which adds on alkyl cyanide to
give iminochloride. In second step, iminochloride
D
+NH3 ¾¾® ¾¾® undergo hydrolysis to give aldehyde.
–H O2
SnCl 2 + 2HCl ¾® SnCl 4 + 2[ H]
Benzoic Amm. Benzamide
acid benzoate
R — C ºº N + 2[ H] ¾® RCH == NH
Aldimine
Q.10 Write short note on the following : (U.P. 2018)
(i) Perkin’s reaction R — CH == NH + HCl ¾® RCH == NH × HCl
Aldimine hydrochloride
(ii) Benzoin condensation
Ans. (i) Perkin’s reaction: On heating with sodium salt R — CH == NH × HCl + H 2O ¾® RCHO + NH 4 Cl
of carboxylic acid containing a - hydrogen and its Q.12 What is Tollen's reagent? Write chemical equation of
anhydride, benzaldehyde forms a, b - unsaturated one example of Tollen's test. (U.P. 2020)
acid. Ans. Tollen’s reagent: Tollens reagent refers to the
For example : When heated with sodium acetate chemical reagent which is used in the detection of an
and acetic anhydride, benzaldehyde forms cinnamic aldehyde functional group, an aromatic aldehyde
acid. functional group or an alfa hydroxy ketone functional
group in a given test substance.
C 6 H 5CH O+H2 CH ×COOCOCH 3 ¾ CH COONa, D
¾3¾ ¾¾ ¾®
- H 2O Tollen’s reagent is a solution of AgNO 3 and NH 3 .
C 6 H 5CH == CHCOOCOCH 3 ¾ -¾ ¾ ¾¾® Example: When acetaldehyde is heated with
CH 3COOH
Tollen’s reagent, it form silver mirror.
C 6 H 5CH == CHCOOH
Cinnamic acid CH 3CHO + Ag 2O ¾® RCOOH + 2Ag
Silver mirror

Long Answer Type Questions


Q.1 Write chemical equations of any two methods for the (b) By Etard reaction: First, chromyl chlorideis
preparation of benzaldehyde. What happens when it dissolved in CCl 4 or CS 2 and the solution is added to
reacts with Grignard reagent, Tollens’ reagent, conc. toluene, when a brown coloured complex is formed
H 2SO 4 and conc. caustic soda solution. (U.P. 2017) which on hydrolysis converts into benzaldehyde.
Ans. (a) By Gattermann’s aldehyde synthesis: In C 6 H 5CH 3 + 2CrO 2Cl 2 ¾ CCl
¾¾ 4
® C 6 H 5CH 3 × 2CrO 2Cl 2
the presence of ZnCl 2 or AlCl 3 , when benzene is Complex
treated with HCN/HCl, and product formed is
hydrolysed benzaldehyde is obtained. ¾ 4¾¾
H 2O
® C 6 H 5CHO + 2H 2CrO 4 + 4HCl
This reaction can also be carried out with Zn(CN) 2 (c) By Rosenmund’s reaction: When benzoyl
and HCl in place of ZnCl 2 and HCN. chloride is reduced with H 2 in presence of
C 6 H 6 ¾(i)HCN
¾ ¾ +¾HCl,AlCl 3
¾¾ ¾® C 6 H 5CHO Pd / BaSO 4 catalyst, benzaldehyde is obtained.
(ii)HOH
C 6 H 5COCl + H 2 ¾ Pd/BaSO
¾¾¾ ¾ 4
® C 6 H 5CHO + HCl
Benzoyl chloride
Mechanism:
+ In this reaction, BaSO4 acts as catalytic poison and
H — C ºº N + HCl ¾ AlCl
¾¾ 3
® H — C == NH + AlCl –4
CH==NH2 CHO benzaldehyde is not further reduced to benzyl
+ H2O, AlCl4–
alcohol.
H
+C (i) Reaction with Grignard reagent : Initially
NH – NH3 ,
–AlCl3, an addition product is formed which on hydrolysis
–HCl Benzaldehyde gives secondary alcohol.
136
OMgI R H O+
C 6 H 5CHO + CH 3 MgI ¾® C6H5CH CH
3
(i) H — C ºº N + RMgI ¾® C==NMgI ¾¾®
Grignard 3 Adduct H
reagent + Addition product
HOH/H
OH I
C6H5CH + Mg R OH
CH3 OH C==O+Mg
1-phenyl alcohol H I
Aldehyde
(1° alcohol)
R¢ H3O+
(ii) Oxidation by Tollen’s reagent : Tollen’s (ii) R ¢ — CN + RMgI ¾® C==NMgI ¾¾¾®
hydrolysis
reagent (ammoniacal AgNO 3 ) oxidises benzaldehyde Alkyl cyanide R
Addition product
to benzoic acid and a silver mirror is formed.
C 6 H 5CHO + Ag 2O ¾® C 6 H 5COOH + 2Ag ¯ R¢ OH
Silver mirror C==O+Mg
(Tollen’s reagent: AgNO 3 + NH 4 OH) R I
Ketone
(iii) Cannizaro’s reaction : This reaction is given Reaction of acetone with conc. H 2SO 4 : When
by those aldehydes only which do not contain a - acetone is heated with conc. H 2SO 4 , mesitylene is
hydrogen. In this reaction, two molecules of obtained.
benzaldehyde react with concentrated solution of H3C CH3
Conc. H2SO4
NaOH. One molecule is oxidised to sodium benzoate 3CH3COCH3 ¾¾¾¾® +3H2O
D
while other molecule is reduced to benzyl alcohol. Acetone
2C 6 H 5CHO + NaOH ¾® C 6 H 5CH 2OH+ C 6 H 5COONa
Benzyl alcohol Sodium
CH3
benzoate Mesitylene
(iv) Sulphonation: On heating with fuming Uses of acetone :
sulphuric acid, m – formyl benzene sulphonic acid is (i) It is used in preparation of CHCl 3 in laboratory.
formed.
CHO CHO (ii) It is also used in the preparation of chloretone.
Conc. H2SO4
Reaction of acetaldehyde with dil. NaOH :
Aldol is obtained.
Heat SO3H dil . NaOH
CH 3CHO + HCH 2CHO ¾ ¾ ¾ ¾® CH 3CH(OH) × CH 2CHO
m-formyl benzene sulphonic acid Aldol
Q.2 Write equation of the reactions for a general method Reaction with Tollen’s reagent : When
of preparation of aldehyde and ketone. How does acetaldehyde is heated with Tollen’s reagent, it form
acetone react with concentrated sulphuric acid? Write silver mirror.
also two uses of acetone. (U.P. 2014) CH 3CHO + Ag 2O ¾® RCOOH + 2Ag
Or Write chemical equation of any two methods of Silver mirror
preparation for aldehydes. How does acetaldehyde Q.3 Describe laboratory method for the preparation of
react with dilute NaOH and Tollen’s reagent? Write formaldehyde. What happens when formaldehyde is
related chemical equations. (U.P. 2015) heated with caustic soda? (U.P. 2010,13,16, 17)
Ans. General methods of preparation of aldehyde Or Describe the method for the preparation of
and ketone : formaldehyde in laboratory. Explain its reactions
1. By oxidation of alcohols : On oxidation with taking place with
K 2Cr2O7 and H 2SO 4 primary alcohols form aldehyde. (i) concentrated NaOH solution and
2 2 7 2K Cr O + H SO
4 (ii) ammonia
RCH 2OH ¾ ¾ ¾¾¾ ¾ ® RCHO + H 2O
Primary [O] Aldehyde with chemical equations. (U.P. 2016, 18)
alcohol Ans. Laboratory method of preparation of
Similarly on oxidations, secondary alcohols form formaldehyde :
ketones. Principle : Formaldehyde is prepared in the
R K2Cr2O7+H2SO4 R laboratory by oxidation of methyl alcohol by oxygen
CHOH ¾¾¾¾¾® C==O+H 2O
R [O]
R Ketone
of the air in the presence of platinised asbestos at
Sec. alcohol 200-250ºC.
2. From Grignard reagent : Grignard reagents 2CH 3OH + O 2 ¾ Pt
¾asbestos
¾¾ ¾® 2HCHO + H 2O
200-250ºC
react with hydrogen cyanide followed by hydrolysis Methyl Formaldehyde
alcohol
give aldehyde while with alkyl cyanide ketone is
formed.
137

Method and labelled diagram : CH 3CH 2OH ¾ Cu


¾® CH 3CHO + H 2
Ethyl alcohol 300ºC Acetaldehyde
2. Laboratory method : In the laboratory
acetaldehyde is prepared by oxidation of ethyl alcohol
with K 2Cr2O7 and dilute H 2SO 4 .
K 2Cr2O7 + 4H 2SO 4 ¾® K 2SO 4 + Cr2 (SO 4 ) 3
+ 4H 2O + 3[O]
CH 3CH 2OH ¾[O]
¾® CH 3CHO + H 2O
Ethyl alcohol Acetaldehyde
or
K 2Cr2O7 + H 2SO4
CH 3CH 2OH ¾ ¾ ¾ ¾ ¾ ¾® CH 3CHO + H 2O
Ethyl alcohol [O] Acetaldehyde

Laboratory method for the preparation of formaldehyde


Reactions :
(i) Reaction with caustic soda (NaOH) :
Formaldehyde reacts with NaOH gives methyl alcohol
and sodium formate.
2HCHO + NaOH ¾® CH 3OH+ HCOONa
Formaldehyde Methyl Sod. formate
alcohol
(ii) Reaction with ammonia : Formaldehyde
reacts with ammonia to form hexam-
ethylenetetramine. Which is also known as
urotropine.
Q.4 Write chemical equations of oxidation and catalytic Laboratory method of preparation of acetaldehyde
dehydrogenation methods for preparation of
acetaldehyde. Also write the equation of condensation Reactions
reaction of acetone. (U.P. 2014, 17) 1. Reaction with dil. NaOH : 3-hydroxybutanal
Or Write any two chemical reactions for the (aldol) is obtained.
preparation of acetaldehyde, give chemical equations H
|
also. How does it react with dil. NaOH and Tollen’s
CH 3CHO + H × CH 2CHO ¾ dil ¾. NaOH
¾ ¾® CH 3 — C — CH 2CHO
reagent? (U.P. 2015) Acetaldehyde |
Or Give laboratory method of preparation of pure OH
acetaldehyde. How does it react hydroxylamine and Aldol
sodium bisulphite? Write chemical equations. 2. Reaction with Tollen’s reagent : With Tollen’s
(U.P. 2015) reagent acetaldehyde gives silver mirror.
Or Write the chemical equation for the method to CH 3CHO + Ag 2O ¾® CH 3COOH + 2Ag
prepare acetaldehyde in the laboratory. What is its Acetaldehyde Tollen’s Silver
reagent mirror
reaction with Tollen’s reagnet? Give equation also.
3. Reaction of hydroxylamine : Acetaldoxime is
(U.P. 2018)
obtained.
Or Describe with diagram the laboratory method of
preparation of pure acetaldehyde. Give also the CH3 CH3
C== O+H2 NOH ¾® C==NOH+H2O
chemical equations involved. Mention its one H H Acetaldoxime
polymerisation reaction. (U.P. 2017)
Or Write two general method of preparation of 4. Reaction with sodium bisulphite :
aldehydes. (U.P. 2020) Acetaldehyde sodium bisulphite is formed.
Ans. Methods of preparation of acetaldehyde CH3 CH3 OH
C==O+NaHSO3 ¾¾® C
1. By catalytic dehydrogenation of ethyl H H SO3Na
alcohol : When vapours of ethyl alcohol is passed
over heat copper catalyst at 300ºC, acetaldehyde is
obtained.
138

Q.5 Write chemical equations of any two methods for the 2. Reaction with chloroform (CHCl 3 ) :
preparation of acetone. Write also equations of its Chloretone is obtained.
reaction with. CH3 CH3 OH
(i) hydrazine C==O+CHCl3 ¾¾® C
CH3 CH3 CCl3
(ii) aromatic aldehyde in presence of dilute alkali. Acetone Chloretone
(U.P. 2016) 3. Reaction with hydrazine : Acetone hydrazone
Or Describe the laboratory method for the preparation is formed.
of acetone and write its reactions with the following : CH3 CH3
(i) Concentrated H 2SO 4 C== O+H 2 N.NH2 ¾¾® C==N.NH2 –H2O
(ii) CHCl 3 CH3 CH3
Acetone hydrazone
(iii) hydroxylamine
Also give necessary chemical equation of the related 4. Reaction with hydroxylamine : Acetoxime is
reactions. (U.P. 2016) formed.
Or Describe laboratory method for the preparation of CH3 CH3
acetone with chemical equations. How does it react C== O+H 2 NOH ¾¾® C==NOH+H2O
CH3 CH3
with hydroxylamine and dil. (BaOH) 2 solution? Write Acetoxime
chemical equations also. (U.P. 2018)
5. Reaction with aromatic aldehyde : Acetone
Or Describe with labelled diagram, the laboratory condense with aromatic aldehyde C 6 H 5CHO to form
method of preparation of acetone. Write the equation 4-phenyl-3-buten-2-one.
of its reaction with the following : (U.P. 2019) -

(i) CHCl 3 , (ii) CaOCl 2 , (iii) PCl 5 C 6 H 5CHO+ CH 3COCH 3 ¾ OH


¾¾®
Benzaldehyde Acetone 100ºC
Ans. Methods of preparation of acetone
C 6 H 5 — CH == CH — COCH 3
1. Laboratory method : In laboratory, acetone is
(6) Reaction with PCl 5 :
prepared by dry distillation of anhydrous calcium
acetate CH3 CH3 Cl
D
C == O + PCl 5 ¾® C + POCl 3
(CH 3COO) 2 Ca ¾¾ ® CH 3COCH 3 + CaCO 3 CH3 CH3 Cl
Calcium acetate Acetone Calcium 2,2-dichloropropane
carbonate
(7) Reaction with bleaching powder :
CaOCl 2 + H 2O ¾® Cl 2 + Ca(OH) 2

2 Ca(OH)
CH 3COCH 3 + 3Cl 2 ¾¾¾® CCl 3COCH 3 + 3HCl
Trichloro acetone

OH CCl COCH hydrolysis


3 3 ¾¾¾¾® 2CHCl 3
Ca +
OH CCl 3COCH 3
+ (CH 3COO) 2 Ca
Q.6 Draw the labelled diagram of the laboratory method
for preparation of formic acid. How does it react with
Tollen’s reagent. Give equations of all reactions.
Or Describe with diagram the laboratory method of
Laboratory method of preparation of Acetone preparation of formic acid. Write down chemical
Reactions : equation of its reaction with Fehling solution.
1. Reaction with conc. H 2SO 4 : When acetone is (U.P. 2018)
heated with conc. H 2SO 4 mesitylene is obtained. Ans. Laboratory method for preparation of formic
acid :
H3C CH3
Conc.H2SO4 Principle : Formic acid is prepared in the laboratory
3CH3COCH3 ¾¾¾¾®
–3H O
Acetone
2 by heating glycerol with oxalic acid at 100 – 110º C.
Reaction proceeds in following steps.
CH3
Mesitylene
139

CH2 OH CH2. OOC .COOH Q.7 Describe with diagram the quick vinegar process for
COO H the preparation of acetic acid and write the equation
CHOH+
100ºC
¾¾® CHOH
D
¾¾® of its reaction with ethyl alcohol. (U.P. 2016)
–H O –CO
COOH
2 2 Ans. Manufacture of acetic acid by quick vinegar
CH2OH CH2OH process :
Glycerol monoxalate Principle : Dilute aqueous solution of ethyl alcohol
(8-10%) is oxidised by air in the presence of
CH2 .COOH CH2OH mycoderma aceti bacteria to acetic acid. This process
110ºC
is known as fermentation.
CHOH+HOH ¾¾® CHOH+HCOOH
Formic acid
C 2 H 5OH+ O 2 ¾ Mycoderma
¾ ¾ ¾ ¾¾ aceti
® CH 3COOH+ H 2O
Ethyl Acetic
CH2OH CH2OH alcohol acid
Glycerol mono Glycerol Procedure : In the quick vinegar process, a wooden
formate
vat is fitted with two wooden plates having holes as
shown in figure. Space between these plates is filled
beach wood shavings moistened with old vinegar
solution which is main source of mycoderma aceti
enzyme.

Laboratory method of preparation of formic acid


Reducing properties : Due to presence of —CHO
group in formic acid, it acts as a strong reducing
Quick vinegar process
agent.
O 10% aqueous solution of ethyl alcohol is dropped
slowly from the top of the vat and air is passed
H—C—OH through the holes near the bottom of the vat. Ethyl
alcohol is oxidised to acetic acid. An 8-10% aqueous
For example :
solution of acetic acid is obtained which is known as
(i) Reduction of Tollen’s reagent : Formic acid vinegar.
reduces Tollen’s reagent in silver mirror.
Reaction with ethyl alcohol : When acetic acid is
HCOOH+ Ag 2O ¾® 2Ag + CO 2 + H 2O heated with ethyl alcohol in the presence of conc.
Formic Silver
acid mirror H 2SO 4 , ethyl acetate is obtained and reaction is called
(ii) Reduction of Fehling solution : Formic acid esterification.
reduces Fehling solution into cuprous oxide (red ppt.). 2Conc . H SO
4
CH3COO H+HO C2H5 ¾¾¾¾®
140ºC CH3COOC2H5+H2O
HCOOH + 2Cu(OH) 2 ¾® Cu 2O + CO 2 + 3H 2O Ethyl acetate
Red ppt.

Question Based on Chemical Reactions


Q.1 How will you obtain following ? Give chemical 3O / H O/H+
2
equations only. Ans. (a) CH 2 == CH 2 ¾¾¾¾® 2HCHO
(a) Formaldehyde from ethylene (U.P. 2014) (b) CH 3COCH 3 + 3Cl 2 ¾® CCl 3 × COCH 3 + 3HCl
(b)Chloroform from acetone. (U.P. 2000, 15) Trichloroacetone

(c) Acetone from acetaldehyde (U.P. 2015, 18) 2CCl 3COCH 3 + Ca(OH) 2 ¾® 2CHCl 3
(d)Formaldehyde from acetaldehyde (U.P. 2011) + (CH 3COO) 2 Ca
(e) Bakelite from formaldehyde (U.P. 2017)
140

(c) CH 3CHO ¾¾¾¾¾®


[O]
CH 3COOH ¾¾¾®
Ca(OH)2 (d) CH 3CHO+ H 2 N × NHCONH 2 ¾¾¾®
+ – H 2O
Na2Cr2O7 / H
CH 3CH== N × NHCONH 2
D
(CH 3COO) 2 Ca ¾® CH 3COCH 3 OH
[O] Sodalime
½
(d) CH 3CHO ¾¾¾¾®
+
CH 3COOH ¾¾¾® (e) CH 3COCH 3 + HCN ¾¾® CH 3 ¾ C ¾ CH 3
K 2Cr2O7 / H
½
2 O / Cu [O] CN
CH 4 ¾¾® CH 3OH ¾¾® HCHO
OH OH
H 2O/H+
½
¾¾¾® CH 3 ¾ C ¾ CH 3
dil. NaOH
(e) nHCHO + m- ¾¾¾® Bakelite ½
COOH
Q.2 What happens when ? (Give equations only) : (f) CH 3COCH 3 + 3I 2 + 4NaOH ¾® CHI 3
(a) Formaldehyde reacts with ammonia. + CH 3COONa+ 3NaI+ 3H 2O
(U.P. 2014, 16, 17, 19) CH3
(b) Formaldehyde reacts with concentrated solution (g) C O + H2 N OH ¾¾¾¾®
CH3 –H2O
of sodium hydroxide. (U.P. 2016, 17)
CH3
(c) Acetaldehyde is heated with iodine and C N OH + H2O
concentrated aqueous alkali solution. CH3
CH3
(U.P. 2016)
(d) Acetaldehyde reacts with semicarbazide. Conc. H2SO4
(h) CH3COCH3 ¾¾¾¾®
(U.P. 2017)
(e) Acetone reacts with hydrogen cyanide and the CH3 CH3
product thus formed is boiled with an acid. Mesitylen
CaOCl
(U.P. 2016, 19) 2
(i) CH 3COCH 3 ¾¾¾® CHCl 3 + (CH 3COO) 2 Ca
(f) Acetone is heated with iodine and aqueous + Ca(OH) 2
solution of caustic soda. (2016, 18, 20) OH
(g) Acetone reacts with hydroxylamine. (j) HCHO+ HCN ¾¾® HCH ƒ
‚
CN
(U.P. 2016, 17)
(k) Benzaldehyde is heated with NaOH :
(h) Acetone is distilled with concentrated sulphuric
acid. (U.P. 2013) Cannizzaro’s reaction
CHO CH2OH COONa
(i) Acetone reacts with bleaching powder.
(U.P. 2016) NaOH /D
¾¾¾® +
(j) Formaldehyde (or aldehyde) reacts with HCN
Benzaldehyde Benzyl alcohol Sodium benzoate
(U.P. 2018, 20)
(k) Benzaldehyde is heated with NaOH? (U.P. 2019) (l) Acetaldehyde is reacted with dilute base:
(l) Acetaldehyde is reacted with dilute base? Aldol is formed in aldol condensation.
OH
(U.P. 2019) |
dil. NaOH
(m) aldehyde reacts with NaHSO 3 (U.P. 2020) CH 3CHO + CH 3CHO ¾¾¾¾® CH 3CH ¾ CH 2CHO
(n) aldehyde reacts with RMgX (U.P. 2020) D /–H O
2
¾¾¾¾® CH 3CH == CHCHO
(o) aldehyde reacts with Felhing’s solution Crotonaldehyde
(U.P. 2020) (m) Addition of sodium bisulphite : Crystalline
Ans. (a) 6HCHO+ 4NH 3 ¾® (CH 2 ) 6 N 4 + 6H 2O solid derivatives are formed when aldehydes or
NaOH(aq. )
Urotropin ketones are shaken with saturated aqueous solution
(b) 2HCHO ¾¾¾¾® CH 3OH+ HCOONa of sodium bisulphite, NaHSO 3 .
(Cannizaro’s reaction)

D
(c) CH 3CHO+ 3I 2 + 4NaOH ¾¾® 3NaI + 3H 2O
+ HCOONa+CHI 3
141
CH 3
C ==O
NaHSO3 HCl ‚ ƒOH
C — OH C == O + NaCl (vii)CH 3COCH 3 + NaHSO 3 ¾® H
ƒ ‚ – +
Carbonyl CH 3 SO 3 Na
compound SO3Na
Q.4 How will you obtain following ? Give chemical
+ SO 2 + H 2O equations only.
(n) Addition of Grignard reagents : Grignard (i) Formic acid from oxalic acid. (U.P. 2017)
reagents add on carbonyl group to give addition (ii) Acetone from acetic acid. (U.P. 2015)
products which on hydrolysis give alcohols. (iii) Acetic acid from ethyl amine. (U.P. 2009)
Depending upon the nature of carbonyl compounds,
(iv) Formic acid from ethane (U.P. 2015)
primary, secondary and tertiary alcohols are formed.
(v) Acetaldehyde from formaldehyde (U.P. 2018)
HOH
C == O + RMgX C — OMgX C — OH (vi) Formic acid from acetic acid (U.P. 2018)
H+
Carbonyl Grignard (vii) Urotropine (hexamethylene tetraamine)
compound reagent R R
Addition Alcohol from formaldehyde (U.P. 2018, 20)
compound 200°C/ D
Ans. (i) (COOH) 2 ¾¾¾® HCOOH+CO 2
(o) Aldehydes reduce Fehling's solution (alkaline
solution of cupric ion complexed with sodium 2 Ca(OH) D/ Dist.
(ii) CH 3COOH ¾¾¾® (CH 3COO) 2 Ca ¾¾¾®
potassium tartarate) to cuprous oxide (red coloured
ppt.) CH 3COCH 3
RCHO + 2CuO ¾® RCOOH + Cu 2O ¯ HNO2
(iii) CH 3CH 2 NH 2 ¾¾® CH 3CH 2OH
Cuprous oxide
(Red ppt.) [O]/K Cr O / H+
2 2 7
¾¾¾¾¾¾® CH 3COOH
Q.3 What happens when ? Give only chemical equations.
Cl /hv NaOH(aq. )
(i) Formic acid is heated with Fehling solution. 2
(iv)C 2 H 6 ¾¾¾® CH 3CH 2Cl ¾¾¾® CH 3CH 2OH
(U.P. 2010, 17, 18, 20) Conc. H SO /180° Ozonolysis
2 4
(ii) Acetic acid is treated with ethyl alcohol. ¾¾¾¾¾¾® CH 2 == CH 2 ¾¾¾® HCHO
(U.P. 2016) [O ]
¾¾¾® HCOOH
(iii) Acetic acid is heated with phosphorous pentoxide. K Cr O Ca(OH)
2 2 7 2
(U.P. 2017) (v) HCHO ¾¾¾® HCOOH ¾¾¾® (HCOO) 2 Ca
(iv) Mixture of calcium acetate and calcium formate is (CH COO) Ca
3 2
¾¾¾¾¾® CH 3CHO
strongly heated. (U.P. 2017) (Acetaldehyde)
(v) Chlorine gas pass through acetic acid. PCl
5
(U.P. 2010) (vi) CH 3COOH ¾¾® CH 3COCl+ POCl 3 + HCl
(vi) Formic acid reacts with lead carbonate. Zn-Hg/HCl KOH (alc.)
¾¾¾¾® CH 3CH 2 ¾ Cl ¾¾¾® CH 2 == CH 2
(U.P. 2018)
4 Alc. KMnO
(vii) Acetone reacts with sodium bisulphite. ¾¾¾¾® 2HCOOH
(U.P. 2018) (vii) 6H × CHO+ 4NH 3 ¾® (CH 2 ) 6 N 4 + 6H 2O
Hexamethylene
Ans. (i) HCOOH+ 2CuO ¾® CO 2 + H 2O+Cu 2O ¯ tetraamine
Red ppt.
Q.5 Organic compound A, having specific smell forms two
H+ /D compounds B and C by reacting with sodium
(ii) CH 3COOH+C 2 H 5OH ¾¾® CH 3COOC 2 H 5
hydroxide. Molecular formula of compound B is
+ H 2O
C 7 H 8O, which on oxidation again forms compound A.
P2O5 /D
(iii) 2CH 3COOH ¾¾¾¾® (CH 3CO) 2 O Compound C on heating with soda lime forms
– H 2O
benzene. Write the structure of organic compounds
D
(iv) (CH 3COO) 2 Ca+(HCOO) 2 Ca ¾¾® 2CH 3CHO A, B and C. Write equations of the related reactions.
(U.P. 2014, 16)
+ 2CaCO 3 + CH 3COCH 3 + HCHO Cannizaro’s
Ans. 2C 6 H 5CHO + NaOH ¾ ¾ ¾ ¾ ¾® C 6 H 5CH 2OH
Cl 2 / P (Red) Benzaldehyde reaction Benzyl alcohol
(v) CH 3COOH ¾¾¾¾® D
ClCH 2COOH ( A) ( B)

(vi) 2HCOOH+ PbCO 3 ¾® (HCOO) 2 Pb+CO 2 + C 6 H 5COONa


Sodium benzoate
+ H 2O ( C)
[O ]
C 6 H 5CH 2OH ¾ ¾ ¾® C 6 H 5CHO+
Oxidation
H 2O
( B) ( A)
142

C 6 H 5COONa + NaOH ¾Decarboxylation


¾ ¾ ¾¾® C 6 H 6 + Na 2CO 3 COOH COO CHO
( C) Benzene
Ca(OH) D/dist.
Structures: (iii) 2
¾¾¾® Ca ¾¾®
CHO CH2OH COONa 2

( A ): ; (B): ; (C): COOH CONH2

Q.6 What happens when: 3 NH /D


(iv) ¾¾¾®
(i) benzoic acid is nitrated? (U.P. 2017)
(ii) toluene reacts with chromylchloride. Q.8 Complete the following reactions :
(U.P. 2013, 19) H2
¾®( B) ¾(¾
(i) CH 3COCl ¾ ¾ C)
¾ ®C 2H 6
(iii) toluene reacts with acidic K 2Cr2O7 solution. Pd BaSO4

(U.P. 2014) (U.P. 2013, 15)


(iv) benzoic acid is reacted with hydrazoic acid in (ii) C 6 H 5COOH + A ¾2¾
¾®4 H SO
C 6 H 5 NH 2 + CO 2 + N 2
presence of conc. H 2SO 4 (U.P. 2018)
(U.P. 2016)
COOH COOH OCOCH3

Conc. HNO /Conc. H SO


3 2 4 anh. AlCl /D
Ans. (i) ¾¾¾¾¾¾¾¾¾® (iii) 3
¾¾¾¾® ? (U.P. 2018)
NO2
H
m-nitrobenzoic acid Ans. (i) CH 3COCl ¾ ¾
¾2
® CH 3CHO ¾ Zn,Hg
¾¾® C 2 H 6
CH3 CHO Pd BaSO4 ( B) Conc.HCl
( C)

H SO
CrO Cl
2 2 (ii) C 6 H 5COOH + N 3 H ¾¾®
2 4
C 6 H 5 NH 2 + N 2 + CO 2
(ii) ¾¾¾® ( A)
OCOCH3 OH OH
CH3 CHO COCH3
anh. AlCl 3/D
(iii) ¾¾¾¾® +
K2Cr2O7/H+
(iii) ¾¾¾¾®
COCH3
(Fries rearrangement)
COOH NH2
Q.9 How will you obtain?
(i) Mesitylene from acetone
(iv) + H3N ¾® +CO2 + N2
(ii) Chloretone with acetone
Q.7 How will you obtain ? (iii) Acetone from isopropyl alcohol (U.P. 2016)
(i) Benzyl chloride from benzaldehyde (U.P. 2010) H3C CH3
Conc. H2SO4
(ii) Aniline from benzoic acid (U.P. 2015) Ans. (i) 3CH3COCH3 ¾¾¾¾® –3H2O
Acetone
(iii) Benzaldehyde from benzoic acid
(U.P. 2016, 17) CH3
Mesitylene
(iv) Benzamide from benzoic acid (U.P. 2017)
(v) Benzoic acid from benzene (U.P. 2018) CH3 CH3 OH
CHO CH3 CHCl2 C==O+CHCl3 ¾¾® C
(ii) CH CH3 CCl3
3
Acetone Chloroform Chloretone
Zn/Hg/HCl Cl2/hv
Ans. (i) ¾¾¾¾® ¾¾®
CH3 Cu CH3
(iii) CHOH ¾¾® C==O+H2
COOH CONH2 NH2 CH3
300ºC
CH3
Isopropyl Acetone
alcohol
NH3/D Br2/KOH
(ii) ¾¾¾® ¾¾¾®
–H2O
143

Q.10 What happens when (write chemical equations only.) n-pentane. Write name and structural formula of the
(U.P. 2014) compound ( A ) and chemical equation of the related
(i) Acetone reacts with chloroform? reaction. (U.P. 2018)
(ii) Calcium formate is heated? Ans. Because C 5 H10O does not give positive, iodoform test
(iii) Formaldehyde reacts with Fehling’s solution? but give n-pentane on reduction. So it is an aldehyde
(U.P. 2016) or a ketone without ¾CH 3 group directly attached to
Ans. (i) Chloretone is obtained. carbonyl group. Thus ( A ) may be
CH 3 × CH 2 × CH 2 × CH 2 CH 3 × CH 2
CH3 CH3 OH ‚ ‚
C==O+CHCl3 ¾¾® C C== O Or C== O
ƒ ƒ
CH3 Chloroform CH3 CCl3 H CH 3 × CH 2
Acetone Chloretone
Red P+HI
(ii) When calcium formate is heated, formaldehyde is Ald / ketone ¾¾¾¾®
Reduction
C 5 H12
( A) ( n-pantane)
obtained.
D Q.14 Write the reagent used for the conversion of the
(HCOO) 2 Ca ¾¾ ® HCHO + CaCO 3
Calcium formate Formaldehyde following functional groups. (U.P. 2018)
(iii) Red ppt. of Cu 2O is obtained. (i) —CHO into ¾CH 2OH (ii) —CN into —COOH
HCHO + Cu(OH) 2 ¾® HCOOH + Cu 2O + H 2O (iii) —COOH into —H (iv) —COOH into —COCl
Formaldehyde Formic acid Red ppt 2 H / Ni
Ans. (i) ¾ CHO ¾¾¾® ¾ CH 2OH
Q.11 Write down the reaction of acetone with the
H O/H+
following : (U.P. 2016, 18) 2
(ii) ¾ CN ¾¾¾® ¾ COOH
(i) Chloroform (ii) Ammonia (iii) Iodine Sodalime
(iii) ¾ COOH ¾¾¾® ¾H
Ans. (i) Chloretone is obtained.
PCl
3
CH3 CH3 OH (iv) ¾ COOH ¾¾® ¾ COCl
KOH
C==O+CHCl3 ¾¾® C Q.15 Identify A in the following reaction : (U.P. 2018)
CH3 Chloroform CH3 CCl3
Acetone Chloretone Conc. H 2SO4
(COOH) 2 + 2C 2 H 5OH ¾¾¾¾®
D
A + 2H 2O
(ii) Acetone reacts with ammonia gives diacetone
amine. COOC 2 H 5
CH 3 Ans. ½ ethyl oxalate
| COOC 2 H 5
2CH 3COCH 3 + NH 3 ¾ –H
¾¾ ® CH 3 — C — CH 2COCH 3 Q.16 What happens when— (U.P. 2019)
2O
|
NH 2 (i) Ethanol is heated with caustic soda ?
Diacetone amine (ii) Propanone reacts with chloroform ?
(iii) Acetone reacts with iodine in the presence of (iii) Propanone is heated with sodium hypochlorite?
NaOH gives iodoform (CHI 3 ).
(iv) Phenyl methyl ketone is heated with
CH 3COCH 3 + 3I 2 + 4NaOH ¾® 2,4-dinitrophenyl hydrazine ?
Acetone
CHI 3 + 3NaI+CH 3COONa + 3H 2O
(v) Benzaldehyde is heated with concentrated NaOH
Q.12 What happens when (Give chemical equations only)? solution ?
(U.P. 2016) Ans. (i) ethanol is heated with caustic soda – aldol
(i) Formaldehyde is heated with concentrated sodium condensation
hydroxide? OH
|
(ii) Acetaldehyde is heated with alkaline iodine? NaOH / D
2CH 3CHO ¾¾¾¾® CH 3 CH ¾ CH 2 CHO
Ans. (i) Formaldehyde on heating with NaOH give methyl Aldol
alcohol and sodium formate. D /- H O
2
2HCHO + NaOH ¾® CH 3OH + HCOONa ¾¾¾® CH 3CH == CHCHO
Formaldehyde Sod. formate
Methyl
alcohol
(ii)propanone reacts with chloroform – chloretone is
formed.
(ii) When acetaldehyde is heated with alkaline iodine, CH3 OH
CH3
iodoform is formed. C == O + CHCl 3 ¾® C
CH 3CHO + 3I 2 + 4NaOH ¾® CH3 CH3 CCl3
CHI 3 + 3NaI + HCOONa + 3H 2O Chloretone
Iodoform
(iii) propanone is heated with sodium hypochlorite
Q.13 Compound A having molecular formula C 5 H10O does (Cl 2 / NaOH) -
not perform iodoform test but on reduction forms
144

CH 3COCH 3 + 3Cl 2 + 4NaOH ¾® CHCl 3 (v) Benzoic acid from bromobenzene :


+3NaCl + 3H 2O + CH 3COONa Br CN COOH
(iv) 2,4–dinitrophenyl hydrazone reacts with phenyl
methyl ketone : KCN HOH /H +
NO2 ¾¾®
- KBr
¾¾¾®
H3C ¾¾® Benzoic acid
- H 2O
C O+H2 N.NH Q.18 What happens when (write chemical equations
NO2
only)— (U.P. 2019)
CH 3 NO2 (i) Methanal reacts with Ammonia ?
| (ii) Methanal is heated with caustic soda ?
¾ C == N . NH ¾
NO2 (iii) Propanone reacts with nitric acid ?
2,4–dinitrophenyl hydrazone (iv) Ethanoic acid is heated with P2O 5 ?
(v) Benzaldehyde reacts with conc. NaOH :
(v) Benzaldehyde is heated with phenyl methyl
Cannizzaro’s reaction
ketone in the presence of caustic soda ?
CHO CH2OH COO–Na+
Ans. (i) Methanal reacts with ammonia :
2 Conc.NaOH
¾ ¾¾ ¾® + 6HCHO + 4NH 3 ¾® (CH 2 ) 6 N 4 + 6H 2O
Hexamethylene
tetramine
Benzyl alcohol Sodium benzoate (ii)Methanal is heated with caustic soda :
Q.17 How the following will be obtained ? (U.P. 2019) Cannizzaro’s reaction
(i) Benzaldehyde from Toluene 2HCHO + NaOH ¾® CH 3OH+ HCOONa
(ii) Benzaldehyde from Benzoyl chloride Methyl Sodium
(iii) Ethanal from Acetonitrile alcohol formate
(iv) Benzoic acid from Phenyl methyl ketone (iii) Propanone reacts with nitric acid :
(v) Benzoic acid from Bromobenzene. Conc.HNO
3
CH 3COCH 3 ¾¾¾¾® CH 3COOH + CO 2 + H 2O
Ans. (i) Benzaldehyde from toluene : Etard’s
reaction (iv) Ethanoic acid is heated with P2O 5 :
CH3 CHO
2 5 P O /D
2CH 3COOH ¾¾¾®
-H O
(CH 3CO) 2 O
2
CrO Cl /CCl
2 2 4
Acetic anhydride
¾¾¾¾¾¾®
(v) Benzaldehyde is heated with phenyl
Benzaldehyde methyl ketone in presence of caustic soda :
OH
(ii)Benzaldehyde from benzoyl chloride :
Rosenmund reduction CHO CH3 CO CH CH2 CO
COCl CHO NaOH /D
+ ¾¾¾¾®
H 2 /Pd - BaSO4
¾¾¾¾¾®
–H2O D

(iii) Ethanal from acetonitrile : Stephan’s CH CH CO


reduction
H /SnCl - HCl
2 2
CH 3CN ¾¾¾¾¾¾® CH 3CH == NH . HCl
Aldimine hydrochloride NH 3 Q.19 What happens when (Give chemical equations
H O
2
only)— (U.P. 2019)
¾¾® CH 3CHO (i) Formaldehyde is heated with 50% NaOH ?
- HCl
(iv) Benzoic acid from phenyl methyl ketone: (ii) Acetaldehyde is heated with dilute NaOH and I 2 ?
COCH3 COOH (iii) Acetaldehyde is heated with Tollen’s reagent?
(iv) Acetone is reacted with chloroform in presence of
Conc.HNO / D
3
¾¾¾¾¾® + CO 2 KOH?
(v) Glycerol is heated with oxalic acid?
Ans. (i) Formaldehyde is heated with 50% NaOH¾
Cannizzaro reaction takes place
145

NaOH / D (iv) Acetamide from acetone :


2HCHO ¾¾¾¾® CH 3OH + HCOONa
Methyl Sodium 2 2 7 K Cr O + Conc.H SO
2 4
alcohol formate CH 3COCH 3 ¾¾¾¾¾¾¾¾®
- CO ,- H O
CH 3COOH
Acetone 2 2
(Disproportionation reaction) NH / D
3
(ii) Acetaldehyde is heated with dil. NaOH and I 2 ¾¾¾®
-H O
CH 3CONH 2
2 Acetamide
¾iodoform reaction takes place.
(v) Cinnamic acid from benzaldehyde :
CH 3CHO + 3I 2 + 4NaOH ¾® CHI 3 ¯ + HCOONa
Acetaldehyde Iodoform Perkin’s reaction
+3NaI + 3H 2O CHO
(iii) Acetaldehyde is heated with Tollen’s reagent¾ (CH CO) O/CH COONa/ D
3 2 3
silver mirror is obtained. ¾¾¾¾¾¾¾¾¾¾®
- H 2 O/ - CH 3 COOH
Tollen ¢s reagent Benzaldehyde
CHCHO + Ag 2O ¾¾¾¾¾¾®
D
CH 3COOH
Acetic acid
C 6 H 5CH == CH ¾ COOH
+ 2Ag ¯ Cinnamic acid
Silver mirror
Q.21 What happens when (Give chemical equations
(iv) Acetone is heated with chloroform in presence of only)— (U.P. 2019)
KOH¾chloretone is formed
CH3 CH3 OH (i) Formaldehyde is heated with Fehling solution?
KOH
C == O + CHCl 3 ¾¾® C (ii) Acetone is distilled with concentrated H 2SO 4 ?
CH3 CH3 CCl3
(iii) Formaldehyde is reacted with CH 3 MgBr and the
Chloretone product is hydrolysed ?
(v) Glycerol is heated with oxalic acid¾at
(iv) Acetic acid (or RCOOH) is heated with PCl 5 ?
100–110ºC, formic acid is obtained.
CH 2OH CH 2OH (U.P. 2020)
| | (v) Benzaldehyde reacts with concentrated NaOH?
C HOH +(COOH) 2 + H 2O ¾®C HOH Ans. (i) formaldehyde is heated with Fehling’s
| Oxalic acid |
CH2OH CH2OH solution :
Glycerol HCHO + 2CuO ¾¾® HCOOH + Cu 2O ¯
F.solution Formic acid Red ppt.
+ H 2O + CO 2 + HCOOH
Formic acid (ii) acetone is distilled with conc.H 2SO 4 :
Q.20 How will you obtain (Given chemical equations 2 4 Conc.H SO
CH 3COCH 3 ¾¾¾¾® C 6 H 3 (CH 3 ) 3
only)— (U.P. 2019) Acetone Mesitylene
(i) Acetone from acetic acid?
(iii) formaldehyde is reacted with CH 3 MgBr
(ii) Salicylaldehyde from phenol?
and the product is hydrolysed:
(iii) Hexamethylene tetramine from formaldehyde?
(iv) Acetamide from acetone?
(v) Cinnamic acid from benzaldehyde? H OH
O MgBr
Ans. (i) Acetone from acetic acid : HOH /H +
Ca(OH)2 / D
HCHO + CH 3 MgBr ¾® H C H ¾¾¾¾®
CH 3COOH ¾¾¾¾¾® (CH 3COO) 2 Ca
Acetic acid Calcium acetate CH3
Distillation
¾¾¾¾® CH 3COCH 3 CH 3CH 2OH+ MgBr(OH)
Acetone
Ethyl alcohol
(ii) Salicylaldehyde from phenol :
(iv) acetic acid is heated with PCl 5 :
Reimmer-Tiemann reaction
OH CH 3COOH + PCl 5 ¾® CH 3COCl + POCl 3 + HCl
OH Acetyl chloride
CHO
CHCl 3 /KOH (v) benzaldehyde reacts with conc. NaOH :
¾¾¾¾¾® +
HOH /H Cannizzaro’s reaction
Phenol Salicylaldehyde
CHO CH2OH COO–Na+
(iii) Hexamethylene tetramine from
formaldehyde : Conc.NaOH +
2 ¾¾¾¾®
6HCHO + 4NH 3 ¾® (CH 2 ) 6 N 4 + 6H 2O Benzyl Sodium
Formaldehyde Ammonia Hexamethylene alcohol benzoate
tetramine
146

Q.22 How will you obtain (Give chemical equations Ans. (i)
only)— (U.P. 2019) O O
(i) Oxalic acid from formic acid? || ||
HCHO or H ¾ C ¾ H HCOOH or H ¾ C ¾ OH
(ii) Ethyl amine from acetic acid?
1. It contains aldehyde group 1. It contains carboxylic acid
(iii) Acetone from acetaldehyde?
( ¾ CHO). group ( ¾ COOH).
(iv) Benzaldehyde from aniline? 2. It forms oxime, hydrazone, 2. It does not form these
(v) Formic acid from oxalic acid? phenyl hydrazone, semicarba- compounds.
Ans. (i) oxalic acid from formic acid : zone, 2,4-dinitrophenyl hydra-
D / 360º C zone.
HCOOH + NaOH ¾¾®
-H O
HCOONa ¾¾¾®
Formic acid 2
3. With NaOH it undergo 3. With NaOH it form sodium
COONa COOH Cannizzaro reaction and form formate.
HOH /H + |
| ¾¾¾® methyl alcohol and sodium
COONa COOH formate.
Oxalic acid
4. With ammonia it form 4. With NH3 it form
(ii)ethylamine from acetic acid : Urotropin [(CH2 )6 N 4] a O
3 NH 2 4 D /H SO medicine for urinary infections ||
CH 3COOH ¾¾® CH 3COONH 4 ¾¾¾®
-H O in goat. H ¾ C ¾ NH2 (formamide).
2
Acetic acid
Reduction
CH 3CN ¾¾¾¾® CH 3CH 2 NH 2 (ii)
Ethyl amine HCHO HCOOH
(iii) acetone from acetaldehyde : Similar reaction : HCOOH + Ag 2O ¾® 2Ag ¯
[O] 2 Ca(OH) Both reduce Tollen’s reagent
CH 3CHO ¾® CH 3COOH ¾¾¾® (CH 3COO) 2 Ca
Calcium acetate HCHO + Ag 2O ¾® 2Ag + H2O + CO 2
Tollen's Silver
Dry distillation reagent mirror
¾¾¾¾¾® CH 3COCH 3
Acetone + HCOOH

(iv) benzaldehyde from aniline : Dissimilar reaction : It does not form oxime with
– It form oxime with hydroxyl hydroxyl amine.
NH2 N2+Cl CN
amine :
NaNO /HCl
H
2 KCN /CuCN C O+ H2 NOH
¾¾¾¾¾® ¾¾¾¾® H
H
C == NOH + H2O
CHO H
Formaldehyde oxime
(i) SnCl 2 /HCl
¾¾¾¾¾®
+ (iii) Aldol condensation : In this reaction two
(ii) H 2 O/H
Benzaldehyde
molecules of aldehyde or ketone condense in presence
of dil. alkali (like NaOH, Ba(OH) 2 etc.). to form
(v) formic acid from oxalic acid : b-hydroxyaldehyde or b-hydroxyketone.
OH
200º C/D |
(COOH) 2 ¾¾¾® HCOOH + CO 2
CH 3CHO + CH 3CHO dil.
¾¾¾®NaOH
CH 3 ¾ CH ¾ CH 2 ¾CHO
Q.23 (i) How would you differentiate between
3-hydroxy butanal (Aldol)
formaldehyde and formic acid with their structures ?
(iv)Two molecules of acetone condense to form aldol
(U.P. 2019) which further dehydrated to give mesityl oxide.
(ii) Write down one similar and dissimilar reactions of CH 3 Ba(OH)2
the above compounds. (U.P. 2019) C O+H CH 2 COCH3
¾¾¾®
CH 3 Acetone Acetone
(iii) Give one reaction of formation of Aldol. OH
(U.P. 2019) |
CH3
C ¾ CH 2COCH 3
(iv) Give one reaction of condensation of Acetone in CH3 Aldol
the formation of Mesityl oxide. (U.P. 2019)
D¯ H + / H 2O
CH3
C == CH ¾ COCH 3 + H 2O
CH3
147

Q.24 Write down the preparations of Benzoic acid from the Ans. (i) Esterification : Monocarboxylic acids react
following compounds. (U.P. 2019) with alcohols in presence of dehydrating agent such
(i) Toluene (ii) Nitrile (iii) Amide as concentrated H 2SO 4 or dry HCl to form esters.
Ans. Preparation of Benzoic acid from :
RCO OH +H OR' RCOOR'+H2O
(1)Toluene : Acid Alcohol Ester
CH3 COOK COOH (ii) Reaction with ammonia : Ammonium salts
Alk.KMnO
4 H O+
3
are formed which give amides on heating and when
¾¾¾¾®
D
¾¾® heated with a dehydrating agent, finally cyanides are
Toluene Benzoic acid formed.
D
RCOOH + NH 3 ¾® RCOONH 4 ¾¾ ® RCONH 2
(2)From Nitrile : Ammonium salt –H 2O Amide
–N
C– CONH2 P2O5, D
¾® RCN + H 2O
¾¾¾
Alkyl cyanide
Dil.HCl Dil.HCl
+ H 2O ¾¾¾® ¾¾¾® (iii) Substitution by halogens : Monocarboxylic
Benzamide acids react with PCl 5 in presence of pyridine to give
acid chlorides.
COOH RCOOH + PCl 5 ¾® RCOCl + POCl 3 + HCl
(iv) Reaction with bases : Salts and water are
+ NH 4 Cl formed, when carboxylic acids react with bases.
Benzoicacid RCOOH + NaOH ¾® RCOONa + H 2O
Salt
(3)From Amide :
(v) Hell-Volhard-Zelinsky (HVZ) reaction :
CONH2 COOH Higher carboxylic acids react with chlorine or
Dil.HCl
bromine to give a -substituted acids. The reaction is
¾¾¾® + NH 4 Cl catalysed by sun-light or P or Fe or I 2 etc. Lower acids
Benzamide Benzoicacid undergo substitution only in presence of sunlight.
Cl
|
Q.25 Complete the reaction and write the name and R — CH 2 × COOH + Cl 2 ¾ P¾¾
hv
® R — CH ×COOH ¾ P¾¾ Cl 2
®
formulae of A, B, C, D and E : (U.P. 2019) –HCl a - monochloro –HCl
acid
NaOH
(i) (HCOO) 2 Ca ¾® A ¾¾®
aq B + CH 3OH Cl
.
|
+ (HCOO) Ca
2 HCN 2 H O R — C ×COOH
(ii) (CH 3COO) 2 Ca ¾¾¾¾¾® C ¾¾® D ¾¾® E |
Cl
HCOO a, a - dichloro
Ans. (i) Ca HCHO+CaCO 3 acid
HCOO NaOH(aq.)
Cannizzaroreaction
Q.27 What happens when (Give chemical equations
only)— (U.P. 2020)
HCOO - Na + + CH 3 OH (i) Benzaldehyde reacts with semicarbazide?
(ii) Ketone reacts with Ethylene glycol?
CH3COO OOCH
(ii) Ca+ Ca D
2CH3CHO+2CaCO (iii) Propanone is heated with Barium hydroxide?
3
CH3COO OOCH (iv) RCOOH is heated with PCl 5 ?
HCN
OH (v) Benzaldehyde is heated with NaOH?
| CH3 OH Ans. (i) C 6 H 5CHO + H 2 N × NHCONH 2 ¾ D/
¾-¾H 2O
¾®
H 2O
CH 3 ¾ CH ¾ COOH ¬¾¾ - NH 3
C
2- hydroxy propanoic acid H CN C 6 H 5CH == N × NHCONH 2 + H 2O
Cyanohydrin Benzaldehyde semicarbazone

CH3 O—CH2
Q.26 What happens when carboxylic acid reacts with : (ii) (CH 3 ) 2 CO + (CH 2OH) 2 ¾® C
(Write chemical equations only) (U.P. 2020) CH3 O—CH2
Ketal
(i) ROH (ii) NH 3 Ba( OH )
2
(iii) PCl 5 (iv) NaOH (iii) 2CH 3COCH 3 ¾ ¾¾® (CH 3 ) 2 C(OH)CH 2COCH 3
Diacetone alcohol
(v) X 2 (Halogen) in presence of Red P/H 2O
148

(iv) RCOOH + PCl 5 ¾® RCOCl + POCl 3 + HCl Ans. (i) 2HCOOH + CaCO 3 ¾® ( HCOO) 2 Ca+ CO 2
Calcium formate
+ H 2O
(v) 2C 6 H 5CHO ¾ NaOH
¾¾®
C 6 H 5CH 2OH + C 6 H 5COONa
Benzyl alcohol Sodium benzoate
(ii) 6HCHO + 4NH 3 ¾ (CH 2 ) 6 N 4 + 6H 2O
(Cannizzaro’s reaction) Urotropine

(iii) CH 3COOH + PCl 5 ¾® CH 3COCl + HCl


Q.28 What happens when (write chemical equations + POCl 3
only)— (U.P. 2020) D
(iv) (CH 3 ) 2 CO + H 2 NNH 2 ¾¾ ®
- H 2O
(i) Acetone reacts with iodine in presence of NaOH?
(ii) Fehling solution is mixed in formic acid? (CH 3 ) 2 C == N × NH 2
Acetone hydrazone
(iii) Benzoic acid is heated with ammonia? COOH COOH
(iv) Tollen's reagent is mixed with acetaldehyde?
(v) Acetic acid is heated with CaO? Br2/CCl4
(v)
Ans. (i) (CH 3 ) 2 CO + 3I 2 + 4NaOH ¾® Br
CHI 3 ¯ + 3NaI + 3H 2O + CH 3COONa m-bromobenzoic acid
Iodoform
Q.30 Complete the following reactions and write the
(ii) HCOOH + 2CuO ¾® Cu 2O ¯ + CO 2 + H 2O names and formulae of A, B, C , D, E : (U.P. 2020)
Fehling Red ppt. (i) LiAlH 4
solution (Cuprous oxide) (i) RCOOH ¾ ¾ ¾ ¾ +
®A
(ii) H 3O

COOH COONH4 CONH2 (ii) RCH2 COOH ¾ (i)


¾X¾ 2 red P
¾¾® B
(ii) H 2O
D/–H2O COOH
(iii) + NH3
conc. HNO3
Benzamide (iii) C
conc. H2SO4
(iv) CH 3CHO + Ag 2O ¾® CH 3COOH + 2Ag ¯
Silver mirror (iv) RCOOH ¾ NH ¾¾3 D
® D ¾¾ ®E
Ans. (i) ( A ) = RCH 2OH
(v) 2CH 3COOH + CaO ¾® (CH 3COO) 2 Ca+ H 2O X
Calcium acetali
|
¾ Dry
¾¾ distillation
¾¾ ¾® CH 3COCH 3 + CaCO 3 (ii) ( B) = RCH— COOH
D Acetone
Q.29 How will you obtain (Give chemical equations only) COOH
(U.P. 2020)
(i) Calcium formate from formic acid? (iii) (C ) =
(ii) Urotropine from formaldehyde? NO2
(iii) Acetyl chloride from acetic acid?
(iv) Acetone hydrazone from acetone? (iv) ( D) = RCOONH 4
(v) m-bromobenzoic acid from benzoic acid? ( E) = RCONH 2
149

Solution of NCERT Text Book Problems


Q.1 What is meant by the following terms ? Give an C O + H 2N OH
–H2O
example in each case ?
(a) Cyanohydrin (b) Semicarbazone C N OH
(c) Hemiacetal (d) Ketal (i) Imine : The functional group containing
(e) 2,4-DNP derivative (f) Acetal Ç
C== N ¾ bond is called imines.
É
(g) Aldol (h) Oxime
e. g ., CH 3C== NH
(i) imine (j) Schiff’s base.
(j) Schiff’s base : Compounds of general structure
Ans. (a) Cyanohydrin : The product obtained by
R 2C== NR ¢ are called Schiff’s bases. They are
addition of HCN on carbonyl group.
OH considered as subclass of imines.
C O + HCN C Q.2 Name the following compounds according to IUPAC
CN
system of nomenclature :
(b) Semicarbazone : The product obtained by
(a) CH 3CH(CH 3 )CH 2CH 2CHO
reaction between carbonyl compound and
(b) CH 3CH == CHCHO
semicarbazide.
(c) CH 3CH(CH 3 )CH 2C(CH 3 ) 2 COCH 3
NH2CONHN H 2 + O C –H2O
NH2CONHN C (d) p - OHCC 6 H 4 CHO
(c) Hemiacetal : The product obtained by addition (e) CH 3CH 2COCH(C 2 H 5 )CH 2CH 2Cl
of alcohol on aldehydic carbonyl group. (f) CH 3COCH 2COCH 3
H HCl gas H OH (g) (CH 3 ) 3 CCH 2COOH
C O + R¢OH C
R R OR¢ Ans. (a) 4-methylpentanal
Hemiacetal (b) but-2-en-1-al
(d) Ketal : The product obtained by action of (c) 3,3,5-trimethylhexan-2-one
alcohol on ketonic carbonyl group. (d) benzene-1, 4-dicarbaldehyde
R HCl gas R OH
C O + R¢OH C (e) 6-chloro-4-ethylhexan-3-one
R R OR¢ (f) pentane-2-, 4-dione
(e) 2, 4-DNP derivative : The product obtained (g) 3,3-dimethylbutanoic acid.
when carbonyl group reacts with 2, 4-dinitrophenyl
Q.3 Draw the structures of the following compounds :
hydrazine.
(a) 3-methylbutanal
C O + H N . NH
2 NO 2 (b) p-methylbenzaldehyde
NO2 (c) 4-chloropentan-2-one
(d) p, p-dihydroxybezophenone
C N . NH NO2 (e) p-nitropropiophenone
–H2O
NO2 (f) 4-methylpent-3-en-2-one
(f) Acetal : The product obtained when hemiacetal (g) 3-bromo-4-phenylpentanoic acid
is allowed to react with alcohol further. (h) hex-2-en-4-ynoic acid
4 3 2 1
H OH R¢OH H OR¢ Ans. (a) CH 3 —CH — CH 2 — CHO
C C
R OR¢ –H2O R OR¢ |
CH3
(g) Aldol : The product obtained when two
molecules of carbonyl compounds (atleast one (b) CH3 CHO
containing a–H) condense in presence of alkali.
O OH
1 2 3 4 5
(c) CH 3 — C — CH 2 — CH— CH 3
CH3 C H + HCH 2CHO CH3CHCH2CHO || |
O Cl
(h) Oxime : The product obtained when carbonyl
group reacts with hydroxylamine (NH 4 OH).
150

O
N—OH
(d) HO C OH CH3 OCH3
(b) (c) C
H OCH3
(e) O2N COCH2CH3
O

1 2 3 4 5 (d) NNH—C—NH2
(f) CH 3 — C — CH == C — CH 3
|| |
O CH3 CH2—CH2
5 4 3 2 1
(g) CH 3 —CH — CH— CH 2 — COOH O O
| |
C 6 H 5 Br (e) CH3—CH2—C—CH2—CH2—CH3
6 5 4 3 2 1
(h) CH 3 — C ºº C— CH == CH— COOH H OH
(f) C
Q.4 Write the IUPAC names of the following ketones and
H OCH3
aldehydes. Wherever possible, give also common
names. Q.6 Predict the products formed when
(a) CH 3CO(CH 2 ) 4 CH 3 cyclohexanecarbaldehyde reacts with following
reagents :
(b) CH 3 (CH 2 ) 5 CHO
(a) PhMgBr and then H 3O +
CHO
(c) (b) Tollens’ reagent
(c) Semicarbazide and weak acid
(d) CH 3CH 2CHBrCH 2CH(CH 3 )CHO (d) Excess ethanol and acid
(e) PhCH == CHCHO (e) Zinc amalgam and dilute hydrochloric acid.
(f) PhCOPh Ans. (a) C 6 H11 — CHO + C 6 H 5 MgBr ¾®
Cyclohexane carbaldehyde
Ans.
H O+
C 6 H11 —CH — OMgBr ¾ ¾3 ¾® C 6 H11 —CH — OH
IUPAC name Common name | |
C 6H 5 C 6H 5
(a) heptan-2-one Methyl n-amyl ketone
Cyclohexyl phenyl carbinol
(b) heptanal —
(b) C 6 H11 — CHO + 2 [Ag(NH 3 ) 2 ]+ + 3OH - ¾®
(c) cyclopentanecarbaldehyde Cyclopentane carbaldehyde Cyclohexane
carbaldehyde
(d) 4-bromo-2-methyl hexanal g-Bromo-a-methyl-caproaldeh-
yde C 6 H11COO - + 2Ag + 4NH 3 + 2H 2O
O
(e) 3-phenylpropenal b-Phenylacrolein ||
(f) diphenylmethanone Benzophenone (c) C 6 H11 — CHO + H 2 NNH C — NH 2 ¾ Weak
¾ ¾acid
¾¾®
Cyclohexane
carbaldehyde
Q.5 Draw structures of the following derivatives : O
(a) The 2,4-dinitrophenylhydrazone of benzaldehyde ||
(b) Cyclopropanone oxime C 6 H11 — CH == NNH C — NH 2 + H 2O
Cyclohexane carbaldehyde semicarbazone
(c) Acetaldehydedimethylacetal HOC 2 H 5
(d) The semicarbazone of cyclobutanone (d) C 6 H11 — CHO + ¾®
Cyclohexane carbaldehyde HOC 2 H 5
(e) The ethylene ketal of hexan-3-one
C 6 H11 —CH — OC 2 H 5 + H 2O
(f) The methyl hemiacetal of formaldehyde. |
NO2 OC 2 H 5
Cyclohexane carbaldehyde diethyl acetal

Ans. (a) CH==NNH NO2


151
Zn-Hg/HCl
(e) C 6 H11 — CHO + 4 [H] ¾ ¾ ¾ ¾ ¾¾®
Clemensen reduction
(II) The compounds (a), (c) and (i) undergo
Cannizzaro reaction. as they do not contain
C 6 H11 — CH 3 + H 2O H-atom at the a-carbon.
Methyl cyclohexane
(a) 2HCHO + conc. NaOH ¾® HCOONa + CH 3OH
Q.7 Which of the following compounds would undergo
aldol condensation, which the Cannizzaro reaction (c) 2C 6 H 5 — CHO + conc. NaOH ¾®
and which neither ? Write the structures of the C 6 H 5COONa + C 6 H 5CH 2OH
Sod. benzoate Benzyl alcohol
expected products of aldol condensation and
Cannizzaro reaction. CH 3
|
(a) Methanal (b) 2-Methylpentanal (i) CH 3 — CH 2 —C — CHO + conc. NaOH ¾®
(c) Benzaldehyde (d) Benzophenone |
CH 3
(e) Cyclohexanone (f) 1-Phenylpropanone
(g) Phenyl acetaldehyde (h) Butan-1-ol CH 3 CH 3
| |
(i) 2,2-Dimethyl butanal CH 3CH 2 —C — COONa + CH 3CH 2 —C — CH 2OH
Ans. (I) The compounds (b), (e), (f) and (g) contain a-H | |
atom and undergo aldol condensation. The other CH 3 CH 3
Sod. 2,2-dimethyl butanoate 2,2-Dimethyl butan-1-ol
compounds do not contain a-hydrogens and do
not undergo aldol condensation. Their reactions (III)Benzophenone and butan-1-ol neither undergo
are as follow. aldol condensation nor Cannizzaro reaction.
(b) 2CH 3 — CH 2 — CH 2 —CH — CHO ¾ Dil.NaOH
¾ ¾¾® Q.8 How will you convert ethanal into the following
| compounds ?
CH 3 (a) Butane-1,3-diol (b) But-2-enal
CH 3 (c) But-2-enoic acid.
7 6 5 4 3 2|
Ans. (a)
CH 3CH 2CH 2 — CH— CHOH —C — CH 2CH 2CH 3 4 3 2 1
| | 2CH 3CHO ¾ Dil.NaOH
¾¾® CH 3 — CHOH— CH 2 — CHO
¾aldol
CH 3 CHO Ethanal condensation 3-Hydroxybutanal
3-Hydroxyl -2, 4-dimethyl -2-propyl heptanal
4 3 2 1
O NaBH
4
¾reduction
¾¾ ¾ ® CH 3 — CHOH— CH 2 CH 2OH
1
Butane-1,3-diol
Dil. NaOH 2
(e) 2 O ¾¾¾¾® 1
(b)
OH 4 3 2 1
2-(1-Hydroxy-1-cyclohexyl) 2CH 3CHO ¾ Dil.NaOH
¾¾® CH 3 — CHOH— CH 2 — CHO
¾aldol
cyclohexan-1-one Ethanal condensation 3-Hydroxybutanal

H O+
D
(f) 2C 6 H 5 — COCH 2CH 3 ¾ Dil.NaOH
¾ ¾¾® ¾¾3
¾¾
-H O
® CH 3 — CH == CH — CHO
2
1-Phenyl propanone But-2-enal
As in (b)
OH CH 3 (c) 2CH 3CHO ¾ ¾ ¾¾® CH 3CH == CHCHO
Ethanal But-2-enal
3| 2½ 1
[Ag(NH 3 )2 ]+ OH -
C 6 H 5 —C — CH — CO— C 6 H 5 ¾¾ ¾ ¾ ¾ ¾¾® CH 3CH == CHCOOH
Tollens’ reagent
4,5| But-2-enoic acid
C 2H 5
Q.9 Write structural formulae and names of four possible
3-Hydroxy -2-methyl -1,3-diphenyl pentane-1-one
aldol condensation products from propanal and
(g) 2C 6 H 5 — CH 2CHO ¾ Dil.NaOH
¾ ¾¾® butanal. In each case, indicate which aldehyde acts as
Phenyl acetaldehyde nucleophile and which as electrophile.
4 3 2
C 6 H 5 — CH 2 — CH —CH — C 6 H 5 Ans. (a) CH 3CH 2CHO + CH 3CH 2CHO ¾®
| | Propanal Propanal
(Electrophile) (Nucleophile)
OH CHO OH CH 3
1
3-Hydroxy -2,4-diphenyl butanal 5 4 |3 |2 1
CH 3CH 2 CH — CH — CHO
3-Hydroxy -2-methylpentanal
152

OH CH 2CH 3
6 5 4 3| 2| 1
(b) CH 3CH 2CH 2CHO + CH 3CH 2CH 2CHO ¾® CH 3CH 2CH 2 — CH — CH — CHO
Butanal (Electrophile) Butanal (Nucleophile) 2-Ethyl -3-hydroxyhexanal

OH CH 2CH 3
5 4 3| 2| 1
(c) CH 3CH 2CHO + CH 3CH 2CH 2CHO ¾® CH 3CH 2 —CH —CH — CHO
Propanal (Electrophile) Butanal (Nucleophile) 2-ethyl -3-hydroxypentanal

OH CH 3
6 5 4 3| 2| 1
(d) CH 3CH 2CH 2CHO + CH 3CH 2CHO ¾® CH 3CH 2CH 2 — CH — CH — CHO
Butanal (Electrophile) Propanal (Nucleophile) 3-hydroxy -2-methylhexanal

Q.10 An organic compound with the molecular formula C 9 H10O forms 2,4-DNP derivative, reduces Tollen’s reagent and
undergoes Cannizzaro reaction. On vigorous oxidation, it gives 1,2-benzenedicarboxylic acid. Identify the compound.
Ans. (i) Since the compound given forms a 2,4-DNP derivative and reduces Tollens’ reagent, so it is an aldehyde.
(ii) It undergoes Cannizzaro reaction, therefore, CHO group is directly attached to the benzene
ring.
(iii) On oxidation, it gives 1,2-benzenedicarboxylic acid, therefore, it is o-ethylbenzaldehyde.
COOH

COOH
1,2-Benzene
dicarboxylic acid NO2
®¾

[O]
COO– CHO CH==NNH NO2+H2O
[Ag(NH3)2]+OH– 2,4-DNP
Ag +
Tollens’ reagent
Silver
mirror C2H5 C2H5 C2H5
2-Ethyl benzoate o-Ethyl benzaldehyde 2,4-DNP derivative

Q.11 An organic compound (A) (molecular formula C 8 H16O 2 ) was hydrolysed with dilute sulphuric acid to give a carboxylic
acid (B) and an alcohol (C). Oxidation of (C) with chromic acid produced (B). (C) on dehydration gives but-1-ene. Write
equations for the reactions involved.
H O+
Ans. CH 3CH 2CH 2COOCH 2CH 2CH 2CH 3 ¾ ¾3 ¾® CH 3CH 2CH 2COOH + CH 3CH 2CH 2CH 2OH
Butyl butanoate ( A ) Butanoic acid ( B ) Butan -1-ol ( C )
Dehydration [O ]
CH 3CH 2CH == CH 2 ¬¾ -¾ ¾¾ ¾ CH 3CH 2CH 2CH 2OH
H 2O
¾® CH 3CH 2CH 2COOH
¾¾
But-1-ene ( C) ( B)

Q.12 Arrange the following compounds in increasing order of their property as indicated :
(a) Acetaldehyde, Acetone, Di-tert-butyl ketone, Methyl tert-butyl-ketone (reactivity towards HCN)
(b) CH 3CH 2CH(Br)COOH, CH 3CH(Br)CH 2COOH, (CH 3 ) 2 CHCOOH, CH 3CH 2CH 2COOH (acid strength)
(c) Benzoic acid, 4-Nitrobenzoic acid, 3,4-Dinitrobenzoic acid, 4-Methoxybenzoic acid (acid strength).
CH 3 CH 3 CH 3
| | |
Ans. (a) CH 3 —C — CO —C — CH 3 < CH 3 —C — CO — CH 3 < CH 3 — CO — CH 3 < CH 3 — CHO
| | |
CH 3 CH 3 CH 3
Di -tert.-butyl ketone Methyl -tert.-butyl ketone

(b) (CH 3 ) 2 CHCOOH < CH 3CH 2CH 2COOH < CH 3CH(Br)CH 2COOH < CH 3CH 2CHBrCOOH
153

Q.13 Give simple chemical tests to distinguish between the following pairs of compounds :
(a) Propanal and propanone (b) Acetophenone and benzophenone
(c) Phenol and benzoic acid (d) Benzoic acid and ethyl benzoate
(e) Pentan-2-one and pentan-3-one (f) Benzaldehyde and acetophenone
(g) Ethanal and propanal
Ans. (a) Propanal and propanone : Iodoform test : propanone gives yellow ppt. with I 2 / NaOH while propanal
does not give this test.
I 2 + 2NaOH ¾¾® NaI+ NaOI+ H 2O
O
½½
CH 3 ¾ C ¾ CH 3 + 3NaOI ¾¾® CHI 3 ¯ + CH 3COONa+ 2NaOH
Iodoform

(b) Acetophenone and benzophenone : Acetophenone (due to presence of ¾COCH 3 group) gives iodoform
test while benzophenone does not.
COCH3 COONa

2 I /NaOH
+ 3NaOI ¾¾¾®
Heat
+ CHI3¯ + 2NaOH

(c) Phenol and benzoic acid : (i) Benzoic acid liberates CO 2 from NaHCO 3 while phenol does not.
COOH COONa

+ NaHCO3 ¾¾¾® + H2O + CO2­

(ii) Br2 / H 2O gives white ppt. with phenol while benzoic acid does not.
OH OH
Br Br
+ 3Br2(aq.) ¾¾¾® + 3HBr

Br
2,4,6-tribromophenol

(d) Benzoic acid and ethyl benzoate : Benzoic acid give CO 2 with NaHCO 3 , while ethyl benzoate does not.
COOH COONa

+ NaHCO3 ¾¾¾® + H2O + CO2­

(e) Pentan-2-one and pentan-3-one : Pentan-2-one gives iodoform test while pentan-3-one does not.
2 NaOH /I
CH 3COCH 2CH 2CH 3 + 3NaOI ¾¾¾¾®
D
CHI 3 ¯ +CH 3CH 2CH 2COONa+ 2NaOH
Iodoform

(f) Benzaldehyde and acetophenone : Acetophenone gives iodoform test while benzaldehyde does not.
COCH3 COONa

2 I /NaOH
+ 3NaOI ¾¾¾® + CHI3¯ + 2NaOH

(g) Ethanal and propanal : Ethanal gives iodoform test while propanal does not.
2 I / NaOH
CH 3CHO+ 3NaOI ¾¾¾¾® CHI 3 ¯ + HCOONa+ 2NaOH
154

Q.14 How will you prepare the following compounds from benzene ? You may use any inorganic reagent and any organic
reagent having not more than one carbon atom.
(a) Methyl benzoate (b) m-Nitrobenzoic acid (c) p-Nitrobenzoic acid
(d) Phenyl acetic acid (e) p-Nitrobenzaldehyde
CH Cl AlCl 3 CH OH H SO4
Ans. (a) C 6 H 6 ¾ ¾3¾ ¾ ¾ ® C 6 H 5 — CH 3 ¾ ¾ [O]
¾¾ ¾® C 6 H 5 — COOH ¾ ¾3¾ ¾ 2¾¾
Strong oxidant
® C 6 H 5COOCH 3
Methyl benzoate
CH 3 Cl AlCl 3 HNO3 H 2 SO4
(b) C 6 H 6 ¾ ¾ ¾ ¾ ¾® C 6 H 5 — CH 3 ¾ ¾ [O]
¾¾ ¾® C 6 H 5
Strong oxidant
— COOH ¾ ¾ ¾ ¾ ¾
¾® m- NO 2 — C 6 H 4 — COOH
m-Nitro benzoic acid
CH 3 Cl Conc. H 2 SO4
(c) C 6H 6 ¾ ¾ ¾¾® C 6 H 5
AlCl 3
— CH 3 ¾ ¾ ¾ ¾ ¾®
Conc. HNO3
p- NO 2 — C 6 H 4 — CH 3 + o- NO 2 — C 6 H 4 — CH 3
p-Nitro toluene o-Nitro toluene

p- NO 2 — C 6 H 4 — CH 3 ¾ ¾ [O]
¾¾ ¾®
Strong oxidant
p- NO 2 — C 6 H 4 — COOH
p-Nitro toluene p-Nitro benzoic acid

CH Cl AlCl Br H O+
3 3
(d) C 6 H 6 ¾¾¾¾® 2
C 6 H 5 — CH 3 ¾Heat
¾
¾ ® C 6 H 5 — CH 2 Br ¾ Alc.KCN
¾ ¾¾® C 6 H 5 — CH 2CN ¾ ¾3 ¾® C 6 H 5CH 2COOH
Benzyl bromide Benzyl cyanide Phenyl acetic acid
As in (c) CrO2 Cl 2 CS2
(e) C 6 H 6 ¾ ¾ ¾¾® p- NO 2 — C 6 H 4 — CH 3 ¾Etard
¾ ¾reaction
¾¾ ¾® p- NO 2 — C 6 H 4 — CHO
p-Nitro toluene p-Nitrobenzaldehyde

Q.15 How will you bring about the following conversions in not more than two steps ?
(a) Propanone to propene (b) Benzoic acid to benzaldehyde
(c) Ethanol to 3-hydroxybutanal (d) Benzene to m-nitroacetophenone
(e) Benzaldehyde to benzophenone (f) Bromobenzene to 1-phenylethanol
(g) Benzaldehyde to 3-phenylpropan-1-ol (h) Benzaldehyde to a hydroxy phenylacetic acid
(i) Benzoic acid to m-nitrobenzyl alcohol.
Ans. (a) Conversion of propanone to propene
4 NaBH 4 Conc. H 2 SO , 444 K
CH3 — CO — CH3 ¾¾¾® CH3 CHOHCH3 ¾¾¾¾¾¾¾®
Dehydration
CH3 CH == CH2 + H2 O
Reduction
Propan -2-ol

(b) Conversion of benzoic acid to benzaldehyde


NaOH/CaO
3 3 CH Cl / AlCl [O]
C6 H5 COOH ¾¾® C6 H6 ¾¾¾¾® C 6 H 5CH 3 ¾¾¾¾® C6 H5 CHO
D CrO2Cl 2 /CCl 4

5 PCl 2 4 H - Pd / BaSO
Or C6 H5 COOH ¾¾® C6 H5 COCl ¾¾¾¾¾® C6 H5 CHO
Benzoyl chloride

(c) Conversion of Ethanol to 3-hydroxybutanal


Oxidation NaOH
CH 3 — CH 2 — OH ¾ ¾ ¾ ¾¾® CH3 CHO ¾¾¾¾¾¾®
Aldol condensation
CH3 CHOHCH2 CHO
KMnO4 dil. H 2 SO4 3-Hydroxybutanal

(d) Conversion of benzene to m-nitroacetophenone


COCH3 COCH3

(CH3CO)2O Anhy. AlCl3 conc. HNO + H SO


3 2 4
¾¾¾¾¾¾¾¾® ¾¾¾¾¾¾ ¾®
Friedel-craft Nitration
acylation NO2
Benzene Acetophenone m-nitroacetophenone

(e) Conversion of benzaldehyde to benzophenone


[O] NaOH/CaO
C6 H5 CHO ¾¾¾¾¾® C6 H5 COOH ¾¾¾® C6 H6 ¾C¾
6H ¾ ¾ AlCl
5COCl ¾¾ 3
® C6 H5 — CO — C6 H5 + HCl
K 2Cr2O7 / H 2SO4 D Benzophenone

(f) Conversion of bromobenzene to 1-phenylethanol


OH
Mg/ dry ether
|
( i ) CH 3 CHO
C 6 H 5 Br ¾¾¾¾¾¾®
Grignard reaction
C 6 H 5 MgBr ¾¾¾¾®
+
C 6 H 5 — CH — CH 3
Bromobenzene Phenyl magnesium ( ii ) H 3 O 1-Phenyl ethanol
bromide
155

(g) Conversion of benzaldehyde to 3-phenylpropan-1-ol


CHO CH==CHCHO CH2CH2CH2OH

dil. NaOH 2H /Ni


+ CH3CHO ¾¾¾¾¾¾® ¾¾¾®
Cross aldol Reduction
condensation
Benzaldehyde 3-phenyl prop-2-ene-1-ol 3-phenyl propan-1-ol

(h) Conversion of benzaldehyde to a-hydroxy phenylacetic acid


OH OH
| H + /H 2O
|
NaCN / HCl
C 6 H 5CHO ¾¾¾®
pH, 9-10
C 6 H 5 — CH — CN ¾¾¾¾® C 6 H 5 — CH — COOH
Hydrolysis
Benzaldehyde Benzaldehyde cyanohydrin a-Hydroxyphenyl acetic acid
(Mandelic acid)

(i) Conversion of benzoic acid to m-nitrobenzyl alcohol


COOH COOH COCl CH2OH

conc. HNO + H SO , heat


3 2 4
SOCl2 (i) NaBH
¾¾¾¾¾¾¾¾¾ ¾® ¾¾¾® 4
¾¾¾¾¾¾¾®
SO2, –HCl
(ii) H3O+, reduction
NO2 NO2 NO2
Benzoic acid
m-nitrobenzoic acid m-nitrobenzyl chloride m-nitrobenzyl alcohol
Q.16 Describe the following :
(a) Acetylation (b) Cannizzaro reaction (U.P. 2020)
(c) Cross aldol condensation (U.P. 2019) (d) Decarboxylation (U.P. 2018)
Ans. (a) Acetylation : Acetylation reaction is that in which compounds containing functional groups like ¾OH, ¾ NH 2 ,
¾ NH, ¾SH etc. react with acetyl chloride to give acyl derivative. Amount of HCl formed in the reaction may be used
to determine the number of active H-atoms.
(b) Cannizzaro’s reaction : Aldehydes lacking a-H (e. g ., HCHO, C 6 H 5CHO) disproportionates when allowed to
react with NaOH to give acid and aldehyde. This reaction is called Cannizzaro reaction.
NaOH
2HCHO ¾¾¾® HCOONa+CH 3OH
NaOH
2C 6 H 5CHO ¾¾¾® C 6 H 5COONa+C 6 H 5CH 2OH
(c) Cross aldol condensation : When two different aldehydes/ketone molecules take part in aldol condensation,
then it is called Cross aldol condensation.
NaOH
CH 3CHO+CH 3CH 2CHO ¾¾¾® CH 3CH 2CH(OH)CH 2CHO +CH 3CH(OH)CHCHO
½
CH 3
+ CH 3CH(OH)CH 2CHO+CH 3CH 2CH(OH)CHCHO
½
CH 3
(d) Decarboxylation : When a carboxylic acid is heated with soda lime, it forms alkane and CO 2 is liberated. This
reaction is called decarboxylation.
Sodalime/ D CaO/ D
CH 3COOH ¾¾¾¾® CH 4 + CO 2 ; CH 3COONa+ NaOH ¾¾¾® CH 4 + Na 2CO 3
D D
C 6 H 5COOH+CaO ¾¾® C 6 H 6 + CaCO 3 ; C 6 H 5COONa+ NaOH ¾¾® C 6 H 6 + Na 2CO 3
Q.17 Complete each synthesis by giving missing starting material, reagent or products :
CH2CH3 COOH
KMnO4 SOCl2
(a) ¾¾¾¾® (b) ¾¾®
Heat
KOH, heat
COOH
O

C
H 2 NCONHNH 2
(c) C 6 H 5CHO ¾ ¾ ¾ ¾ ¾¾® (d) ¾®
156

O (f) Cyanohydrin formation occurs at the aldehyde


[Ag(NH ) ]+
3 2
(e) ¾¾¾¾® group.
OH
CHO
CHO CHO CH—CN
NaCN/HCl
(f) ¾¾¾¾® NaCN/HCl
¾¾¾¾®
COOH COOH COOH
Dil.NaOH 2-Formyl benzoic acid 2-[1-Hydroxycyanomethyl]
(g) C 6 H 5CHO + CH 3CH 2CHO ¾ ¾ ¾¾® benzoic acid
4 (i) NaBH CH3
(h) CH 3COCH 2COOC 2 H 5 ¾ ¾ ¾+¾
¾ ®
(ii) H
Dil. NaOH
3 CrO (g) C6H5CH O + H2 C—CHO ¾¾¾¾¾®
Claisen-Schmidt
(i) OH ¾¾® Benzaldehyde Propanal condensation

CH3
(j) CH2 ¾® CHO 3 2 1
C6 H5 CH==C—CHO + H 2O
2-Methyl-3-phenyl-prop-2-enal
(i) O
3
(k) ¾¾¾¾®
(ii) Zn-H O 2 O (h) Only keto group is reduced by NaBH 4 .
2

O
Ans. (a) Side chain is oxidised to —COOH. ||
(i) NaBH 4
CH2CH3 COOK CH 3 — C — CH 2COOC 2 H 5 ¾ ¾ ¾+¾ ¾®
(ii) H
KMnO4 Ethyl -3-oxobutanoate
¾¾¾¾®
KOH, heat OH
|
Ethylbenzene Pot. benzoate
CH 3 —CH — CH 2COOC 2 H 5
Ethyl -3-hydroxy butanoate
COOH COCl
SOCl2 CrO3–H2SO4
(b) ¾¾¾®
Heat (i) OH ¾¾¾¾¾® O
COOH COCl Cyclohexanol Cyclohexanone
Phthalic acid Phthaloyl chloride

(c) More nucleophilic NH 2 NH part of H 2 NHNCONH 2 B2H6/THF


(j) CH2 CH B ¾
reacts of form semicarbazone.
Methylene 3
C 6 H 5CHO + H 2 NNHCONH 2 ¾® cyclohexane
Benzaldehyde Semicarbazide H2O2 NaOH
C5H5N+HCrO3Cl–(PCC)
C 6 H 5CH == NNHCONH 2 + H 2O CH2OH
Benzaldehyde semicarbazone

COCl
Anhyd. AlCl 3
(d) + ¾¾¾¾¾¾¾¾® CHO
Friedel-Crafts acylation
Cyclohexane
Benzene Benzoyl chloride carbaldehyde
O
(i) O3
C (k) (ii) Zn–H2O 2 O

+ HCl Cyclohexylidene cyclohexane Cyclohexanone


Q.18 Give plausible explanation for each of the following :
Benzophenone (a) Cyclohexanone forms cyanohydrin in good yield
(e) Only aldehydes are oxidised by Tollens’ reagent. but 2,2,6-trimethylcyclohexanone does not.
O O (b) There are two NH 2 groups in semicarbazide.
[Ag(NH3)2]+ However, only one is involved in the formation of
¾¾¾¾®
Oxidation semicarbazones.
CHO COO–
4-Oxocyclohexane 4-Oxocyclohexane
(c) During the preparation of esters from a
carbaldehyde carboxylate anion carboxylic acid and an alcohol in the presence of
157

an acid catalyst, the water or the ester should be ethanoic and propanoic acid. Write the possible
removed as soon as it is formed. structure of the compound.
O Ans. Let us calculate the molecular formula of the
HO CN
compound.
HCN % of O = 100 - 69.77 - 11.63 = 18.60
Ans. (a)
Divide the % of each element by its atomic mass.
Cyclohexanone Cyclohexanone
cyanohydrin 69.77 11.63 18.6
C: = 5.88 , H: = 11.63 , O: = 1.16
12 1 16
Due to steric hindrance.
Now divide each value by the minimum value 1.16.
O
5.88 11.63 1.16
H3C C: = 5 , H: = 10 , O: =1
a CH3 1.16 1.16 1.16
a HCN
CH3 ¾¾® No reaction The empirical formula is C 5 H10O.
2,2,6-Trimethylcyclohexanone Empirical mass = 5 ´ 12 + 10 ´ 1 + 1 ´ 16 = 86
(b) Because NH 2 gp. attached to —NH —gp. does Molecular mass given = 86
not take part in resonance. Therefore, molecular mass of the compound = 86 and
O O- molecular formula C 5 H10O.
.. || .. .. + | .. .. The given reactions suggest that :
H 2 N— C — NH— NH 2 ¬® H 2 N == C — NH— NH 2
(i) Since the compound forms an addition compound
(c) Esterification is a reversible process. with sodium hydrogen sulphite, the compound may
¾®
R — COOH + R ¢OH ®¾ RCOOR ¢ + H 2O be an aldehyde or ketone.
Ester
One of the product ester or water should be removed (ii) Since the compound does not reduce Tollens’
from the reaction mixture to prevent the hydrolysis of reagent it cannot be aldehyde. Therefore, the
the ester in backward direction. compound is a ketone.
Q.19 An organic compound contains 69.77% carbon, (iii)Since the compound gives +ve iodoform test, it
11.63% hydrogen and rest oxygen. The molecular contains the group CH 3 — CO (methyl ketone).
mass of the compound is 86. It does not reduce (iv) Since the compound on oxidation gives a mixture
Tollens’ reagent but forms an addition compound of ethanoic acid and propanoic acid, the compound is
with sodium hydrogen sulphite and gives positive CH 3COCH 2CH 2CH 3 . This formula explains all the
iodoform test. On vigorous oxidation, it gives reactions.

OH
|
[O] NaHSO3
CH 3COOH + CH 3CH 2COOH ¬K¾Cr¾O¾ ¾ ¾ ¾ CH 3COCH 2CH 2CH 3 ¾ ¾ ¾¾ ® CH 3 —C — CH 2CH 2CH 3
2 2 7 + H 2 SO4
|
SO 3 Na
CH 3COCH 2CH 2CH 3 + 3I 2 + 4NaOH ¾® CHI 3 + CH 3CH 2CH 2COONa + 3NaI + 3H 2O
Q.20 Although phenoxide ion has more number of resonating structures than carboxylate ion, carboxylic acid is a stronger
acid than phenol. Why ?
Ans. Because carboxylate ion is more stable than phenoxide ion.
O– O O O O– O –d(1/4)
(1/4)
– – –d –d(1/4)


–d
(1/4)

O O– O–d(1/2)
R C R C R C
O– O O–d(1/2)
In carboxylate ion – charge is equally distributed on two electronegative O-atoms.
158

Chapter

13 ORGANIC COMPOUNDS
CONTAINING NITROGEN
Syllabus : Amines: Nomenclature, classification, structure, methods of preparation of ethyl amine and aniline, physical and
chemical properties, uses, distinction among primary, secondary and tertiary amines.
*Cyanides and Isocyanides: Will be studied at suitable places as reference.
*Diazonium salt: Methods of preparation, Chemical reactions and synthetic, Synthetic, Importance in
Organic Chemistry.
* Topics Deleted for Examination 2021

Objective Questions
1. Product obtained when nitrobenzene is reduced in (c) CH 3 — NH — CH 3 (d) C 6 H 5 NH 2
strong acidic medium is : (U.P. 2017) 6. The aqueous solution of amines is : (U.P. 2019)
(a) aniline (a) acidic (b) basic
(b) phenyl hydroxyl amine (c) amphoteric (d) neutral
(c) p-aminophenol 7. The reagent used to prepare amine from amide is :
(d) azobenzene (U.P. 2019)
2. Major product of the reaction is : (a) Br2 KOH (b) NaOH CaO
Br KOH
C 6 H 5CONH 2 ¾ ¾2 ¾¾® (U.P. 2017) (c) HCl ZnCl 2 (d) K 2Cr2O7 H 2SO 4
(a) C 6 H 6 (b) C 6 H 5CHO 8. Which of the following compounds reduces Tollen’s
(c) C 6 H 5 NH 2 (d) C 6 H 5COOK reagent ? (U.P. 2019)
(d) both aliphatic and aromatic 1º amines (a) C 6 H 5 NO 2 (b) C 6 H 5 NO
3. Which of the following will not give primary amines? (c) C 6 H 5 NH 2 (d) C 6 H 5 NHOH
(a) CH 3CN ¾ ¾ ¾
LiAlH 4
¾® 9. The reagent used to convert amides into amines is:
LiAlH (U.P. 2020)
(b) CH 3 NC ¾ ¾ ¾
¾4
® (a) Br2 /KOH (b) K 2Cr2O7 /H 2SO 4
LiAlH
(c) CH 3CONH 2 ¾ ¾ ¾
¾4
® (c) HCl/ZnCl 2 (d) NaOH/Ca(OH) 2
(d) CH 3CONH 2 ¾ ¾2 ¾¾®
Br KOH 10. Most alkaline amine is : (U.P. 2020)
(a) C2 H5 NH2 (b) (CH3 ) 2 NH
4. The reaction between
R - NH 2 + CHCl 3 + KOH ( alc ) is known as : (c) CH3 NH2 (d) (C2 H5 ) 2 NH
(U.P. 2019) 11. (CH3 ) 3 — N is a: (U.P. 2020)
(a) Coupling reaction (a) Primary amine (b) Secondary amine
(b) Carbylamine reaction (c) Tertiary amine (d) None of these
(c) Hoffmann bromamide reaction 12. Which one of the following molecules contains one
(d) Schmidt reaction lone pair of electrons on the central atom?
5. Which of the following amines does not give (U.P. 2020)
carbylamine reaction ? (U.P. 2019) (a) Cl 2 (b) CH4 (c) CHCl 3 (d) NH3
(a) CH 3CH 2 NH 2 (b) CH 3 NH 2

Answers
1. (a) 2. (c) 3. (b) 4. (b) 5. (c) 6. (b) 7. (a) 8. (d) 9. (a) 10. (d)
11. (c) 12. (d)
159

Very Short Answer Type Questions


Q.1 Arrange CH 3 NH 2 ,(CH 3 ) 2 NH,(CH 3 ) 3 N and NH 3 in p-H 2 N — C 6 H 4 — NH 2 , m-H 2 N — C 6 H 4 — NH 2 ,
decreasing order of their basic nature. (U.P. 2017) C 6 H 5 NH 2 (U.P. 2017)
Ans. (CH 3 ) 2 NH > CH 3 NH 2 > (CH 3 ) 3 N > NH 3 Ans. p-H 2 N — C 6 H 4 — NH 2 > m-H 2 N — C 6 H 4 — NH 2
Q.2 Arrange following in decreasing order of their basic > C 6 H 5 NH 2
strength:
Short Answer Type Questions
Q.1 Methyl amine is more basic than ammonia, explain. Ans. (i) C 6 H 5 NH 2 is less basic than aliphatic amine like
(U.P. 2005) CH 3 NH 2 as lone pair of electrons of nitrogen atom of
Ans. Basic nature of amines and ammonia is due to —NH 2 group participates in resonance with benzene
presence of lone pair of electrons on nitrogen atom. nucleus. Hence, its availability for protonation is less
In methyl amine + I effect of methyl group (—CH 3 ) than in CH 3 NH 2 .
increases electron density on nitrogen atom. As a + + +
result nitrogen atom can make a bond with proton by NH2 NH2 NH2 NH2

donating electron pair. On the other hand + I effect in –
not produced in NH 3 due to absence of alkyl group.
H H –
CH3—N—H
> ; H—N—H
> The solubility of CH 3 NH 2 is higher than C 6 H 5 NH 2 in
Methyl amine Ammonia water due to its ability to form hydrogen bonds with
Q.2 Ethyl amine is stronger base than methyl amine, water.
why? (ii) ArN +2 Cl - + H 3 PO 2 + H 2O ¾®
Ans. + I effect of ethyl amine is greater than that of methyl ArH + N 2 + H 3 PO 3 + HCl
amine. Hence, electron density on nitrogen atom is Q.5 Write chemical equation for the formation of
greater in ethyl amine. Thus, nitrogen atom in ethyl N,N-diethyl ethanamine from chloroethane.
amine has greater tendency to donate lone pair of (U.P. 2019)
electrons for bond formation. CH 3 CH 3
Q.3 Write notes on the following : Ans. CH 3CH 2Cl + HN ¾¾® CH 3CH 2 N
(i) Gattermann reaction CH 3 - HCl CH 3
N, N-dimethyl ethanamine
(ii) Sandmeyer’s reaction (U.P. 2015)
Ans. (i) Gattermann reaction : In this reaction, Q.6 Write chemical equation for the preparation of
benzene diazonium chloride is treated with copper aniline by Hoffman Bromamide reaction and also
powder and halogen acid (HCl or HBr) to form the write chemical equations of the reactions taking place
corresponding haloarenes as the product. between aniline with acetyl chloride and aniline with
+
N 2Cl– Cl carbon disulphide. (U.P. 2019)
Ans. (i) Preparation of aniline by Hoffmann
Cu powder bromamide reaction :
+HCl ¾¾¾¾® +N2
CONH2 NH
Chlorobenzene 2
Benzene
diazonium chloride
Br + 4 KOH
(ii) Sandmeyer’s reaction : In this reaction ¾ ¾2 ¾ ¾
¾® + 2KBr + K 2CO 3 + 2H 2O
benzene diazonium chloride is treated with halogen Benzamide
Aniline
acid in the presence of cuprous halides to form the
corresponding haloarenes as the product. (ii) Reaction of aniline with acetyl chloride :
+ NH2 NHCOCH3
N 2Cl– X
Cu X
2 2
¾¾¾® +N2 + CH 3COCl ¾® + HCl
HX

Q.4 (i) CH 3 NH 2 is a stronger base than C 6 H 5 NH 2 , why? (iii) Reaction of aniline with CS 2 :
Write notes on the solubility of CH 3 NH 2 and NH2 S
C 6 H 5 NH 2 in water. (U.P. 2015) ||
KOH(alc.)
(ii) Complete the following reaction 2 + CS 2 ¾¾¾® ¾ NH ¾ C ¾ NH ¾
ArN +2 Cl - +....+ H 2O ¾® ArH + N 2 +....+ HCl D /- H S 2
Diphenyl thiourea
(U.P. 2015)
160

Long Answer Type Questions


Q.1 Define amines. What are primary, secondary and Or Write down the chemical equation for the
tertiary amines? Give an example of each. preparation of Ethyl amine and also write the
(U.P. 2015, 17) chemical equation of its reaction with the following:
Ans. The alkyl derivatives of ammonia are called amines (U.P. 2020)
which are obtained by replacement of one or more (i) Chloroform and alcoholic KOH
hydrogen atoms of ammonia by same number of alkyl (ii) Nitrous acid
group (— R ). Their general formula is C n H 2n+1 NH 2 . Ans. Hoffmann bromamide reaction : It is a
NH 3 ¾ –H
¾® R. NH 2 laboratory method of preparation of ethyl amine.
+R (Primary amine) When amide is heated with the mixture of bromine
Amines in which the nitrogen atom is directly bonded and caustic potash, primary amines are obtained and
to one or more alkyl groups are called aliphatic reaction is called Hoffmann bromamide
amines, e. g ., reaction. Ethyl amine is prepared by Hoffmann
bromamide method.
CH 3 CH 3
| | For example :
··
CH 3 NH 2 ; CH 3 — NH ; CH 3 —N — CH 3 C 2 H 5CONH 2 + Br2 + 4KOH ¾®
·· ·· Propanamide

Classification of aliphatic amines : Aliphatic C 2 H 5 NH 2 + 2KBr + K 2CO 3 + 2H 2O


Ethyl amine
amines are of three types :
(1) Primary amines or 1º amines : These are
obtained by replacing one hydrogen atom of ammonia
by an alkyl group. Primary amine has the functional
group —NH 2 (amine gr.)
For example :
NH 3 ¾ –H
¾® RNH 2 , e. g ., CH 3 NH 2
+R Methyl amine

(2) Secondary amines or 2º amines : Secondary


amines are formed by replacing two hydrogen atoms
of ammonia by two alkyl groups. Secondary amine has
the functional group.
NH
Reactions :
Imino group
(1) Reaction with acetyl chloride : Ethyl amine
For example :
NH 3 ¾ –2H
¾ ¾® R 2 NH; e. g ., (CH 3 ) 2 NH reacts with acetyl chloride to form N-ethyl acetamide
+2R Sec. amine Dimethyl amine C 2 H 5 NH 2 + ClCOCH 3 ¾® C 2 H 5 NHCOCH 3 + HCl
Ethyl amine Acetyl chloride N - ethyl acetamide
(3) Tertiary amines or 3º amines : Tertiary
amines are formed by the replacement of all the three (2) Reaction with Grignard’s reagent : Ethyl
hydrogen atoms of ammonia by three alkyl group. amine reacts with Grignard reagents to form alkane
Tertiary amine has the functional group. ® N (Tertiary methane.
nitrogen or tertiary amino gp.) C2H5NH
NH 3 ¾ –3H
¾¾® R 3 N; e. g ., (CH 3 ) 3 N C2H5NH2+CH3MgBr ¾¾® CH4+Mg
+3R Grignard Methane Br
Trimethyl amine reagent
Q.2 What do you understand by Hoffmann bromamide (3) Reaction with nitrous acid (HNO 2 ) : Ethyl
reaction? How will you prepare ethyl amine from this alcohol and nitrogen gas are formed.
method? Give equations. C 2 H 5 NH 2 + HNO 2 ¾® C 2 H 5OH + N 2 + H 2O
Or Describe laboratory method of preparation of Ethyl amine Ethyl alcohol
ethyl amine with proper diagram and chemical (4) Reaction with CHCl 3 /KOH:
equations. How does ethyl amine react (i) With acetyl D
C 2 H 5 NH 2 + CHCl 3 + 3KOH ¾¾ ®
chloride ?
C 2 H 5 NC + 3KCl + 3H 2O
(ii) With Grignard’s reagent ? (U.P. 2015, 19)
161

Q.3 Write chemical equation for the formation of N, Q.5 Write down the chemical equation for the preparation
N-dimethyl phenyl methanamine from benzyl of ethylamine by Schmidt reaction and also write the
chloride. (U.P. 2019) chemical equations for the reactions of ethylamine
Or (i) How would you prepare Aniline from with acetaldehyde and nitrous acid. (U.P. 2019)
chlorobenzene ? (U.P. 2019) Or Write the chemical equation for the preparation of
(ii) Discuss the solubility of aliphatic amines (1º ,2º methylamine by Schmidt reaction. Write chemical
and 3º C ). (U.P. 2019) equations for the reactions taking place on heating (i)
Ans. Dimethylamine reacts with benzyl chloride to form Methylamine and (ii) Aniline with chloroform in the
N,N-dimethyl phenyl methanamine. presence of alcoholic KOH. (U.P. 2020)
CH3 Ans. Schmidt reaction : When carboxylic acids react
CH2Cl CH2N with hydrazoic acid (N 3 H) in presence of conc.
CH3
CH3 H 2SO 4 , p-amines are formed and this reaction is
+ HN ¾¾®
CH3 - HCl called Schmidt reaction.
Conc.H SO
2 4
RCOOH + N 3 H ¾¾¾¾® RNH 2 + CO 2­ + N 2­ By
(i) Aniline from chlorobenzene : When
chlorobenzene is heated with ammonia solution in using Schmidt reaction, ethylamine may be prepared
presence of cuprous oxide at high (P) and 200ºC, by action of hydrazoic acid on propanoic acid.
aniline is obtained. 2
CH 3CH 2COOH + N 3 H ¾¾¾¾®4Conc.H SO
CH 3CH 2 NH 2
2C 6 H 5Cl + 2NH 3 + Cu 2O ¾® 2C 6 H 5 NH 2 Ethylamine
+2CuCl + H 2O
+CO 2 + N 2
(ii) Solubility of 1º, 2º,3º aliphatic amines :
Conc. H 2 SO4
1º,2º,3º aliphatic amines form hydrogen bond so CH 3COOH + N 3 H ¾¾¾¾® CH 3 NH 2 + CO 2 + N 2
lower aliphatic amines are water soluble but as the Methylamine
size of alkyl group (hydrophobic part) increases, the Reaction of ethylamine with
solubility decreases. However, they are soluble in (i) Reaction with acetaldehyde :
organic solvents like C 6 H 6 , ether etc. H
H |
| C2H5N H 2+O C ¾ CH 3 ¾¾®
- H 2O
R ¾ N ¾ H - - - H ¾ O- - -
| C 2 H 5 N == CH ¾ CH 3
H Ethylidine ethylamine
Q.4 Write chemical equation for the reaction of excess (ii) Reaction with nitrous acid :
Chloroethane with Ethanamine and Chloroethane C 2 H 5 NH 2 + HNO 2 ¾® C 2 H 5OH + N 2 + H 2O
with Benzylamine. (U.P. 2020) Ethyl alcohol
D
Ans. CH 3CH 2 NH 2 + 2CH 3CH 2Cl ¾® (CH 3CH 2 ) 3 N CH 3 NH 2 + CHCl 3 + 3KOH ¾¾ ® CH 3 NC + 3KCl + 3H 2O
Excess Triethyl amine D
C 6 H 5 NH 2 + CHCl 3 + 3KOH ¾¾® C 6 H 5 NC + 3KCl + 3H 2O
C 6 H 5CH 2 NH 2 + 2CH 3CH 2Cl ¾®
C 6 H 5CH 2 N(C 2 H 5 ) 2
N,N -diethyl benzylamine

Question Based on Chemical Reactions


Q.1 Complete the following reactions : (ii) Ethylamine reacts with NaNO 2 and HCl.
Conc.H 2SO4 (U.P. 2013)
(i) C 2 H 5COOH + A ¾¾¾¾® C 2 H 5 NH 2 + N 2 + CO 2
(U.P. 2016) (iii) Ethylamine reacts with Grignard’s reagent.
Br KON HNO2
(ii) CH 3CONH 2 ¾ ¾2 ¾¾® A ¾ ¾¾ ® (U.P. 2013)
Ans. (i) C 2 H 5 NH 2 + CS 2 + HgCl 2 ¾® C 2 H 5 NCS
B + N 2 + H 2O Ethyl isothiocyanate
(U.P. 2016, 17)
+ HgS + 2HCl
Ans. (i) A = N 3 H (ii) A = CH 3 NH 2 , B = CH 3OCH 3
NaNO2 / HCl
Q.2 Giving chemical equations, mention what happens (ii) C 2 H 5 NH 2 ¾¾¾¾® C 2 H 5OH
when ? (iii) C 2 H 5 NH 2 + RMgX ¾® RH + C 2 H 5 NHMgX
(i) Ethylamine is heated with CS 2 and mercuric
chloride. (U.P. 2008)
162

Q.3 How will you obtain (Give chemical equations only) : Ans. (i) A = HNO 2 , (ii) B = Br2 KOH ,

(i) Methylamine from ethylamine ? (U.P. 2017) N+
2Cl
(ii) Ethylamine from ammonium acetate?
(U.P. 2010) (iii) C = H 2O H + (iv) D =
HNO2 [O ]
Ans. (i) C 2 H 5 NH 2 ¾¾® C 2 H 5OH ¾® CH 3COOH
| NH /D (v) A = C 6 H 5 NH 2 , B = C 6 H 5 N 2Cl, C = C 6 H 5OH,
3
¯
Br / NaOH ( NO 2 ) 3
D = C 6H 2ƒ
2
CH 3 NH 2 ¬¾¾¾¾ CH 3CONH 2
‚
D P2O5 NH 2
(ii) CH 3COONH 4 ¾® CH 3CONH 2 ¾¾¾®
-H O 2 (vi) A = C 6 H 5CN, B = C 6 H 5COOH
4 [H ] (vii) A = C 6 H 5 NH 2 , B = C 6 H 5 N 2Cl, C = C 6 H 5OH
CH 3CN ¾¾¾® CH 3CH 2 NH 2
Reduction (viii) A = C 6 H 5 N 2Cl, B = C 6 H 5Cl
Q.4 Write the formula and name of A, B and C in the (ix) A = C 6 H 5 NH 2
following reactions: (U.P. 2014) Q.6 Complete the following reactions.
+
OH (i) Ar — N ºº N — Cl - + A ¾ Cu
¾¾ CN
® B + N 2 + KCl
Anhydrous ZnCl 2 HNO PCl
(i) + NH3 ¾ ¾ ¾ ¾ ¾ ¾
¾® ( A ) (ii) CH 3 NH 2 ¾ ¾¾
2
® A ¾ ¾¾
5
®
B ¾ KCN
¾¾® C ¾ Sn
¾HCl
¾¾® D
NH2 N == NCl Cl (U.P. 2014)
+
( B) ( C) Ans. (i) Ar — N ºº N — Cl + KCN ¾ CuCN
¾¾®
(ii) ¾¾
¾® ¾¾
¾® ( A)
0-5°C
C 6 H 5CN + N 2 + KCl
( B)
NH2
HNO2 PCl 5
(ii) CH 3 NH 2 ¾ ¾¾® CH 3OH ¾ ¾¾® CH 3Cl
Ans. (i) ( A ): ( A) ( B)
KCN Sn HCl
¾ ¾¾® CH 3CN ¾ ¾ ¾
¾® CH 3CH 2 NH 2
( C) ( D)
(ii) ( B): NaNO 2 HCl
Q.7 Complete the following reactions.
(C ): Cu 2Cl 2 HCl
NO2
Q.5 Write the name and formula of A, B, C and D in the
following reactions. (U.P. 2015) Fe/HCl2 2 NaNO /HCl H O
A B (i) ¾¾¾® A ¾¾¾¾®
0ºC B ¾¾®
Heat C
(i) RNH 2 ¾¾ ® ROH (ii) RCONH 2 ¾¾ ® RNH 2
NH2
(ii) Ethylamine + Phosgene ¾® A + 2HCl
C NaNO2 HCl (U.P. 2014)
(iii) RCN ¾¾® RCOOH (iv) ¾¾ ¾¾® D
0- 5°C
Cl NH2 N==N+Cl–
Br KOH NaNO HCl A B
(v) C 6 H 5CONH 2 ¾ ¾2 ¾¾® A ¾ ¾0¾ 2
¾¾® B
- 5°C (iii) ¾¾® ¾¾®
Conc.H SO conc.HNO
¾ Steam
¾¾® C ¾ ¾ ¾ ¾ 2 4
¾ ¾¾¾ 3
¾ ®D NO2 NH2
(U.P. 2017)
P2O5 H 2O Fe/HCl NaNO /HCl
2
(vi) C 6 H 5CONH 2 ¾ ¾¾® A ¾ ¾¾ + ® B (U.P. 2017) Ans. (i) ¾¾¾® ¾¾¾¾®
0ºC
H
HCl NaNO HCl conc. (A)
(vii) C 6 H 5 NO 2 ¾ Fe
¾¾ ¾® A ¾ ¾ ¾20¾
°C
¾® B
¾¾
H O H+ +
¾ ¾2 ¾¾® C (U.P. 2017) N 2Cl OH
NaNO2 HCl CuCN KCN
(viii) C 6 H 5 NH 2 ¾ ¾ ¾
0°C
¾¾® A ¾ ¾ ¾ ¾ ¾® B H O
2
¾¾®
heat (Phenol)
(U.P. 2017) (B) (C)
6 [H]
(ix) C 6 H 5 NO 2 ¾Sn¾HCl
¾® A + 2H 2O (U.P. 2017)
163

(ii) C 2 H 5 NH 2 + COCl 2 ¾® C 2 H 5 NCO + 2HCl 2 5 PO 4 [H ]


Ethylamine Phosgene Ethyl isocyanate
CH 3CONH 2 ¾¾® CH 3 N ¾¾® CH 3CH 2 NH 2
- H 2O Sn / HCl

(iii) A = NH 3 , B = HNO 2 or NaNO 2 /HCl (0-5°C) Q.12 How will you convert: (U.P. 2018)
Cl NH2 N2Cl (i) propanamide into ethylamine
NH3 NaNO2/HCl (ii) aniline into acetanilide
–HCl 2 Br / KOH
Ans. (i) CH 3CH 2CONH 2 ¾¾¾® CH 3CH 2 NH 2
Q.8 Write the formulae and names of A,B,C and in the NH2 NHCOCH3
following reactions :
CH3COCl
A
(i) RNH 2 ¾¾ ® ROH (ii) –HCl
B
(ii) RCONH 2 ¾¾ ® RNH 2
Q.13 Write chemical equation to convert benzene
C
(iii) RCN ¾¾ ® RCOOH sulphonic acid into aniline. Write chemical equations
NH2 of carbylamine and diazotization reactions of aniline.
(U.P. 2020)
NaNO /HCl
(iv) 2
¾¾¾¾¾®
0–5ºC D (U.P. 2015) SO3H SO2O–Na+ NH2
NaOH NaNH2/fusion
Ans. (i) A = HNO 2 (nitrous acid) Ans.
(ii) B = Br2 KOH (Bromine and caustic potash) Benzene Aniline
(iii) C = dil. HCl (or H 3O + ) sulphonic acid

(iv) Deleted for examination 2021.


NH2
Q.9 Identify A in the following reaction. (U.P. 2018)
C 2 H 5 NH 2 + CS 2 + HgCl 2 ¾® A + HgS + 2HCl
+CHCl 3 +3NaOH ¾®
Ans. A = C 2 H 5 NCS
C 2 H 5 NH 2 + CS 2 + HgCl 2 ¾® C 2 H 5 NCS + HgS + 2HCl
C 6 H 5 NC + 3NaCl + 3H 2O
Q.10 Identify A and B in the following reaction. Phenyl
isocyanide
(U.P. 2018)
HNO2 +
B
C 2 H 5 NH 2 ¾¾® A ¾¾® CH 3COOC 2 H 5 NH2 N2 Cl–

Ans. A = C 2 H 5OH, B = CH 3COOH NaNO2/HCl


HNO CH3COOH 0-5°C
2
C2H5NH2 ¾¾® C2H5OH
Q.14 Complete the following reaction and write the names
CH3COOC2H5 and formulae of A, B, C : (U.P. 2020)
A PCl 5
Q.11 Identify compounds A and B in the following CH3 NH2 ¾¾® CH3 OH ¾ ¾¾®
reactions and write the reactions also. (U.P. 2018) B ¾ KCN
¾¾® C ¾ Sn
¾HCl
¾
¾® C2 H5 NH2
P2O5 4 [H ]
CH 3CONH 2 ¾¾® A ¾¾® B Ans. (A) HNO 2
Sn / HCl
(B) CH 3Cl
Ans. A = CH 3CN, B = CH 3CH 2 NH 2
(C) CH 3CN
164

Solution of NCERT Text Book Problems


Q.1 Write IUPAC names of the following compounds and (iii) Aq. solution of methylamine furnishes OH – ions to
classify them into primary, secondary and tertiary precipitize Fe 3+ as Fe(OH) 3 .
amines. (iv) In the presence of nitrating mixture, the ¾ NH 2
(i) (CH 3 ) 2 CHNH 2 (ii) CH 3 (CH 2 ) 2 NH 2 +
(iii) CH 3 NHCH(CH 3 ) 2 (iv) (CH 3 ) 3 C ¾ NH 2 group gets protonated to form ¾ N H 3 . The
+
(v) C 6 H 5 NHCH 3 (vi) (CH 3CH 2 ) 2 NCH 3 ¾ N H 3 group is electron withdrawing. It is meta
(vii) m- BrC 6 H 4 NH 2
directing and deactivates the benzene ring.
Ans. (i) Propan-2-amine (1º),
(v) Aniline is a Lewis base and react with AlCl 3 which
(ii) Propan-1-amine (1º),
is Lewis acid.
(iii) N-Methylpropan-2-amine (2º) +
(iv) 2-Methylpropan-2-amine (1º), C 6 H 5 NH 2 + AlCl 3 ¾® C 6 H 5 NH 2 × AlCl -3
(v) N-Methylbenzenamine or N-methylaniline (2º) (vi) The positive charge on N-atom in diazonium ion
(vi) N-Ethyl-N-methylethanamine (3º) gets dispersed over the benzene ring due to
(vii) 3-Bromobenzenamine or 3-Bromoaniline (1º) resonance.
Q.2 Give one chemical test to distinguish between the (vii) Because it gives pure primary (1º) amine.
following pairs of compounds : Q.4 Arrange the following :
(i) Methylamine and dimethylamine (a) in decreasing order of pK b values : C 2 H 5 NH 2 ,
(ii) Secondary and tertiary amines C 6 H 5 NHCH 3 ,(C 2 H 5 ) 2 NH and C 6 H 5 NH 2 .
(iii) Ethylamine and aniline (b) in decreasing order of basic strength :
(iv) Aniline and benzylamine C 6 H 5 NH 2 ,C 6 H 5 N(CH 3 ) 2 ,(C 2 H 5 ) 2 NH and
(v) Aniline and N-methylaniline CH 3 NH 2
Ans. (i) Methylamine gives carbylamine test, while (c) increasing order of basic strength : (i) Aniline,
dimethylamine does not p-nitroaniline and p-toluidine,
CH 3 NH 2 + CHCl 3 + 3KOH ¾® CH 3 NC (ii) C 6 H 5 NH 2 , C 6 H 5 NHCH 3 , C 6 H 5CH 2 NH 2
Methyl isocyanide (d) decreasing order of basic strength in the gas
(offensive smell)
phase :
+ 3KCl + 3H 2O
C 2 H 5 NH 2 ,(C 2 H 5 ) 2 NH,(C 2 H 5 ) 3 N and NH 3
(ii) Secondary amines react with G.R. whereas tertiary (e) increasing order of boiling point :
amines do not. C 2 H 5OH, (CH 3 ) 2 NH,C 2 H 5 NH 2
(iii) Aniline gives azo dye test whereas ethylamine, (f) increasing order of solubility in water :
benzylamine and N-methylaniline do not. C 6 H 5 NH 2 ,(C 2 H 5 ) 2 NH, C 2 H 5 NH 2
Q.3 Account for the following : Ans. (a) (C 2 H 5 ) 2 NH > C 2 H 5 NH 2 > C 6 H 5 NHCH 3
(i) pK b of aniline is more than that of methylamine. > C 6 H 5 NH 2
(ii) Ethylamine is soluble in water whereas aniline is C 6 H 5 NH 2 > C 6 H 5 NHCH 3 > C 2 H 5 NH 2
not. > (C 2 H 5 ) 2 NH
(iii) Methylamine in water reacts with ferric chloride to (b) (C 2 H 5 ) 2 NH > CH 3 NH 2 > C 6 H 5 N(CH 3 ) 2
precipitate hydrated ferric oxide. > C 6 H 5 NH 2
(iv) Although amino group is o, p-directing in aromatic (c) (i) p-nitroaniline < aniline < p-toluidine
substitution reactions, aniline on nitration gives a (ii) C 6 H 5 NH 2 < C 6 H 5 NHCH 3 < C 6 H 5CH 2 NH 2
substantial amount of m-nitroaniline.
(d) (C 2 H 5 ) 3 N > (C 2 H 5 ) 2 NH > C 2 H 5 NH 2 > NH 3
(v) Aniline does not undergo Friedel-Crafts reaction.
(e) (CH 3 ) 2 NH < C 2 H 5 NH 2 < C 2 H 5OH
(vi) Diazonium salts of aromatic amines are more
(f) C 6 H 5 NH 2 < (C 2 H 5 ) 2 NH < C 2 H 5 NH 2
stable than those of aliphatic amines. Q.5 How will you convert ?
(vii) Gabriel phthalimide synthesis is preferred for (i) Ethanoic acid into methanamine
synthesising primary amines. (ii) Hexanenitrile into 1-aminopentane
Ans. (i) Because aniline is less basic than methylamine. (iii) Methanol to ethanoic acid
Therefore, its value of pK b is more. (iv) Ethanamine into methanamine
(ii) Because ethylamine forms hydrogen bonds with (v) Ethanoic acid into propanoic acid
water and dissolves in it. (vi) Methanamine into ethanamine
(vii) Nitromethane into dimethylamine
(viii) Propanoic acid into ethanoic acid.
165

NH 3 2 D Br / KOH
Ans. (i) CH 3COOH ¾¾® CH 3COONH 4 ¾¾®
-H O
CH 3CONH 2 ¾¾¾® CH 3 NH 2
2

HOH /H+ NH 3 / D
2 Br / KOH
(ii) CH 3 (CH 2 ) 4 CN ¾¾¾® CH 3 (CH 2 ) 3 CH 2COOH ¾¾® CH 3 (CH 2 ) 3 CH 2CONH 2 ¾¾¾® CH 3 (CH 2 ) 3 CH 2 NH 2
3 PCl NaCN HOH /H +
(iii) CH 3OH ¾¾® CH 3Cl ¾¾® CH 3CN ¾¾¾® CH 3COOH
2 HNO 3 [O]
2 NH /D Br / KOH
(iv) CH 3CH 2 NH 2 ¾¾® CH 3CH 2OH ¾¾® CH 3COOH ¾¾® CH 3CONH 2 ¾¾¾® CH 3 NH 2
[H] 5 PCl 2 KCN H O/H+
(v) CH 3COOH ¾¾® CH 3CH 2OH ¾¾® CH 3CH 2Cl ¾¾® CH 3CH 2CN ¾¾¾® CH 3CH 2COOH
( i ) HNO2 KCN PCl 5 [H ]
(vi) CH 3 NH 2 ¾¾¾® CH 3OH ¾¾® CH 3Cl ¾¾® CH 3CN ¾¾® CH 3CH 2 NH 2
( ii ) HOH

[H] 3 CH Cl
(vii) CH 3 NO 2 ¾¾® CH 3 NH 2 ¾¾¾®
– HCl
CH 3 NHCH 3
NH 3 /D 2 HNO [O]
(viii) CH 3CH 2COOH ¾¾¾® CH 3CH 2CONH 2 ¾® CH 3CH 2 NH 2 ¾¾® CH 3CH 2OH ¾¾® CH 3COOH
Q.6 Describe the method for the identification of primary, secondary and tertiary amines. Also write chemical equations of
the reactions involved.
Ans.
S.No. Test Primary amine Secondary amine Tertiary amine
1. Functional group Amino group (—NH2 ) Imino group ( NH) |
Tertiary nitrogen (— N )
|

2. Reaction with CHCl 3 and alc. KOH Foul smelling carbylamine is No reaction No reaction
formed.
3. Reaction with CS2 and MgCl 2 Alkyl isothiocyanate having No reaction No reaction
smell like mustard oil is formed.
4. Reaction with CH3COCl Amide derivative is formed. Amide is formed No reaction
5. Reaction with diethyl oxalate Solid oxamide is formed Liquid oxamic ester No reaction
is formed
6. Reaction with Hinsberg reagent N-alkyl sulphonamide is formed N, N-dialkyl sulphonamide is No reaction
which is soluble in an alkali. formed formed which is
insoluble in alkali.
7. Reaction with KMnO 4 Aldimine is formed. Tetra alkyl hydrazine is No reaction
formed
8. Reaction with HNO 2 Alcohol is formed with evolution Nitrosoamine is formed Alcohol and nitrosoamine are
of N 2 gas. formed.

Q.7. Write short notes on the following :


(i) Carbylamine reaction (U.P. 2014,15,19) (ii) Diazotisation,
(iii) Hofmann-bromamide reaction (U.P. 2015,18, 19) (iv) Coupling reaction (v) Ammonolysis, (vi) Acetylation,
(vii) Gabriel phthalimide synthesis.
Ans. (i) Carbylamine reaction : Ethyl amine (and all other primary amines) react with chloroform and alcoholic
potassium hydroxide to form foul smelling isocyanides. Due to foul smell of isocyanides, they can be easily detected.
This reaction is used to test the presence of —NH 2 group (i.e., p-amines). Secondary and tertiary amines do not give
this reaction.
D
C 2 H 5 NH 2 + CHCl 3 + 3KOH ¾¾ ® C 2 H 5 NC + 3KCl + 3H 2O
Chloroform Ethyl isocyanide
(ii) Diazotisation: When aniline reacts with nitrous acid, —NH 2 group is replaced by —N 2Cl group and a solid,
unstable diazo compound is formed. The reaction is named as diazotisation.
Aniline reacts with HNO 2 (Na NO 2 / HCl) at 0° C to form benzene diazonium chloride.
166

secondary, tertiary amines and quaternary


0-5°C ammonium salt is obtained. The reaction is named as
ammonolysis of alkyl halide.

C 2H 5I + NH 3 ¾® C 2 H 5 NH 2 + HI
Ethyl iodide Ethyl amine
0-5°C (1°amine)

C 2 H 5 NH 2 + C 2 H 5 I ¾® (C 2 H 5 ) 3 NH + HI
Diethyl amine
(2°amine)

(C 2 H 5 ) 2 NH + C 2 H 5 I ¾® (C 2 H 5 ) 2 N + HI
0-5°C Triethyl amine
(3° amine)

(C 2 H 5 ) 3 N + C 2 H 5 I ¾® (C 2 H 5 ) 4 N + I –
Tetraethyl
ammonium
At high temperature, phenol is formed in place of iodide
diazonium salt. (vi) Acetylation: Amines react with acetyl chloride
and acetic anhydride to form N-ethyl acetamide.
NaNO2/HCl C 2 H 5 NH 2 + CH 3COCl ¾® C 2 H 5 NHCOCH 3 + HCl
10°C Acetyl chloride N - ethyl acetamide

C 2 H 5 NH 2 + (CH 3CO) 2 O ¾® C 2 H 5 NHCOCH 3


(iii) Hofmann bromamide reaction: When Acetic N - ethyl acetamide
benzamide is treated with bromine in presence of anhydride

KOH, aniline is obtained. This reaction also occur +CH 3COOH


with NaOBr, thus, is also named as Hofmann (vii) Gabriel phthalimide reaction : Following
bromamide reaction. steps are involved in this reaction :
C 6 H 5CONH 2 + Br2 + 4KOH ¾® C 6 H 5 NH 2 + 2KBr (i) Phthalimide reacts with caustic potash to form
Benzamide
potassium phthalimide.
+ K 2CO 3 + 2H 2O (ii) On heating mixture of potassium phthalimide
(iv) Diazo coupling reaction: Diazonium salts and ethyl iodide, N-ethyl phthalimide is formed.
readily undergo coupling reaction with phenols, (iii) Hydrolysis of N-ethyl phthalimide yields ethyl
naphthols and aromatic amines to form highly amine and phthalic acid.
coloured azo compounds.
CO KOH
CO C2H5I
e.g., Benzene diazonium chloride couples with
NH NK
phenol in weakly alkaline solution to form –H2O
CO KI
CO
p -hydroxyazobenzene or with dimethyl aniline in OH
weakly acid solution to form p -dimethyl amino CO H HCl
COOH
azobenzene. N—C2H5 + C2H5NH2
2H2O
CO H COOH Ethyl amine
OH
+ – C6H5OH
N == N Cl ¾¾¾®

N-ethyl phthalimide Phthalic acid
OH

Q.8 Accomplish the following conversions :


N == N OH (i) Nitrobenzene to benzoic acid
(ii) Benzene to m-bromophenol
– C6H5N(CH3)2 (iii) Benzoic acid to aniline
N == N+Cl ¾¾¾¾®
Slightly acidic
(iv) Aniline to 2, 4, 6-tribromofluorobenzene
CH3 (v) Benzyl chloride to 2-phenylethanamine
N == N N (vi) Chlorobenzene to p-chloroaniline
CH3 (vii) Aniline to p-bromoaniline
(v) Ammonolysis : When mixture of ethyl iodide (viii) Benzamide to toluene
(or other alkyl halides) and alcoholic ammonia is (ix) Aniline to benzyl alcohol
heated in a closed tube, mixture of primary,
167

Ans.
NO2 NH2 N+
2Cl
– CN COOH

Sn/HCl NaNO2/HCl CuCN/HCl HOH/H+


(i) ¾¾® ¾¾¾® ¾¾¾® ¾¾®
[H] 0-5°C

NO2 NO2 NH2 N+


2Cl
– OH

HNO3/H2SO4 Br2/Fe NaNO2/HCl HOH


Sn/HCl
(ii) ¾¾¾® ¾¾® ¾¾® 0-5°C
Br Br Br Br

COOH CONH2 NH2

3NH /D 2Br /KOH


(iii) ¾¾® ¾¾¾®

NH2 NH2 N2BF4– F


Br Br NaNO2/HBF4
Br Br Br Br
Br2/H2O D
(iv) ¾¾¾® ¾¾¾® ¾®
0-5°C

Br Br Br
CH2Cl CH2CN CH2CH2NH2

KCN Sn/HCl
(v) ¾¾® ¾¾¾®
[H]

Cl Cl Cl

Conc. HNO3 Sn/HCl


(vi) ¾¾¾® ¾¾®
60-70°C 6[H]

NO2 NH2

NH2 NHCOCH3 NHCOCH3 NH2

CH COCl
3
Br2 HOH
(vii) ¾¾® ¾® ¾®
–HCl

Br Br
CONH2 COOH CH3

HNO
2 NaOH/CaO 3 CH Cl/AlCl
3
(viii) ¾¾® ¾¾¾® ¾¾¾®

NH2 N+
2Cl

CN CH2NH2 CH2OH

NaNO2/HCl KCN Reduction HNO2


(ix) ¾¾¾® ¾® ¾¾¾® ¾®
0-5°C LiAlH4

Q.9 Give the structures of A, B and C in the following reactions :


NaCN 2 OH - NaOH + Br CuCN 3 H 2O/H + NH
(i) CH 3CH 2 I ¾¾® A ¾¾¾¾¾® B ¾¾¾¾® C (ii) C 6 H 5 N 2Cl ¾¾® A ¾¾¾® B ¾¾® C
Partial hydrolysis D
168

KCN
4 2 LiAlH HNO Fe/HCl NaNO2 + HCl H 2O/H +
(iii) CH 3CH 2 Br ¾¾® A ¾¾¾® B ¾¾¾® C (iv) C 6 H 5 NO 2 ¾¾¾® A ¾¾¾¾¾® B ¾¾¾® C
0ºC 273 K
NH 3 NaOBr NaNO2 /HCl Fe/HCl HNO2
6 5 C H OH
(v) CH 3COOH ¾¾® A ¾¾® B ¾¾¾¾¾® C (vi) C 6 H 5 NO 2 ¾¾¾® A ¾¾¾® B ¾¾¾® C
D 273 K
Ans. (i) CH 3CH 2CN( A ), CH 3CH 2CONH 2 ( B), CH 3CH 2 NH 2 (C )
(ii) C 6 H 5CN( A ), C 6 H 5COOH( B), C 6 H 5COONH 4 and then C 6 H 5CONH 2 (C ).
(iii) CH 3CH 2CN( A ), CH 3CH 2CH 2 NH 2 ( B), CH 3CH 2CH 2OH(C )
+
(iv) C 6 H 5 NH 2 ( A ), C 6 H 5 N 2 Cl - ( B), C 6 H 5OH(C )
(v) CH 3CONH 2 ( A ), CH 3 NH 2 ( B), CH 3OH(C )

(vi) C 6 H 5 NH 2 ( A ), C 6 H 5 N == NCl( B); C6H5—N==N OH (C)


Q.10 An aromatic compound ‘ A ’ on treatment with aqueous ammonia and heating forms compound ‘ B’ which on the
heating with Br2 and KOH forms a compound ‘C’ of molecule formula C 6 H7 N. Write the structures and IUPAC names of
compound A, B, C.
aq. NH 3 2 Br + KOH
Ans. C 6 H 5COOH ¾¾¾® C 6 H 5CONH 2 ¾¾¾® C 6 H 5 NH 2
( A) Heat ( B) ( C)
Q.11 Complete the following reactions :
(i) C 6 H 5 NH 2 + CHCl 3 + alc. KOH ¾® (ii) C 6 H 5 N 2Cl + H 3 PO 2 + H 2O ¾®
(iii) C 6 H 5 NH 2 + H 2SO 4 (conc. ) ¾® (iv) C 6 H 5 N 2Cl + C 2 H 5OH ¾®
(v) C 6 H 5 NH 2 + Br2 ( aq.) ¾® (vi) C 6 H 5 NH 2 + (CH 3CO) 2 O ¾®
4 (i) HBF
(vii) C 6 H 5 N 2Cl ¾¾¾¾¾®
(ii) NaNO2 /Cu, D
Ans. (i) Carbylamine reaction. C 6 H 5 NC, 3KCl and 3H 2O is formed.
(ii) C 6 H 6 is formed by reduction.
+
(iii) Salt C 6 H 5 NH 3× HSO -4 (anilinium hydrogen sulphate) is formed.
(iv) C 6 H 6 is formed by reduction.
(v) 2, 4, 6-tribromo aniline is formed.
(vi) C 6 H 5 NHCOCH 3 + CH 3COOH is formed.
(vii) C 6 H 5 NO 2 , BF3 and NaF are formed.
Q.12 Why aromatic primary amines cannot be prepared by Gabriel phthalimide synthesis ?
Ans. Dut to resonance, C—X bond in aryl halide has some double bond character.
Q.13 How do aromatic and aliphatic amines reacts with nitrous acid ?
+
Ans. (a) Aryl primary amines form diazonium salt, e.g., C 6 H 5 NH 2 gives C 6 H 5 N 2Cl – with NaNO 2 and HCl.
(b) Aliphatic amines also form diazonium salt but they are unstable. They decompose to give alcohol.
NaNO2 + HCl + H O
R ¾ NH 2 ¾¾¾¾® [ R ¾ N 2Cl – ] ¾¾®
2
R ¾ OH + N 2 + HCl
273- 278 K

(c) Aralkyl primary amines give aromatic alcohols.


HNO2
C 6 H 5CH 2 NH 2 ¾¾® C 6 H 5CH 2OH + N 2 + H 2O
Benzyl amine Benzyl alcohol
Q.14 Give plausible explanation for each of the following :
(i) Why are amines less acidic than alcohols ?
(ii) Why are primary amines higher boiling than tertiary amines ?
(iii) Why are aliphatic amines stronger bases than aromatic amines ?
Ans. (i) Because of higher electronegativity of O-atom than N-atom, H-atom attached to O-atom acquires more positive
charge.
(ii) Due to intermolecular H-bonding in p-amines.
(iii) In aromatic amines, lone pair of N-atom participates in resonance with benzene ring, so it is less available for
protonation.
vvv
169
Chapter

14 BIOMOLECULES
Syllabus : Carbohydrates : Classification (Aldose and ketose), Monosaccharides (Glucose and Fruct ose), D-L configuration,
*oligosaccharides (Sucrose, Lactose and Maltose), polysaccharides (Starch, Cellulose and Glycogen),
importance
Proteins : Elementary idea of amino acids, introduction, peptide bond, polypept ides, protein, structure of protein: primary,
secondary, tertiary and quaternary structures (elementary idea only). Denaturatio n of proteins, *enzymes
*Vitamins: Classification and function
Nucleic acids : DNA and RNA
* Topics Deleted for Examination 2021

Objective Questions
1. Glucose on heating with Fehling’s solution shows: (c) diastase (d) all of these
(U.P. 2010) 7. Which of the following compound does not give biuret
(a) oxidation (b) reduction test? (U.P. 2014)
(c) decomposition (d) condensation (a) Carbohydrate (b) Polypeptide
2. Pair, which gives positive Tollen’s test is : (U.P. 2016) (c) Urea (d) Proteins
(a) glucose and sucrose (b) fructose and sucrose 8. Which of the following bases is not found in DNA ?
(c) glucose and fructose (d) all of these (U.P. 2019)
3. Product obtain by hydrolysis of inulin is : (U.P. 2017) (a) Thiamine (b) Cytosine
(a) glucose (b) fructose (c) Uracil (d) Adenine
(c) lactose (d) both (a) and (b) 9. The bases present in RNA are : (U.P. 2019)
4. The carbohydrate, which acts as reserve glucose is our (a) Adenine, Guanine, Cytosine, Thymine
body is: (U.P. 2013, 17) (b) Guanine, Cytosine, Thymine, Uracil
(a) sucrose (b) starch (c) Cytosine, Thymine, Adenine, Uracil
(c) glycogen (d) fructose (d) Adenine, Guanine, Cytosine, Uracil.
5. Which of the following is the sweetest sugar ? 10. Cellulose on complete hydrolysis gives : (U.P. 2019)
(U.P. 2019) (a) L-Glucose (b) D-Fructose
(a) Glucose (b) Fructose (c) D-Ribose (d) D-Glucose
(c) Lactose (d) Sucrose 11. Which one is a disaccharide ? (U.P. 2020)
6. Glucose is converted into ethyl alcohol in the presence (a) Glucose (b) Fructose
of enzyme : (U.P. 2016) (c) Sucrose (d) Xylose
(a) invertase (b) zymase

Answers
1. (a) 2. (c) 3. (b) 4. (c) 5. (b) 6. (b) 7. (a) 8. (c) 9. (d) 10. (d)
11. (c

Very Short Answer Type Questions

Q.1 Write one function of RNA. (U.P. 2017) Ans. Those sugars which reduces Fehling’s solution to red
Ans. To help in synthesis of proteins. Cu 2O and Tollen’s reagent to silver mirror are called
Q.2 What are reducing sugars ? Support your answer with reducing sugars.
chemical equations. (U.P. 2015) For example :
170
CHO COOH Ans. This is because in presence of an alkali, fructose
| | undergoes rearrangement and converts into glucose
(CHOH) 4 + 2CuO ¾® (CHOH) 4 + Cu 2O containing aldehyde group. This reaction is known as
Fehling Red ppt.
| sol. | enolisation.
CH 2OH CH 2OH Q.5 What do you understand by monosaccharides and
Gluconic
acid polysaccharides? Explain with examples.
Q.3 With the help of chemical equation, prove that (U.P. 2018)
glucose molecule possesses five —OH groups. Ans. Monosaccharides : The carbohydrates which are
(U.P. 2014, 15, 16, 17, 18) not hydrolysed further are called monosaccharides,
Or Prove the presence of an aldehyde group and five e.g., glucose, fructose etc.
—OH groups in glucose molecule giving chemical Polysaccharides : The carbohydrates which give
equations. (U.P. 2017) several (>10) monosaccharide units on hydrolysis
Ans. Glucose reacts with acetyl chloride or acetic are called polysaccharides, e. g ., cellulose, starch.
anhydride to form penta acetyl glucose. This confirms Q.6 Write the structural formula of D(+) and L (–)
the presence of five —OH groups. glucose. (U.P. 2017)
C 6 H7 O(OH) 5+ 5 (CH 3CO) 2 O ¾® C 6 H7 O(OCOCH 3 ) 5 CHO CHO
Glucose Acetic Penta acetyl glucose ½ ½
anhydride (CHOH) 3 (CHOH) 3
+ 5CH 3COOH
Ans. ½ ½
Oxidation by Fehling’s solution and Tollen’s reagent
CH ¾ OH HO ¾ CH
prove the presence of —CHO group.
½ ½
Q.4 Ketones do not reduce Fehling solution and Tollen’s CH 2OH CH 2OH
reagent while fructose containing ketonic group does, D-glucose L -glucose
why? (U.P. 2011, 16)
Short Answer Type Questions
Q.1 What is Molisch’s test? Which types of compounds are Ans. Fehling solution is a mixture of Fehling solution A and
identified by this test? (U.P. 2011,12, 17) Fehling solution B. Fehling solution A is aqueous
Ans. Molisch’s test : This is a general test of solution of copper sulphate and Fehling solution B is
carbohydrates. When carbohydrates treated with aqueous solution of sodium potassium tartarate.
Molisch’s reagent (1% alcoholic solution of Reactive component in Fehling solution is CuO. It
a-naphthol) in the presence of conc. H 2SO 4 , the oxidises glucose (monosaccharide and aldehyde)
formation of a voilet ring at the junction of two
forming red precipitate of Cu 2 O.
liquids confirm the presence of a carbohydrate.
Thus, carbohydrates are identified by this OHC × (CHOH) 4 × CH 2OH + 2CuO ¾®
Glucose Fehling
test. Solution
Q.2 Write structural formula possible for molecular HOOC(CHOH) 4 × CH 2OH + Cu 2O ¯
Red ppt.
formula C6 H12 O6 . (U.P. 2017)
Ans. A large number of structural formulae for C6 H12 O6 Q.4 Explain the amphoteric nature of amino acids.
formula is possible. However, following two Or Explain the zwitter ion. (U.P. 2014, 16, 18)
structural formulae of glucose and fructose are Ans. In amino acids, —NH 2 group is basic while —COOH
important: group. is acidic. In solution, —COOH group liberates
CHO CH2OH ..
H + to form —COO - while — N H 2 accepts it to form
H—C—OH CO +
— N H 3 . In most of the amino acids —R group. is
HO—C—H HO—C—H non-ionisable and due to formation of zwitter ion,
amino acids are amphoteric in nature.
H—C—OH H—C—OH R — CH — COOH a R — CH — COO -
| +|
H—C—OH H—C—OH NH 2 NH 3
CH2OH CH2OH In solutions, basic nature of amino acids is due to
+
D-glucose D-fructose —COO - group. while acidic nature is due to — N H 3
Q.3 What is Fehling solution? What happens when it is group.
heated with glucose? Give chemical equations also. Q.5 Describe the double helix structure of DNA with the
(U.P. 2017) help of a diagram. (U.P. 2015)
171

Ans. Watson and Crick in 1953 proposed a double Fructose adds HCN to form Cyanohydrin, proving the
helicle model of DNA. According to this, DNA presence of ‚
ƒ C==O group in fructose.
molecule consists of two deoxyribonuceleotide chains
CH 2OH CH 2OH
that are spirally coiled around each other on a
common axis. The coiling is generally right handed ½ ½ OH
ƒ
C== O C‚
and show major and minor grooves alternately. The
two chains are held together by the formation of ½ + HCN ¾¾¾® ½ CN
hydrogen bonds as shown in Figure. (CHOH) 3 (CHOH) 3
½ ½
CH 2OH CH 2OH
One molecule of fructose reacts with 5 molecules of
CH 3COCl proving the presence of 5 —OH groups.
CH 2OH CH 2OCOCH 3
½ ½
C== O C== O
5CH 3COCl
½ ¾¾¾¾®
–5 HCl ½
(CHOH) 3 (CHOCOCH 3 ) 3
½ ½
CH 2OH CH 2OCOCH 3
Penta acetyl fructose
Q.8 Although glucose contains —CHO group but it
neither give test with schiff’s reagent nor reacts with
NaHSO 3 and NH 3 , explain why ? (U.P. 2015)
Ans. Because it has cyclic structure in which —CHO group
is not free.
H OH
C
(i) Schiff ’s reagent
(CHOH)3 O ¾¾¾¾¾¾¾® X (No reaction)
(ii) NaHSO3
(iii) NH3
CH
Fig. Double helical structure
of Watson and Crick
CH2OH
Q.6 What is Tollen’s reagent? What happens when it Q.9 (i) What do you understand by DNA fingerprinting?
reacts with glucose? Give proper chemical equation. What are its applications? (U.P. 2014)
(U.P. 2003, 17) (ii) What is the difference between DNA and RNA?
Ans. Tollen’s reagent : Ammonical silver nitrate Give one example of both. (U.P. 2014,15)
(AgNO 3 + NH 4 OH) is called Tollen’s reagent which is Ans. (i) DNA Fingerprinting : We all know that every
weak oxidising agent. Actual oxidant present in it is individual has unique fingerprints and fingerprints of
Ag 2O. Glucose is oxidised to gluconic acid by Tollen’s no two person match. They may be altered through
reagent and silver mirror is formed. surgery. In DNA every individual has its own
CHO COOH sequence of bases and the information regarding this
½ ½ is called DNA fingerprinting.
Ag2O
(CHOH) 4 ¾¾® (CHOH) 4 + 2Ag ¯ Applications of DNA fingerprinting : DNA
½ ½ fingerprinting is used in
CH 2OH CH 2OH (i) identification of dead body
Q.7 How fructose is obtained from inulin? Confirm the (ii) deciding paternity of a person
presence of carbonyl and —OH groups in fructose (iii) in the forensic laboratories to establish the
giving proper equations. (U.P. 2004, 07, 17) identities of individuals.
Ans. Inulin, on hydrolysis yields fructose
dil. H 2SO4
(C 6 H10O 5 ) n + nH 2O ¾¾¾® nC 6 H12O 6
Fructose
172

(ii) Difference between DNA and RNA For example :


NH2 NH2
S.No. RNA (Ribonucleic acid) DNA (Deoxyribonucleic CH2 ; CH3CH
acid) COOH COOH
Glycine Alanine
1. The pentose sugar present The pentose sugar present in
in RNA is D-ribose. DNA is D-2-deoxyribose. 2. Acidic amino acids : The amino acids in which,
the carboxyl groups are in excess than amino groups
2. It is formed by DNA and It may replicate to form
can not replicate itself. another strand of DNA.
are called acidic amino acids.
For example :
3. RNA is single stranded and D N A h a s d o u b l e h e l i c a l NH2
linear structure. structure. (i) HOOC.CH2CH
COOH
4. It regulat es pro tein It transfers the characters from Aspartic acid
synthesis. one generation to the other. NH2
Q.10 What are amino acids? How are they classified? (ii) HOOC.CH2CH2CH
COOH
Explain with example. (U.P. 2015, 20) Glutamic acid
Ans. Amino acids : The organic compounds that contain 3. Basic amino acids : In basic amino acids, amino
both a carboxylic group (—COOH) as well as an groups have greater proportions, e. g .,
amine group (—NH 2 ) are called amino acids. NH2
For example : (i) H2N.(CH2)4CH
COOH
Carboxylic group Lysine
H
¾

Q.11 What are sugars and non-sugars? Give their physical


®

R—C—COOH properties and give two examples of each?


® (U.P. 2018)
¾
a-Carbon atomNH2 ¬¾ Amine group Ans. Those carbohydrates which are soluble in water and
Where R may be alkyl, aryl or any other group. sweet in taste are called sugars. They are white
Classification of amino acids : On the basis of crystalline solids, e. g ., glucose, fructose, sucrose etc.
number of —NH 2 and —COOH groups amino acids On the other hand, the carbohydrates which are
are classified into following three types. tasteless and insoluble in water are called
1. Neutral amino acids : Those amino acids in non-sugars. They are also white solids, e. g .,
which the carboxyl groups and amino groups are cellulose, starch, inulin etc.
equal are called neutral amino acids.
Long Answer Type Questions
Q.1 What are monosaccharides? Write the name and Classification of carbohydrates.
formula of a disaccharide. How will you prove the (1) Monosaccharides : Monosaccharides are
presence of 5-OH groups in glucose? Give chemical simplest carbohydrates which can not hydrolysed into
equations wherever it is necessary. (U.P. 2015) smaller products.
Or What are carbohydrates? Classify them and give For example : Glucose, fructose etc.
name and formula of one carbohydrate from each CHO CH 2OH
class. (U.P. 2010,11,13,14, 16) | |
Or What do you understand by monosaccharides, (CHOH) 4 C == O
disaccharides and polysaccharides. Explain with | |
examples. How will you distinguish between glucose CH 2OH (CHOH) 3
Glucose |
and starch? (U.P. 2020)
Ans. Carbohydrates : The polyhydroxy aldehydes and CH 2OH
ketones or substances which give polyhydroxy Fructose

aldehydes or ketones on hydrolysis and contain at (2) Oligosaccharides : The carbohydrates which
least one asymmetric carbon atom are called form two to ten molecules of monosaccharides on
carbohydrates. hydrolysis, are known as oligosaccharides. The main
For example : CHO oligosaccharides are :
| (i) Disaccharides : The oligosaccharides which on
(CHOH) 4 hydrolysis give two molecules of monosaccharides are
| called disaccharides, e. g ., sucrose, maltose, lactose etc.
CH 2OH C 12 H 22O11 + H 2O ¾® C 6 H12O 6 + C 6 H12O 6
Glucose Sucrose Glucose Fructose
173

(ii) Trisaccharides : Carbohydrates giving three (5) In lives, carbohydrate is stored as glycogen,
molecules of monosaccharides on hydrolysis are called which is converted to glucose and provide energy.
trisaccharides, e. g ., raffinose. Q.3 (i) Write D-and L-configurations of a-amino acids.
C 18 H 32O16 + 2H 2O ¾ Hydrolysis
¾¾¾ ¾® 3C 6 H12O 6 (U.P. 2015)
Raffinose Monosaccharide (ii) What are peptides and peptide bonds? Explain
(iii) Polysaccharides : These are the carbohydrates with one example. (U.P. 2015, 17)
which give a large number of monosaccharides on (iii) What do you understand by denaturation of
hydrolysis are called polysaccharides, e. g ., starch, proteins? (UP 2014)
cellulose etc. Ans. (i) D-L configuration of amino acids : Due to
(C 6 H10O 5 ) n + nH 2O ¾® nC 6 H12O 6 presence of asymmetric carbon atom (except glycine),
Starch Glucose a-amino acids exist in two stereoisomeric forms and
Presence of 5-OH groups in glucose : Glucose have D and L configuration. In the D-form the amino
reacts with acetyl chloride to give pentaacetyl glucose group (–NH 2 ) towards the right while in the L-form, it
which confirms the presence of five —OH groups. is present on the left.
CHO CHO For example :
| | COOH COOH
(CHOH) 4 + 5CH 3COCl ¾® (CHO.COCH 3 ) 4 + 5HCl | |
| | H — C — NH 2 H 2 N — C — H
CH 2OH CH 2O.COCH 3 | |
Glucose Pentaacetyl glucose CH 3 CH 3
D-form L -form
Difference between glucose and starch: (Alanine) (Alanine)

S.No. Glucose Starch (ii) Peptides and peptide bonds : The bond
1. Crystalline Amorphous
formed between two amino acids by the elimination of
a water molecule is called a peptide bond
2. Water soluble Forms colloid (—CO — NH—)
3. Reducing sugar Non-reducing sugar O O
4. No reaction with I2 Blue colour with I2 H2N—CH2—C—OH+H—NH—CH—C—OH ¾¾®
–H O 2
Glycine
Q.2 Write short notes on the following: (U.P. 2015) CH3
(i) Polypeptides O
(ii) Importance of carbohydrates
Ans. (i) Polypeptides : Peptides are the compounds H2N—CH2—CO—NH—CH—C—OH (peptide)
¾®

formed when two or more molecules of a-amino acids


CH3
combine with the loss of molecules of H 2O and they
Peptide bond
linked with help of bonds called peptide bonds
(—CO — NH). When a large number of The product formed by linking amino acid
molecules through peptide bond
a-amino acids (10 to 100 molecules) are
(—CO — NH—) is called a peptide.
joined together with peptide bond the
(iii) Denaturation of proteins : The structure of
molecule formed is called polypeptides.
the natural proteins is responsible for their biological
H2N—CH—CO—NH—CH—CO—NH—CH—COOH activity. These structure are maintained by various
attractive forces between different parts of the
R R R polypeptide chains. The breaking of these forces by a
n
Thus the peptides containing 10 to 100 physical or chemical change makes the proteins to lose
all or part of their biological activity. This is called
amino acids are known as polypeptides.
denaturation of proteins.
(ii) Importance of carbohydrates :
The denaturation of proteins can be done by adding
(1) They play important role in metabolic reactions.
chemical such as acids, bases, organic solvents etc. It
(2) Carbohydrates acts as body fuel and provide
can also be done by heat and ultraviolet light.
energy.
For example : The heating of white of an egg gives a
C 6 H12O 6 + 6O 2 ¾® 6CO 2 + 6H 2O + energy
hard and rubbery insoluble mass.
(3) Cellulose constitute the cell wall in plant cells.
Q.4 Explain the presence of > C == O, — CHO and five
(4) Monosaccharide ribose is major component of
—OH groups in glucose with the equations of related
ATP which is currency of energy in living beings.
reactions.
Also give two chemical test of glucose and fructose
each. (U.P. 2019)
174

Or Define carbohydrates. Write one equation for each to 1. Primary structure 2. Secondary structure
prove that glucose molecule contains 5 — OH gps., 3. Tertiary structure 4. Quaternary structure
—CHO and C == O. Write difference between 1. Primary Structure : Proteins are constituted by
glucose and sucrose. (U.P. 2020) one or more polypeptide chains and in each
Ans. (i) Glucose reduces Tollen’s reagent which polypeptide chain a large number of a-amino acids are
confirms the presence of ¾CHO group in it. arranged in a specific sequence. This specific sequence
CHO COOH of a-amino acids shows primary structure of proteins.
Ag2O If sequence of amino acids in primary structure is
(CHOH)4 (CHOH)4 changed. The properties and functions of proteins are
also changed. Primary structure of proteins is
CH2OH CH2OH
analysed by sequential enzyme or acid hydrolysis.
gluconic acid
H O R''
(ii) Glucose adds HCN to form cyanohydrin which R' H | || | H
confirms the presence of C == O in it. C N C C
N C C N C
CN | || R
H C O CH H | ||
OH H O H O
(CHOH)4 +HCN (CHOH)4 primary-structure of protein

CH2OH CH2OH For example : Normal haemoglobin molecule is


constituted by 574 amino acid units. If sequence of
Glucose cyanohydrin
even one amino acid is changed, the molecule become
(iii) Glucose form pentaacetyl derivative with acetyl defective and person gets affected with a disease
chloride which proves presence of 5 ¾ OH groups. named sickle cell anaemia. In this disease, R.B.Cs.
CHO CHO attain abnormal shape and sometimes even bursts.
(CHOH)4+5CH3COCl (CHOCOCH3)4+5HCl
CH2OH CH2OCOCH3
Chemical Tests of Glucose and Fructose :
A. Glucose : (i) When solution of phenyl
hydrazine in acetic acid is added to glucose,
yellow coloured crystalline glucosazone is
formed which melts at 205° C.
(ii) Molisch’s test: This is a general test of
carbohydrates. 2 mL of glucose solution is taken
in a test tube. 2 mL of alcoholic solution of
a -naphthol is added to glucose solution. 1 mL of
conc. H 2SO 4 is added dropwise. The formation
of a violet ring at the junction of two liquids
confirms the presence of a carbohydrate.
B. Fructose : (i) Pinoff’s test: When aqueous
solution of fructose is heated with ammonium
molybdate and few drops of acetic acid, blue
colouration appears. Fig. Primary structure of Proteins
(ii) Selivenoff’s test: On adding solution of Normal Haemoglobin :
resorcinol in HCl to aqueous solution of fructose Val-His-Leu-Thr-Pro-Glu-Glu-Lys
red colouration or precipitate is formed which is Defected Haemoglobin :
soluble in alcohol. This test is not given by
Val-His-Leu-Thr-Pro-Val-Glu-Lys
glucose, hence the test is used to differentiate
In defected haemoglobin, glutamic acid is replaced
between fructose and glucose.
by valine.
Q.5 Explain the primary, secondary, tertiary and
During the synthesis of proteins, chain of polypeptides
quaternary structures of proteins and also explain the
gets coiled due to formation of disulphide bonds
denaturation of proteins. (U.P. 2019)
between cysteine.
Or What are proteins ? How are they classified ? Discuss
2. Secondary Structure : In secondary structure
briefly about their primary, secondary and tertiary long, elastic polypeptide chains are coiled to form
structures. (U.P. 2019) relatively rigid structure. In secondary structure,
Ans. Structure of proteins : Proteins have different peptide bonds and H-bonds formed between
complex, three dimensional structure. Their structure
is studied at four levels.
175
‚ ‚C == O and —N—H groups are
C == O and —N—H bonds and planar. In 1951, Note: 1. In peptide bond,
ƒ ƒ
Linus Pauling and Corey studied the secondary trans to each other.
structure of proteins with the help of X-rays and told 2. a-helical structure arises as a result of resonating
that primary polypeptide chains are coiled in two structures of peptide bond and H-bonds between
‚C == O and —NH groups.
ways to form secondary structure. ƒ
The secondary structure of protein depend upon the 3. All the amino acids in polypeptides have
size of alkyl group, ( R ). If R is bulky, then it attains L-configuration.
a-helixal structure and if R is relatively smaller, then it 4. Helix is always right handed.
attains b-pleated sheet structure.
(i) a-Helix Structure : (ii) b-pleated Sheet Structure :
If alkyl group R is bulky, then O-atom of In b-pleated sheet structure, alkyl group ( R ) is
‚ relatively smaller. In this structure, two segments of
C == O group one amino acid forms H-bond,
ƒ polypeptide chains come close and gets linked in
H-atom of —N—H group of fourth amino acid. It leads disordered zig-zag sequence.
to coiling of polypeptide chain into a ring and this
structure is called a-coiled or right handed or a-helix
structure. Such proteins are flexible and may be
stretched. During stretching weak H-bonds, which are
responsible for helical structure gets broken; and
length of helix increases like spring. If this strain is
removed, H-bonds are reformed and again a-helical
structure is obtained.

(a) Parallel

Fig. (a) Right handed a-helix (b) H-bonds in helix


a-helix is called 3.613 helix also, because each turn of
helix contain average 3.6 amino acids and in different
‚
parts of helix, H-bonds formed between C == O and
ƒ
—NH groups form 13 membered ring. Distance
between two successive turns is 54 pm. Myosin (found
in muscles) and keratin (found in nails, hairs, wool
(b) Antiparallel
etc.) have a-helical structure.
Fig. Structure of pleated b-sheet
‚
In this structure, O-atom ofC == O groups of all the
ƒ
amino acids form H-bond with H-atom of —NH group
present in front of it. Thus in both the segments,
176

amino acid unit are in front of each other. b with organic solvents, they get coagulated or
-pleated structure appears like folded cloth, which are precipitated. This process is called denaturation of
arranged in 3-D structure. proteins. In denaturation soluble proteins (globular)
b-pleated sheet structures may be of two types. are converted into insoluble protein (fibrous). During
In one type of structure, polypeptide chains are the process, biological activity of protein is lost. Thus
arranged in parallel while in another type of structure the process in which physical as well as
they are arranged in antiparallel. Keratin in hairs has biological properties of proteins are
parallel b-pleated sheet structure while silk protein changed, is called denaturation of proteins.
fibroin has antiparallel b-pleated
sheet structure.
3. Tertiary Structure : In Denaturation
tertiary structure, polypeptide Reagent
chains are folded to from 3D
structure. Tertiary structure Denatured proteins
decides the overall structure of Native proteins
protein molecule.
Denaturation of Proteins
In the formation of tertiary
Characteristics of Denaturation :
structure, H-bonds, ionic bonds,
(i) In denaturation, secondary and tertiary
hydrophobic interactions, van der
structures of protein changes but primary
Waals’ forces, disulphide bridge
structure remains unchanged.
bonds etc. play important role.
(ii) When pH is changed, H-bonds and salt bridges
Two important tertiary structures
are broken, e. g . , in polypeptide, polylysine,
of proteins are fibrous and globular
contains molecules of amino acid lysine, whose
structure. Water soluble proteins
amino group is present at one end. In acidic
have globular structure. In most of Tertiary medium, all the amino groups of side chain get
the cases, tertiary structure attains structure of collagen
protonated and attain positive charge. Due to
spherical shape. Triple helix
electrostatic repulsion, well organised coiled
structure of collagen is shown in Figure.
structure is changes to linear structure.
4. Quaternary Structure : Some proteins have
(iii) On heating protein molecule, thermal vibrations
two or more than two types of polypeptide chains
increase, resulting in the breaking of H-bonds
known as submit or protomer. Protomers may be same
and salt bridges. It leads to loss of biological
or different held together with H-bonds, electrostatic
activity of protein molecule.
or van der Waals’ interactions.
(iv) During denaturation, entropy increases.
Quaternary structure of protein refers to number of
Examples of Denaturation :
protomers and relative interstitial arrangement. Thus
(i) Most common example of denaturation of
at this level, proteins are multimeric and these
protein is coagulation of albumin present in
molecules are quite large and complex.
white part of egg. When egg is heated through
Four levels of different structures of proteins are
boiling water, then soluble globular albumin gets
shown in Figure, in which each dot (sphere) refers to
converted into fibrous protein. Water molecules
an amino acid.
present there form H-bond with protein. It is
irreversible process.
(ii) When lactic acid is added in milk, it converts into
curd. It is also irreversible process.
(iii) Addition of lemon juice (citric acid) to milk
coagulates it and cheese is obtained. In this
denaturation process, globular milk protein is
changed to lactoalbumin.
Q.6 What is protein? Write its main sources and its
(a) (b) (c) (d)
significance for human body. What do you mean by
Structure of Haemoglobin
(a) primary, (b) secondary , (c) tertiary , (d) quarternary denaturation of protein? Explain. (U.P. 2019)
Denaturation of proteins : All the native proteins Or What are proteins ? Give their main sources. Write
found in living systems have specific 3D structure and primary structure of protein. Also give main functions
are biologically active. If proteins solutions are heated of protein in our body. (U.P. 2019)
or treated with acidic, basic or salt solution or treated
177

Or Define amino acids and proteins. Write important Nucleic acids are biopolymers of high molecular mass
sources and functions of proteins. Explain the whose monomeric units are nucleotides, that is why
denaturation of proteins. (U.P. 2020) they are called polynucleotides also.
Or What is importance of proteins in our diet? What are Chemical composition of nucleic acids :
their important sources. What do you know about Chemically nucleic acids are polynucleotides and their
secondary structure of proteins? What is denaturation chemical composition may be expressed by following
of proteins? (U.P. 2020) chart.
Or What are proteins? Write their important uses. Nucleic acid (Polynucleotides)
(U.P. 2016)
Or What are proteins? Write their important uses.
(U.P. 2018) Nucleoside phosphate group
Or What are the function of proteins in the human (Joins the nucleosides)
body? Mention their two important functions? What
is the effect of their deficiency? Pentose sugar Nitrogenous base
(U.P. 2010, 17)
Or What are the sources of proteins ? What is the
importance of proteins for our body? Ribose Deoxyribose Purines Pyrimidines
(U.P. 2013, 15, 16, 17) (RNA) (DNA)
Ans. Proteins : Proteins are complex polymers of
Adenine Guanine
a-amino acids having high molecular mass in which
monomeric units of a-amino acids are linked through
peptide bonds. Thiamine Cytosine Uracil
Main Sources : (DNA) (RNA)
(i) Animal Sources : milk, cheese, meat, fish etc. In simple way, it may be represented as
(ii) Plant Sources : Kidney bean, Soyabean, fruits, 5'-end
vegetables.
Phosphate
Significance to Human body : Proteins are most
abundant biomolecules found in living beings. They
are present in almost all the parts of body and serve as Sugar Base
basis for structural and functional unit of life.
e.g., Keratin : skin, hairs, nails etc. Phosphate
Myosin : muscles
Collagen : bones, teeth, blood etc. Sugar Base
Elastin : elastic tissue and ligaments.
Denaturation of Proteins : The structure of the 3'-end
natural proteins is responsible for their biological Q.8 Give definition of monosaccharide and disaccharide
activity. These structure are maintained by various and give one example for each. Give one reaction for
attractive forces between different parts of the each group to detect the presence of —CHO and
polypeptide chains. The breaking of these forces by a > C == O groups. Differentiate between glucose and
physical or chemical change makes the proteins to lose fructose. (U.P. 2019)
all or part of their biological activity. This is called Or What are monosaccharides ? How are they classified ?
denaturation of proteins. Explain the ring structures of glucose and fructose.
The denaturation of proteins can be done by adding (U.P. 2019)
chemical such as acids, bases, organic solvents etc. It Ans. Monosaccharides : Those carbohydrates
can also be done by heat and ultraviolet light. which can’t be hydrolysed further are called
For example : The heating of white of an egg gives a monosaccharides e.g., glucose, fructose.
hard and rubbery insoluble mass. Disaccharides : Those carbohydrates which on
Q.7 What are nucleic acids ? Explain the chemical hydrolysis gives two monosaccharides.
composition of nucleic acids. (U.P. 2019) HOH /H +
e.g., Sucrose ¾¾¾® glucose + fructose
Ans. Nucleic acids are colourless complex
HOH /H +
compounds of C,H,N,O and P. Being acidic in nature, Maltose ¾¾¾® glucose + glucose
they are soluble in dilute alkali solutions but insoluble Tests for ¾CHO and C == O group :
in organic solvents, water and dilute acids. They are
called nucleic acids because they are found in nucleus
of cell.
178

(i) ¾CHO group reduces Fehling’s solution and equations of its reactions with hydroxyl amine and
Tollen’s reagent. acetic anhydride and explain what is proved by these
¾ CHO + 2CuO ¾® ¾ COOH + Cu 2O ¯ reactions. Also write the main uses of glucose.
F.S. Red ppt. (U.P. 2020)
¾ CHO + Ag 2O ¾® ¾ COOH + 2Ag ¯ Ans. Methods of preparation: (1) Laboratory
T.R. method : In laboratory, glucose is prepared by
(ii) C == O group adds HCN and gives oxime with hydrolysis of sucrose. Sucrose is dissolved in 90%
hydroxylamine alcohol and small amount of 4% alcoholic
OH hydrochloric acid is added. The resulting solution is
C == O+ HCN ¾® C heated at 50°C to yield mixture of glucose and
CN fructose.
Cyanohydrin
C 12 H 22O11 + H 2O ¾ Hydrolysis
¾¾¾ ¾® C 6 H12O 6 + C 6 H12O 6
C== O + H 2 N ¾ OH ¾¾®
-H O
C == N ¾ OH Sucrose Glucose Fructose
2 Oxime (Cane sugar)

Difference between glucose and fructose: (2) Industrial manufacture : Commercially,


glucose is prepared by hydrolysis of starch. Starch is
S. No. Test Glucose Fructose mixed with water to form a paste. The paste is heated
1. Melting point 146°C 104°C with dilute sulphuric acid at 4-5 atmospheric pressure,
when glucose is obtained.
2. Nature Dextro-rotatory Laevo-rotatory (C 6 H10O 5 ) n + nH 2O ¾ dil.
¾H¾ 2SO4
¾® nC 6 H12O 6
¾
Starch Glucose
3. Reaction with Soluble calcium Insoluble calcium
milk of lime. glucosate formed. fructosate Reactions with Hydroxyl amine and acetic
formed. (1) Action of hydroxylamine: Glucose reacts with
hydroxylamine to form glucose oxime.
4. Reaction with Light blue colour Light green-blue
ammonium appears colour appears.
HOH 2C(CHOH) 4 CHO + H 2 N — OH ¾®
Glucose
molybdate and
acetic acid. HOH 2C(CHOH) 4 CH == N — OH + H 2O
Glucoseoxime
5. Reaction with No reaction Gives red
conc. HCl and colouration. (2) Acetylation: Glucose forms pentaacetyl
resorcinol. derivatives with acid chlorides or acid anhydrides in
presence of anhydrous zinc chloride. It proves that
6. Reduction Sorbitol formed. Mixture of one molecule of glucose contains five —OH groups.
sorbitol and CHO CHO
mannitol formed. | |
, ZnCl 2
7. Oxidation with Saccharic acid Tartaric and (CHOH) 4 + 5(CH 3CO) 2 O ¾ D¾ ¾¾® (CHOCOCH 3 ) 4
- 5CH 3COOH
conc. HNO3. formed. glycollic acids | Acetic anhydride
|
formed. CH 2OH CH 2OCOCH 3
Glucose Glucose pentaacetate
8. Reaction with Gluconic acid No reaction
Br2 water. formed.
Uses of Glucose:
Q.9 Write five chemical equations in favour of Fisher (1) Most important use of glucose is as source of energy.
structure of glucose. (U.P. 2020) Glucose provides energy to weak and patient human beings
Ans. 1. It does not give addition reaction with NH 3 when given orally or as an intravenous injection.
and NaHSO 3 . (2) Calcium glucosate is used in treatment of calcium
2. It does not react with Schiff’s base. deficiency.
3. It shows muta-rotation due to a and b forms.
(3) Glucose is used in the synthesis of vitamin C
4. It forms pentaacetyl derivative with CH 3COCl
C 6 H12O 6 + 5CH 3COCl ¾® C 6 H7 O(OCOCH 3 ) 5 (ascorbic acid).
+ 5HCl (4) It is used as a food for patients and children.
5. It gives n-hexane with red P/HI (5) It is used as sweetening agent in preparation of
C 6 H12O 6 ¾ Red
¾ ¾¾P/HI
® C 6 H14 confectionary.
Q.10Write down two methods of preparation of glucose
giving chemical equation. Write the chemical
179

Question Based on Chemical Reactions


Q.1 How will you obtain glucosazone, gluconic acid, CH2OH CH2O.COCH3
n-hexane and sorbitol from glucose?
CO CO
(U.P. 2003, 04, 06, 08, 10, 18)
+5(CH3CO)2O ¾® +5CH3COOH
Ans. (i) Glucose into glucosazone : (CHOH)3 (CHO.COCH3)3
CHO CH = N × NHC 6 H 5 CH2OH CH2OCOCH3
½ Fructose Pentaacetyl
½
CHOH fructose
C6H 5NHNH 2
CHOH C6H 5NHNH 2
½ ¾¾¾¾¾® ½ ¾¾¾¾¾¾® (ii) Fructose reacts with HCN to form cyanohydrine.
–H 2O –NH 3, – C6H 5NH 2
(CHOH) 3 (CHOH) 3 CH2OH CH2OH
½ ½ OH
CH 2OH CO C
CH 2OH CN
+HCN ¾¾®
(CHOH)3 (CHOH)3
CH== N × NHC 6 H 5 CH== N × NHC 6 H 5
½ ½ CH2OH CH2OH
C==O C== N × NHC 6 H 5 cyanohydrine
C6H 5NHNH 2
½ ¾¾¾¾¾® ½ (iii) Fructose reacts with hydroxyleamine to form
(CHOH) 3 (CHOH) 3 oxime.
½ ½ CH2OH CH2OH
CH 2OH CH 2OH
Glucosazone C==O+H2 NOH C==NOH
(ii) Glucose into gluconic acid : ¾¾®
–H O
(CHOH)3 2 (CHOH)3
CHO COOH
½ ½ CH2OH CH2OH
Br2 / H 2O, [O]
(CHOH) 4 ¾¾¾¾¾® (CHOH) 4 Fructose oxime
½ ½ Q.3 What happens when (give chemical equations only)
CH 2OH CH 2OH (U.P. 2014)
Gluconic acid
(i) Glucose reacts with bromine water
(iii) Glucose into n-hexane :
CHO (ii) Glucose reacts with HCN
½ (iii) Glucose reacts with phenyl hydrazine
Red P/HI (Reduction) Ans. (i) Glucose reacts with bromine water to form
(CHOH) 4 ¾¾¾¾¾¾¾® C 6 H14
100°C gluconic acid.
½ n-hexane
CH 2OH CHO COOH
| |
(iv) Glucose into sorbitol : (CHOH) 4 ¾ Br H 2O
¾2 ¾ ¾® (CHOH) 4
CHO CH 2OH | |
½ ½ CH 2OH CH 2OH
Na-Hg
(CHOH) 4 + 2[ H] ¾¾¾® (CHOH) 4 Glucose Gluconicacid
½ ½ (ii) Glucose reacts with HCN to form glucose
CH 2OH CH 2OH cyanohydrine.
Q.2 What happens when (Give chemical equations OH
only) (U.P. 2014) CHO H—C
(i) Fructose reacts with acetic anhydride CN
(CHOH)4 +HCN ¾¾® (CHOH)4
(ii) Fructose reacts with HCN
(iii) Fructose reacts with hydroxylamine CH2OH CH2OH
Ans. (i) Fructose reacts with acetic anhydride to form Glucose cyanohydrine
pentaacetyl fructose. This indicates the presence (iii) Glucose reacts with phenyl hydrazine to form
of five —OH groups. glucosazone.
180
CHO CH==N.NH.C6H5 glucose containing —CHO group. Therefore fructose
reduce the Fehling sol and Tollen’s reagent.
CHOH C==N.NH.C6H5 Q.5 What happens when? Give chemical equations also.
+3C6H5NH.NH2 ¾¾®
(CHOH)3 (CHOH)3 (i) Cane sugar is hydrolysed with dilute sulphuric
Phenyl
hydrazine acid. (U.P. 2010)
CH2OH CH2OH Or Sucrose is hydrolysed in presence of mineral
Glucosazone acid. (U.P. 2017)
+C6H5NH2+NH3+2H2O (ii) Fructose reacts with hydroxylamine.
(U.P. 2017)
Q.4 (i) Give only one reaction proving presence of carbony
(iii) Fructose reacts with phenyl hydrazine.
C==O and aldehyde groups (—CHO) in glucose. (U.P. 2010)
(U.P. 2014) dil. H SO
2 4
(ii) Prove the presence of a straight chain of 6 carbon Ans. (i) C 6 H12O 6 + H 2O ¾¾¾¾® C 6 H12O 6 + C 6 H12O 6
Glucose Fructose
atoms in glucose. (U.P. 2018) CH 2OH
(iii) Ketones do not reduce Fehling solution and
½
Tollen’s reagent while fructose containing ketonic C==O
group does, why? (U.P. 2011,16) (ii) ½ + H 2 N × NHC 6 H 5 ¾¾¾®
Ans. (i)(a) Glucose reacts with hydroxylamine to form (CHOH) 3
–H 2O

oxime which indicates glucose contain carbony group.


½
CHO H—C==NOH CH 2OH
(CHOH)4+NH2OH ¾¾® (CHOH)4+H2O CH 2OH
½
CH2OH CH2OH C== N × NHC 6 H 5
Glucose oxime ½
(b) On heating glucose with Fehling solution give ppt. (CHOH) 3
of Cu 2O, which indicates that glucose has aldehyde ½
group. CH 2OH
Hydrazone
CHO COOH

(CHOH)4+2CuO ¾¾® (CHOH)4+Cu2O CH 2OH CH 2OH


½ ½
CH2OH CH2OH C==O C== N ¾ OH
(ii) Glucose, on heating with HI and red P, gives (iii) ½ + H 2 NOH ¾¾¾® ½
–H 2O
n-hexane. This proves the presence of straigth chain of (CHOH) 3 (CHOH) 3
six carbon atom in glucose. ½ ½
C 6 H12O 6 ¾ HI
¾Red
¾¾P
® C 6 H14 CH 2OH CH 2OH
100ºC n- hexane Oxime
(iii) This is because in the presence of an akali,
fructose undergoes rearrangement and converts into
181

Solution of NCERT Text Book Problems

Q.1 What are monosaccharides ? Q.6 What is glycogen ? How is it different from starch ?
Ans. They are common carbohydrates and can’t be Ans. Deleted for examination 2021.
hydrolysed further. Glucose and fructose are most Q.7 What are the hydrolysis products of : (i) sucrose and
common examples of monosaccharides. In (ii) lactose ?
monosaccharides, —OH groups are present along Ans. (a) a -D (+)-glucose and a -D (–)-fructose.
with aldehydic (—CHO) and ketonic ( ‚ ƒ C == O ) (b) a -D(+)-glucose and a -D(+)-galactose.
groups. If carbonyl groups is aldehydic, then they are Q.8 What is the basic structural difference between starch
named as aldoses and if it is ketonic then as and cellulose ?
ketoses. Aldehyde group is monovalent and hence it Ans. Deleted for examination 2021.
is always terminal in position. It is always at C-1. Q.9 What happens when D-glucose is treated with the
Ketonic group is bivalent and is always subterminal. following reagents ?
In natural ketoses, it lies at C-2 generally. (i) HI (ii) Bromine water (iii) HNO 3
Q.2 What are reducing sugars ? Ans. (i) n-hexane is obtained
Ans. Sugars are divided into two groups on the basis of CHO
their reducing nature: |
HI / D
Reducing sugar : Those sugars which reduce (CHOH) 4 ¾¾® C H 3 (CH 2 ) 4C H 3
Fehling solution or Tollen’s reagent are known as |
CH 2OH
reducing sugars.
For example : Glucose, fructose, lactose etc. (ii) Gluconic acid is obtained
CHO COOH
Non-reducing sugars : Those sugars which do not | |
reduce Fehling solution or Tollen’s reagent are [O]/Br2 water
(CHOH) 4 ¾¾¾¾¾® (CHOH) 4
termed as non-reducing sugars. | |
For example: sucrose, polysaccharides. CH 2OH CH 2OH
Q.3 Write two main functions of carbohydrates in plants. (iii) Gluconic acid is formed
Ans. (i) To supply energy in the form of food. CHO COOH
(ii) To provide mechanical support as cell wall which | [O], HNO3
|
(CHOH) 4 ¾¾¾¾¾® (CHOH) 4
is made up of cellulose.
| |
Q.4 Classify the following into monosaccharides and
CH 2OH CH 2OH
disaccharides. Ribose, 2-deoxyribose, maltose,
Q.10 Enumerate the reactions of D-glucose which cannot
galactose, fructose and lactose.
be explained by its open chain structure.
Ans. Monosaccharides : Ribose, 2-deoxyribose,
Ans. Cyclic structure of glucose: Open chain
galactose and fructose.
structure of glucose could not explain following facts:
Disaccharides : Maltose and lactose.
(i) It does not give addition reaction with NaHSO3
Q.5 What do you understand by the term glycosidic and NH3 .
linkage ?
(ii) Glucose exists in a- and b - forms.
Ans. When two monosaccharides are linked together
(iii) Aqueous solution of glucose shows mutarotation.
through —O — atom, the bond formed is called
glycosidic bond. In disaccharide, the two (iv) Glucose does not react with Schiff’s reagent.
monosaccharides are joined together by glycosidic On the basis of above mentioned facts, it may be
bond as shown below. concluded that free —CHO group is not present in
CH2OH CH2OH glucose. Aldehyde group exists as hemiacetal cyclic
O O structure. The cyclic structure also exists in two
H H H H
H H isomeric forms, that is, a - D -glucose and
OH H O OH H OH b - D -glucose. These two cyclic forms exist in
HO equilibrium state through an open chain structure,
hence, glucose shows mutarotation. In the honour of
H OH H OH
Fischer, these formulae are known as Fischer
Glycosidic bond projection formulae.
182

OH CHO H (ii) Primary structure : See long answer type


questions No. 6.
H—C H—C—OH HO—C
(iii) Denaturation : See long answer type
H—C—OH questions No. 6.
H—C—OH HO—C—H
Q.13 What are the common types of secondary structure of
HO—C—H O
º H—C—OH º HO—C—H O proteins ?
Or What do you understand by the secondary structure
H—C—OH H—C—OH H—C—OH
of protein? Explain. Write the factors which stabilize
H—C CH2OH H—C the structure of protein. (U.P. 2014, 16)
D-glucose Ans. See long answer type questions No. 6.
CH2OH CH2OH
+52.5°
b-D-glucose Q.14 What type of bonding helps in stabilising the a-helix
a-D-glucose
[a]D=+111°
+19.5° structure of proteins ?
Q.11 What are essential and non-essential amino acids ? Ans. H-bonding
Give two examples of each type. Q.15 Differentiate between globular and fibrous proteins.
Ans. The amino acids which can synthesised by the body Ans. Based on Molecular Structure :
are called non-essential amino acids, e. g ., glycine. On the basis of molecular structure, proteins are
The amino acids which can not synthesise by the divided into two groups:
body are known as essential amino acids, e. g ., (a) Fibrous proteins : Such proteins are made up
leucine and valine. of linear, thread like molecules. In these molecules,
Q.12 Define the following as related to proteins : polypeptide chains are held together with H-bonds
(i) Peptide linkage (ii) Primary structure and disulphide bonds. They are insoluble in water
(iii) Denaturation but soluble in strong acids and bases. They functions
Ans. (i) Peptide linkage : Two or more similar or chiefly as constitutional proteins.
different amino acids condense together to form For example : Keratin is present in skin, hairs,
peptides. In the formation of peptide bond, —NH 2 nails, wool, feathers, horns etc. Myosin is present in
group of one amino acid is condensed with —COOH muscles. Collagen is present in bones, cartilages,
of adjacent amino acid to form —CONH — (peptide teeth, blood, walls of blood vessels etc. Elastin is
bond or amide bond) bond. In this reaction, H 2O present in elastic tissues and ligaments.
molecule is eliminated. Peptide bonds bind amino (b) Globular proteins : In these proteins,
acids with each other in order to form protein polypeptides attain spherical shape and polypeptides
molecule. are held together with relatively weaker H-bonds.
O O They are soluble in water, alkalies, salt solutions and
|| || Condensation acid solutions. Chiefly they act as functional proteins.
H 2 N —CH— C — OH + H N —CH — C — OH ¾ ¾ ¾ ¾®
- H 2O For example :
| | |
R H R (i)Enzyme pepsin is secreted in stomach and helps
Amino acids in digestion.
O O (ii) Hormone insulin is secreted from pancreas and
|| || regulates the metabolism of glucose.
H2N —CH— C — N —CH— C — OH
| | | (iii) g-globulin is present in blood and protects the
R H R body against infections.
Dipeptide Peptide bond (iv) Haemoglobin, present in blood, transports the
O 2.
CH3
| Q.16 How do you explain the amphoteric behaviour of
Condensation
H2N — CH2 — COOH+ H2N —CH — COOH ¾ ¾ ¾ ¾®
-H O
amino acids ?
2
Glycine Alanine
CH3
Ans. Amino acids react with both acids and bases, so they
| are amphoteric in nature.
H2N — CH2 — CO — NH —CH — COOH +H + +
(Gly -Ala) or (G-A) HOOC ¾ R ¾ NH 2 ¾¾® HOOC ¾ R ¾ NH 3
In dipeptides, both the ends have free functional OH –

groups, which condense further with amino acid ¾¾¾¾® OOC ¾ R ¾ NH 2
molecules to form tri, tetra, penta peptides etc.
183

Q.17 What are enzymes ? By using prefix deoxy-before the name of ribose
Ans. Deleted for examination 2021. nucleoside, name of deoxyribose nucleoside is
Q.18 What is the effect of denaturation on the structure of obtained, e. g ., deoxyadenosine, deoxyguanosine etc.
proteins ? NH2
Ans. Highly ordered structure gets unfolded. N
N
Q.19 How are vitamins classified ? Name the vitamin Base
responsible for the coagulation of blood ? N N
5'
HOH2C O
Ans. Deleted for examination 2021. N-glycosidic bond
Q.20 Why are vitamin A and vitamin C essential to us ? 4' H H 1'

Give their important sources. H H


3' 2'
Ans. Deleted for examination 2021. OH OH
Q.21 What are nucleic acids ? Mention their two important D-ribose nucleoside (RNA)
functions. (U.P. 2016) NH2
Ans. 1. Nucleic acids are colourless complex compounds
N
of C, H, N, O and P. N
Base
2. They are acidic in nature. N
5' N
3. They are soluble in dilute alkali solutions HOH2C O
N-glycosidic bond
4. They are insoluble in organic solvents, water and 4' 1'
H H
dilute acids. H
5. They have rich content of phosphorus. Properties 3' 2'
OH H
of nuclein proves them to be acidic and they are
D-2-deoxyribase nucleoside
associated with nucleus. Thus, they are called
nucleic acids. Note: In purine nucleoside, purine is bonded at 9' position while in
6. Nucleic acids are biopolymers of high molecular pyrimidines it is at 3' position. Sugar is always as
5-membered ring i . e. , furanose form. Some nucleosides and
mass. Monomeric units are nucleotides and thus
theri components are given below.
they are known as polynucleotides also.
7. Nucleic acid obtained from yeast is called
zymonucleic acid and phytonucleic acid. Nucleoside Sugar Base
8. In living being, two types of nucleic acids are 1. Adenosine (A) = ribose + adenine
found. 2. Guanosine (G) = ribose + guanine
3. Cytidine (C) = ribose + cytosine
(i) De-oxyribonucleic acid (DNA)
4. Uridine (U) = ribose + uracil
(ii) Ribonucleic acid (RNA)
5. Deoxythiamidine (T) = deoxyribose + thiamine
Two important functions:
(B) Nucleotides are made by nucleoside and
(i) Replication phosphoric acid ( H 3 PO 4 ), thus nucleotides are esters
(ii) Protein synthesis of nucleosides and phosphoric acid.
Q.22 What is the difference between a nucleoside and a
NH2
nucleotide ?
Ans. Nucleosides and Nucleotides : N Base
N
(A) In nucleoside, purine or pyrimidine base O
N
(nitrogenous base) are attached to pentose sugar 5' N
(D-ribose or D-2-deoxyribose) through b-N-glycosidic O—P—O—CH2 O
bond at C 1 of pentose sugar. 4' 1'
Nucleoside ® Purine or pyrimidine base + H H
Phosphate unit 3' 2'
pentose sugar
OH OH
To differentiate between base and pentose sugar,
Sugar
carbon atoms of pentose sugar are assigned by 1, 2, 3,
Nucleotide Adenosine-5-
4, 5. In nucleoside, purine or pyrimidine bases are
monophosphate
attached to sugar through b-glycosidic bond of at 1' of
pantose sugar.
184
.......
.......
Nucleotides ® Nucleoside + Phosphoric acid bonds (C ....... G). A pairs with T through two
In nucleotides, phosphate group is attached to —OH ....... T). Due to this pairing
hydrogen bonds (A .......
group of C-5 of pentose sugar through ester linkage principle, the sequences of bases in one strand
which is called 5-phospho ester bond. automatically fixes the sequence of bases in other
Nucleotides are represented by English capital letters. strand. Thus, the two strands are not identical.
In deoxy series d- used an prefix e. g ., Q.24 Write the important structural and functional
AMP = adenosine monophosphate differences between DNA and RNA.
d - AMP = deoxy adenosine monophosphate Or Write short note on DNA. (U.P. 2018)
Nucleotides are linked together through Ans. There are four structural difference between RNA
phosphodiester bond formed at C-3 and C-5 pentose and DNA :
sugar. (i) The sugar present in DNA is 2-deoxy ribose
(5' end)
O O whereas in RNA it is ribose.
Base Base
5' 5' (ii) DNA contains cytosine and thiamine as
O—P—O—CH2 O O—P—O—CH2 O
pyrimidine bases whereas RNA contains cytosine
4' 1' 4' 1' and uracil.
O H H O H H
H H H H (iii) DNA has double stranded a -helix structure
3' 2' 3' 2'
O whereas RNA has single stranded a -helix
OH OH OH
Phosphodi- structure.
O O—P==O ester (iv) DNA controls the transmission of heredity effects
Base
5' bond whereas RNA controls synthesis of proteins.
O—P—O—CH2 O O
Base There are two functional difference between RNA
4' 1'
H H 5' CH2 O and DNA :
H 3' 2'
H (i) DNA controls transmission of heredity whereas
5' H H 1'
OH OH RNA controls synthesis of proteins.
H H
3' 2' (ii) DNA has the property of replication whereas
OH OH RNA usually does not replicate.
(3' end)
Q.25 What are the different types of RNA found in the cell?
Fig. Dinucleotide
Ans. RNA molecules are of three types:
In simple way, it may be represented as
5'-end (i) Ribosomal RNA (r-RNA) : It is found in
ribosomes and takes part in the synthesis of
Phosphate polypeptide chains of proteins. Each ribosome
contains about 65% r-RNA and rest 35% of proteins.
Sugar Base About 80% of total RNA content of cell is r-RNA.
(ii) Messenger RNA (m-RNA) : m-RNA is found
Phosphate in ribosomes and cytoplasm. m-RNA carries the
genetic messages from DNA to ribosomes; where
Sugar Base protein is synthesized. m-RNA molecules get
degraded regularly and that is why their rate of
3'-end synthesis is higher. About 5% of total RNA content is
Note: At 175°C and high pressure, if nucleotide is hydrolysed by m-RNA.
aq. NH 3, it gives nucleoside and phosphoric acid, which (iii) Transfer RNA (t-RNA) : t-RNA molecules are
proves that base molecule is always attached to sugar carries of amino acids, t-RNA is capable to read the
directly. Because nucleosides are not reducing, thus codons and to identify the respective amino acids.
aldehyde group in sugar is not free i.e., nucleosides are
glycosides.
Thus in protein synthesis, t-RNA acts as adaptor and
fitter. t-RNA is smallest (» 54-100
Q.23 The two strands in DNA are not identical but are nucleotides/molecule) among all the RNA and are
complementary. Explain. water soluble. Due to their solubility in water, it is
Ans. The two strands are complementary to each other called soluble RNA (s-RNA) also. About 15% of total
because hydrogen bonds are formed between specific RNA content of cell is t-RNA.
pairs of bases. C pairs with G through three hydrogen
185

Chapter

15 POLYMERS
(This Chapter has been Deleted from Syllabus for
Session 2020-21 Due to COVID-19 Epidemic)

Chapter

16 CHEMISTRY IN EVERYDAY LIFE


(This Chapter has been Deleted from Syllabus for
Session 2020-21 Due to COVID-19 Epidemic)
186

Model Test Paper-1


Chemistry (Class-12)
(Only Paper)
Time : 3.15 Hrs. M.M. 70
Instruction : Initial 15 minutes are allotted to read question paper.
Note : All the questions are compulsory. Marks assigned to each questions are given against it.

1. Each section of this question contains four options. (C) In which crystal defect, density of crystal
Select the correct option and write it in answer book. decreases and why? 2
1´ 6 = 6 (D) On the basis of electronic configuration, explain
(A) Crystal having metal deficiency defect is : the inert nature of inert gases. 2
(a) NaCl (b) FeO 4. (A) (i) Explain Raoult’s law 2
(c) KCl (d) ZnO (ii) Write E.C. of 29 Cu 1
(B) In molar solution, 1 mole solute remains (B) Giving chemical equations, explain the method of
dissolved in : conversion of benzene sulphonic acid into
(a) 1000 g solvent (b) 1 L solution aniline. Write the chemical equation of
carbylamine reaction and diazotisation reaction
(c) 1 L solvent (d) 22.4 L solution
of aniline. 1×3=3
(C) Among Cu, Ag, Fe and Zn which metal may
(C) Explain lanthnoid contraction with reason. 3
displace all from their salt solution?
(D) Explain Werner’s theory of co-ordination
(a) Cu (b) Ag
compounds. 3
(c) Zn (d) Fe 5. (A) Decomposition of NH 3 on the surface of Pt is zero
(D) Unit of rate for zero order reaction is : order reaction. If value of K is 2.5 ´ 10 –4 mol L–1
(a) mol × second (b) mol × L × time –1 sec –1 , then calculate the rate of formation of N 2
–1 1
(c) L × mol time (d) mol × L–1 time –1 and H 2 . 4
(E) Aliphatic primary amine reacts with HNO 2 to (B) Write the IUPAC name of the following: 4
give: (i) K 3[Cr(CN ) 6 ] (ii) [ Ag( NH 3 ) 2 ][ Ag(CN ) 2 ]
(a) alcohol (b) alkyl nitrite (iii) K 2[ HgI 4 ]
(c) s-amine (d) nitroalkane (C) (i) Explain Brownian moment with reason. 2
(F) Ascorbic acid is : (ii) Write the name of sweetest sugar. 2
(a) enzyme (b) vitamin (D) Explain the mechanism of one electrophilic
(c) protein (d) hormones substitution reaction of aryl halide by taking
2. (A) Sodium metal crystallizes as end centred cubic suitable example. 4
crystal with edge length of unit cell a = 4.29 Å. 6. (A) Write the chemical equation of two methods of
Calculate the radius of Na-atoms. 2 preparation of ethanol with the help of Grignard
(B) Calculate the osmotic pressure of 3% urea reagent. Also write two methods of preparation of
solution at 30°C. diethyl ether from ethyl alcohol. 3+2=5
(Solution constant = 0.082 L × atm K –1 mol –1 ) 2 Or
(C) At 25°C, specific conductance of 0.2 M KCl Explain the method of preparation of phenol by
solution is 0.0248 S × cm –1 . Calculate its molar Grignard reagent. Raschig process and Dow’s
conductance. 2 process by giving suitable equations. Write the
(D) What are peptizing agents? Write the name of chemical equations of benzoylation and
suitable peptizing agent for ferric hydroxide sol.2 acetylation of phenol. 1×5=5
3. (A) On dissolution of 15 g of a substance into 150 g (B) What happens when : (Write chemical equations
water, depression in freezing point is 1.2°C. only) 1×5=5
Calculate the molar mass of solute. (Molal (i) acetone reacts with HCN?
depression constant of water is 1.86). 2 (ii) acetone reacts with NaHSO 3 ?
(B) What is Nernst equation? What is the relation (iii) acetone reacts with phenyl hydrazine?
between standard electrode potential and
(iv) formaldehyde reacts with NH 3 ?
electrode potential? 2
187

(v) benzaldehyde reacts with hot H 2SO 4 ? Or


Or Giving suitable diagram, explain the Birkeland-
How will you obtain : (Write chemical equations Eyde process of manufacturing of HNO 3 . Write
only) 1×5=5 the chemical equation of reaction of HNO 3 with
(i) benzophenone from acetone? Zn and Sn. 3+1+1=5
(ii) acetaldehyde from acetyl chloride? (B) Giving suitable examples, explain mono, di and
(iii) bakelite from formaldehyde? polysaccharides. Write their significance in our
daily life. 3+2=5
(iv) chloretone from acetone?
Or
(v) mesitylene from acetone? With the help of chemical reactions, prove that
7. (A) Give suitable reaction and diagram, write the glucose contains. 1+4=5
method of preparation of HNO 3 by Ostwald’s (i) 5 — OH gps. (ii) —CHO gp.
method. Write the equation of reaction when
(iii) Straight 6C-chain (iv) Cyclic structure
HNO 3 reacts with phosphorus, sulphur and
iodine. 2+1+1+1=5

Model Test Paper-2


Chemistry (Class-12)
(Only Paper)
Time : 3.15 Hrs. M.M. 70
Instruction : All the questions are compulsory. Marks assigned to each question are given in front of it.

1. Each section of this question contains four options. (C) Write chemical equations for the preparation of
Select the correct option and write it in answer book. four interhalogen compounds made by chlorine,
1´ 6 = 6 bromine, iodine with fluorine. 2
(A) For a zero order reaction, A + B ¾® C , rate of (D) Write the IUPAC name of following complexes: 2
reaction is : (i) K 3[Cr(CN ) 6 ] (ii) [ Ni( NH 3 ) 6 ]Cl 2
(a) Rate = K[ A]0[ B]0 (b) Rate = K[ A]1[ B]0 3. (A) Copper is crystallized as face centred cubic (fcc)
(c) Rate = K[ A]0[ B]1 (d) None of these lattice. Its atomic radii is 128 pm. Calculate its
density if Cu=63.5. 2
(B) Plastic is : (B) At 298K, Henry constant for dissolution of
(a) ionic solid (b) metallic solid methane in benzene is 4.27 ´ 10 5 mm Hg.
(c) amorphous solid (d) molecular solid Calculate the solubility of methane in benzene at
(C) Which of the fluoride of xenon is impossible ? 298 K and 760 mm Hg. 2
(a) XeF2 (b) XeF4 (C) Calculate mole fractions of each in a mixture of
(c) XeF6 (d) XeF3 72 g of water and 92 g of ethyl alcohol (C 2 H 5OH).
(D) Which of the following has maximum number of 2
unpaired electrons? (D) Concentration of a first order reaction is reduced
(a) Fe 2+ (b) Ti 3+ to half in 72 seconds. Calculate the rate constant
(c) V 3+ (d) Mg 2+ of reaction. 2
4. (A) Transition elements show different oxidation
(E) Tollen’s reagent is :
states, why? 3
(a) Ag 2O (b) Cu 2O o
(B) 2Ce 4 + + Co ¾® 2Ce 3+ + Co 2+ ; E Cell = 1.89 V.
(c) Cu-citrate (d) None of these o
If E Co2+ = -0.28 V , them find the value
(F) Which of the following is a colligative property? / Co
1M
(a) Surface tension (b) Osmotic pressure o
(c) Optical rotation (d) Viscosity E Ce 4 + / Ce 3 + . 3
1M 1M
2. (A) Explain why, lyophilic sols are more stable than
lyophobic sols? 2 (C) Explain: 1½+1½=3
(B) Calculate the volume occupied by atoms in fcc (i) Gold number (ii) Brownian movement
unit cell of a metal if atomic radius is r. 2
188

(D) (i) Explain the structure of [ Fe(CN ) 6 ]4- using Chloroform is stored in dark coloured bottles
VBT. 2 after adding small amount of ethyl alcohol, why?
(ii) Write the nitrogenous bases present in DNA. 5
1 (B) (i) Establish the ring structure of glucose. 3
5. (A) Write short note on the following : 1×4=4 (ii) Explain conductivity and molar conductivity.
(i) Carbylamine reaction 2
Or
(ii) Hoffmann bromamide reaction
(i) With the help of suitable diagram, explain the
(iii) Schmidt reaction Watson-crick model of DNA. 3
(iv) Hoffmann mustard oil reaction (ii) With the help of suitable diagram, explain
(B) Discuss the uses of noble gases in detail. 4 standard hydrogen electrode. 2
(C) Write short note on the following : 2×2=4 7. (A) Describe laboratory method of preparation of
(i) Raoult’s law for solutions of two liquids pure diethyl ether. Given equations and diagram
(ii) Distinction between molecularity and order of apparatus also. Mention its important
(D) (i) What are ligands? How are they classified on properties and uses. Why does it called as
the basis of charge? 2 continuous etherification method? 5
(ii) DNA fingerprinting and its uses. 2 Or
6. (A) What happens when : 1´ 5 = 5 Write three general methods of preparation of
(i) chlorobenzene is heated with aq. KOH or phenol. Explain the acidic nature of phenol. 5
aq. NaOH at 200 atm and 300°C? (B) Write the chemical equations to prepare
formaldehyde, acetaldehyde and acetone from
(ii) chloroform is heated with conc. HNO 3 ?
Grignard reagent. How will you distinguish
(iii) ethyl bromide reacts with potassium between formaldehyde and acetaldehyde? 5
thiocyanate? Or
(iv) benzene diazonium chloride is heated with How will you obtain : 1×5=5
copper powder and HCl? (i) cinnamic acid from benzaldehyde
(v) chlorobenzene is heated with chloral? (ii) benzamide from benzoic acid
Or
(iii) mesitylene from acetone
Explain the laboratory method of preparation of
chloroform by using suitable diagram. (iv) urotropin from formaldehyde
(v) methyl amine from acetic acid

Model Test Paper-3


Chemistry (Class-12)
(Only Paper)
Time : 3.15 Hrs. M.M. 70
Instruction : All the questions are compulsory. Marks assigned to each question are given in front of it.

1. Each section of this question contains four options. (C) Which of the following is s-amine ?
Select the correct option and write it in answer book. (a) CH 3 NH 2 (b) C 6 H 5 NHCH 3
1´ 6 = 6 (c) C 6 H 6 NH 2 (d) (CH 3 ) 3 N
(A) Which of the following will change with (D) Among the following, sweetest sugar is :
temperature? (a) sucrose (b) glucose
(a) molality (b) mole fraction (c) fructose (d) maltose
(c) mass% (d) molarity (E) Which of the following triad of metals show
(B) Correct order of reactivity of Mg, Cu, Na and Au ferrous metals?
is: (a) Fe, Co, Ni (b) Ru, Rh, Pd
(a) Au > Cu > Mg > Na (c) Os, Ir, Pt (d) Cr, Mn, Cu
(b) Mg > Cu > Au > Na (F) Purple of cassius is :
(c) Na > Mg > Cu > Au (a) Fe(OH 3 ) sol (b) Au sol
(d) Cu > Mg > Na > Au (c) sulphur sol (d) As 2S 3 sol
189

2. (A) Lithium borohydride ( LiBH 4 ) crystallizes in an (ii) On the basis of VBT, explain the structure and
orthorhombic system with 4 molecules per unit magnetic nature of Ni(CO) 4 . 2
cell. The unit cell dimensions are: (C) Write short note on the following :
a = 6.81 Å , b = 4.43 Å and c = 7.17 Å. If the (i) Gold number 2
molecular weight of LiBH 4 is 21.76 g mol -1 . (ii) Adsorption and absorption 1
Calculate the density of the crystal. 2 (iii) Hardy Schulz rule 1
(B) On dissolving 13.6 gram of a substance in 20 (D) Describe with diagram the laboratory method of
gram water, freezing pint is depressed by 3.7°C. preparation of formic acid. Write down chemical
Compute the molecular mass of solute. Molal equation of its reaction with Fehling solution. 4
depression constant of water is 6. (A) How will you obtain p, s and t-alcohols by using
-1
1.863 K. molality 2 Grignard reagent? Explain with the help of
(C) Write the name and formula of sugars found in equations. 5
DNA and RNA. 2 Or
(D) Write one method of preparation, one reducing Write three general method of preparation of
reaction and one use of SO 2 . 2 ether. How does ether reacts with : 5
3. (A) Write Raoult’s law and its limitation. 2 (i) PCl 5 (ii) (CH 3CO) 2 O
(B) Write the IUPAC name of the following: 1+1=2 (B) On the basis of E.C. explain the position of gp. 18
(i) [Cr( NH 3 ) 5 CO 3 ]Cl (ii) Fe(CO) 5 elements in periodic table. 5
(C) Write two tests used for distinguishing between Or
aldehydes and ketones. 2 Draw the shape of the following : 1×5=5
1 (i) XeF2 (ii) XeF4 (iii) XeF6 (iv) PCl 5 (v) ClF3
(D) In 500 mL solution of HCl, equivalent mass of
10 7. (A) Explain the first order reaction. If 1/3 of a
HCl is present. Calculate the normality of reaction is completed in 100s then how much
solution. 2 time will it remain 1/9 part? 5
4. (A) (i) Transition metals and their compounds are Or
good catalysts, explain. 1½ At 15°C half-life of nitramide by first order
(ii) Write the general electronic configuration reaction is 2.1 hrs. 5
and their most important oxidation state. NH 2 NO 2( aq.) ¾® N 2O( g ) + H 2O( l )
If 6.2g NH 2 NO 2 is decomposed then calculate :

(B) An element crystallizes as bcc lattice. Edge length (i) time taken in 99% decomposition.
of its unit cell is 288 pm and density is 7.2 g cm –3 . (ii) volume of dry N 2O at NTP at this moment.
How many atoms are there in its 208 g. 3 (B) Giving suitable equations, explain the following:
(C) How will you obtain : (Write equations only) 1×5=5
1½+1½=3 (i) Wurtz-Fittig reaction
(i) ethyl amine from methyl amine (ii) Frankland’s reaction
(ii) p-aminobenzene from aniline (iii) Saytzeff’s rule
(D) Explain the structure of a nucleotide fragment of (iv) Peroxide effect
DNA with its components and then arrangement. (v) Dehydrohalogenation
3 Or
5. (A) (i) What is electrochemical series? Write its two (i) Sodium and compound A react with each other to
uses. 2 form ethane. Write the structural formula of
(ii) Specific conductance of 0.04 N solution of a compound A and equation of the reaction. 2½
weak acid is 4.24 ´ 10 –4 Mho × cm –1 . At this (ii) Write the structures of the major organic product
dilution degree of dissociation of acid is in each of the following reactions: 2½
0.0612. Calculate the equivalent (a) CH 3CH 2CH 2Cl + NaI ¾ Acetone
¾¾ ¾®
Heat
conductance of weak acid at this dilution. 2
(B) (i) What do you understand by effective atomic (b) (CH 3 ) 3 CBr + KOH ¾ Ethanol,
¾ ¾ ¾¾ heat
®
number? Calculate EAN of Fe in K 3[ Fe(CN) 6 ]. (c) CH 3CH( Br )CH 2CH 3 + NaOH ¾ Water
¾¾®
2
190

Model Test Paper-4


Chemistry (Class-12)
(Only Paper)
Time : 3.15 Hrs. M.M. 70
Instruction : All the questions are compulsory. Marks assigned to each question are given in front of it.

1. Each section of this question contains four options. 3. (A) Na-metal crystallizes as body centred cubic lattice
Select the correct option and write it in answer book. with edge length 4.29 Å. Calculate the radii of
1´ 6 = 6 Na-atom and distance between two nearest
(A) Graphite is : atoms. 2
(a) ionic solid (b) metallic solid (B) 18.1 g of a substance are dissolved in 100 g water.
(c) non-crystalline solid (d) molecular solid Vapour pressure of solution was found to be 87
(B) Which of the following mixture is always mm Hg. At the same temperature, vapour
homogenous? pressure of pure water is 92 mm Hg. Calculate the
(a) solid + liquid (b) liquid + liquid molecular mass of substance. 2
(C) Write the IUPAC name of the following: 2
(c) solid + solid (d) gas + gas
o (i) K[Cr( H 2O) 2 × (C 2O 4 ) 2 ] (ii) K 2[ PdCl 4 ]
(C) At 25°C, the E Red values are :
(D) Write short note on Xe compounds. 2
o
Zn 2+ ( aq. ) + 2e ¾® Zn ( s); E Red = - 0.76 V 4. (A) For the reaction, Ester + H + ¾® Acid + Alcohol,
Cu 2+ ( aq. ) + 2e ¾® Cu ( s); o
E Red = + 0.34 V dx
= K [Ester] [ H 3O + ]0 . What will be the effect
dt
Zn( s) + Cu 2+ ( aq. ) ¾® Cu ( s) + Zn 2+ ( aq. )
on the rate of reaction if: 3
o
E Cell for the reaction is: (i) Concentration of ester is doubled?
(a) -1.10 V (b) +1.10 V (ii) Concentration of H + ion is doubled?
(c) 1.34 V (d) 0.76 V (B) How will you obtain ethylamine from
(D) Which of the following is lyphobic colloid? propanamide? Write chemical equations also.
(a) gelatin (b) sulphur Write the equations of carbylamine reaction and
(c) starch (d) gum mustard oil reactions. 3
(C) Why the transition elements are also called as
(E) Divers respire by mixture of:
d-block elements? Explain on the basis of their
(a) O 2 + N 2 (b) O 2 + He
electronic configuration. How do their ionization
(c) O 2 + H 2 (d) O 2 + Ar
potential values change in any d-block series? 3
(F) Reason of lanthanoid contraction is: (D) (i) Write the formula of invert sugar. 1
(a) negligible screening effect of f-orbitals (ii) p-amines are soluble in water, why? 1
(b) increasing nuclear charge (iii) What is Fehling’s solution? 1
(c) decreasing nuclear charge 5. (A) Write short note on the following : 2´2 = 4
(d) decreasing screening effect (i) Gold number
2. (A) Calculate the osmotic pressure of 6% aqueous
(ii) At infinite dilution, molar conductivity of
solution of urea at 27°C.
acetic acid is 39.07 and of 0.1 M acetic acid
( R = 0.0821 L atm K -1 mol -1 ) 2
is 5.2 mho.cm 2 mol -1 , Calculate:
(B) At 25°C, specific conductance of 0.2 M KCl
(a) degree of dissociation of acetic acid
solution is 0.0248 S cm -1 . Calculate its molar
(b) [ H + ] solution
conductance. 2
(B) Explain the following with reference to
(C) Silver crystallizes as cubic close packed (ccp)
coordination compounds : 2×2=4
lattice. X-ray analysis reveals that edge length of
its unit cell is 408.6 pm. Calculate the density of (i) Coordination complex or ionization sphere
silver. (M Ag = 107.9) 2 (ii) Ligands and their classification
(D) Explain the difference between physical and (C) Write short note on the following : 2×2=4
chemical adsorption by giving suitable examples. (i) Pseudounimolecular reactions
2
191

(ii) Density of 2.03 M aqueous solution of acetic (iv) benzene from phenol?
acid (CH 3COOH) is 1.017 g/mL. Molecular (v) benzoic acid from aniline?
mass of acetic acid is 60. Compute the 7. (A) Primary alkyl halide ( A ) C 4 H 9 Br reacted with
molality of solution. alcoholic KOH to give compound ( B). Compound
(D) Explain the following with reason : 2×2=4 ( B) is reacted with HBr to give (C ) which is an
(i) Boiling point of ammonia is higher than isomer of ( A ). When ( A ) reacted with Na metal, it
phosphine. gave a compound ( D) C 8 H18 that was different
(ii) Sugar turns black in presence of conc. than the compound when n-butyl bromide was
H 2SO 4 . reacted with sodium. Give the structural formula
6. (A) Write two general methods of preparation of of ( A ) and write the equation for all the reactions.
phenol. How will you obtain following from 3+1+1=5
phenol : 5 Or
(i) chlorobenzene (ii) aniline (iii) benzaldehyde How will you convert chlorobenzene into?
Or 1×5=5
What are monohydric alcohols? Why their boiling (i) benzene (ii) phenol (iii) toluene
points are high? Describe industrial method of (iv) DDT (v) aniline
preparation of methanol by destructive (B) What happens when? Give chemical equations
distillation of wood giving diagram. Give reaction also. 1×5=5
of methanol with concentrated sulphuric acid (i) Glucose is reacted with hydroxylamine.
also. 5 (ii) Glucose reacts with an excess of phenyl
(B) What happens when : 1×5=5 hydrazine.
(i) acetone reacts with ethyleneglycol ? (iii) Cane sugar is hydrolysed with dilute
(ii) formaldehyde reacts with NH 3 ? sulphuric acid.
(iii) acetaldehyde reacts with hydrazine ? (iv) Aqueous solution of glucose is heated with
(iv) acetone reacts with chloroform ? Fehling solution.
(v) acetic acid reacts with hydrazoic acid ? (v) Glucose reacts with ammonical solution of
Or silver nitrate.
How will you obtain : 1×5=5 Or
(i) mesitylene from acetone? How will you obtain glucosazone, gluconic acid,
n-hexane and sorbitol from glucose? 5
(ii) acetaldehyde from methyl cyanide?
(iii) benzoic acid from benzaldehyde?

Model Test Paper-5


Chemistry (Class-12)
(Only Paper)
Time : 3.15 Hrs. M.M. 70
Instruction : All the questions are compulsory. Marks assigned to each question are given in front of it.

1. Each section of this question contains four options. (c) van der Waals (d) H-bond
Select the correct option and write it in answer book. (D) Anhydride of HNO 3 is:
1´ 6 = 6 (a) NO 2 (b) NO
(A) The vacant space in body-centred cubic lattice (c) N 2O 5 (d) N 2O
(bcc) unit cell is about: (E) Among the following, which functional groups
(a) 32% (b) 10% (c) 23% (d) 46% may be present in carbohydrates is/are:
(B) Bidentate ligand ethylenediammine is expressed (a) —OH (b) —CHO
by : (c) —CO — (d) All of these
(a) en (b) ox (c) et (d) py (F) Calcium acetate on heating gives:
(C) Forces involved in physical adsorption is : (a) Acetic anhydride (b) Acetone
(a) ionic (b) covalent (c) Acetaldehyde (d) Ethyl alcohol
192

2. (A) Ethylamine is basic in nature, why ? 2 Na-atoms are at centre. What will be the
(B) Explain why molarity varies with temperature molecular formula of compound? 2
while molality does not? 2 6. (A) What do you understand by secondary structure
(C) Differentiate between Schottky and Frenkel of proteins? Write the factors which stabilises the
defects. 2 structure of proteins. 5
(D) Calculate the mole fraction of solute in 1 molal Or
solution. 2 Explain the following with reference to proteins.
3. (A) Define specific conductance and molar 5
conductance of an electrolytic solution. 2 (i) Peptide bonds (ii) Primary structure
(B) An organic compound A, on reduction gives (iii) Denaturation (iv) Zwitter ion
amine C 2 H7 N, which reacts with CHCl 3 /KOH to
(v) Isoelectric point
give a foul smelling compound B. Write the name
(B) Giving suitable diagram, explain the laboratory
and structural formula of A and B. 2
method of preparation of chloroform. Write its
(C) Explain what are primary and secondary
two important properties and uses. 5
valencies in coordination complexes? 2
Or
(D) What is aqua-regia? How does it react with Au? 2
Explain the following reactions with suitable
4. (A) Prove that for a first order reaction, time required
example. 5
for 3/4 completion is 2 times to the half time. 3
(B) (i) A solution of 0.591 gram of a substance (mol. (i) Finklsteins reaction (ii) Swart’s reaction
mass = 58) is prepared is 100 gram acetic acid. (iii) Darzen reaction
Freezing point of resulting solution is 0.32°C less (iv) Hunsdicker reaction (v) Fittig reaction
than that of pure acetic acid (1.66°C). Calculate 7. (A) Write two general methods of preparation of
the value of molal depression constant of acetic phenol. How will you obtain following from
acid. 2 phenol : 5
(ii) Write effect of temperature on adsorption. 1 (i) chlorobenzene (ii) picric acid
(C) What is the difference between reversible and (iii) salicylic acid?
irreversible colloids? Give example of each. 3 Write the related equations also.
(D) On the basis of spontaneity of following Or
reactions, arrange Mg, Zn, Cu and Ag in What happens when : (Write equations only) 5
decreasing order of their electrode potentials. 3
(i) ethyl alcohol reacts with acetic acid in
Cu + 2Ag + ¾® Cu 2+ + 2Ag
presence of conc. H 2SO 4 ?
Mg + Zn 2+ ¾® Mg 2+ + Zn (ii) diethyl ether reacts with cold HI?
Zn + Cu 2+ ¾® Zn 2+ + Cu (iii) methyl alcohol reacts with salicylic acid in
5. (A) On the basis of electronic configuration, explain presence of conc. H 2SO 4 ?
the position of N, P, As, Sb and Bi in periodic (iv) benzene diazonium chloride is heated with
table. 4 steam?
(B) (i) Explain why transition metals show variable (v) phenol reacts with Br2 water?
valencies? 2 (B) Write two general methods of preparation of
(ii) What is lanthanide contraction? Write its acetone. How does chloroform and conc. H 2SO 4
two effects. 2 react with acetone? Write chemical equations
(C) Define coordination number. Also determine the also. 5
oxidation number of copper in [Cu( NH 3 ) 4 ]SO 4 . Or
4 How will you obtain acetic acid by quick vinegar
(D) (i) Half life of a first order reaction is 60 min. method? Give diagram and equations also. How
How much time will it take in 90% will you obtain ethyl alcohol and acetone from
completion? 2 acetic acid? 5
(ii) In a cubic crystal, tungsten (W) and oxygen
(O) atoms are at corners of cube while

You might also like